You are on page 1of 728

IBPS RRB PO Prelims – Ultra Practice Bundle PDF

IBPS RRB PO Prelims – Reasoning Ability

S.no Topic Page number


1. Seating- Linear
Questions 6
Solutions 14
2. Seating- Circle
Questions 23
Solutions 30
3. Seating- Square

Page 1 of 728

Subscribe the Xpress Video Course & Mock Test Package for Bank & Insurance Exams
If there are any suggestions/ errors in our PDFs Feel Free to contact us via this email: admin@exampundit.in
IBPS RRB PO Prelims – Ultra Practice Bundle PDF
Questions 43
Solutions 52
4. Seating- Rectangle
Questions 65
Solutions 74
5. Seating- Triangle
Questions 94
Solutions 103
6. Certain Number - Linear
Questions 112
Solutions 120
7. Blood Relation
Questions 135
Solutions 143
8. Number Sequence
Questions 151
Solutions 158
9. Alphabet Series
Questions 164
Solutions 172
10. Coding Decoding
Questions 182
Solutions 191
11. Inequality
Questions 196
Solutions 208
12. Direction
Questions 217

Page 2 of 728

Subscribe the Xpress Video Course & Mock Test Package for Bank & Insurance Exams
If there are any suggestions/ errors in our PDFs Feel Free to contact us via this email: admin@exampundit.in
IBPS RRB PO Prelims – Ultra Practice Bundle PDF
Solutions 225
13. Alpha Numeric Series
Questions 241
Solutions 249
14. Data Sufficiency
Questions 258
Solutions 278
15. Puzzle- Days
Questions 299
Solutions 309
16. Seating- Parallel Row
Questions 341
Solutions 350
17. Puzzle- Floor with Flat
Questions 358
Solutions 367
18. Certain Number- Circle
Questions 378
Solutions 384
19. Puzzle- Floor
Questions 398
Solutions 407
20. Puzzle- Box/Stack
Questions 419
Solutions 427
21. Puzzle- Month
Questions 440
Solutions 449

Page 3 of 728

Subscribe the Xpress Video Course & Mock Test Package for Bank & Insurance Exams
If there are any suggestions/ errors in our PDFs Feel Free to contact us via this email: admin@exampundit.in
IBPS RRB PO Prelims – Ultra Practice Bundle PDF
22. Puzzle- Month and Date
Questions 461
Solutions 470
23. Puzzle- Random
Questions 481
Solutions 490
24. Syllogism
Questions 502
Solutions 515
25. Conditional Coding
Questions 524
Solutions 534

IBPS RRB PO Prelims – Quantitative Aptitude


S.no Topic Page number
1. Approximation
Questions 539
Solutions 544

2. Simplification
Questions 550
Solutions 556

3. Missing Number
Questions 560
Solutions 565

4. Wrong Number
Questions 572
Solutions 578

Page 4 of 728

Subscribe the Xpress Video Course & Mock Test Package for Bank & Insurance Exams
If there are any suggestions/ errors in our PDFs Feel Free to contact us via this email: admin@exampundit.in
IBPS RRB PO Prelims – Ultra Practice Bundle PDF
5. Quadratic Equation
Questions 585
Solutions 593

6. DI – Table
Questions 609
Solutions 622

7. DI- Bar
Questions 631
Solutions 646

8. DI- Line
Questions 653
Solutions 668

9. DI- Pie
Questions 682
Solutions 702

10. Caselet DI
Questions 708
Solutions 718

Page 5 of 728

Subscribe the Xpress Video Course & Mock Test Package for Bank & Insurance Exams
If there are any suggestions/ errors in our PDFs Feel Free to contact us via this email: admin@exampundit.in
IBPS RRB PO Prelims – Ultra Practice Bundle PDF
1). Linear Seating Arrangement
Directions (1-5): Study the following information (a) B
carefully and answer the questions given below: (b) D
There are six persons A, B, C, D, E, and F are sitting in a (c) A
linear line from west to east but not necessarily in the (d) E
same order. Among them, some are facing north and (e) None of these
some are facing south direction. F, who does not face 4. What is the position of F with respect to A?
south direction, sits immediate right to B. B sits second (a) Second to the left
to the right of D. C and E facing the opposite direction to (b) Third to the left
each other.D is not an immediate neighbor of A. Two (c) Second to the right
persons are sitting between B and A, who faces south (d) Fourth to the right
direction. Only one person sits between E and F. Both E (e) None of these
and F facing the same direction. B and C facing the 5. C faces in which direction?
opposite direction to D. (a) North
1. What is the position of F with respect to D? (b) South
(a) Third to left (c) Both (a) and (d)
(b) Immediate right (d) Same as B faces
(c) Immediate left (e) Cannot be determined
(d) Second to right Directions (6-10): Study the following information
(e) None of these carefully and answer the questions given below:
2. How many persons facing North direction? There are eight persons A, B, C, D, E, F, G, and H are
(a) One sitting in a row facing towards the north but not
(b) Four necessarily in the same order. No two persons are sitting
(c) Two together according to the English alphabetical order (for
(d) Three ex- A can’t sit with B and B can’t sit with A and C and
(e) None of these so on). C does not sit at the end of the row. B sits on the
3. Who is sitting second to the right of C? immediate right of F. H sits third to the right of D, who
Page 6 of 728

Subscribe the Xpress Video Course & Mock Test Package for Bank & Insurance Exams
If there are any suggestions/ errors in our PDFs Feel Free to contact us via this email: admin@exampundit.in
IBPS RRB PO Prelims – Ultra Practice Bundle PDF
does not sit at the end of the row. Only one person sits (c) Three
between C and H. Only one person sits between G and E. (d) One
6. Four of the following five options are alike in a (e) Two
certain way, find the odd one out. 10. Which of the following statement is true?
(a) D, B (a) D sits fourth to the left of A.
(b) D, C (b) F sits third to the right of E.
(c) A, F (c) B sits fourth to the left of G.
(d) E, G (d) E sits immediate right of C.
(e) H, C (e) None of these
7. Who among the following is sitting at any of the Directions (11-15): Study the following information
extreme ends? carefully and answer the questions given below:
(I) A There are eight friends J, K, L, M, N, O, P, and Q who
(II) G are sitting in a straight line but not necessarily in the
(III) E same order. Some of them are facing south while some
(a) Only I are facing north. Immediate neighbors of K facing
(b) Only III opposite directions to each other. People sitting at the
(c) Only II and III extreme ends face the opposite directions. M faces south
(d) Only I and II direction. Q does not sit near to L. L sits fourth to the left
(e) Only II of N. N sits at one of the extreme ends of the line. Both
8. What is the position of G with respect to C? the immediate neighbors of L facing north direction. K
(a) Immediate left sits second to the left of P. P is not an immediate
(b) Third to the right neighbor of L. Neither P nor J sits at the extreme ends of
(c) Immediate right the line. J faces the opposite direction to M. Both the
(d) Fourth to the left immediate neighbors of J facing north direction. Q sits
(e) None of these second to the left of O. Immediate neighbors of M face
9. How many persons sit between G and A? opposite directions.
(a) Five 11. Which of the following pairs represents seated at
(b) Six the extreme ends of the line?
Page 7 of 728

Subscribe the Xpress Video Course & Mock Test Package for Bank & Insurance Exams
If there are any suggestions/ errors in our PDFs Feel Free to contact us via this email: admin@exampundit.in
IBPS RRB PO Prelims – Ultra Practice Bundle PDF
(a) M, N 15. Who amongst the following sits third to the left of
(b) Q, L P?
(c) Q, N (a) J
(d) J, L (b) M
(e) L, K (c) L
12. How many persons are seated between M and N? (d) K
(a) Four (e) None of these
(b) More than four Directions (16-20): Study the following information
(c) One carefully and answer the questions given below:
(d) Three There are eight persons A, B, C, D, E, F, G, and H are
(e) Two sitting in a row but not necessarily in the same order.
13. If each of the persons is made to sit in alphabetical Some of them are facing south while some are facing
order from right to left the positions of how many will north. Not More than two persons facing the same
remain unchanged as compared to the original direction sit together. E, who sits to the immediate left of
seating arrangement? G. C faces north. C is not an immediate neighbor of A. F
(a) One and A faces the same direction. B neither sits to the left
(b) Two of D nor faces north. E faces south direction. Three
(c) Four persons sit between A and F. F sits at one of the ends. D
(d) None sits at one of the extreme ends and faces north. H sits
(e) Three second to the right of A and faces the opposite direction
14. Who amongst the following sits exactly between O of A. G sits third to the right of H.
and K? 16. If each of the persons is made to sit in alphabetical
(a) N order from right to left the positions of how many will
(b) Q remain unchanged as compared to the original
(c) P seating arrangement?
(d) L (a) One
(e) M (b) Two
(c) Four
Page 8 of 728

Subscribe the Xpress Video Course & Mock Test Package for Bank & Insurance Exams
If there are any suggestions/ errors in our PDFs Feel Free to contact us via this email: admin@exampundit.in
IBPS RRB PO Prelims – Ultra Practice Bundle PDF
(d) None Directions (21-25): Study the following information
(e) Three carefully and answer the questions given below:
17. Who sits immediate left of C? There are eight friends S, T, U, V, W, X, Y, and Z are
(a) H sitting in a linear table and all are facing towards the
(b) B south but not necessarily in the same order. There are
(c) G three friends sit between S and V. T and S sit together. Y
(d) F sits immediate left of X. U is immediate right of the W.
(e) E Z sits at one of the extreme ends. V sits third to the right
18. How many persons sit between G and B? of the Z, who is an immediate neighbor of Y. X does not
(a) Three sit near to U.
(b) Four 21. What is the position of V with respect to T?
(c) One (a) Third to left
(d) No one (b) Immediate right
(e) None of these (c) Immediate left
19. Who among the following sits exactly between D (d) Second to left
and G? (e) None of these
(a) A 22. Who sits immediately left of W?
(b) C (a) V
(c) F (b) S
(d) D (c) Z
(e) E (d) X
20. What is the position of C with respect to B? (e) T
(a) Third to left 23. How many persons sit between Y and Z?
(b) Immediate right (a) Three
(c) Immediate left (b) Four
(d) Second to left (c) One
(e) None of these (d) No one
(e) None of these
Page 9 of 728

Subscribe the Xpress Video Course & Mock Test Package for Bank & Insurance Exams
If there are any suggestions/ errors in our PDFs Feel Free to contact us via this email: admin@exampundit.in
IBPS RRB PO Prelims – Ultra Practice Bundle PDF
24. Who among the following sits exactly between W 26. Four of the following five options are alike in a
and X? certain way, find the odd one out.
(a) V (a) P
(b) S (b) V
(c) Z (c) S
(d) T (d) U
(e) U (e) R
25. How many persons sit between Z and W? 27. Who is sitting exactly between T and S?
(a) One (a) Q
(b) Two (b) U
(c) Four (c) W
(d) None (d) P
(e) Three (e) V
Directions (26-30): Study the following information 28. What is the position of U with respect to V?
carefully and answer the questions given below: (a) Immediate left
There are eight friends P, Q, R, S, T, U, V, and W are (b) Third to the right
sitting in a linear table. Five of them are facing North (c) Immediate right
and three are facing South but not necessarily in the (d) Fourth to the left
same order. Q faces south. V is the immediate neighbor (e) None of these
of the one who is facing north. There are four people 29. How many persons sit between T and S?
between P and R. T sits fifth to the right of V. P and S (a) Five
are sitting at the extreme ends. T faces the direction (b) Six
opposite to that of W. V sits third to the right of U. As (c) Three
many persons sit between T and W is same as the (d) One
number of persons sits to the right of R. P is sitting third (e) Two
to the left of the U. Q is sitting second to the left of W. V 30. Which of the following statement is true?
is not an immediate neighbor of P. P and S are facing the (a) U sits fourth to the left of T.
opposite direction to V. (b) S sits third to the right of V.
Page 10 of 728

Subscribe the Xpress Video Course & Mock Test Package for Bank & Insurance Exams
If there are any suggestions/ errors in our PDFs Feel Free to contact us via this email: admin@exampundit.in
IBPS RRB PO Prelims – Ultra Practice Bundle PDF
(c) R sits fourth to the left of T. 33. If each of the persons is made to sit in alphabetical
(d) P sits immediate right of U. order from right to left the positions of how many will
(e) None of these remain unchanged as compared to the original
Directions (31-35): Study the following information seating arrangement?
carefully and answer the questions given below: (a) One
Nine people P, Q, R, S, T, U, V, W, and X are sitting in a (b) Two
linear table facing north direction but not necessarily in (c) Four
the same order. Two persons sit between X and V. W sits (d) None
just to the right of R. One person sits between W and P. (e) Three
Two persons sit between the Q and P. Q does not sit just 34. Who amongst the following sits exactly between X
to the right of W. Three persons sit between X and W. X and U?
sits fourth from the right end. R sits just to the left of W. (a) T
X sits just to the right of T. U sits immediately left to the (b) Q
V. (c) P
31. Which of the following pairs represents the (d) W
persons seated at the extreme ends of the line? (e) V
(a) R, V 35. Who amongst the following sits third to the right
(b) P, T of S?
(c) U, X (a) U
(d) P, V (b) Q
(e) R, T (c) R
32. How many persons are seated between P and U? (d) X
(a) Four (e) None of these
(b) More than four Directions (36-40): Study the following information
(c) One carefully and answer the questions given below:
(d) Three There are seven people P, Q, R, S, T, U, and V sit in a
(e) Two linear table. Only two persons are facing south direction
and others are facing north direction but not necessarily
Page 11 of 728

Subscribe the Xpress Video Course & Mock Test Package for Bank & Insurance Exams
If there are any suggestions/ errors in our PDFs Feel Free to contact us via this email: admin@exampundit.in
IBPS RRB PO Prelims – Ultra Practice Bundle PDF
in the same order. Q faces the north direction and sits (c) Second to the right
third to the right of S. There are three persons sits (d) Fourth to the right
between P and R. V sits third to the right of Q. U sits (e) None of these
immediate right of R. P and R facing the same direction. 40. P faces in which direction?
U and V facing the same direction as S. P sit immediate (a) North
left of T who is an immediate neighbor of Q. S sits at (b) South
one of the extreme ends of the table. (c) Same as R faces
36. What is the position of T with respect to V? (d) Both (b) and (c)
(a) Fourth to left (e) Cannot be determined
(b) Immediate right Directions (41-45): Study the following information
(c) Immediate left carefully and answer the questions given below:
(d) Second to right There are seven people J, K, L, M, N, O, and P are sitting
(e) None of these in a linear table. Some of them facing north and some of
37. How many persons sit between S and Q? them facing south direction but not necessarily in the
(a) One same order. O sits immediate right of P who does not sit
(b) Four adjacent to K. L and K are immediate neighbors of N. P
(c) Two sits immediate left of O. The persons who sit at the
(d) Three extreme ends facing the opposite direction to each other.
(e) None of these O does not face south direction. M is an immediate
38. Who is sitting second to the right of T? neighbor of J. There are more than two persons sit
(a) P between J and L. J and K are facing the opposite
(b) U direction to M. N does not face south direction. J sits
(c) S second from one of the extreme ends. Three persons sit
(d) R between J and N. K sits second to the right of L.
(e) None of these 41. What is the position of P with respect to K?
39. What is the position of P with respect to R? (a) Third to left
(a) Second to the left (b) Immediate right
(b) Third to the left (c) Immediate left
Page 12 of 728

Subscribe the Xpress Video Course & Mock Test Package for Bank & Insurance Exams
If there are any suggestions/ errors in our PDFs Feel Free to contact us via this email: admin@exampundit.in
IBPS RRB PO Prelims – Ultra Practice Bundle PDF
(d) Second to right There are eight people A, B, C, D, E, F, G, and H are
(e) None of these sitting in a linear table facing north direction but not
42. How many persons facing north direction? necessarily in the same order. Two people are sitting
(a) One between C and D. F is sitting second to the left of C. B is
(b) Four sitting to the immediate left of D. H is sitting at the
(c) Two extreme left end. H is sitting second to the left of D.
(d) Three Only one person is sitting between F and C. A is an
(e) None of these immediate neighbor of C. E and G are immediate
43. Who is sitting second to the left of J? neighbors. There are two persons sit between G and F. G
(a) O sits immediate left of E.
(b) K 46. Four of the following five options are alike in a
(c) N certain way, find the odd one out.
(d) L (a) H, B
(e) None of these (b) A, C
44. What is the position of J with respect to O? (c) D, F
(a) Second to the left (d) C, G
(b) Third to the left (e) H, C
(c) Second to the right 47. Who among the following is sitting at any of the
(d) Fourth to the right extreme ends?
(e) None of these (I) A
45. M faces in which direction? (II) G
(a) North (III) E
(b) South (a) Only I
(c) Same as P faces (b) Only III
(d) Both (a) and (c) (c) Only II and III
(e) Cannot be determined (d) Only I and II
Directions (46-50): Study the following information (e) Only II
carefully and answer the questions given below: 48. What is the position of G with respect to C?
Page 13 of 728

Subscribe the Xpress Video Course & Mock Test Package for Bank & Insurance Exams
If there are any suggestions/ errors in our PDFs Feel Free to contact us via this email: admin@exampundit.in
IBPS RRB PO Prelims – Ultra Practice Bundle PDF
(a) Immediate left (d) One
(b) Third to the right (e) Two
(c) Immediate right 50. Which of the following statement is true?
(d) Fourth to the left (a) D sits fourth to the left of A.
(e) None of these (b) F sits third to the right of E.
49. How many persons sit between G and A? (c) B sits fourth to the left of G.
(a) Five (d) E sits immediate right of C.
(b) Six (e) None of these
(c) Three

1). Linear Seating Arrangement - Solutions with Explanation


SOLUTION (1-5): 3. F, who does not face south direction, sits
immediate right to B.
Explanation in detail:
4. B sits second to the right of D.
1. Two persons are sitting between B and A. 5. D is not an immediate neighbor of A.
2. A faces south direction. 6. B facing the opposite direction to D.

CASE1: CASE1:

CASE2:
CASE2:

Page 14 of 728

Subscribe the Xpress Video Course & Mock Test Package for Bank & Insurance Exams
If there are any suggestions/ errors in our PDFs Feel Free to contact us via this email: admin@exampundit.in
IBPS RRB PO Prelims – Ultra Practice Bundle PDF
7. Only one person sits between E and F. 1. H sits third to the right of D.
8. Both E and F facing the same direction. 2. D does not sit at the end of the row.
9. C facing the opposite direction to D.

CASE1:

3. Only one person sits between C and H.


4. C does not sit at the end of the row.
CASE2:
NOTE: No two persons are sitting together
according to the English alphabetical order (for
ex- A can’t sit with B and B can’t sit with A and C
and so on).
10. C and E are facing the opposite direction to each
other.

So CASE1 is canceled out.

Final arrangement:
5. Only one person sits between G and E.
6. B sits on the immediate right of F.

1. b
2. d
3. a
Final arrangement:
4. d
5. c

SOLUTION (6-10):

Explanation in detail: 6. b

Page 15 of 728

Subscribe the Xpress Video Course & Mock Test Package for Bank & Insurance Exams
If there are any suggestions/ errors in our PDFs Feel Free to contact us via this email: admin@exampundit.in
IBPS RRB PO Prelims – Ultra Practice Bundle PDF
7. d
8. c
9. b
10. c

SOLUTION (11-15): CASE2:

Explanation in detail:

1. N sits at one of the extreme ends of the line.


2. L sits fourth to the left of N. 8. Q sits second to the left of O. Immediate
3. Both the immediate neighbors of L facing north neighbors of M face opposite directions.
direction. 9. Q does not sit near to L.

CASE1: CASE1:

CASE2:
CASE2:

There is a condition for Q and L that they are not


adjacent to each other, So CASE1 is canceled out.
4. K sits second to the left of P.
5. P is not an immediate neighbor of L. 10. Both the immediate neighbors of J facing north
6. P does sit at the extreme end of the line. direction.
7. People sitting at the extreme ends face the 11. Immediate neighbors of K facing opposite
opposite directions. directions to each other.
12. M faces to south direction.
CASE1:
Page 16 of 728

Subscribe the Xpress Video Course & Mock Test Package for Bank & Insurance Exams
If there are any suggestions/ errors in our PDFs Feel Free to contact us via this email: admin@exampundit.in
IBPS RRB PO Prelims – Ultra Practice Bundle PDF
13. Neither P nor J sits at the extreme end of the CASE2:
line.
14. J faces opposite direction to M.

CASE2:
4. B neither sits to the left of D nor faces north.

CASE1:

Final arrangement:

CASE2:

11. c
12. b
13. b
There is no place for B in CASE1 so CASE1 is
14. d
canceled out.
15. c
5. H sits second to the right of A and faces the
SOLUTION (16-20):
opposite direction of A.
Explanation in detail: 6. G sits third to the right of H.

1. Three persons sit between A and F. CASE2(a):


2. F sits at one of the ends.
3. D sits at one of the extreme ends and faces north.

CASE1:
CASE2(b):

Page 17 of 728

Subscribe the Xpress Video Course & Mock Test Package for Bank & Insurance Exams
If there are any suggestions/ errors in our PDFs Feel Free to contact us via this email: admin@exampundit.in
IBPS RRB PO Prelims – Ultra Practice Bundle PDF
18. c
19. e
20. d
7. E, who sits to the immediate left of G.
8. C faces north. SOLUTION (21-25):

9. C is not an immediate neighbor of A. Explanation in detail:


10. F and A faces the same direction.
1. Z sits at one of the extreme ends.
11. B neither sits to the left of D nor faces north.
2. V sits third to the right of the Z.
12. E faces south direction.
3. Z is an immediate neighbor of Y.
CASE2(a):

4. There are three friends sit between S and V.


5. T and S sit together.
CASE2 (b):

6. Y sits immediate left of X.


7. U is the immediate right of the W.
Note: Not More than two persons facing the same 8. X does not sit near to U.
direction sit together. So case2a is canceled out.

Final arrangement:

21. a
22. a
23. d
16. d
24. a
17. a
25. e
Page 18 of 728

Subscribe the Xpress Video Course & Mock Test Package for Bank & Insurance Exams
If there are any suggestions/ errors in our PDFs Feel Free to contact us via this email: admin@exampundit.in
IBPS RRB PO Prelims – Ultra Practice Bundle PDF
SOLUTION (26-30):

Explanation in detail:

1. P and S are sitting at the extreme ends.


Final arrangement:
2. There are four people between P and R.
3. P is sitting third to the left of the U.
4. U facing in the north direction.

26. b
27. c
5. V sits third to the right of U. 28. b
6. T sits fifth to the right of V. 29. a
30. c

SOLUTION (31-35):

Explanation in detail:

7. V is the immediate neighbor of who facing north. 1. X sits fourth from the right end.

8. T faces the direction opposite to that of W. 2. Two persons sit between X and V.

9. There are as many persons sit between T and W 3. U sits immediately left to the V.

as the number of persons sits to the right of R. 4. Three persons sit between X and W.

10. Q is sitting second to the left of W.


11. V is not an immediate neighbor of P.
12. P and S are facing the opposite direction to V.
13. Q faces south.
5. W sits just to the right of R.
6. One person sits between W and P.
7. Two persons sit between the Q and P.

Page 19 of 728

Subscribe the Xpress Video Course & Mock Test Package for Bank & Insurance Exams
If there are any suggestions/ errors in our PDFs Feel Free to contact us via this email: admin@exampundit.in
IBPS RRB PO Prelims – Ultra Practice Bundle PDF

8. X sits just to the right of T. 4. P sits immediate left of T who is an immediate


9. Q does not sit just to the right of W. neighbor of Q.
5. There are three persons sits between P and R.

CASE1:

Final arrangement:

CASE2:

31. a
32. d
6. U sits immediate right of R.
33. b
34. b 7. P and R facing the same direction.

35. d 8. U and V facing the same direction as S.

SOLUTION (36-40): CASE1:

Explanation in detail:

1. S sits at one of the extreme ends of the table.


2. Q faces the north direction and sits third to the
CASE2:
right of S.
3. V sits third to the right of Q.

Page 20 of 728

Subscribe the Xpress Video Course & Mock Test Package for Bank & Insurance Exams
If there are any suggestions/ errors in our PDFs Feel Free to contact us via this email: admin@exampundit.in
IBPS RRB PO Prelims – Ultra Practice Bundle PDF
NOTE: Only two persons are facing south direction
and others are facing north direction.

So, CASE1 is canceled out.


4. L and K are immediate neighbors of N.
Final arrangement: 5. K sits second to the right of L.

CASE1:

36. a
37. c
CASE2:
38. b
39. d
40. d

SOLUTION (41-45):
6. O sits immediate right of P who does not sit
Explanation in detail: adjacent to K.
7. P sits immediate left of O.
1. J sits second from one of the extreme ends.
8. The persons who sit at the extreme ends facing
2. Three persons sit between J and N.
the opposite direction to each other.
3. There are more than two persons sit between J
and L. CASE1:

CASE1:

CASE2:

CASE2:
Page 21 of 728

Subscribe the Xpress Video Course & Mock Test Package for Bank & Insurance Exams
If there are any suggestions/ errors in our PDFs Feel Free to contact us via this email: admin@exampundit.in
IBPS RRB PO Prelims – Ultra Practice Bundle PDF

9. O does not face south direction. 4. Only one person is sitting between F and C.
5. F is sitting second to the left of C.
So CASE1 is canceled out.
6. B is sitting to the immediate left of D.
10. J and K are facing the opposite direction to M.
11. N does not face south direction.

CASE2:

7. A is an immediate neighbor of C.
8. E and G are immediate neighbors.

Final arrangement: 9. There are two persons sit between G and F.


10. G sits immediate left of E.

41. d
42. b
Final arrangement:
43. a
44. a
45. d

SOLUTION (46-50): 46. e

Explanation in detail: 47. b


48. c
1. H is sitting at the extreme left end.
49. d
2. H is sitting second to the left of D.
50. e
3. Two people are sitting between C and D.
Page 22 of 728

Subscribe the Xpress Video Course & Mock Test Package for Bank & Insurance Exams
If there are any suggestions/ errors in our PDFs Feel Free to contact us via this email: admin@exampundit.in
IBPS RRB PO Prelims – Ultra Practice Bundle PDF
2). Circle Seating Arrangement
Directions (1-5): Answer the questions based on the (c) G
information given below: (d) B
There are eight persons A, B, C, D, E, F, G and H are (e) E
sitting around a circular table facing to the centre but not 4. Who sits third to the left of C?
necessarily in the same order. F sits third to right of A. C (a) F
sits second to the left of D. D is not an immediate (b) E
neighbor of A and F. D is an immediate neighbor of H. (c) D
There are three persons sit between A and C. Only one (d) A
person sit between H and B. E and G are immediate (e) B
neighbors. Only one person sits between H and B. B sit 5. E sits second to the left of?
opposite to F. G sits to immediate right of E. (a) B
1. What is the position of C with respect to H? (b) D
(a) third to right (c) F
(b) second to left (d) A
(c) third to left (e) None of these
(d) Immediate left Directions (6-10): Answer the questions based on the
(e) None of these information given below:
2. What is the position of F with respect to E? There are eight friends A, B, C, D, E, F, G and H sitting
(a) second to right around a circular table and all are facing towards the
(b) immediate left centre but not necessarily in the same order.
(c) third to right E is sitting third to the left of G. C is to the immediate
(d) second to left right of E. B is sitting fourth to the right of H. H is not an
(e) immediate right immediate neighbor of G. Neither B nor H is an
3. Who sits immediate left of H? immediate neighbor of E. B is sitting second to the left of
(a) D F. H is sitting second to the left of E. A is sitting exactly
(b) C

Page 23 of 728

Subscribe the Xpress Video Course & Mock Test Package for Bank & Insurance Exams
If there are any suggestions/ errors in our PDFs Feel Free to contact us via this email: admin@exampundit.in
IBPS RRB PO Prelims – Ultra Practice Bundle PDF
between F and H. D is sitting second to the right of the (b) A
A. C is sitting third to the left of F. (c) H
6. What is the position of B with respect to H? (d) C
(a) third to right (e) G
(b) second to left Directions (11-15): Answer the questions based on the
(c) third to left information given below:
(d) fourth to right There are eight persons J, K, L, M, N, O, P and Q sitting
(e) Cannot be determined around a circular table and all are facing away from the
7. What is the position of D with respect to E? centre but not necessarily in the same order.
(a) second to right J and L can never sit together. There are two persons sits
(b) immediate left between the K and M. M sits second to the right of O. Q
(c) third to right is also an immediate neighbor of M. J cannot sit with Q.
(d) second to left There are two persons sitting between L and N. K and N
(e) immediate right are immediate neighbors of each other. J sits on the
8. Who sits opposite to A? immediate left of O. J sits second to the right K. N and P
(a) C are not immediate neighbors. P sits on the immediate
(b) G right of Q.
(c) B 11. Who sits to the immediate left of L?
(d) D (a) M
(e) Cannot be determined (b) O
9. Who is sitting third to the left of C? (c) K
(a) D (d) J
(b) A (e) None of these
(c) H 12. Who sits second to the left of P?
(d) G (a) O
(e) B (b) L
10. B sits opposite to whom? (c) Q
(a) D (d) M
Page 24 of 728

Subscribe the Xpress Video Course & Mock Test Package for Bank & Insurance Exams
If there are any suggestions/ errors in our PDFs Feel Free to contact us via this email: admin@exampundit.in
IBPS RRB PO Prelims – Ultra Practice Bundle PDF
(e) None of these centre. E sits third to the right of F. C sits third to the left
13. N is sitting third to the left of? of F. B sits opposite to F. D is not the immediate
(a) L neighbor of F and faces away from the centre. A sit
(b) M second to the left of C. G sits between F and H. B faces
(c) K away from the centre. A and B are not neighbours.
(d) J 16. What is the position of A with respect to B?
(e) None of these (a) third to right
14. Who is sitting second to the right of O? (b) second to left
(a) O (c) third to left
(b) J (d) fourth to right
(c) M (e) Cannot be determined
(d) L 17. What is the position of H with respect to E?
(e) None of these (a) second to right
15. How many persons sit between Q and J? (b) immediate left
(a) One (c) third to right
(b) Four (d) second to left
(c)Three (e) immediate right
(d) Six 18. Who sits fourth to the left of H?
(e) None of these (a) A
Directions (16-20): Answer the questions based on the (b) D
information given below: (c) G
There are eight friends A, B, C, D, E, F, G and H sitting (d) B
around a circular table and Four of them are facing the (e) E
centre and other four are facing away from the centre but 19. Who sits second to the right of C?
not necessarily in the same order. E faces towards the (a) F
centre. Both the immediate neighbors of E faces away (b) A
from the centre. D faces away from the centre. Both the (c) H
immediate neighbors of D do not face away from the (d) E
Page 25 of 728

Subscribe the Xpress Video Course & Mock Test Package for Bank & Insurance Exams
If there are any suggestions/ errors in our PDFs Feel Free to contact us via this email: admin@exampundit.in
IBPS RRB PO Prelims – Ultra Practice Bundle PDF
(e) B (c) J
20. Who sits opposite to the A? (d) L
(a) F (e) None of these
(b) C 23. Who is sitting third to the left of K?
(c) E (a) O
(d) B (b) L
(e) G (c) P
Directions (21-25): Answer the questions based on the (d) M
information given below: (e) None of these
There are eight persons J, K, L, M, N, O, P and Q sitting 24. Who is sitting second to the right of L?
around a circular table and all are facing towards the (a) J
centre but not necessarily in the same order. (b) M
J sits third to the right of L. Only two people sit between (c) K
the L and Q. P and the K are immediate neighbors of (d) P
each other. P is not an immediate neighbor of L. Q sits (e) None of these
second to the left of N. Q is an immediate neighbor of O 25. Who sits opposite to one who sits immediate right
and M. L sits to the immediate right of N. O is not an of J?
immediate neighbor of J. P is not an immediate neighbor (a) P
of L. (b) L
21. Who is to the immediate left of L? (c) J
(a) M (d) M
(b) P (e) None of these
(c) O
(d) J Directions (26-30): Answer the questions based on the
(e) N information given below:
22. Who sits second to the left of Q? There are eight persons A, B, C, D, E, F, G and H are
(a) N sitting around a circular table and all of them facing
(b) P outside the centre but not necessarily in the same order.
Page 26 of 728

Subscribe the Xpress Video Course & Mock Test Package for Bank & Insurance Exams
If there are any suggestions/ errors in our PDFs Feel Free to contact us via this email: admin@exampundit.in
IBPS RRB PO Prelims – Ultra Practice Bundle PDF
E sits opposite to D. A sit second to left of D. B sits third 30. How many persons sit between G and E from
to left of A. F sits second to left of B. C is not an right of E?
immediate neighbor of A. H sits second to the left of G. (a) Six
A sit opposite to C. (b) Four
26. What is the position of D with respect to H? (c) None
(a) third to right (d) Two
(b) second to left (e) Three
(c) third to left
(d) fourth to right Directions (31-35): Answer the questions based on the
(e) Cannot be determined information given below:
27.What is the position of G with D? There are eight persons P, Q, R, S, T, U, V and W are
(a) second to right sitting around a circular table and all of them facing
(b) third to right outside the centre but not necessarily in the same order.
(c) immediate left R sits second to the left of P. Only one person is sitting
(d) second to left between R and U. Q is sitting second to the left of U. S
(e) immediate right sits third to right of Q. T sits second to right of S. Only
28. Who sits third to the right of C? one person is sitting between T and V. W is sitting
(a) E second to the right of V.
(b) G 31. Who sits to the immediate right of P?
(c) B (a) V
(d) D (b) Q
(e) H (c) R
29. Who sits immediate right of D? (d) T
(a) A (e) P
(b) F 32. Who among the following sits fourth to the right
(c) B of P?
(d) G (a) S
(e) C (b) Q
Page 27 of 728

Subscribe the Xpress Video Course & Mock Test Package for Bank & Insurance Exams
If there are any suggestions/ errors in our PDFs Feel Free to contact us via this email: admin@exampundit.in
IBPS RRB PO Prelims – Ultra Practice Bundle PDF
(c) R sitting between V and T. Three persons are sitting
(d) T between Q and V. P is sitting third to the left of Q. U is
(e) U sitting fifth to the left of W. R is not an immediate
33. Who sits third to the left of S? neighbour of Q. One person is sitting between R and P. S
(a) R is sitting third to the right of R. U is sitting fourth to the
(b) U left of the S.
(c) T 36. Who sits to the immediate left of Q?
(d) S (a) R
(e) Q (b) W
34. Who is an immediate neighbor of P and Q? (c) T
(a) S (d) S
(b) V (e) U
(c) R 37. Four of the following five are alike in a certain
(d) T way based on the given arrangement and hence form
(e) None of these a group. Which is the one that doesn’t belong to that
35. How many persons sit between who likes U and T group?
from left of T? (a) WR
(a) One (b) TP
(b) Four (c) TR
(c) Six (d) SU
(d) Two (e) PT
(e) Three 38. Who sits second to the right of P?
Directions (36-40): Answer the questions based on the (a) S
information given below: (b) W
Eight people P, Q, R, S, T, U, V and W are sitting around (c) R
a circular table facing the centre, not necessarily in the (d) Q
same order. Only one person is sitting between T and W. (e) T
T is sitting exactly opposite to the P. Two persons are 39. Who is second to the left of W?
Page 28 of 728

Subscribe the Xpress Video Course & Mock Test Package for Bank & Insurance Exams
If there are any suggestions/ errors in our PDFs Feel Free to contact us via this email: admin@exampundit.in
IBPS RRB PO Prelims – Ultra Practice Bundle PDF
(a) Q (e) None of these
(b) U 42. What is the position of W with V?
(c) W (a) Immediate right
(d) P (b) Immediate left
(e) S (c) Third to the right
40. How many persons sit between V and Q? (d) Third to the left
(a) One (e) None of these
(b) Four 43. What is T’s position with respect to U?
(c) Six (a) Third to the right
(d) Two (b) Second to the right
(e) Three (c) Immediate right
Directions (41-45): Answer the questions based on the (d) Second to the left
information given below: (e) Third to the left
There are eight persons P, Q, R, S, T, U, V and W are 44. Who is sitting third to the right of R?
sitting around a circular table four of them are facing the (a) U
centre and other four are facing away from the centre but (b) P
not necessarily in the same order. (c) Q
T and P are facing same direction. T is sitting between (d) W
the S and U. U faces the centre and sits third to the right (e) T
of R. S faces the W. V sits third to the right of Q. V is 45. Which one of the following statements is true
facing outside the centre and an immediate neighbor of according to the above-mentioned arrangement?
U. W is an immediate neighbor of P and V. T sits to (a) P is to the immediate left of S.
immediate left of S. Q faces inside. (b) W sits second to the right of Q.
41. Who is sitting third to the right of R? (c) V sits second to right of P.
(a) W (d) T sits fourth to the right of W.
(b) S (e) None of these
(c) P Directions (46-50): Answer the questions based on the
(d) U information given below:
Page 29 of 728

Subscribe the Xpress Video Course & Mock Test Package for Bank & Insurance Exams
If there are any suggestions/ errors in our PDFs Feel Free to contact us via this email: admin@exampundit.in
IBPS RRB PO Prelims – Ultra Practice Bundle PDF
There are eight persons A, B, C, D, E, F, G and H are (e) None of these
facing away from the centre but not necessarily in the 48. Who is sitting third to the left of B?
same order. Only one person sits between G and E. F and (a) A
A are immediate neighbors of each other. Only one (b) G
person sits between A and C. C sits third to the right of (c) D
H. E is not an immediate neighbor of either C or H. D is (d) F
an immediate neighbor of E. Three people sit between H (e) None of these
and B. F sits third to the left of E. 49. Who is immediate neighbors of E?
46. Who is to the immediate left of B? (a) B and D
(a) B (b) C and D
(b) C (c) G and H
(c) D (d) B and A
(d) G (e) None of these
(e) None of these 50. Who is sitting fourth to the right of G?
47. Who is in front of immediate left of H? (a) E
(a) C (b) D
(b) E (c) F
(c) G (d) B
(d) A (e) C

2). Circle Seating Arrangement - Solutions with Explanation


SOLUTION (1-5):

Explanation in detail:

1. F sits third to right of A.


2. C sits second to the left of D.
3. D is not an immediate neighbor of A and F.

Page 30 of 728

Subscribe the Xpress Video Course & Mock Test Package for Bank & Insurance Exams
If there are any suggestions/ errors in our PDFs Feel Free to contact us via this email: admin@exampundit.in
IBPS RRB PO Prelims – Ultra Practice Bundle PDF

9. Only one person sits between H and B.

4. B sit opposite to F.
5. G sits to immediate right of E.
6. G and E are immediate neighbors.

Final arrangement:

7. D is an immediate neighbor of H.
8. There are three persons sit between A and C.

Page 31 of 728

Subscribe the Xpress Video Course & Mock Test Package for Bank & Insurance Exams
If there are any suggestions/ errors in our PDFs Feel Free to contact us via this email: admin@exampundit.in
IBPS RRB PO Prelims – Ultra Practice Bundle PDF

6. F is sitting third to the the right of C.


1. d 7. B is sitting second to the left of F.
2. a 8. H is sitting second to the left of E.
3. b 9. A is sitting exactly between F and H.
4. b 10. D is sitting second to the right of the A.
5. c 11. C is sitting third to the left of F.

SOLUTION6-10): D

Explanation in detail: E H

1. E is sitting third to the left of G.


2. C is to the immediate right of E. C A
3. B is sitting fourth to the right of H.
4. H is not an immediate neighbor of G.
5. Neither B nor H is an immediate neighbor of E. B F
G

Final arrangement:
6. d
7. b
8. a
9. c
10. c
SOLUTION (11-15):

Page 32 of 728

Subscribe the Xpress Video Course & Mock Test Package for Bank & Insurance Exams
If there are any suggestions/ errors in our PDFs Feel Free to contact us via this email: admin@exampundit.in
IBPS RRB PO Prelims – Ultra Practice Bundle PDF
Explanation in detail:

1. M sits second to the right of O.


2. Q is an immediate neighbor of M.
3. P sits on the immediate right of Q.
4. There is only one person sitting between P and N
and that person can never be O.

Final arrangement:

5. There are two person sits between the persons K


and M.
6. J sits second to the right of M.
7. There are two persons sitting between L and N.
8. J and L can never sit together.

11. b
12. d
13. a
14. c
15. c
SOLUTION (16-20):

Explanation in detail:

Page 33 of 728

Subscribe the Xpress Video Course & Mock Test Package for Bank & Insurance Exams
If there are any suggestions/ errors in our PDFs Feel Free to contact us via this email: admin@exampundit.in
IBPS RRB PO Prelims – Ultra Practice Bundle PDF
1. E faces towards the centre.
2. E sits third to the right of F.
3. Both the immediate neighbors of E faces away
from the centre.
4. B sits opposite to F.
5. C sits third to the left of F.

8. D faces away from the centre.


9. Both the immediate neighbors of D do not face
away from the centre.

There is no such space for D in CASE1 So CASE1 is


6. A sit second to the left of C. cancelled out
7. G sits between F and H.

Page 34 of 728

Subscribe the Xpress Video Course & Mock Test Package for Bank & Insurance Exams
If there are any suggestions/ errors in our PDFs Feel Free to contact us via this email: admin@exampundit.in
IBPS RRB PO Prelims – Ultra Practice Bundle PDF
10. D is not the immediate neighbor of F and faces 18. b
away from the centre. 19. d
11. B faces away from the centre. 20. c
SOLUTION (21-25):

Explanation in detail:

1. J sits third to the right of L.


2. Only two people sit between the L and Q.
3. P and K are immediate neighbors of each other.
4. P is not an immediate neighbor of L.

16. a
17. e 5. Q sits second to the left of N.

Page 35 of 728

Subscribe the Xpress Video Course & Mock Test Package for Bank & Insurance Exams
If there are any suggestions/ errors in our PDFs Feel Free to contact us via this email: admin@exampundit.in
IBPS RRB PO Prelims – Ultra Practice Bundle PDF
6. N is not an immediate neighbor Q. 10. O is not an immediate neighbor of J.
7. Q is an immediate neighbor of O and M. 11. P is not an immediate neighbor of Q.
8. M sits third to the right of K.

Final arrangement:

21. e
22. c
23. a
24. d
There is no such space for M and O. So CASE2 25. b
is cancelled out SOLUTION (26-30):

9. L sits to the immediate right of N. Explanation in detail:


Page 36 of 728

Subscribe the Xpress Video Course & Mock Test Package for Bank & Insurance Exams
If there are any suggestions/ errors in our PDFs Feel Free to contact us via this email: admin@exampundit.in
IBPS RRB PO Prelims – Ultra Practice Bundle PDF
1. E sits opposite to D.
2. A sit second to left of D.

Final arrangement:
3. B and sits third to left of A.
4. F sits second to left of B.
5. C is not an immediate neighbor of A.

26. a
27. c
6. H and sits second to the left of G.
28. e
7. A sit opposite to the C.
29. b
8. A sit second to the left of D.
30. d
SOLUTION (31-35):
Page 37 of 728

Subscribe the Xpress Video Course & Mock Test Package for Bank & Insurance Exams
If there are any suggestions/ errors in our PDFs Feel Free to contact us via this email: admin@exampundit.in
IBPS RRB PO Prelims – Ultra Practice Bundle PDF
Explanation in detail: 7. W is sitting second to the right of V.

1. R sits second to the left of P.


2. Only one person is sitting between R and U.
3. Q is sitting second to the left of U.

4. S sits third to right of Q. Final arrangement:


5. T sits second to right of S.
6. Only one person is sitting between T and V.

31. a
Page 38 of 728

Subscribe the Xpress Video Course & Mock Test Package for Bank & Insurance Exams
If there are any suggestions/ errors in our PDFs Feel Free to contact us via this email: admin@exampundit.in
IBPS RRB PO Prelims – Ultra Practice Bundle PDF
32. e
33. e
34. b
35. d
SOLUTION (36-40):

Explanation in detail:

1. S is sitting third to the right of R.


2. U is sitting fourth to the left of the S.
3. Only one person is sitting between R and P.
4. T is sitting exactly opposite to the P.

5. Two persons are sitting between V and T.

6. Three persons are sitting between Q and V.

CASE2:

Page 39 of 728

Subscribe the Xpress Video Course & Mock Test Package for Bank & Insurance Exams
If there are any suggestions/ errors in our PDFs Feel Free to contact us via this email: admin@exampundit.in
IBPS RRB PO Prelims – Ultra Practice Bundle PDF

7. R is not an immediate neighbour of Q.

R is an immediate neighbour of Q CASE1 so case1 is


cancelled out.

8. Only one person is sitting between T and W.

9. U is sitting fifth to the left of W.

Page 40 of 728

Subscribe the Xpress Video Course & Mock Test Package for Bank & Insurance Exams
If there are any suggestions/ errors in our PDFs Feel Free to contact us via this email: admin@exampundit.in
IBPS RRB PO Prelims – Ultra Practice Bundle PDF
40. e
SOLUTION (41-45):

Explanation in detail:

1. T sits to immediate left of S.


2. T is sitting between the S and U.
3. S faces the W.

Note: S faces W means S facing towards the


centre.

Final arrangement:

4. W is an immediate neighbor of P and V.

5. V sits third to the right of Q.

36. b
37. c
38. b
39. d
Page 41 of 728

Subscribe the Xpress Video Course & Mock Test Package for Bank & Insurance Exams
If there are any suggestions/ errors in our PDFs Feel Free to contact us via this email: admin@exampundit.in
IBPS RRB PO Prelims – Ultra Practice Bundle PDF

41. d
42. a
6. T and P are facing same direction. 43. c
44. a
7. U faces the centre and sits third to the right of 45. b
R. SOLUTION (46-50):

8. V is facing outside the centre and an immediate Explanation in detail:


neighbor of U.
1. C sits third to the right of H.
Only four persons Facing inside and only four facing 2. Three people sit between H and B.
outside. So, W is facing inside the circle. 3. F sits third to the left of E.
4. E is not an immediate neighbor of either C or H.

5. D is an immediate neighbor of E.
6. Only one person sits between G and E.
7. F and A are immediate neighbors of each other.
Page 42 of 728

Subscribe the Xpress Video Course & Mock Test Package for Bank & Insurance Exams
If there are any suggestions/ errors in our PDFs Feel Free to contact us via this email: admin@exampundit.in
IBPS RRB PO Prelims – Ultra Practice Bundle PDF
8. Only one person sits between A and C.
9. C sits third to the right of H.

46. b
47. a
48. a
Final arrangement: 49. a
50. e

3). Square Seating Arrangement


Directions (1-5): Answer the questions based on the immediate right of H. G sits second to the left of C. G is
information given below: an immediate neighbor of neither H nor B.
A, B, C, D, E, F, G, and H are sitting around a square 1. Who among the following person sits opposite to
table in such a way that four of them sit at four corners of E?
the table while four of them sit in the middle of each of (a) G
the four sides. The one who sits at the four corners faces (b) A
the center while those sit in the middle of the sides face (c) H
outside but not necessarily in the same order. E does not (d) B
face inside. C sits third to the left of F. D sits on the (e) D
immediate left of H. E is not an immediate neighbor of 2. What is the position of D with respect to A?
A. Only two persons sit between C and H. A sits on the (a) Second to the left
(b) Second to the right

Page 43 of 728

Subscribe the Xpress Video Course & Mock Test Package for Bank & Insurance Exams
If there are any suggestions/ errors in our PDFs Feel Free to contact us via this email: admin@exampundit.in
IBPS RRB PO Prelims – Ultra Practice Bundle PDF
(c) Third to the right the table while four of them sit in the middle of each of
(d) Immediate left the four sides. The one who sits in the middle of the sides
(e) Third to the left faces the center while those sit at the four corners face
3. Four of the following five are alike in a certain way outside but not necessarily in the same order. B sits third
and hence form a group. Which of the following does to the right of F. There is only one person sit between B
not belong to the group? and H. B sits at the corner of the table. A is an immediate
(a) G neighbor of D and F. D is not an immediate neighbor of
(b) A H. E sits second to the right of C. B is not an immediate
(c) C neighbor of E.
(d) B 6. Who among the following person sits opposite to
(e) D G?
4. How many persons sit between F and A when (a) H
counted from the left of A? (b) A
(a) One (c) C
(b) Three (d) B
(c) Two (e) D
(d) Four 7. What is the position of E with respect to B?
(e) Six (a) Second to the left
5. What is the position of H with respect to B? (b) Second to the right
(a) Third to the left (c) Third to the right
(b) Sixth to the right (d) Immediate left
(c) Seventh to the left (e) Third to the left
(d) Seventh to the right 8. Four of the following five are alike in a certain way
(e) Sixth to the left and hence form a group. Which of the following does
Directions (6-10): Answer the questions based on the not belong to the group?
information given below: (a) H
A, B, C, D, E, F, G, and H are sitting around a square (b) A
table in such a way that four of them sit at four corners of (c) B
Page 44 of 728

Subscribe the Xpress Video Course & Mock Test Package for Bank & Insurance Exams
If there are any suggestions/ errors in our PDFs Feel Free to contact us via this email: admin@exampundit.in
IBPS RRB PO Prelims – Ultra Practice Bundle PDF
(d) F immediate right of D. D is an immediate neighbor of F.
(e) G C sits second to the left of B.
9. How many persons sit between D and H, if counted 11. Which of the following statement is false?
from right of D? I. G is facing outside.
(a) One II. E is facing outside
(b) Three III. H is facing inside
(c) Two
(d) Four (a) Both I and II
(e) Six (b) Only III
10. Who among the following person sits fifth to the (c) Only II
left of H? (d) Both II and III
(a) G (e) Only I
(b) A 12. Who sits exactly between C and D.
(c) C I. B
(d) B II. F
(e) D III. E
Directions (11-15): Answer the questions based on the (a) Both I and II
information given below: (b) Only III
A, B, C, D, E, F, G, and H are sitting around a square (c) Only II
table in such a way that four of them sit at four corners of (d) Both I and III
the table while four of them sit in the middle of each of (e) Only I
the four sides. The one who sits in the middle of the sides 13. Who sits immediate right of C?
faces the center while those sit at the four corners face (a) A
outside but not necessarily in the same order. No two (b) E
successive people sit with each other according to (c) H
alphabetical order (such as A does not with B, B does not (d) D
with A and C so on). F faces towards the center. G sits (e) G
fourth to the left of F. H sits third to the right of E. A sits
Page 45 of 728

Subscribe the Xpress Video Course & Mock Test Package for Bank & Insurance Exams
If there are any suggestions/ errors in our PDFs Feel Free to contact us via this email: admin@exampundit.in
IBPS RRB PO Prelims – Ultra Practice Bundle PDF
14. How many persons sit between F and E, if II. A
counted from right of E? III. B
(a) One
(b) Three (a) Both I and II
(c) Two (b) Only III
(d) Four (c) Only II
(e) Six (d) Both II and III
15. Who among the following person sits Sixth to the (e) Only I
left of G? 17. Who sits exactly between C and F from the right
(a) A of F?
(b) D (a) G
(c) C (b) A
(d) B (c) E
(e) H (d) D
Directions (16-20): Answer the questions based on the (e) H
information given below: 18. Who sits third to the right of H?
A, B, C, D, E, F, G, and H are sitting around a square (a) A
table in such a way that four of them sit at four corners of (b) E
the table while four of them sit in the middle of each of (c) B
the four sides. The one who sits in the middle of the sides (d) D
faces the center while those sit at the four corners face (e) G
outside but not necessarily in the same order. D sits 19. How many persons sit between G and D, if
second to the left of C. C is an immediate neighbor of B. counted from left of G?
F is an immediate neighbor of B and A. G sits third to (a) One
the right of F. E is an immediate neighbor of G. H faces (b) Three
towards the center. (c) Two
16. Who is an immediate neighbor of E? (d) Four
I. G (e) None
Page 46 of 728

Subscribe the Xpress Video Course & Mock Test Package for Bank & Insurance Exams
If there are any suggestions/ errors in our PDFs Feel Free to contact us via this email: admin@exampundit.in
IBPS RRB PO Prelims – Ultra Practice Bundle PDF
20. Who among the following person sits immediate (c) Third to the right
right to the C? (d) Immediate left
(a) A (e) Third to the left
(b) D 23. Four of the following five are alike in a certain
(c) F way and hence form a group. Which of the following
(d) B does not belong to the group?
(e) H (a) N
Directions (21-25): Answer the questions based on the (b) Q
information given below: (c) L
J, K, L, M, N, O, P and Q are sitting around a square (d) M
table in such a way that four of them sit at four corners of (e) P
the table while four of them sit in the middle of each of 24. How many persons sit between K and P when
the four sides. All of them are facing to the center but not counted from the left of P?
necessarily in the same order. There are two persons sit (a) None
between K and Q. Q is an immediate neighbor of L. P (b) Three
sits to the immediate left of L. There are two persons sit (c) Two
between P and J. N sits second to the left of M. J is not (d) Four
an immediate neighbor of Q. K sits in the middle of the (e) Six
table. 25. What is the position of Q with respect to P?
21. Who among the following sits opposite to L? (a) Third to the left
(a) Q (b) Sixth to the right
(b) J (c) Seventh to the left
(c) O (d) Seventh to the right
(d) P (e) Sixth to the left
(e) M Directions (26-30): Answer the questions based on the
22. What is the position of J with respect to M? information given below:
(a) Second to the left A, B, C, D, E, F, G, and H are sitting around a square
(b) Second to the right table in such a way that four of them sit at four corners of
Page 47 of 728

Subscribe the Xpress Video Course & Mock Test Package for Bank & Insurance Exams
If there are any suggestions/ errors in our PDFs Feel Free to contact us via this email: admin@exampundit.in
IBPS RRB PO Prelims – Ultra Practice Bundle PDF
the table while four of them sit in the middle of each of (c) B
the four sides. The one who sits at the four corners faces (d) D
the center while those sit in the middle of the sides face (e) G
outside. F does not face inside. C sits third to the left of 29. How many persons sit between E and D, if
F. D sits on the immediate left of H. E is not an counted from left of E?
immediate neighbor of A. Only two persons sit between (a) One
C and H. A sits on the immediate right of H. C sits (b) Three
second to the right of G. (c) Two
26. Who is an immediate neighbor of F? (d) Four
I. G (e) None
II. C 30. Who among the following person sits immediate
III. H right to the one who sits third to the left of F?
(a) A
(a) Both I and II (b) D
(b) Only III (c) G
(c) Only II (d) B
(d) Both II and III (e) H
(e) Only I Directions (31-35): Answer the questions based on the
27. Who sits exactly between B and F? information given below:
(a) G There are eight person P, Q, R, S, T, U, V, and W are
(b) A sitting around a square table with two person sits on each
(c) E side at an equal distance. Four of them are facing the
(d) D center and other four are facing away from the center.
(e) C None of the immediate neighbors are facing the same
28. Who sits opposite to the one who sits third to the direction. T sits to the immediate left of S and faces
right of E? outside the center. S faces the center. T and P are facing
(a) A the same direction. T is sitting between S and U. S and T
(b) C sit on the same side of the table. V is facing outside the
Page 48 of 728

Subscribe the Xpress Video Course & Mock Test Package for Bank & Insurance Exams
If there are any suggestions/ errors in our PDFs Feel Free to contact us via this email: admin@exampundit.in
IBPS RRB PO Prelims – Ultra Practice Bundle PDF
center and an immediate neighbor of U. W is an (a) None
immediate neighbor of P and V. U faces the center and (b) Three
sits third to the right of R. V sits third to the right of Q. (c) Two
W faces the same direction as U. W and P sits one of the (d) Four
four sides. (e) Six
31. Who among the following sits opposite to second 35. What is the position of T with respect to P?
to the right of T? (a) Third to the left
(a) Q (b) Fourth to the right
(b) P (c) Seventh to the left
(c) O (d) Sixth to the right
(d) W (e) Sixth to the left
(e) U Directions (36-40): Answer the questions based on the
32. What is the position of Q with respect to W? information given below:
(a) Second to the left There are eight person J, K, L, M, N, O, P, and Q are
(b) Second to the right sitting around a square table in such a way that four of
(c) Third to the right them sit at four corners of the table while four of them sit
(d) Immediate left in the middle of each of the four sides. The one who sits
(e) Third to the left in the middle of the sides faces the center while those sit
33. Four of the following five are alike in a certain at the four corners face outside but not necessarily in the
way and hence form a group. Which of the following same order. K is sitting third to the left of M. J is sitting
does not belong to the group? second to the left of O. M is an immediate neighbor of O.
(a) T J is not an immediate neighbor of M. There are three
(b) R persons sit between Q and L. L is not an immediate
(c) V neighbor of K. P sits third to the right of N. N is not an
(d) S immediate neighbor of M. P does not sit at any corner of
(e) P the table.
34. How many persons sit between W and V when 36. Who among the following person sits opposite to
counted from the left of V? O?
Page 49 of 728

Subscribe the Xpress Video Course & Mock Test Package for Bank & Insurance Exams
If there are any suggestions/ errors in our PDFs Feel Free to contact us via this email: admin@exampundit.in
IBPS RRB PO Prelims – Ultra Practice Bundle PDF
(a) M (a) K
(b) L (b) L
(c) J (c) P
(d) K (d) M
(e) Q (e) Q
37. What is the position of P with respect to L? Directions (41-45): Answer the questions based on the
(a) Second to the left information given below:
(b) Second to the right There are eight person P, Q, R, S, T, U, V, and W are
(c) Third to the right sitting around a square table in such a way that four of
(d) Immediate left them sit at four corners of the table while four of them sit
(e) Third to the left in the middle of each of the four sides. The one who sits
38. Four of the following five are alike in a certain in the middle of the sides faces the center while those sit
way and hence form a group. Which of the following at the four corners face outside but not necessarily in the
does not belong to the group? same order. There are three persons sit between P and Q.
(a) M R sits to the immediate right of P. Only three persons sit
(b) L between R and T. S sits second to the left of T. U sits to
(c) Q the immediate right of V. W sits fourth to the left of U.
(d) O W sits at one of the corners of the table.
(e) N 41. Who among the following person sits opposite to
39. How many persons sit between Q and K, if V?
counted from left of K? (a) W
(a) One (b) Q
(b) Three (c) S
(c) None (d) R
(d) Four (e) U
(e) Six 42. What is the position of U with respect to T?
40. Who among the following person sits sixth to the (a) Second to the left
left of N? (b) Second to the right
Page 50 of 728

Subscribe the Xpress Video Course & Mock Test Package for Bank & Insurance Exams
If there are any suggestions/ errors in our PDFs Feel Free to contact us via this email: admin@exampundit.in
IBPS RRB PO Prelims – Ultra Practice Bundle PDF
(c) Third to the right them sit at four corners of the table while four of them sit
(d) Immediate left in the middle of each of the four sides. The one who sits
(e) Third to the left in the middle of the sides faces the center while those sit
43. Four of the following five are alike in a certain at the four corners face outside but not necessarily in the
way and hence form a group. Which of the following same order. P sits immediate right of Q. Only one person
does not belong to the group? sits between P and T. T is not an immediate neighbor of
(a) S Q. Only two persons sit between S and T. Q sits fourth to
(b) T the left of R. U sits second to the left of W. X faces
(c) V towards the center of the table.
(d) Q 46. Who among the following person sits opposite to
(e) R the third to the right of U?
44. How many persons sit between T and V when (a) X
counted from the left of T? (b) W
(a) Five (c) S
(b) Three (d) Q
(c) Two (e) P
(d) Four 47. What is the position of T with respect to P?
(e) Six (a) Second to the left
45. What is the position of Q with respect to R? (b) Second to the right
(a) Third to the left (c) Third to the right
(b) Sixth to the right (d) Immediate left
(c) Seventh to the left (e) Third to the left
(d) Seventh to the right 48. Four of the following five are alike in a certain
(e) Sixth to the left way and hence form a group. Which of the following
Directions (46-50): Answer the questions based on the does not belong to the group?
information given below: (a) SX
There are eight person P, Q, R, S, T, U, V, and W are (b) UT
sitting around a square table in such a way that four of (c) QR
Page 51 of 728

Subscribe the Xpress Video Course & Mock Test Package for Bank & Insurance Exams
If there are any suggestions/ errors in our PDFs Feel Free to contact us via this email: admin@exampundit.in
IBPS RRB PO Prelims – Ultra Practice Bundle PDF
(d) PU (e) Six
(e) WP 50. Who among the following person sits second to
49. How many persons sit between U and R, if the right of W?
counted from left of U? (a) U
(a) One (b) X
(b) Three (c) V
(c) None (d) S
(d) Four (e) T

3). Square Seating Arrangement - Solution with Explanation

SOLUTION (1-5):

Explanation in detail:

1. C sits third to the left of F.


2. Only two persons sit between C and H.
3. A sit on the immediate right of H.

4. D sits on the immediate left of H.


5. G sits second to the left of C.

Page 52 of 728

Subscribe the Xpress Video Course & Mock Test Package for Bank & Insurance Exams
If there are any suggestions/ errors in our PDFs Feel Free to contact us via this email: admin@exampundit.in
IBPS RRB PO Prelims – Ultra Practice Bundle PDF

H is facing inside in CASE2, so CASE2 is canceled


out.

6. E is not an immediate neighbor of A.


7. G is an immediate neighbor of neither H nor B.
8. E does not face inside.

1. c

2. b

3. d

4. d

5. b

Page 53 of 728

Subscribe the Xpress Video Course & Mock Test Package for Bank & Insurance Exams
If there are any suggestions/ errors in our PDFs Feel Free to contact us via this email: admin@exampundit.in
IBPS RRB PO Prelims – Ultra Practice Bundle PDF
SOLUTION (6-10):

Explanation in detail:

1. B sits third to the right of F.


2. There is only one person sit between B and H.
3. B sits at the corner of the table.

6. E sits second to the right of C.


7. B is not an immediate neighbor of E.

4. A is an immediate neighbor of D and F.


5. D is not an immediate neighbor of H.

Page 54 of 728

Subscribe the Xpress Video Course & Mock Test Package for Bank & Insurance Exams
If there are any suggestions/ errors in our PDFs Feel Free to contact us via this email: admin@exampundit.in
IBPS RRB PO Prelims – Ultra Practice Bundle PDF
1. F faces towards the center.
2. G sits fourth to the left of F.

There is no place for E, So CASE2 is canceled out

3. A sits immediate right of D.


4. D is an immediate neighbor of F.

6. d

7. e

8. d

9. c

10. e 5. C sits second to the left of B.


SOLUTION (11-15): 6. H sits third to the right of E.
Note: No two successive people sit with each other
Explanation in detail:
according to alphabetical order (such as A does not
with B, B does not with A and C so on).
Page 55 of 728

Subscribe the Xpress Video Course & Mock Test Package for Bank & Insurance Exams
If there are any suggestions/ errors in our PDFs Feel Free to contact us via this email: admin@exampundit.in
IBPS RRB PO Prelims – Ultra Practice Bundle PDF
Final arrangement:

Case2:

11. e

12. d

13. c

14. d

15. a

SOLUTION (16-20): 4. F is an immediate neighbor of B and A.

Explanation in detail: 5. G sits third to the right of F.


6. B sits second to the right of A.
1. D sits second to the left of C. Case1:
2. C is an immediate neighbor of B.
3. B is not an immediate neighbor of D.

Case1:

Page 56 of 728

Subscribe the Xpress Video Course & Mock Test Package for Bank & Insurance Exams
If there are any suggestions/ errors in our PDFs Feel Free to contact us via this email: admin@exampundit.in
IBPS RRB PO Prelims – Ultra Practice Bundle PDF

Case2:

Case2:

7. E is an immediate neighbor of G.
8. H faces towards the center.
Case1:
So CASE2 is canceled out.

Page 57 of 728

Subscribe the Xpress Video Course & Mock Test Package for Bank & Insurance Exams
If there are any suggestions/ errors in our PDFs Feel Free to contact us via this email: admin@exampundit.in
IBPS RRB PO Prelims – Ultra Practice Bundle PDF
Final arrangement:

Case2:

16. a

17. a

18. b

19. e

20. d 4. There are two persons sit between P and J.


SOLUTION (21-25): 5. J is not an immediate neighbor of Q.
6. N sits second to the left of M.
Explanation in detail:
Case1:
1. There are two persons sit between K and Q.
2. Q is an immediate neighbor of L.
3. P sits to the immediate left of L.
Case1:

Page 58 of 728

Subscribe the Xpress Video Course & Mock Test Package for Bank & Insurance Exams
If there are any suggestions/ errors in our PDFs Feel Free to contact us via this email: admin@exampundit.in
IBPS RRB PO Prelims – Ultra Practice Bundle PDF
Case2: SOLUTION (26-30):

Explanation in detail:

1. F does not face inside.


2. C sits third to the left of F.

So, Case2 is canceled out.


Final arrangement:

3. Only two persons sit between C and H.


4. D sits on the immediate left of H.

21. b
22. d
5. A sits on the immediate right of H.
23. c
24. a 6. C sits second to the right of G.
25. e

Page 59 of 728

Subscribe the Xpress Video Course & Mock Test Package for Bank & Insurance Exams
If there are any suggestions/ errors in our PDFs Feel Free to contact us via this email: admin@exampundit.in
IBPS RRB PO Prelims – Ultra Practice Bundle PDF
SOLUTION (31-35):

Explanation in detail:

1. T sits to the immediate left of S and faces outside the


center.
2. S faces the center.
3. T is sitting between S and U.
4. S and T sit on the same side of the table.

7. E is not an immediate neighbor of A.


So, Final arrangement:

5. V is facing outside the center and an immediate


neighbor of U.
6. W is an immediate neighbor of P and V.

26. e
27. c
28. b
29. d
30. d
Page 60 of 728

Subscribe the Xpress Video Course & Mock Test Package for Bank & Insurance Exams
If there are any suggestions/ errors in our PDFs Feel Free to contact us via this email: admin@exampundit.in
IBPS RRB PO Prelims – Ultra Practice Bundle PDF
7. U faces the center and sits third to the right of R. 4. J is not an immediate neighbor of M.
8. V sits third to the right of Q.
9. W faces the same direction as U.
10. T and P are facing the same direction.

31. a

32. a
5. There are three persons sit between Q and L.
33. d 6. L is not an immediate neighbor of K.
34. e

35. b

SOLUTION (36-40):

Explanation in detail:

1. K is sitting third to the left of M.


2. J is sitting second to the left of O.
3. M is an immediate neighbor of O.

Page 61 of 728

Subscribe the Xpress Video Course & Mock Test Package for Bank & Insurance Exams
If there are any suggestions/ errors in our PDFs Feel Free to contact us via this email: admin@exampundit.in
IBPS RRB PO Prelims – Ultra Practice Bundle PDF
39. c

40. e

SOLUTION (41-45):

Explanation in detail:

1. There are three persons sit between P and Q.


2. R sits to the immediate right of P.
3. Only three persons sit between R and T.

7. P sits third to the right of N.


8. N is not an immediate neighbor of M.
9. P does not sit at any corner of the table.

So Case1 is canceled out.


Final arrangement:

36. d

37. e

38. d
4. S sits second to the left of T.
Page 62 of 728

Subscribe the Xpress Video Course & Mock Test Package for Bank & Insurance Exams
If there are any suggestions/ errors in our PDFs Feel Free to contact us via this email: admin@exampundit.in
IBPS RRB PO Prelims – Ultra Practice Bundle PDF
5. U sits to the immediate right of V.
6. W sits fourth to the left of U.

41. c

42. c

43. d

44. a

45. a

SOLUTION (46-50):

Explanation in detail:

1. P and Q sits immediately left to each other.


2. Only one person sits between P and T.
3. T is not an immediate neighbor of Q.

7. W sits at one of the corners of the table.


So, Case2 is canceled out.

Final arrangement:

Page 63 of 728

Subscribe the Xpress Video Course & Mock Test Package for Bank & Insurance Exams
If there are any suggestions/ errors in our PDFs Feel Free to contact us via this email: admin@exampundit.in
IBPS RRB PO Prelims – Ultra Practice Bundle PDF

4. Only two persons sit between S and T.


5. Q sits fourth to the left of R.

6. U sits second to the left of W.

Page 64 of 728

Subscribe the Xpress Video Course & Mock Test Package for Bank & Insurance Exams
If there are any suggestions/ errors in our PDFs Feel Free to contact us via this email: admin@exampundit.in
IBPS RRB PO Prelims – Ultra Practice Bundle PDF
Final arrangement:

46. d

47. a

48. d

49. d
7. X faces towards the center of the table.
50. e
So, Case1 is canceled out.

4). Rectangle Based Sitting Arrangement


Directions (1-5): Study the following information Jacob sits second to the left of Henry and both of them
carefully and answer the questions given below: sit at the corner of the table. James is an immediate
Seven Persons i.e. James. Henry, Kevin, Jacob, Gavin, neighbour of Jacob. Tyler faces James who faces inside.
Jaxon and Tyler are sitting around a rectangular table. Gavin sits between Jacob and Henry. Two person sits
Some of them are facing the centre and some of them are between Jaxon and Gavin. No person sits between James
facing outside the centre. One of the seat is vacant. and Kevin who faces same direction of James.
Person facing the same side are not sit together. Four of 1. Who among the following sits to the immediate left
them sitting at the corner of the table and four are sitting of Kevin?
at the middle of the table A) Person sits to the immediate right of Tyler.
Page 65 of 728

Subscribe the Xpress Video Course & Mock Test Package for Bank & Insurance Exams
If there are any suggestions/ errors in our PDFs Feel Free to contact us via this email: admin@exampundit.in
IBPS RRB PO Prelims – Ultra Practice Bundle PDF
B) Person sits to the immediate right of Jaxon.
C) Person who faces Jacob Directions (6-10): Study the following information
D) None carefully and answer the questions given below:
E) Cannot be determined
Eight people are sitting around a rectangular table. Four
2. What is the position of Gavin with respect to of them facing inside while four of them facing outside
James? the centre. Four of them sitting at the corner of the table
A) Gavin who sits third to the left of James and four are sitting at the middle of the table
B) Gavin sits Second to the right of James. Piku is sitting third to the left of Rafi who is sitting at the
C) Gavin sits second to the left of James corner of the table. Lyra and Myra are sitting opposite to
D) Either A or C each other and facing to each other. Arya is faces
E) None of these towards north-west direction. Immediate neighbour of
Paul faces opposite direction to each other. Rafi does not
3. Which of the following statements is true regarding face inside the centre. Immediate neighbour of Lyra
Jaxon? faces inside the centre. Immediate neighbour of Myra
A) Jaxon sits two places away from Henry. faces Arya and Nick. John, who faces west direction, sits
B) Jaxon faces James third to the right of paul. John is not an immediate
C) Jaxon sits between Tyler and Kevin neighbour of Myra and Nick.
D) Both A and C
E) Both B and C 6. Who among the following sits exactly between
Myra and Lyra?
4. Who among the following sits third to the right of A) Person sits between Rafi and Arya
Gavin? B) Person sits between Paul and Nick
A) Kevin C) Both John and Piku
B) No one D) Both A and B
C) James E) All A, B and C
D) Jacob
E) Jaxon 7. Who among the following faces Nick?
A) Person sits to the immediate right of John
5. Person sits second to the left of __ sits second to the B) Person sits to the immediate left of Myra
right of __. C) Person Sits second to the left of Arya
A) Jaxon and James D) All of the above
B) Jacob and Gavin E) None of the above
C) Tyler and James
D) Henry and Kevin 8. Which of the following statements is/are true
E) None of these regarding Lyra?

Page 66 of 728

Subscribe the Xpress Video Course & Mock Test Package for Bank & Insurance Exams
If there are any suggestions/ errors in our PDFs Feel Free to contact us via this email: admin@exampundit.in
IBPS RRB PO Prelims – Ultra Practice Bundle PDF
A) Lyra sits third to the left of Paul 11. How many persons are seated between Aura and
B) Lyra sits third to the right of Rafi Arvi when counted anticlockwise direction from
C) Lyra sits exactly between Nick and Arya Aura?
D) All are true A) 5
E) Both A and C B) 4
C) 3
9. How many persons sit between Piku and Rafi when D) 2
counted anticlockwise from Piku? E) None
A) 2
B) 3 12. Who among the following faces the person who
C) 4 sits Immediate left of Nova?
D) 5 A) Person sits to the immediate right of Alia
E)) None B) Person sits to the immediate left of Aura
C) Person who faces Emma
10. Who sits fourth to the right of John? D) Either A or B
A) Piku E) Either A or C
B) Paul
C) Nick 13. Who sits second to the left of Emma?
D) Myra A) Person who faces Arvi
E) Lyra B) Tara
C) Person who sits immediate left of Nova
Directions (11-15): Study the following information D) Both A and B
carefully and answer the questions given below: E) Both A and C
Eight persons i.e. Nova, Aura, Sima, Tara, Alia, Dipa,
Arvi and Emma are sitting around a rectangular table. 14. Which of the following statements is/are not true
Four of them sitting at the corner of the table and four regarding Alia?
are sitting at the middle of the table. Those who are A) Alia sits third to the right of Emma.
sitting at the corner facing outside the centre and those B) Alia sits exactly between Sima and Dipa.
who are sitting at the middle facing inside the centre. C) Alia sits fourth to the right Nova.
Nova sits second to the right of Sima and second to the D) None is true
left of the Aura. Emma sits opposite to Dipa who is not E) Both B and C are not true
an immediate neighbour of Nova. Tara sits to the
immediate right of Nova. Sima and Aura are sitting 15. Tara sits second to the right of ____ and Fourth to
diagonally opposite to each other. Alia sits immediate the right of_____
left of Dipa. A) Dipa and Sima
B) Alia and Arvi

Page 67 of 728

Subscribe the Xpress Video Course & Mock Test Package for Bank & Insurance Exams
If there are any suggestions/ errors in our PDFs Feel Free to contact us via this email: admin@exampundit.in
IBPS RRB PO Prelims – Ultra Practice Bundle PDF
C) Dipa and Arvi
D) Sima and Dipa 18. Find odd man out?
E) None of these A) Jeevan
B) Mehul
Directions (16-20): Study the following information C) Anil
carefully and answer the questions given below: D) Prem
Eight persons i.e. Anil, Mehul, Mehek, Heer, Prem, E) Mehek
Abeer, Jeevan and Pran are sitting around a Rectangular
table. Four of them sitting at the corner of the table and 19. Who among the following faces Jeevan?
facing towards the centre and remaining four are sitting A) Pran
at the middle of the table and facing outside the centre. B) Mehul
Mehek sits third to the right of Abeer. Anil and Mehul C) Anil
sits diagonally opposite to each other and facing the D) Prem
centre. Mehek is not the neighbour of Anil. Mehul is not E) Mehek
an immediate neighbour of Mehek. Heer sits to the
immediate right of Mehek. Three persons are sitting 20. Who sits opposite to Abeer?
between Heer and Prem. Pran faces outside the centre. A) Pran
Jeevan sits between Abeer and Prem. Abeer sits third to B) Mehul
the left of Anil. Mehul is an immediate neighbour of C) Anil
Heer. D) Prem
E) Mehek
16. What is the position of Anil with respect to
Mehek? Directions (21-25): Study the following information
A) 6th to the left carefully and answer the questions given below:
B) 3rd to the right Ten dancers i.e. Alexa, Avery, Sofia, Elena, Layla,
C) 2nd to the left Maria, Hazel, Ellie, Clara and Alice are sitting around a
D) 2nd to the right rectangular table such that three persons are sitting on the
E) None of these longer side and two persons are sitting on the shorter
side. All of them are facing inside the centre.
17. How many people seated between Mehul and Maria faces Ellie who sits on the longer side of the table.
Mehek when counted from the left of Mehek? Alexa and Clara the neighbours of Maria. Alexa sits to
A) 4 the right of Clara. Only one person sits between Alice
B) 5 and Layla. Avery sits to the immediate right of Alexa.
C) More then 5 Two person sits between Avery and Ellie. Clara faces
D) None Layla. Alice and Ellie are immediate neighbours of each
E) None of these

Page 68 of 728

Subscribe the Xpress Video Course & Mock Test Package for Bank & Insurance Exams
If there are any suggestions/ errors in our PDFs Feel Free to contact us via this email: admin@exampundit.in
IBPS RRB PO Prelims – Ultra Practice Bundle PDF
other. Three person sits between Sofia and Alexa. Sofia C) Elena and Alice
and Hazel are immediate neighbours of each other. D) Both A and B
E) Both A and C
21. Who among the following faces Hazel?
A) Person sits to the immediate right of Alexa Directions (26-30): Study the following information
B) Person sits to the immediate left of Elena. carefully and answer the questions given below:
C) Person who sits third from the left of Ellie Eight girls Bipin, Chanda, Dana, Eva, Faizal, Gauri,
D) Both A and C Harsha and Aanchal are sitting around a rectangular table
E) All of the above facing the centre but not necessarily in the same order.
There was one person sitting at each corner and one in
22. Who among the following pair sits on the shorter the middle of each sides.
side? Dana was third to the right of the girl whose neighbour
A) Sofia and Layla sits at one of the corners. Aanchal was third to the left of
B) Elena and Maria Faizal and both of them are not the neighbours of Dana.
C) Avery and Hazel Bipin sits third to the right of Eva. Harsha is the
D) Avery and Alice neighbour of Gauri. Chanda was 2nd to the right of the
E) Hazel and Alexa one, who is sitting opposite to Aanchal. Bipin’s
neighbour is sitting on one of the middle sides of the
23. Which of the following statements is/ are true? table. Harsha is sitting diagonally opposite to the one,
A) Clara sits on the shorter side of the table who is neighbour of Eva. Gauri is an immediate
B) Ellie faces Maria neighbour of the one, who is sitting to the immediate left
C) Hazel sits to the immediate left of the Clara of Eva. Eva faces the one, who is the immediate
D) Both B and C neighbours of Harsha and Bipin.
E) Both A and B
24. How many persons sits between the Clara and 26. Who among the following was 2nd to the right of
Elena? the one, who is to the immediate left of Chanda?
A) 3 A) Harsha
B) 4 B) Gauri
C) 5 C) Dana
D) Either A or B D) Bipin
E) Either A or C E) Aanchal
25. Who among the following pair sits to the
immediate left and immediate right of Ellie 27. Four of the following bears a relationship and
respectively? hence forms a group, who among the following is not
A) Alice and Layla a part of that group?
B) Alice and the person sits immediate left of Sofia A) Eva

Page 69 of 728

Subscribe the Xpress Video Course & Mock Test Package for Bank & Insurance Exams
If there are any suggestions/ errors in our PDFs Feel Free to contact us via this email: admin@exampundit.in
IBPS RRB PO Prelims – Ultra Practice Bundle PDF
B) Aanchal direction. Friends sitting on same side face the opposite
C) Gauri direction. Immediate neighbours of person sitting at
D) Chanda corner faces the same direction.
E) Bipin Suresh, who sits at the longer side sits 4th to the right of
Wasim, who does not sit on the smaller side. One friend
28. Who among the following is sitting opposite to sits between Wasim and Naren. Tina sits 4th to the right
Bipin? of Ollie who is immediate neighbout of pradeep.
A) Immediate right of Gauri Pradeep, who sits at one of the corners sits 4th to the right
B) Immediate left of Eva of Uday, who faces the center. Wasim and Roma are
C) Both A and B neither immediate neighbours nor faces the same
D) Cannot Be Determined direction. Neither Suresh nor Wasim is an immediate
E) None of These neighbour of Pradeep. Both Tina and Vartika are not an
immediate neighbours of Naren but have one person
29. In which of the following groups is 3rd person is between them. Roma sits 3rd to the right of Naren, who
sitting exactly between the 1st and the 2nd person? sits at corner and faces outside, and both faces the same
A) Dana, Faizal and Gauri direction.
B) Faizal, Harsha and Gauri
C) Harsha, Bipin and Chanda 31. Who sits to the immediate right of Xmas?
D) Bipin, Faizal and Aanchal A) Naren
E) Chanda, Eva and Faizal B) Wasim
C) Vartika
30. Who among the following is sitting opposite to D) Uday
Gauri? E) Roma
A) Aanchal
B) Third to the left of Eva 32. How many people faces outside?
C) Second to the left of Aanchal A) Six
D) Either B or C B) Seven
E) None of These C) Eight
D) Nine
Directions (31-35): Study the following information E) Cannot Be Determined
carefully and answer the questions given below:
Ten friends Naren, Ollie, Pradeep, Roma, Suresh, Tina, 33. Who among the following faces the same
Uday, Vartika, Wasim and Xmas are sitting around a direction?
rectangular table such that 4 friends sit at 4 corners and 2 A) Suresh and Roma
each on longer sides and 1 each on shorter sides of the B) Wasim and Vartika
table. All friends sitting at corner faces the same C) Naren and Pradeep

Page 70 of 728

Subscribe the Xpress Video Course & Mock Test Package for Bank & Insurance Exams
If there are any suggestions/ errors in our PDFs Feel Free to contact us via this email: admin@exampundit.in
IBPS RRB PO Prelims – Ultra Practice Bundle PDF
D) Uday and Ollie inside, Fredricks faces outside, and, vice-versa). Neither
E) Pradeep and Wasim Cherry and Harry, nor Dawkins and Harry, sit together.
34. Who among the following doesn’t sit at the Cherry is to immediate left of Berry. Angelo always sits
corners? on the longer side of the table.
A) Uday
B) Tina 36. Which among the following pair is the immediate
C) Vartika neighbour of Harry?
D) Naren A) Garry and Angelo
E) Pradeep B) Eve and Garry
C) Angelo and Berry
35. How many friends sit between Roma and Xmas D) Cherry and Dawkins
when counted clockwise from Roma? E) Cannot Be Determined
A) Seven
B) Six 37. How many persons are sitting between Angelo
C) Five and Berry, counting from right of Angelo?
D) Eight A) Six
E) Nine B) Four
C) Either None or Six
Directions (36-40): Study the following information D) None
carefully and answer the questions given below: E) Either Four or Two

Eight persons Angelo, Harry, Garry, Berry, Cherry, 38. Who among the following is/are the immediate
Fredricks, Dawkins and Eve were sitting around a neighbour of Eve?
rectangular table such that four of them sit at corner A) Fredricks
while the other four sits in the middle of the sides. The B) The one, who sits immediate right of Cherry
one sitting at the corners face inside while other face C) Both A and B
outside. D) Garry
Both the immediate neighbour of Eve are sitting at the E) None of These
corners. Angelo, who sits on one of the middle sides, and
is to the immediate left of the one, who sits at one of the 39. Fill in the respective blanks.
corners. Neither Cherry nor Harry sit immediate next to Garry sits ______ to Harry and Berry sits ____ to
Fredricks. Garry sits second to the right of Eve. Dawkins Cherry respectively.
sits third to the right of Angelo. Berry and Fredricks, A) Second to the right, Third to the left
both didn’t sit in the middle of the sides and they are B) Third to the left, Second to the right
both diagonally opposite to each other. Fredricks faces in C) Either A or B
opposite direction as compared to Eve (If Eve faces D) Immediate Right, Immediate Left

Page 71 of 728

Subscribe the Xpress Video Course & Mock Test Package for Bank & Insurance Exams
If there are any suggestions/ errors in our PDFs Feel Free to contact us via this email: admin@exampundit.in
IBPS RRB PO Prelims – Ultra Practice Bundle PDF
E) Second to the left, Immediate Right E) Both A and B

40. Who among the following are immediate 42. Find the number of persons sits between Aman-
neighbours of Cherry? Farhin and Himmat-Danny respectively when
I. Immediate Right of Angelo counted from the right of the latter?
II. Immediate left of Eve A) 3 and 4
III. Berry and Dawkins B) 2 and 3
A) Only I C) 4 and 3
B) Only II D) 3 and 2
C) Only III E) None of these
D) Both I and II
E) All of the Above 43. Who among the following faces Chandu?
A) Person sits second to the right of Aman
Directions (41-45): Study the following information B) Person sits third to the left of Evana
carefully and answer the questions given below: C) Person sits immediate right of the one who sits
There are eight people Aman, Himmat, Gaurav, Bhuwan, diagonally opposite to Bhuwan
Chandu, Danny, Farhin, and Evana seated around D) Both A and C
rectangular table such that exactly one person is seated E) Both A and B
on each edge and each corner all facing towards the
center but not necessarily in the same order. 44. __ sits fourth to the right of Danny, sits exactly
Danny is to the immediate left of Chandu. Bhuwan sits at between __ and __.
one of the corners. Gaurav, was opposite to the person, A) Gaurav, Farhin- Aman
who sits to the immediate left of Bhuwan and was seated B) Himmat, Aman - Gaurav
at one of the edges. Bhuwan was to the immediate left of C) Gaurav, Evana - Aman
Aman. Danny is at one of the corners and 3rd to the right D) Farhin, Himmat- Evana
of Gaurav. Himmat was adjacent to the person who sits E) None of these.
to the immediate right of Bhuwan. The person who sits 45. Which of the following statements is/are not true?
to the immediate left of Chandu is to the immediate right A) Evana faces the person sits to the immediate right of
of Evana. Farhin sits to the immediate right of Gaurav. Bhuwan
B) Chandu sits exactly between Aman and Danny
41. Who among the following sits exactly between C) Gaurav sits second to the right of Evana
Farhin and Bhuwan? D) Both A and B
A) Person sits second to the left of Chandu E) Both B and C
B) Person sits to the immediate right of Evana
C) Person sits to the immediate left of Gaurav Directions (46-50): Study the following information
D) Both B and C carefully and answer the questions given below:

Page 72 of 728

Subscribe the Xpress Video Course & Mock Test Package for Bank & Insurance Exams
If there are any suggestions/ errors in our PDFs Feel Free to contact us via this email: admin@exampundit.in
IBPS RRB PO Prelims – Ultra Practice Bundle PDF

Pihu, Sonam, Tia, Manish, Akash, Naren, Cocco and 47. If Jack and Manish interchange their position
Jack are sitting around a rectangular table in such a way then who sits third to the left of Manish??
that four of them sit at four corners while the other four A) Person sits exactly between Tia and Pihu
sits in the middle of the each of the four sides. The one, B) The person who diagonally faces Akash.
who sit at four corners face the center of the table while C) Person who sits fifth to the left of Naren.
those sit in the middle of the sides faces outside. D) Both A and B
Akash sits 3rd to the left of Tia. Tia faces outside. Only E) All of the above
two persons sit between Akash and Pihu. The person,
who is immediate right of Pihu is also immediate left of 48. How many persons are sitting between the person
Jack. The person, who is immediate left of Naren is also sits to the immediate right of Coco and immediate left
second to the right of Manish who sits at one of corners. of Coco when counted from right of Jack?
Manish is neither an immediate neighbour of Pihu nor A) 5
Akash. Only one person sits between Cocco and Akash. B) 4
Sonam sits to the immediate left of the one, who is C) 3
sitting to the immediate right of Cocco. Naren is not an D) 2
immediate neighbour of Cocco. Manish is an immediate E) None of these
neighbour of Naren. Sonam is an immediate neighbour
of Tia. 49. Which of the following statements is/are true
regarding Akash and Tia?
46. Which among the following pair sit to the A) Akash and Tia sits third to the left of each other.
immediate right and immediate left of Sonam B) Akash and Tia faces opposite direction of each other.
respectively? C) Akash and Tia sits fifth to the right of each other.
A) Person sits immediate right of Coco and the person D) Both A and C
sits immediate left of Manish E) All A, B and C
B) Person sits immediate right of Manish and the person 50. Which among the following does not belong to the
sits immediate left of Coco. group?
C) Person sits immediate left of Manish and the person A) Person sits immediate right of Naren.
sits immediate right of Coco. B) Person sits immediate right of Jack.
D) Person sits exactly between Naren-Pihu and the C) Person sits immediate right of Tia.
person sits exactly between Tia- Coco. D) Person sits immediate right of Coco.
E) None of these E) Person sits immediate right of Pihu.

Page 73 of 728

Subscribe the Xpress Video Course & Mock Test Package for Bank & Insurance Exams
If there are any suggestions/ errors in our PDFs Feel Free to contact us via this email: admin@exampundit.in
IBPS RRB PO Prelims – Ultra Practice Bundle PDF
4). Rectangle Based Sitting Arrangement - Detailed Explanation with
Solutions:
Answers (1-5):

1) D
2) C
3) D
4) B
5) C

Solutions (1-5):

Step 1: Jacob sits second to the left of Henry and both of


them sit at the corner of the table.
As per this line two cases will form:

Step 2: James is an immediate neighbour of Jacob.


As per this line two more cases wil form:

Page 74 of 728

Subscribe the Xpress Video Course & Mock Test Package for Bank & Insurance Exams
If there are any suggestions/ errors in our PDFs Feel Free to contact us via this email: admin@exampundit.in
IBPS RRB PO Prelims – Ultra Practice Bundle PDF

Step 3: . Tyler faces James. Gavin sits between Jacob


and Henry.
As per these lines Case1 and Case2 will be cancelled:

Step 4: Two person sits between Jaxon and Gavin. No


one sits between Kevin and James.
Person facing the same side are not sit together.
As per the last line case 1(a) will be cancelled:

Page 75 of 728

Subscribe the Xpress Video Course & Mock Test Package for Bank & Insurance Exams
If there are any suggestions/ errors in our PDFs Feel Free to contact us via this email: admin@exampundit.in
IBPS RRB PO Prelims – Ultra Practice Bundle PDF

This is the final answer.

Answers (6-10):

6) E
7) B
8) D
9) C
10) A

Solutions (6-10):

Step 1: Arya is faces towards north-west direction. Piku


is sitting third to the left of Rafi who is sitting at the
corner of the table. Rafi does not face inside the centre.
As per these lines three cases will form:

Step 2: Lyra and Myra are sitting opposite to each other


and facing to each other. Immediate neighbour of Lyra
faces inside the centre. Immediate neighbour of Myra
faces Arya and Nick.
As per these lines Case 2 will cancel:

Page 76 of 728

Subscribe the Xpress Video Course & Mock Test Package for Bank & Insurance Exams
If there are any suggestions/ errors in our PDFs Feel Free to contact us via this email: admin@exampundit.in
IBPS RRB PO Prelims – Ultra Practice Bundle PDF

Case 1 is the final answer.

Answers (11-15):

11) B
12) B
13) D
14) B
15) C

Solutions (11-15):

Step 3: Immediate neighbour of Paul faces opposite


Step 1: Nova sits second to the right of Sima and second
direction. Two person sit between Paul and John. John is
to the left of the Aura.
not an immediate neighbour of Myra and Nick.
As per this line two case will form:
As per these lines Case 3 will be eliminated:

Page 77 of 728

Subscribe the Xpress Video Course & Mock Test Package for Bank & Insurance Exams
If there are any suggestions/ errors in our PDFs Feel Free to contact us via this email: admin@exampundit.in
IBPS RRB PO Prelims – Ultra Practice Bundle PDF

Step 3: Emma sits opposite to Dipa who is not an


immediate neighbour of Nova.

Step 2: Tara sits to the immediate right of Nova.

Page 78 of 728

Subscribe the Xpress Video Course & Mock Test Package for Bank & Insurance Exams
If there are any suggestions/ errors in our PDFs Feel Free to contact us via this email: admin@exampundit.in
IBPS RRB PO Prelims – Ultra Practice Bundle PDF
Step 4: Sima and Aura are sitting diagonally opposite to As per this line two case will arise:
each other. Alia sits immediate left of Dipa.
As per these lines Case 1 will be cancelled:

Step 2: Anil and Mehul sits diagonally opposite to each


other and facing the centre. Mehek is not the neighbour
of Anil.

This is the final arrangement.

Answers (16-20):

16) C
17) B
18) D
19) E
20) A

Solutions (16-20):

Step 1: Mehek sits third to the right of Abeer.


Page 79 of 728

Subscribe the Xpress Video Course & Mock Test Package for Bank & Insurance Exams
If there are any suggestions/ errors in our PDFs Feel Free to contact us via this email: admin@exampundit.in
IBPS RRB PO Prelims – Ultra Practice Bundle PDF

Step 3: Mehul is not an immediate neighbour of Mehek.


As per these lines Case 1 will cancel: Step 5: Pran faces outside the centre. Jeevan sits
between Abeer and Prem.

Step 6: Abeer sits third to the left of Anil. Mehul is an


immediate neighbour of Heer.

So the final arrangement is:

Step 4: Heer sits to the immediate right of Mehek. Three


persons are sitting between Heer and Prem.

Page 80 of 728

Subscribe the Xpress Video Course & Mock Test Package for Bank & Insurance Exams
If there are any suggestions/ errors in our PDFs Feel Free to contact us via this email: admin@exampundit.in
IBPS RRB PO Prelims – Ultra Practice Bundle PDF
Answers (21-25):

21) E
22) C
23) D
24) E
25) D

Solutions (21-25):

Step 1: Maria faces Ellie who sits on the longer side of


the table. Step 2: Immediate neighbours of Maria likes Aerial and
As per this line three possibility will arise: Belly dance. Person likes Aerial dance sits to the right of
the person likes Belly dance.

Page 81 of 728

Subscribe the Xpress Video Course & Mock Test Package for Bank & Insurance Exams
If there are any suggestions/ errors in our PDFs Feel Free to contact us via this email: admin@exampundit.in
IBPS RRB PO Prelims – Ultra Practice Bundle PDF

Step 3: Avery sits to the immediate right of Alexa. Two Step 4: Clara faces Layla. Only one person sits between
person sits between Avery and Ellie. Alice and Layla.
As per these lines Case 1 and Case 3 will cancel: As per these lines one more case of Case 2 will form:

Page 82 of 728

Subscribe the Xpress Video Course & Mock Test Package for Bank & Insurance Exams
If there are any suggestions/ errors in our PDFs Feel Free to contact us via this email: admin@exampundit.in
IBPS RRB PO Prelims – Ultra Practice Bundle PDF
Step 5: Alice and Ellie are immediate neighbours of each Solutions (26-30):
other. Three person sits between Sofia and Alexa. Sofia
and Hazel are immediate neighbours of each other. STEP I: Dana was third to the right of the girl whose
As per these lines Case 2(a) will be cancelled: neighbour sits at one of the corners

As per this statement there will be two cases getting


framed and they are as follows:

CASE I:

CASE II:

STEP II: Aanchal was third to the left of Faizal and both
of them are not the neighbours of Dana.
This is the final answer.
As per this statement, the arrangement will look like this:
Answers (26-30):
CASE I:
26) D
27) E
28) C
29) B
30) C
Page 83 of 728

Subscribe the Xpress Video Course & Mock Test Package for Bank & Insurance Exams
If there are any suggestions/ errors in our PDFs Feel Free to contact us via this email: admin@exampundit.in
IBPS RRB PO Prelims – Ultra Practice Bundle PDF

STEP IV: Harsha is the neighbour of Gauri.


CASE II:
As per this statement, the arrangement will look like this:

CASE II:

STEP III: Chanda was 2nd to the right of the one, who
is sitting opposite to Aanchal.
CASE II (A):
As per this statement, CASE I will get eliminated and
we will continue with CASE II and the arrangement of
CASE II will look like this:
CASE I:

STEP V: Eva faces the one, who is the immediate


neighbours of Harsha and Bipin.

As per this statement, CASE II (A) will get eliminated


and we will continue with CASE II and the final
CASE II:
arrangement will look like this:
Page 84 of 728

Subscribe the Xpress Video Course & Mock Test Package for Bank & Insurance Exams
If there are any suggestions/ errors in our PDFs Feel Free to contact us via this email: admin@exampundit.in
IBPS RRB PO Prelims – Ultra Practice Bundle PDF
CASE II: CASE I:

CASE II (A): CASE II:

Answers (31-35): CASE III:

31) A
32) B
33) C
34) A
35) C

Solutions (31-35):

STEP I: Suresh, who sits at the longer side sits 4th to the
right of Wasim, who does not sit on the smaller side. CASE IV:

As per this statement there will be four cases being


framed and they are as follows:

Page 85 of 728

Subscribe the Xpress Video Course & Mock Test Package for Bank & Insurance Exams
If there are any suggestions/ errors in our PDFs Feel Free to contact us via this email: admin@exampundit.in
IBPS RRB PO Prelims – Ultra Practice Bundle PDF
CASE III:

STEP II: All friends sitting at corner faces the same


direction. Friends sitting on same side face the opposite
direction. Immediate neighbours of person sitting at
corner faces the same direction.
CASE IV:
As per these statements, the arrangements will look like
this:

CASE I:

STEP III: One friend sits between Wasim and Naren.


Roma sits 3rd to the right of Naren and both faces the
same direction. Wasim and Roma are neither immediate
neighbours nor faces the same direction.
CASE II:
As per these statements, CASE I and CASE IV will get
eliminated and we will continue with CASE II and
CASE III and the arrangement will look like this:

CASE I:

Page 86 of 728

Subscribe the Xpress Video Course & Mock Test Package for Bank & Insurance Exams
If there are any suggestions/ errors in our PDFs Feel Free to contact us via this email: admin@exampundit.in
IBPS RRB PO Prelims – Ultra Practice Bundle PDF

CASE II:
STEP IV: All friends sitting at corner faces the same
direction.

As per this statement, the arrangement will look like this:

CASE II:

CASE III:

CASE III:

CASE IV:

Page 87 of 728

Subscribe the Xpress Video Course & Mock Test Package for Bank & Insurance Exams
If there are any suggestions/ errors in our PDFs Feel Free to contact us via this email: admin@exampundit.in
IBPS RRB PO Prelims – Ultra Practice Bundle PDF
STEP V: Pradeep, who sits at one of the corners sits 4th STEP VI: Both Tina and Vartika are not an immediate
to the right of Uday, who faces the center. Neither neighbours of Naren but have one person between them.
Suresh nor Wasim is an immediate neighbour of Roshni.
As per this statements, CASE III and CASE III (A)
As per these statements, the arrangements will look like will get eliminated and we will proceed with CASE II
this: and the arrangement will look like this:

CASE II: CASE II:

CASE III:
CASE III:

CASE III (A):


CASE III (A):

STEP VII: Tina sits 4th to the right of Ollie.


Page 88 of 728

Subscribe the Xpress Video Course & Mock Test Package for Bank & Insurance Exams
If there are any suggestions/ errors in our PDFs Feel Free to contact us via this email: admin@exampundit.in
IBPS RRB PO Prelims – Ultra Practice Bundle PDF
As per this statement, the final arrangement will look STEP II: Garry sits second to the right of Eve.
like this:
As per this statement, the arrangement will look like this:

Answers (36-40): STEP III: Angelo, who sits on one of the middle sides,
and is to the immediate left of the one, who sits at one of
36) A the corners. Dawkins sits third to the right of Angelo.
37) D
38) C As per these statements, there will be two cases getting
39) D framed and they will look like this:
40) E
CASE I (A):
Solutions (36-40):

STEP I: Both the immediate neighbour of Eve are sitting


at the corners.

As per this statement, Eve will in the middle of the side,


and the arrangement will look like this:

CASE I (B):

Page 89 of 728

Subscribe the Xpress Video Course & Mock Test Package for Bank & Insurance Exams
If there are any suggestions/ errors in our PDFs Feel Free to contact us via this email: admin@exampundit.in
IBPS RRB PO Prelims – Ultra Practice Bundle PDF
CASE I (B) (1):

STEP IV: Berry and Fredricks, both didn’t sit in the


middle of the sides and they are both diagonally opposite
CASE I (B) (2):
to each other. Fredricks faces in opposite direction as
compared to Eve (If Eve faces inside, Fredricks faces
outside, and, vice-versa).

As per these statements, the arrangements will look like


this:

CASE I (A) (1)

STEP V: Neither Cherry nor Harry sit immediate next to


Fredricks. Cherry and Harry do not sit together. Cherry is
to immediate left of Berry.

As per these statements, CASE I (A) (1) and CASE I


(B) (2) will get eliminated and we will continue with
CASE I (A) (2) and CASE I (B) (1) and the
arrangements will look like this:
CASE I (A) (2):
CASE I (A) (1):

Page 90 of 728

Subscribe the Xpress Video Course & Mock Test Package for Bank & Insurance Exams
If there are any suggestions/ errors in our PDFs Feel Free to contact us via this email: admin@exampundit.in
IBPS RRB PO Prelims – Ultra Practice Bundle PDF
STEP VI: Angelo always sits on the longer side of the
table.

As per this statement, CASE I (B) (1) will get eliminated


and the final arrangement will look like this:

CASE I (A) (2):

CASE I (A) (2):

Answers (41-45):

CASE I (B) (1): 41) D


42) C
43) A
44) B
45) E

Solutions (41-45):

STEP I: Bhuwan sits at one of the corners.

CASE I (B) (2):


As per this statement, the arrangement will look like this:

Page 91 of 728

Subscribe the Xpress Video Course & Mock Test Package for Bank & Insurance Exams
If there are any suggestions/ errors in our PDFs Feel Free to contact us via this email: admin@exampundit.in
IBPS RRB PO Prelims – Ultra Practice Bundle PDF
STEP II: Gaurav, was opposite to the person, who sits to STEP V: Himmat was adjacent to the person who sits to
the immediate left of Bhuwan and was seated at one of the immediate right of Bhuwan, who was not Saksham or
the edges. Sagar.

As per this statement, the arrangement will look like this: As per this statement, the arrangement will look like this:

STEP VI: Farhin sits to the immediate right of Gaurav.


STEP III: Bhuwan was to the immediate left of Aman.

As per this statement, the arrangement will look like this:


As per this statement, the arrangement will look like this:

STEP IV: Danny is at one of the corners and 3rd to the


STEP VII: The person who sits to the immediate left of
right of Gaurav.
Chandu is to the immediate right of Evana.
As per this statement, the arrangement will look like this:
As per this statement, the final arrangement will look
like this:

Page 92 of 728

Subscribe the Xpress Video Course & Mock Test Package for Bank & Insurance Exams
If there are any suggestions/ errors in our PDFs Feel Free to contact us via this email: admin@exampundit.in
IBPS RRB PO Prelims – Ultra Practice Bundle PDF

Answers (46-50): STEP III: The person, who is immediate right of Pihu is
also immediate left of Jack
46) C
47) E As per this statement, the arrangement will look like this:
48) A
49) E
50) D

Solutions (46-50):

STEP I: Akash sits 3rd to the left of Tia. Tia faces


outside.

As per these statements, the arrangement will be like


this: STEP IV: Manish is neither an immediate neighbour of
Pihu nor Akash. The person, who is immediate left of
Naren is also second to the right of Manish.

As per these statements, the arrangement will look like


this:

STEP II: Only two persons sit between Akash and Pihu.

As per this statement, the arrangement will look like this:

Page 93 of 728

Subscribe the Xpress Video Course & Mock Test Package for Bank & Insurance Exams
If there are any suggestions/ errors in our PDFs Feel Free to contact us via this email: admin@exampundit.in
IBPS RRB PO Prelims – Ultra Practice Bundle PDF
As per this statement, the final arrangement will look
like this:

STEP V: Only one person sits between Cocco and


Akash

5). Triangular Seating


Direction (1-5): Answer the questions based on the 1) Who sits to the immediate left of P?
information given below. a. R
Six people P, Q, R, S, T and U are sitting around a b. S
triangular table in such a way that three are sitting at the c. T
corner of the table and three are sitting in the middle of d. U
the sides of the table. Also, none of the persons sits e. None of these
adjacent to each other according to their names in 2) How many persons sit between P and S on table,
English alphabetical order. For e.g., P can’t sit adjacent when counted from the left of S?
to Q, Q can’t sit adjacent to R and so on. The ones sitting a. Two
at the corner of the table are facing outside the table and b. Three
the ones who sit in the middle of the sides are facing c. One
towards the center of the table. d. More than Three
P sits at the corner of the table. R is an immediate e. None of these
neighbor of P. One person sits between R and T and also 3) Who sits third to the left of U?
one person sits between T and S. S sits to the immediate a. P
left of Q. b. T
Page 94 of 728

Subscribe the Xpress Video Course & Mock Test Package for Bank & Insurance Exams
If there are any suggestions/ errors in our PDFs Feel Free to contact us via this email: admin@exampundit.in
IBPS RRB PO Prelims – Ultra Practice Bundle PDF
c. R Chinu is not the immediate neighbour of Raghav. Anuj is
d. S seated second to the right of Sima. Raghav and Anuj are
e. None of these seated just opposite to each other. Ashi is seated second
4) If in the arrangement T is related to Q and R is to the left of Sima. There is only one person between
related to P in the same way then who among the Sherin and Raghav. There is only one person between
following is related to S? Sima and Anuj. Sherin is seated immediate left to Anuj.
a. Q There are two persons between Sima and Chinu. Anuj
b. T doesn’t sit at corner of the table.
c. U 6) How many persons are there between Sima and
d. P Ashi from the right of Sima?
e. None of these a. 2
5) If all the persons sit according to the English b. 1
alphabetically order starting from the right of P then c. 3
which among the following remains the same in the d. 4
new arrangement excluding P? e. 5
a. Q 7) Who is seated just next to Anuj?
b. S a. Raghav
c. R b. Sima
d. U c. Ashi
e. T c. Either Ashi or Sima
Directions (6-10): Study the following information e. Both Sherin and Chinu
carefully and answer the following questions. 8) Who is seated just opposite to Ashi?
6 friends Raghav, Sima, Ashi, Sherin, Chinu, Anuj are a. Raghav
seated around a triangular table such that that three are b. Sima
sitting at the corner of the table and three are sitting in c. Ashi
the middle of the sides of the tableand facing towards the d. Either Ashi or Sima
centre but not necessarily in the same order. Rest of the e. Sherin
information is given below. 9) How many persons sit between Raghav and Anuj?
Page 95 of 728

Subscribe the Xpress Video Course & Mock Test Package for Bank & Insurance Exams
If there are any suggestions/ errors in our PDFs Feel Free to contact us via this email: admin@exampundit.in
IBPS RRB PO Prelims – Ultra Practice Bundle PDF
a. 1 White. R does not like White. M doesn’t sit at corner of
b. 2 the table.
c. 3 11) Who among the following likes Green colour?
d. 4 a. O
e. 5 b. M
10) What is the position of Sima with respect to c. N
Chinu? d. R
a. third to the left e. Q
b. fourth to the right 12) Who sits second to the left of O?
c. second to the left a. Q
d. second to the right b. M
e. None of these c. P
Directions (11-15): Study the following information d. N
and answer the given questions. e. None of the above
Six persons M, N, O, P, Q and R are sitting in a triangle 13) Find the odd one out.
such that that three are sitting at the corner of the table a. O, P
and three are sitting in the middle of the sides of the b. N, R
table, facing away from the centre. All six persons like c. M, Q
different colours among Red, Blue, Green, Yellow, d. P, Q
Black and White. They are not seated necessarily in the e. P, M
mentioned order. 14) Who likes Black Color?
The one who likes Black colour sits second to the left of a. O
M. R sits adjacent to the one who likes black. N sits b. M
immediate right of the one, who likes Red. M does not c. N
like Red. N is not adjacent to the one who like Black. P d. R
likes Blue. O does not like Red. The one who likes e. Q
Yellow is to the immediate right of the one who likes 15) How many persons sits between M and Q when
counted from left of Q?
Page 96 of 728

Subscribe the Xpress Video Course & Mock Test Package for Bank & Insurance Exams
If there are any suggestions/ errors in our PDFs Feel Free to contact us via this email: admin@exampundit.in
IBPS RRB PO Prelims – Ultra Practice Bundle PDF
a. 1 b. R
b. 2 c. Q
c. 3 d. U
d. 4 e. None of these
e. 0 18) Who among the following sits third to the right of
Directions (16-20): Answer the questions based on the R?
information given below. a. S
Six persons P, Q, R, S, T and U are sitting around a b. P
triangular table. Three persons sit on each corner and c. T
three persons sit on each of the sides of the table. Some d. U
of them are facing towards the center while some are e. None of these
facing away from the center. 19) If all are sitting according to the English
R sits second to the left of P but both are facing opposite alphabetical order starting from the left of P, then the
directions with respect to each other. The one who sits to position of how many persons remains unchanged
the immediate left of R sits second to the right of T, who (excluding P)?
is not an immediate neighbor of R. S sits third to the a. One
right of U, who faces away from the center. Q and S both b. Three
face the same direction as P who faces opposite direction c. Four
with respect to U. S sits to the immediate left of P. P d. None
doesn’t sit at corner of the table. e. Two
16) Who sits second to the left of Q? 20) Four of the following five are related to each
a. T other in some way and thus formed a group. Choose
b. S the one who does not belong to that group.
c. V a. P and S
d. R b. S and R
e. None of these c. U and R
17) Who sits third to the left of S? d. P and Q
a. T e. T and U
Page 97 of 728

Subscribe the Xpress Video Course & Mock Test Package for Bank & Insurance Exams
If there are any suggestions/ errors in our PDFs Feel Free to contact us via this email: admin@exampundit.in
IBPS RRB PO Prelims – Ultra Practice Bundle PDF
Direction (21-25) : Answer the questions based on the b. E
information given below. c. A
Six persons A, B, C, D, E and F are sitting around a d. D
triangular table facing the centre with equal distance e. None of the above
between adjacent persons. Such that that three are sitting 24) Who sits to the immediate left of F?
at the corner of the table and three are sitting in the a. B
middle of the sides of the table, b. D
A sit to the immediate right of B. Only one person sits c. E
between A and D (either from the left or the right side). F d. A
sits third to the right of D. E is an immediate neighbor of e. None of the above
F. A doesn’t sit at corner of the table. 25) Who sits 2nd to the right of D?
21) Who sits to the immediate right of A? a. B
a. F b. E
b. C c. F
c. B d. A
d. E e. None of the above
e. None of the above Directions (26-30) : Answer the questions based on
22) If all the persons are sitting according to the the information given below.
English alphabetical order starting from the right of Six persons, A, B, C, D, E and F are sitting around a
A, then the position of how many persons remains the triangular table. All of them are facing the centre. Such
same (excluding A)? that that three are sitting at the corner of the table and
a. Two three are sitting in the middle of the sides of the table,
b. One F’s seat is 2nd to the left of B’s seat who sits on the
c. None corner of the triangle. One person is sitting between F
d. Four and C, who doesn’t sit adjacent to B. A is sitting opposite
e. Three to C. Neither A nor D sits adjacent to E’s seat. E is the
23) Who sits 3rd to right of C? immediate neighbor of B.
a. F
Page 98 of 728

Subscribe the Xpress Video Course & Mock Test Package for Bank & Insurance Exams
If there are any suggestions/ errors in our PDFs Feel Free to contact us via this email: admin@exampundit.in
IBPS RRB PO Prelims – Ultra Practice Bundle PDF
26) How many persons are sitting between B and A, c. Either (a) or (b)
when counted from the left of A? d. C
a. 1 e. None of these
b. 4 Directions (31-35): Answer the questions based on the
c. 3 information given below.
d. Cannot be determined Six persons A, B, C, D, E, F sit around a triangular table.
e. None of these Only A and F face outside while others face towards the
27) _____ is sitting opposite to E. centre. Such that that three are sitting at the corner of the
a. B table and three are sitting in the middle of the sides of the
b. F table,
c. C A and B sit 3rd to the right of each other. One person sits
d. Either (a) or (b) between B and E. F sits 3rd to the right of E.C sits third
e. None of these to the right of D. D sits immediate right of A. A doesn’t
28) What is the position of B’s seat with respect to D? sit at middle of the table.
a. 2nd to the left 31) Who sits to the immediate right of C?
b. 2nd to the right a. E
c. Either (a) or (b) b. A
d. 3rd to the left c. F
e. None of these d. B
29) Who is definitely sitting adjacent to C? e. None of these
a. B 32) Who sits 3rd to the left of D?
b. E a. B
c. A b. E
d. D c. F
e. Both (b) and (d) d. A
30) Who is sitting opposite to A? e. None of these
a. B 33) What is the position of D with respect to E?
b. E a. 3rd to the left
Page 99 of 728

Subscribe the Xpress Video Course & Mock Test Package for Bank & Insurance Exams
If there are any suggestions/ errors in our PDFs Feel Free to contact us via this email: admin@exampundit.in
IBPS RRB PO Prelims – Ultra Practice Bundle PDF
b. Immediate right other. F likes Vadapav sits second to the right of B, who
c. Immediate left does not like Idli. The one who likes Idli sits
d. 2nd to the left immediately to the left of the one who likes Noodles. A
e. None of these doesn’t sit at corner of the table.
34) __ and __ sit 2nd to the left of each other. 36) Who among the following likes Idli?
a. A, D a. A
b. E, A b. B
c. E, F c. D
d. C, A d. E
e. None of these e. F
35) How many persons sit between A and B, if 37) How many people sit between B and E, when
counted from the right of B? counted in a clockwise direction from B?
a. Three a. None
b. One b. One
c. Four c. Two
d. Two d. Three
e. None of these e. Cannot be determined
Direction (36-40): Read the given information 38) What is the position of F with respect to C?
carefully and answer the questions asked below. a. Immediate to the left
Six people A, B, C, D, E and F are sitting around a b. Second to the left
triangular table facing towards center, but not necessarily c. Second to the right
in the same order. Three persons sit on each corner and d. Immediate to the right
three persons sit on each of the sides of the table. Each of e. Third to the right
them likes different food namely Dosa, Noodles, Idli, 39) Who among the following likes Noodles?
Burger, Pizza and Vadapav. a. A
The one who like Pizza sits second to the right of A. C b. B
likes Dosa and sits third to the left of E, who does not c. E
like Pizza. B and D are immediate neighbors of each d. D
Page 100 of 728

Subscribe the Xpress Video Course & Mock Test Package for Bank & Insurance Exams
If there are any suggestions/ errors in our PDFs Feel Free to contact us via this email: admin@exampundit.in
IBPS RRB PO Prelims – Ultra Practice Bundle PDF
e. F d. UK
40) Who sits opposite to the one who likes Vadapav? e. None of the above
a. A 42) Who is from France?
b. D a. S
c. E b. T
d. B c. W
e. None of these d. V
Directions (41-45): Answer the questions based on the e. None of the above
information given below. 43) What is the position of the person from USA with
Seven persons Q, R, S, T, U, V and W sit around a respect to Q?
triangular table facing outside the centre such that six a. 2nd to the right
persons sit on the sides of the triangle with two on each b. 3rd to the right
side and one person sitting on the corner which is c. 3rd to the left
opposite to the base of the triangle. Each of them d. 2nd to the left
belongs to different countries among Kenya, India, e. None of the above
Finland, USA, Bhutan, UK and France. 44) ___ sits 2nd to the right of the person from
T sits 2nd to the left of the person from Kenya but not on Finland.
the corner. W sits 3rd to the left of T who sits immediate a. T
left of R. U sits immediate left of the person from Bhutan b. U
and belongs to Kenya. Person from India sits 2nd to the c. W
left of S. V is not from India. Person from France sits d. R
adjacent to the person from India. R is from UK and sits e. None of the above
on the corner of the triangle. V sits to the immediate left 45) Person from ___ sits 4th to the right of R.
of the person from Finland. a. USA
41) T is from which country? b. Finland
a. Finland c. India
b. France d. France
c. USA e. None of the above
Page 101 of 728

Subscribe the Xpress Video Course & Mock Test Package for Bank & Insurance Exams
If there are any suggestions/ errors in our PDFs Feel Free to contact us via this email: admin@exampundit.in
IBPS RRB PO Prelims – Ultra Practice Bundle PDF
Directions (46-50) : Answer the questions based on e. Cannot be determined
the information given below. 48) __________ is/are sitting in the middle of R and S,
Six persons are sitting at the sides of a Triangular table when counted from the right of S.
such that persons sit on the sides of the triangle with two I. T
on each side. Some of them are facing the centre and II. P
others are facing away from the centre. III. U
U is sitting 2nd to the left of S and both of them are a. Both I and II
facing the same direction. Immediate neighbors of S are b. Only III
facing the opposite direction of S. P is sitting c. Both II and III
immediately right of T and both of them are not sitting d. None of the above
adjacent to U. Q is sitting 2nd to the left of R, who is e. Only I
facing the centre. R and U sitting on the same side. 49) Who is sitting 2nd to the left of Q?
46) How many persons are sitting between S and T, if a. S
counted from the left of S? b. U
a. 1 c. P
b. 4 d. T
c. 3 e. None of these
d. Either (a) or (b) 50) Find the odd one out.
e. None of these a. T, P
47) _______ is sitting immediate right of P. b. R, T
a. U c. Q, S
b. R d. U, R
c. Q e. None of these
d. S

Page 102 of 728

Subscribe the Xpress Video Course & Mock Test Package for Bank & Insurance Exams
If there are any suggestions/ errors in our PDFs Feel Free to contact us via this email: admin@exampundit.in
IBPS RRB PO Prelims – Ultra Practice Bundle PDF

5). Triangular Seating - Solutions with Detailed Explanation


Answers
1. c
2. a
3. b
4. c
5. b
Solution 1-5

1. P sits at the corner of the table.


2. R is an immediate neighbor of P.
Since, no one sits together with their name according to
3. One person sits between R and T and also T and S. the English alphabetical order and S sits to the immediate
Case I. left of Q.
Hence case II is invalid.
The final arrangement is as follows.

Case II.

Answers

6. c

7. e

8. e
Page 103 of 728

Subscribe the Xpress Video Course & Mock Test Package for Bank & Insurance Exams
If there are any suggestions/ errors in our PDFs Feel Free to contact us via this email: admin@exampundit.in
IBPS RRB PO Prelims – Ultra Practice Bundle PDF
9. b 7) There are two persons between Sima and Chinu.

10. a 8) Ashi is seated second to the left of Sima.


(Since only place left is immediate right to Chinu, hence,
Solution 6-10
Ashi must be seated there.)
1) There is only one person between Sima and Anuj. Hence, the final arrangement is as follows:
2) Anuj is seated second to the right of Sima. Anuj
doesn’t sit at corner of the table.

Answers
11. b
3) Raghav and Anuj are seated just opposite to each 12. d
other.
4) There is only one person between Sherin and Raghav. 13. d

5) Sherin is seated immediate left to Anuj. 14. a

15. e

Solution 11-15

1) The one who likes Black sits second to the left of M.


2) R sits adjacent to one, who likes Black. M doesn’t sit
at corner of the table.
Case-1

6) Chinu is not the immediate neighbour of Raghav.

Page 104 of 728

Subscribe the Xpress Video Course & Mock Test Package for Bank & Insurance Exams
If there are any suggestions/ errors in our PDFs Feel Free to contact us via this email: admin@exampundit.in
IBPS RRB PO Prelims – Ultra Practice Bundle PDF

Case-2
Case-2

3) N sits immediate right of the one who likes Red.


4) M does not like Red.
5) N is not adjacent to one, who like Black.
6) P likes Blue. 7) O does not like Red colour.
Case-1 8) The one, who likes Yellow color is immediate right of
one, who likes White colour.
9) R does not like White colour. So, case 2 is rejected.
Therefore, M must like Green colour.

Page 105 of 728

Subscribe the Xpress Video Course & Mock Test Package for Bank & Insurance Exams
If there are any suggestions/ errors in our PDFs Feel Free to contact us via this email: admin@exampundit.in
IBPS RRB PO Prelims – Ultra Practice Bundle PDF

Case II:
Answers

16. a

17. d

18. c

19. a

20. b

Answers 16-20

1. R sits second to the left of P but both are facing


opposite direction with respect to each other so, P faces
either towards or away from the center.
3. S sits third to the right of U, who faces away from the
2. The one who sits to the immediate left of R sits second
center.
to the right of T, who is not an immediate neighbor of R.
P doesn’t sit at corner of the table. 4. Q and S both face the same direction as P who faces
opposite direction with respect to U.
Case I:
5. S sits to the immediate left of P.
Hence, Case-2 is eliminated.

Page 106 of 728

Subscribe the Xpress Video Course & Mock Test Package for Bank & Insurance Exams
If there are any suggestions/ errors in our PDFs Feel Free to contact us via this email: admin@exampundit.in
IBPS RRB PO Prelims – Ultra Practice Bundle PDF
Case-2

2. F sits third to the right of D.


Answers
3. E is an immediate neighbor of F.
21. a
Case-1
22. c

23. c

24. d

25. d

Answers 21-25

1. Only one person sits between A and D. A doesn’t sit at


corner of the table.
Case-1
Case-2

Page 107 of 728

Subscribe the Xpress Video Course & Mock Test Package for Bank & Insurance Exams
If there are any suggestions/ errors in our PDFs Feel Free to contact us via this email: admin@exampundit.in
IBPS RRB PO Prelims – Ultra Practice Bundle PDF
4. A sits to the immediate right of B. 2. One person is sitting between F and C, who doesn’t sit
adjacent to B.
Hence, Case-2 is eliminated.

Answers 3. A is sitting opposite to C.


26. b

27. b

28. d

29. e

30. d

Answers 26-30

1. F’s seat is 2nd to the left of B’s seat who sits on the
4. Neither A nor D sits adjacent to E’s seat.
corner of the triangle.
5. E is the immediate neighbor of B.

Page 108 of 728

Subscribe the Xpress Video Course & Mock Test Package for Bank & Insurance Exams
If there are any suggestions/ errors in our PDFs Feel Free to contact us via this email: admin@exampundit.in
IBPS RRB PO Prelims – Ultra Practice Bundle PDF
Answers Case I:

31. d

32. e

33. d

34. d

35. d

Solution 31-35 Case II:


1. A and B sit 3rd to the right of each other. A
doesn’t sit at middle of the table.
2. One person sits between B and E, so E sits 2 nd to the
left of B or E sits 2nd to the right of B:
Case I:

4. C sits 3rd to the right of D.


5. D sits immediate right of A, so case I is rejected.
Case II: The final seating arrangement is given below:

3. F sits 3rd to the right of E.


Page 109 of 728

Subscribe the Xpress Video Course & Mock Test Package for Bank & Insurance Exams
If there are any suggestions/ errors in our PDFs Feel Free to contact us via this email: admin@exampundit.in
IBPS RRB PO Prelims – Ultra Practice Bundle PDF
Answers

36. a

37. b

38. d

39. c

40. b

Solution 36-40
4) F likes Vadapav sits second to the right of B, who
1) The one who like Pizza sits second to the right of A. A does not like Idli.
doesn’t sit at corner of the table.
5) The one who likes Idli sits immediately to the left of
2) C likes Dosa and sits third to the left of E, who does the one who likes Noodles.
not like Pizza.
Note: B does not like Idli so this condition eliminates
3) B and D are immediate neighbors of each other. case 2.
Case-1 Final arrangement:

Answers

Case-2 41. c

42. d

43. c

44. d
Page 110 of 728

Subscribe the Xpress Video Course & Mock Test Package for Bank & Insurance Exams
If there are any suggestions/ errors in our PDFs Feel Free to contact us via this email: admin@exampundit.in
IBPS RRB PO Prelims – Ultra Practice Bundle PDF
45. d 6. Person from France sits adjacent to the person from
India.
Solution 41-45
7. V is not from India.
1. T sits 2nd to the left of the person from Kenya who
8. V sits immediate left of the person from Finland
does not sit on the corner.
2. W sits 3rd to the left of T who sits immediate left of R.
3. R is from UK and sits on the corner of the triangle.

4. U sits immediate left of the person from Bhutan and Answers


belongs to Kenya.
46. c
nd
5. Person from India sits 2 to the left of S.
47. d

48. a

49. c

50. b

Solution 46-50

1. U is sitting 2nd to the left of S and both of them are


facing the same direction.
2. Immediate neighbour of S are facing the opposite
direction of S.
3. P is sitting immediately right of T and both of them
are not sitting adjacent to U.

Page 111 of 728

Subscribe the Xpress Video Course & Mock Test Package for Bank & Insurance Exams
If there are any suggestions/ errors in our PDFs Feel Free to contact us via this email: admin@exampundit.in
IBPS RRB PO Prelims – Ultra Practice Bundle PDF
Case-1 4. Q is sitting 2nd to the left of R, who is facing the
centre.
5. R and U sitting on the same side.
The final arrangement is as follows:

Case-2

6). Certain Number of Persons Linear


Directions (1-5): Study the following information immediate neighbours. Less than seventeen persons sit in
carefully and answer the questions given below: the row. Five person sits between the Tanu and the joey.
A certain number of persons are sitting in a row facing Tanu does not sit at the end of the row.
North. Not more than two known persons sit adjacent to 1. How many persons sit between Sonu and Prem?
each other. A) 9
B) 8
Four persons sit between Joey and Prem. Renu is an C) 7
immediate neighbour of Prem. Ross sits fourth to the left D) 6
of the Renu. Joey sits in the middle of the row. Sonu who E) 5
sits immediate right of the Pari sits three places away
from Joey. Number of Person who sits to the right of 2. Number of persons sits to the right of Ross is equal
Prem is one less than the number of persons sits to the to the number of persons sits to the left of ________
left of Pari. Joey does not sits to the right of the Ross and A) An unknown Person who sits to the immediate right
less than two persons sit between them but not of Sonu.
Page 112 of 728

Subscribe the Xpress Video Course & Mock Test Package for Bank & Insurance Exams
If there are any suggestions/ errors in our PDFs Feel Free to contact us via this email: admin@exampundit.in
IBPS RRB PO Prelims – Ultra Practice Bundle PDF
B) An unknown Person who sits second to the left of sits forth to the left of S. U sits between S and T. V sits
Joey. second to the right of U. W is third from any of the end.
C) An unknown Person who sits second to the right of S is eight from the left end of the row
Pari. 6. How many persons sitting in a row?
D) Both A and C A) 20
E) All A, B and C B) 17
3. Which of the following statements is/are true? C) 23
A) Three persons sit between Sonu and Tanu D) 18
B) Three persons sit between Ross and Renu E) None of these
C) Three persons sit between Joey and Pari 7. Who is 3rd to the right of Q?
D) Both B and C A) W
E) Both A and B B) V
C) P
4. Who among the following sits ninth to the left of D) S
the one who sits third to the left of the Prem? E) None of these
A) Tanu 8. Who is to the immediate left of P?
B) No one A) W
C) Sonu B) Q
D) Pari C) T
E) None of the above D) S
E) NONE
5. If a new person, Aman, sits at the extreme left end 9. Who is third to the right of S?
then who among the following sits seventh to the right A) T
of Aman? B) W
A) Joey. C) V
B) Ross D) R
C) Sonu E) None of these
D) Prem 10. What is the position of R with respect to S?
E) None of these A) Third to the right
B) Second to the right
Directions (6-10): Study the following information C) Third to the left
carefully and answer the questions given below: D) Second to the left
A certain number of persons are sitting in the row. All of E) Fourth to the left
them are facing towards south. Q sits sixth from the right Directions (11-15): Study the following information
of S. T sits forth to left of Q. Only two persons sit carefully and answer the questions given below:
between Q and P. Six persons sit between W and V. R

Page 113 of 728

Subscribe the Xpress Video Course & Mock Test Package for Bank & Insurance Exams
If there are any suggestions/ errors in our PDFs Feel Free to contact us via this email: admin@exampundit.in
IBPS RRB PO Prelims – Ultra Practice Bundle PDF
A certain number of persons are sitting in a row facing A) none
North. Less than fourteen persons sit in the row. B) 1
Only one person sits to the right of Jose. Two persons sit C) 6
between Jack and Nora. Alex is an immediate neighbour D) 3
of Jack. One person sits between Alex and Lucy. Four E) 2
persons sit between Lucy and Jose. One person sits
between Lucy and Ruby. Three persons sit between Jose 15. Total number of persons seated in the row?
and Mary. Ruby sits third to the left of Jose. Three A) 13
persons sit between Nora and Alex. Mary sits third to the B) 14
right of Alex. C) 12
D) 11
11. What is the position of Lucy in the row? E) 15
A) Sixth from the right end
B) Exactly middle of the row Directions (16-20): Study the following information
C) Fifth from the right end carefully and answer the questions given below:
D) Either A or B A certain number of persons are sitting in a row facing
E) None of the above South.
A and B sits at the both end of the row. Three person sits
12. Which of the following statements is/are not true between A and F, F sits to the left of A. Four person sits
regarding Ruby? between G and B. Number of Person sits between G and
A) Ruby sits second to the right of Lucy. F is two more than the number of persons sits to the right
B) Four persons sit between Ruby and Jack. of F. C sits exactly between F and B. Three person sits
C) Ruby sits fifth from the right end between C and D. Two person sits between D and H.
D) All are true Three person sits between H and E. Number of person
E) None is true sits to the left of E is one less than the number of person
sits to the right of F. J is an immediate neighbour of A.
13. What is the position of Alex with respect to the Number of person sits to the left of I is equal to the
Jose and the Nora respectively? number of person sits to the right of J.
A) Fourth to the right and seventh to the left
B) Sixth to the left and Third to the right 16. Who among the following sits exactly middle of
C) Seventh to the left and Fourth to the right the row?
D) Eighth to the left and Fourth to the right A) C
E) Seventh to the left and Third to the right B) Second right person of C
C) No one
14. Number of persons sits between Mary and Alex is D) D
_____ E) None of these

Page 114 of 728

Subscribe the Xpress Video Course & Mock Test Package for Bank & Insurance Exams
If there are any suggestions/ errors in our PDFs Feel Free to contact us via this email: admin@exampundit.in
IBPS RRB PO Prelims – Ultra Practice Bundle PDF
17. How many persons are there in the row? Tinku sits fourth to the right of Pinky and fourth to the
A) 18 left of Bobby. One person lives between Bobby and
B) 19 Mikku. One person lives between Pinky and Jimmy.
C) 17 Miku sits fifth to the right of Rinku who sits third to the
D) 21 right of Jimmy. Pinku sits to the immediate left of the
E) 22 Bobby. Tinku and Rinku are immediate neighbours.
Pinky sits fourth to the left of Tinku. Mikku sits third to
18. How many persons are sitting between I and D? the right of Pinku. Not more than eleven persons sit in
A) 9 the row.
B) 10
C) 12 21. Who among the following sits exactly between the
D) 13 row?
E) None of these A) Rinku
B) Tinku
Q19. Find the number of persons sits to the right of D C) Jimmy
and the number of people sits D) Pinku
to the left of E respectively: E) None of these
A) 3 and 8
B) 6 and 3 22. Which of the following statements is/are true
C) 5 and 7 regarding Pinku?
D) 7 and 5 A) Pinku sits second to the right of Rinku
E) None of these B) Pinku sits fourth to the left of Mikku
C) Pinku sits fourth from the right end
20. Which of the following statements is/are is not D) Both A and B
true? E) Both A and C
A) H sits exactly between F and B
B) Five person sits between C and I Q23. How many persons are there in the row?
C) D sits fourth to the right of G A) 8
D) None is true B) 9
E) All are true C) 11
D) 10
Directions (21-25): Study the following information E) None of these
carefully and answer the questions given below:
A certain number of persons are sitting in a row facing 24. Number of Person sits to the left of Jimmy is
north. equal to the number of persons sits to the right
of________

Page 115 of 728

Subscribe the Xpress Video Course & Mock Test Package for Bank & Insurance Exams
If there are any suggestions/ errors in our PDFs Feel Free to contact us via this email: admin@exampundit.in
IBPS RRB PO Prelims – Ultra Practice Bundle PDF
A) Bobby E) 12
B) Person sits third from the right end
C) Person sits to the immediate right of Pinku 27. What is the position of Xmas from the right end?
D) All A, B and C A) Ninth from the Right
E) Both B and C B) Fifth from the Right
C) Fourth to the Right
25. Which among the following pairs sits at the D) Fifth to the Right
Extreme ends of the row? E) Seventh from the Right
A) Jimmy and Mikku
B) Pinky and Mikku 28. If Dany sits exactly between Xmas and Queen,
C) Pinky and Bobby then what is the position of Dany with respect to
D) Person sits second to the left of Jimmy and Mikku Clarie?
E) Both B and D A) Fifth to the Left
B) Sixth to the Right
Directions (26-30): Study the following information C) Fourth to the Right
carefully and answer the questions given below: D) Fifth to the Right
A certain number of persons are sitting in a row facing E) Seventh to the Right
North.
Sahil sits fourth to the right of PK. Queen is an 29. How many persons sit between Vivek and Fredie?
immediate neighbour of Sahil. Only three persons sit A) Two
between the one, who is an immediate neighbour of Sahil B) None
and Amit, who sits third from the right end. The one, C) More Than Three
who is third from the right end is not an immediate D) One
neighbour of PK. Only one person sits between the one, E) Three
who is third from the right end and Fredie. Five persons
sit between Fredie and Vivek. Xmas is an immediate 30. What is the position of the one, who sits
neighbour of PK. Five persons sit between Clarie and immediate right of Vivek with respect the one, who
Xmas. Amit is third from the right end. Less than sits third from the right end?
16persons sit in the row. Xmas sits to the right ofVivek. A) Fifth to the Left
B) Sixth to the Right
26. If Clarie sits second from the left end, then how C) Fourth to the Left
many persons sit in the row? D) Fifth to the Right
A) 14 E) Seventh to the Left
B) 16
C) 15 Directions (31-35): Study the following information
D) 13 carefully and answer the questions given below:

Page 116 of 728

Subscribe the Xpress Video Course & Mock Test Package for Bank & Insurance Exams
If there are any suggestions/ errors in our PDFs Feel Free to contact us via this email: admin@exampundit.in
IBPS RRB PO Prelims – Ultra Practice Bundle PDF
A certain number of persons sit in a row adjacent to each 34. What is the position of the one, who likes Lily
other. Some of them like different chocolates and other from the end?
like different flowers. persons who likes chocolates faces A) Seventh
South and persons who likes flowers faces North. B) Eleventh
Six persons sit between the one, who like Dairy Milk C) Tenth
(DM) and the one, who like 5-Star. The one, who likes D) Ninth
Lily sit third to the left of the one, who likes 5-Star. Two E) Eighth
persons sit between the one, who likes Lily and the one,
who likes Sunflower (SF), who is not a neighbour of the 35. If the one, who likes Orange sits third to the left of
one, who likes Dairy Milk (DM). Only one person sits to the one, who likes Dairy Milk (DM) then how many
the right of the one, who likes Sunflower. No-one sits to persons sit between the one, who likes 5-Star and the
the right of the one, who likes Dairy Milk (DM). one, who likes Orange?
A) Seven
31. How many persons are sitting in the row? B) Three
A) 15 C) Two
B) 17 D) Nine
C) 13 E) Eight
D) 18
E) 11 Directions (36-40): Study the following information
carefully and answer the questions given below:
32. How many persons sit between the one, who likes Certain number of persons are sitting in a row facing
Dairy Milk (DM) and the one, who likes Lily? North.
A) Fifteen N sits fourth to the right of H. Five persons sit between N
B) Seven and C. G sits at one of the positions left to H. The
C) Thirteen number of persons sitting between N and F are same as
D) Nine between H and G. K is 2nd from one of the extreme ends.
E) Eleven Four persons sit between H and F. No one sits to the
right of M, who is to the immediate right of K. C is 3 rd to
33. How many persons sit to the right of the one, who the left of K. Two persons sit between J and F, who is to
likes Sunflower (SF)? the right of J.
A) None
B) Two 36. How many persons are sitting in a row?
C) Five A) 22
D) One B) 23
E) Three C) 24
D) 25

Page 117 of 728

Subscribe the Xpress Video Course & Mock Test Package for Bank & Insurance Exams
If there are any suggestions/ errors in our PDFs Feel Free to contact us via this email: admin@exampundit.in
IBPS RRB PO Prelims – Ultra Practice Bundle PDF
E) 26 Certain number of persons is sitting in a north facing row
equidistant from each other.
37. How many persons are sitting between H and G? Damien sits adjacent to Chinki, who sits third from left
A) Four end of the row. Eva sits to the immediate right of
B) Five Damien. Jacob sits to the immediate right of Iris. Only 4
C) Six persons sit between Gagan and Lavlesh, who sits to the
D) Seven right of Fredie. The number of persons who sit on the
E) Eight right side of Fredie is 1 more than sitting to the left side
of him. Iris is not an immediate neighbor of Chinki.
38. What is the position of F from the left end? Bobby does not sit on the extreme ends of the row and he
A) Sixth sits to the left of Eva. Only 2 persons sit between Gagan
B) Fifth and Jacob. There are more than 2 persons between Fredie
C) Fourth and Bobby. Fredie sits to the left of Gagan, who does not
D) Third sit adjacent to Kavita, and Bobby sits to the left of
E) Second Chinki. Abhas is sitting at the left end of the row. Only 1
person sits between Damien and Fredie
39. How many persons are sitting between G and N? 41. Who among the following sits third to the left of
A) Ten the one who sits second to the left of Jacob?
B) Eleven A) Person sits second to the right of Gagan
C) Twelve B) Person sits sixth from the left end
D) Thirteen C) Person sits second to the right of Chinki
E) Fourteen D) Person sits exactly between Bobby and Fredie
E) None of these
40. Which of the following represents the person
sitting at the extreme end? 42. Which of the following statements is/are not true
I. G regarding Gagan?
II. M A) Gagan sits immediate right of Harish
III. C B) Gagan sits second to the left of the one who sits to the
A) Only I immediate left of Jacob
B) Only II C) Gagan sits second to the right of Eva and third to the
C) Only III left of Iris
D) Both II and III D) Both A and C
E) Both I and II E) Both B and C

Directions (41-45): Study the following information


carefully and answer the questions given below:

Page 118 of 728

Subscribe the Xpress Video Course & Mock Test Package for Bank & Insurance Exams
If there are any suggestions/ errors in our PDFs Feel Free to contact us via this email: admin@exampundit.in
IBPS RRB PO Prelims – Ultra Practice Bundle PDF
43. Number of persons sits to the left of __ is equal to immediate neighbour of C. A and I do not sit at the end
the number of persons sits to the right of ___ of the row.
respectively.
A) Jacob and Chinki 46. Find odd one out?
B) Damien and Iris A) B, F
C) Kavita and Bobby B) E, G
D) Both A and C C) F, B
E) All of the above D) C, F
E) A, D
44. Who among the following pair sits fifth from the
extreme right and extreme left respectively? 47. If all the persons sit in the alphabetical order from
A) Person sits immediate left of Iris and the person sits left to right then position of how many persons
immediate right of Damien remain unchanged?
B) Eva and Harish A) 3
C) Iris and Eva B) 2
D) Both A and B C) 0
E) None of these D) 1
E) 4
45. How many persons are sitting between Abhas and
Kavita? 48. Who among the following sits to the immediate
A) 10 left of the person who sits third to the left of H?
B) 8 A) Person sits sixth from the left end of the row.
C) 11 B) Person sits sixth to the left of D
D) 7 C) Person sits fourth to the left of E
E) 9 D) Both A and B
Directions (46-50): Study the following information E) Both A and C
carefully and answer the questions given below:
A certain number of persons are sitting in a row facing 49. __ Sits fifth to the left of C and sits __ to the left of
north. Less than seventeen persons sit in the row. B.
Three persons sit between G and B. E is an immediate A) E, Eighth
neighbour of G. A sit second to the left of E. H sits B) G, Ninth
second to the right of B. A and H are not an immediate C) E, Tenth
neighbours of each other. One person sits between C and D) A, Seventh
B. D sits second to the right of H. I sits fourth to the right E) None of these
of H. F sits fourth to the right of E who is not an
50. Who sits exactly between H and I?

Page 119 of 728

Subscribe the Xpress Video Course & Mock Test Package for Bank & Insurance Exams
If there are any suggestions/ errors in our PDFs Feel Free to contact us via this email: admin@exampundit.in
IBPS RRB PO Prelims – Ultra Practice Bundle PDF
A) Person sits second to the right of H D) Both A and B
B) Person sits third from the right end E) Both A and C
C) Person sits fifth to the right of B

6). Certain Number of Persons Linear - Detailed Explanation with


Answers
Answers (1-5):

1) C
2) E
3) D
4) B
5) A

Solutions (1-5):
Step I: Four persons sit between Joey and Prem.
As per this line two cases will arrive; joey sits either left
or right to the prem:

Step 2: Renu is an immediate neighbour of Prem.


Now, according to this line two more cases of above two
case will form: Step 3: Ross sits fourth to the left of the Renu.
As per this line Case 1 will be cancelled:

Page 120 of 728

Subscribe the Xpress Video Course & Mock Test Package for Bank & Insurance Exams
If there are any suggestions/ errors in our PDFs Feel Free to contact us via this email: admin@exampundit.in
IBPS RRB PO Prelims – Ultra Practice Bundle PDF
As per these lines Case 2, and Case 4 will cancel because
total number of person is less than 17 so, Pari can’t be
seated to the left of the Ross in these two cases:

Subcase of Case 3 will form because Number of persons


towards right of Prem is not known:

Step 4: Sonu who sits immediate right of the Pari sits


three places away from Joey.

Step 6: Joey sits in the middle of the row.


Five person sits between the Tanu and the Joey.
Tanu does not sit at the end of the row.

Joey does not sits to the right of the Ross and less than
two persons sit between them but not immediate
neighbours.

Less than seventeen persons sit in the row.

Step 5: Number of Person who sits to the right of Prem As per these lines Case 3 will be cancel:
is one less than the number of persons sits to the left of
Pari. Pari does not sits to the right of the Ross.

Page 121 of 728

Subscribe the Xpress Video Course & Mock Test Package for Bank & Insurance Exams
If there are any suggestions/ errors in our PDFs Feel Free to contact us via this email: admin@exampundit.in
IBPS RRB PO Prelims – Ultra Practice Bundle PDF
Answers (11-15):

11) B
12) D
13) C
14) E
15) A
Case 3(a) is the final answer.
Solutions (11-15):
Answers (6-10): Step I: Two persons sit between Jack and Nora.
6) A As per this line two case will arrive:
7) C
8) E
9) C
10)E
Solution (6-10)
1. S is eight from the left end of the row
2. R sits forth to the left of S.
3. Q sits sixth from the right of S.
4. T sits forth to left of Q.

Step 2: Alex is an immediate neighbour of Jack.


As per this line two more case will form:
5. U sits between S and T.
6. V sits second to the right of U.
7. Only two persons sit between Q and P.
8. W is third from any of the end.
9. Six persons sit between W and V.

Page 122 of 728

Subscribe the Xpress Video Course & Mock Test Package for Bank & Insurance Exams
If there are any suggestions/ errors in our PDFs Feel Free to contact us via this email: admin@exampundit.in
IBPS RRB PO Prelims – Ultra Practice Bundle PDF

Step 4: Four persons sit between Lucy and Jose. One


person sits between Lucy and Ruby. Only one person sits
to the right of Jose. Ruby sits third to the left of Jose.
As per these lines Case 1(a), Cade 1(b), Case 1 and Case
2 will be cancelled:

Step 3: One person sits between Alex and Lucy.


As per this line two more case will arrive:

Step 5: Three persons sit between Jose and Mary. Three


persons sit between Nora and Alex. Mary sits third to the
right of Alex.
Page 123 of 728

Subscribe the Xpress Video Course & Mock Test Package for Bank & Insurance Exams
If there are any suggestions/ errors in our PDFs Feel Free to contact us via this email: admin@exampundit.in
IBPS RRB PO Prelims – Ultra Practice Bundle PDF
As per these lines Case 2(a) will be cancelled and the
final arrangement is
Case 2(b):

Step 3: Four person sits between G and B. Number of


Person sits between G and F is two more than the
number of persons sits to the right of F. C sits exactly
between F and B.

Answers (16-20):

16) B Step 4: Three person sits between C and D.


17) C As per this line two possibility arise:
18) E
19) B
20) D

Solutions (16-20):
Step 1: A and B sits at the Either end of the row.
Two cases will form:
Step 4: Two person sits between D and H. Three person
sits between H and E. Number of people sits to the left of
E is one less than the number of persons sits to the right
of F.
As per these lines Case 1 will be eliminated:

Step 2: Three person sits between A and F, F sits to the


left of A.
As per this line Case 2 will cancel:

Page 124 of 728

Subscribe the Xpress Video Course & Mock Test Package for Bank & Insurance Exams
If there are any suggestions/ errors in our PDFs Feel Free to contact us via this email: admin@exampundit.in
IBPS RRB PO Prelims – Ultra Practice Bundle PDF
Step 5: J is an immediate neighbour of A. Number of
persons sits to the left of I is equal to the number of
people sits to the right of J.
The final result is:

Answers (21-25):

21) A
22) E Step 3: One person lives between Pinky and Jimmy.
23) C As per this line two more case each of Case 1 and Case
24) D 1(a) will form:
25) E

Solutions (21-25):
Step 1: Tinku sits fourth to the right of Pinky and fourth
to the left of Bobby.

Step 2: One person lives between Bobby and Mikku.


As per this line two case will form:

Step 4: Mikku sits fifth to the right of Rinku who sits


third to the right of Jimmy.
As per this line Case 1 and Case 1(b) will be eliminated:

Page 125 of 728

Subscribe the Xpress Video Course & Mock Test Package for Bank & Insurance Exams
If there are any suggestions/ errors in our PDFs Feel Free to contact us via this email: admin@exampundit.in
IBPS RRB PO Prelims – Ultra Practice Bundle PDF
30) E

Solutions (26-30):

STEP I: Sahil sits fourth to the right of PK. Queen is an


immediate neighbour of Sahil.
As per these statements the arrangement will have two
cases and they will look like this:

CASE I:

Step 5: .Pinku sits to the immediate left of Bobby.


Mikku sits third to the right of Pinku.
As per these lines Case 1(c) will cancel:
CASE II:

STEP II: Only three persons sit between the one, who is
an immediate neighbour of Sahil and the one, who sits
The final arrangement is: third from the right end. Amit is third from the right end.

As per these statements, the above two cases will look


like this:

CASE I:
Answers (26-30):

26) B
27) A
28) E
29) C
Page 126 of 728

Subscribe the Xpress Video Course & Mock Test Package for Bank & Insurance Exams
If there are any suggestions/ errors in our PDFs Feel Free to contact us via this email: admin@exampundit.in
IBPS RRB PO Prelims – Ultra Practice Bundle PDF
CASE II:
STEP IV: Five persons sit between Fredie and Vivek.

As per this statement, CASE I and CASE II will get


eliminated and we will continue with the remaining cases
and, the arrangement will look like this:
STEP III: Only one person sits between the one, who is CASE I:
third from the right end and Fredie.

As per this statement, the arrangement will look like this:

CASE I:

CASE I (A):

CASE I (A):

CASE II:

CASE II:
CASE II (A):

CASE II (A):

STEP V: Xmas is an immediate neighbour of PK. Five


persons sit between Clarie and Xmas.Clarie is not an
immediate neighbour of Amit.

Page 127 of 728

Subscribe the Xpress Video Course & Mock Test Package for Bank & Insurance Exams
If there are any suggestions/ errors in our PDFs Feel Free to contact us via this email: admin@exampundit.in
IBPS RRB PO Prelims – Ultra Practice Bundle PDF
As per these statements, the arrangement will look like STEP I: Six persons sit between the one, who like Dairy
this: Milk (DM) and the one, who like 5-Star.

CASE I (A): As per this statement there will be two cases and they
will look like this:

CASE I:

CASE II (A):

CASE II:

STEP VI: Less than 18 persons sit in the row.

As per this statement, CASE II (A) will get cancelled


and the final arrangement will look like this: STEP II: The one, who likes Lily sit third to the left of
the one, who likes 5-Star.
CASE I (A):
As per this statement the arrangement will look like this:
CASE I:

Answers (31-35):

31) A
32) D CASE II:
33) D
34) B
35) B

Solutions (31-35):

Page 128 of 728

Subscribe the Xpress Video Course & Mock Test Package for Bank & Insurance Exams
If there are any suggestions/ errors in our PDFs Feel Free to contact us via this email: admin@exampundit.in
IBPS RRB PO Prelims – Ultra Practice Bundle PDF
STEP III: Two persons sit between the one, who likes 36) C
Lily and the one, who likes Sunflower (SF), who is not a 37) E
neighbour of the one, who likes Dairy Milk (DM). 38) B
39) C
As per this statement CASE II will get eliminated as the 40) E
one, who likes Sunflower (SF) cannot be a neighbour of
the one, who likes Dairy-Milk and we will continue with Solutions (36-40):
CASE I and it will look like this:
CASE I: STEP I: N sits fourth to the right of H. Five persons sit
between N and C.

As per these statements, there will be two cases getting


framed and they are as follows:

CASE II: CASE I:

STEP IV: Only one person sits to the right of the one,
who likes Sunflower. No-one sits to the right of the one, CASE II:
who likes Dairy Milk (DM).

As per these statements the final arrangement will look


like this:

CASE I:

STEP II: Four persons sit between H and F.

As per these statements, these two cases will further get


split into two more cases and they are as follows:

Answers (36-40): CASE I:

Page 129 of 728

Subscribe the Xpress Video Course & Mock Test Package for Bank & Insurance Exams
If there are any suggestions/ errors in our PDFs Feel Free to contact us via this email: admin@exampundit.in
IBPS RRB PO Prelims – Ultra Practice Bundle PDF

CASE I (A): CASE I (A):

CASE II:
CASE II:

CASE II (A):
CASE II (A):

STEP III: G sits at one of the positions left to H. The STEP IV: C is 3rd to the left of K. K is 2 nd from one of
number of persons sitting between N and F are same as the extreme ends. No one sits to the right of M, who is to
between H and G. the immediate right of K. H is not an immediate
neighbour of G.
As per these statements, the arrangements will look like
this: As per these statements CASE I, CASE I (A) and
CASE II will get eliminated and we will proceed with
CASE I: CASE II (A) and it will look like this:

CASE I:
Page 130 of 728

Subscribe the Xpress Video Course & Mock Test Package for Bank & Insurance Exams
If there are any suggestions/ errors in our PDFs Feel Free to contact us via this email: admin@exampundit.in
IBPS RRB PO Prelims – Ultra Practice Bundle PDF

41) C
42) D
43) B
44) A
45) E
CASE I (A):
Solutions (41-45):
STEP I: Damien sits adjacent to Chinki, who sits third
from left end of the row. Abhas is sitting at the left end
of the row. Eva sits to the immediate right of Damien.

As per these statements, the sitting arrangement will look


CASE II: like this:

STEP II: Only 1 person sits between Damien and


CASE II (A):
Fredie, who does not sit adjacent to Chinki. Only 4
persons sit between Chinki and Harish.

As per these statements, the arrangement will be like


this:
STEP V: Two persons sit between J and F, who is to the
right of J.

As per this statement, the sitting arrangement will look


like this:
STEP III: Bobby does not sit on the extreme ends of the
row and he sits to the left of Eva. Only 4 persons sit
between Gagan and Lavlesh, who sits to the right of
Fredie. Fredie sits to the left of Gagan and Bobby sits to
Answers (41-45):
the left of Chinki, who does not sit adjacent to Jacob.
Page 131 of 728

Subscribe the Xpress Video Course & Mock Test Package for Bank & Insurance Exams
If there are any suggestions/ errors in our PDFs Feel Free to contact us via this email: admin@exampundit.in
IBPS RRB PO Prelims – Ultra Practice Bundle PDF
Only 2 persons sit between Gagan and Jacob. Lavlesh
sits at extreme end of the row.

As per these statements, the arrangement will look like


this:

CASE II:

STEP IV: Jacob sits to the right of Iris but not


immediate right. The number of persons who sit on the
right side of Fredie is 1 more than sitting to the left side
of him. STEP II: E is an immediate neighbour of G.

As per these statements, the final arrangement will look As per this statement, there will be two more cases
like this: getting framed and they will look like this:

CASE I:

Answers (46-50):

46) E
CASE I (A):
47) B
48) B
49) D
50) A

Solutions (46-50):
CASE II:
STEP I: Three persons sit between G and B.

As per this statement, there will be two cases getting


framed and they will look like this:
CASE I:
Page 132 of 728

Subscribe the Xpress Video Course & Mock Test Package for Bank & Insurance Exams
If there are any suggestions/ errors in our PDFs Feel Free to contact us via this email: admin@exampundit.in
IBPS RRB PO Prelims – Ultra Practice Bundle PDF

CASE II (A):

STEP IV: H sits second to the right of B. A and H are


not an immediate neighbours of each other.

As per these statements, CASE II and CASE II (A) will


STEP III: A sit second to the left of E.
get eliminated and we will continue with CASE I and
CASE I (A) and they will look like this:
As per this statement, the arrangement will look like this:

CASE I: CASE I:

CASE I (A):
CASE I (A):

CASE II: STEP V: One person sits between C and B.

As per this statement, the arrangement will look like this:

CASE I:

CASE II (A):

Page 133 of 728

Subscribe the Xpress Video Course & Mock Test Package for Bank & Insurance Exams
If there are any suggestions/ errors in our PDFs Feel Free to contact us via this email: admin@exampundit.in
IBPS RRB PO Prelims – Ultra Practice Bundle PDF
CASE I (A):

STEP VIII: F sits fourth to the right of E who is not an


immediate neighbour of C.

STEP VI: D sits second to the right of H.


As per this statement, CASE I (A) will get eliminated
As per this statement, the arrangement will look like this:
and we will continue with CASE I and it will look like
this:
CASE I:
CASE I:

CASE I (A):
CASE I (A):

STEP VII: I sit fourth to the right of H.


STEP IX: A and I do not sit at the end of the row. Less
As per this statement, the arrangement will look like this: than seventeen persons sit in the row.

CASE I: As per these statements, the final arrangement will look


like this:

CASE I (A):

CASE I (A):

Page 134 of 728

Subscribe the Xpress Video Course & Mock Test Package for Bank & Insurance Exams
If there are any suggestions/ errors in our PDFs Feel Free to contact us via this email: admin@exampundit.in
IBPS RRB PO Prelims – Ultra Practice Bundle PDF

7). Blood Relation Questions


Directions (1-3): These questions are based on the b. J
following information. c. B
Eleven members in a family – A, B, C, D, E, F, G, H, I, J d. H
and X are related to each other. The family is a three- e. I
generation family. Directions (4-6): Study the given information and
A is brother of C. B is the daughter of A. J is the sister answer the questions that follow.
of B whose mother is H. I is the father of H. E, who is Eight members in a family - Sudha, Ravi, Shardul,
the son of D, is brother of H. F, who has only one son Sakshi, Teju, Ramu, Deepak and Ramya went to Shimla
and is married to X. G is the husband of C. The mother - to beat their summer heat. Family consists of 2 couples.
in - law of G is X. Ramu has four children. Out of them, only one is
1) Who is the son of X? married.
a. A The married son of Ramu has a daughter named Sakshi.
b. B Sudha is grandmother of Sakshi. Sakshi’s mother is the
c. C wife of Shardul who has an only sister named Ramya.
d. D Ravi is the younger brother of Shardul, who is not eldest.
e. E Teju is the wife of the brother of Ravi. Ramu is a male.
2) How is J related to C? 4) How Sakshi is related to Ravi?
a. Daughter a. Sister
b. Niece b. Daughter
c. Nephew c. Sister in law
d. Son d. Niece
e. Cannot be determined e. Wife
3) Who is the daughter of D? 5) Who is Teju in this family?
a. E a. Ravi’s sister
Page 135 of 728

Subscribe the Xpress Video Course & Mock Test Package for Bank & Insurance Exams
If there are any suggestions/ errors in our PDFs Feel Free to contact us via this email: admin@exampundit.in
IBPS RRB PO Prelims – Ultra Practice Bundle PDF
b. Sudha’s Mother in law d. Brother
c. Deepak’s wife e. None of these.
d. Shardul’s daughter 8) Who among the following doesn’t have more than
e. Ravi’s sister in law 13 coins?
6) Who is the elder brother of Shardul? I. B
a. Ramu II. A
b. Ravi III. C
c. Deepak a. Both I and II
d. Cannot be determined b. Only II
e. None of the above c. Both II and III
Directions (7-9): Answer the questions based on the d. Only III
information given below. e. Both I and III
There are six members A, B, C, D, E and F in a family 9) The person with 16 coins is the ____ of person with
which consists of only one married couple and three lowest number of coins.
generations. Each member has different number of coins a. Sister-in-law
among 4, 8, 11, 13, 16 and 19. b. Sister
Person with 4 coins is the paternal uncle of D. The only c. Son
sister of D has two coins more than the son of C. The d. Niece
sibling of the person with 13 coins has one daughter. C is e. None of these.
a female member. No female has 8 coins. Person with 16 Direction (10-12): Read the following information
coins is elder than F, who has 19 coins such that both are carefully to answer the question that follows:
not in same generation. Neither A nor E is married to C. In a family of 3 generations and 7 members- A to G,
E is younger than A such that both are not in same there are 2 couples. Only married couples can have
generation. children.
7) How is E related to F? A is the father of E. The sibling of D is the uncle of F. B
a. Father has 2 children and both are of different gender. The
b. Aunt father-in-law of C is the grandfather of G. No couple has
c. Grand father more than 2 children.
Page 136 of 728

Subscribe the Xpress Video Course & Mock Test Package for Bank & Insurance Exams
If there are any suggestions/ errors in our PDFs Feel Free to contact us via this email: admin@exampundit.in
IBPS RRB PO Prelims – Ultra Practice Bundle PDF
10) How is B related to D? b. Sister
a. Father c. Granddaughter
b. Mother d. Husband
c. Sister e. Can’t be determined
d. Daughter 14) How S is related to T?
e. Can’t be determined a. Granddaughter
11) How is E related to C? b. Son
a. Brother c. Mother
b. Son d. Sister-in-law
c. Son-in-law e. Mother-in-law
d. Brother-in-law 15) How T is related to U?
e. Can’t be determined a. Husband
12) How is G related to D? b. Son
a. Son c. Father
b. Daughter d. Father-in-law
c. Brother e. None of these
d. Sister Directions (16-18): These questions are based on the
e. Can’t be determined. following information.
Directions (13- 15): These questions are based on the P is son of R. U is sister of P. L has 3 children out of
following information. whom 2 are married. X is Q’s daughter. V is daughter-in-
There are eight members in the family P, Q, R, S, T, U, law of R, but not married to P. Q is sister-in-law of S and
V and W with the three generations. mother of K. L is S’s father. S is brother of U. K is
S is the mother of V. W is the niece of R. P is the father grandson of L.
of T. Q has only two married sons. V is the grandson of 16) How is V related to L?
Q. U is the sister-in-law of R, who is the father of V. a. Brother
There is no single parent in the family. b. Sister
13) How is W related to P? c. Daughter-in-law
a. Grandson d. Sister-in-law
Page 137 of 728

Subscribe the Xpress Video Course & Mock Test Package for Bank & Insurance Exams
If there are any suggestions/ errors in our PDFs Feel Free to contact us via this email: admin@exampundit.in
IBPS RRB PO Prelims – Ultra Practice Bundle PDF
e. None of these 20) How is S related to U if gender of S and Q is
17) How is X related to S? same?
a. Niece a. Son
b. Brother b. Daughter
c. Son c. Father
d. Nephew d. Sister
e. None of these e. None of these
18) Which of the following statement is true? 21) How many male members in the family if S is the
a. P is the father of K only daughter of R?
b. P and V are brother and sister a. One
c. U and X are married couple b. Two
d. L is grandmother of X c. Three
e. Q is the brother-in-law of U d. Four
Directions (19-21): These questions are based on the e. None of these
following information. Directions (22-24): Study the following information
There are six members P, Q, R, S, T and U in the family carefully and answer the questions that follow:
of three generations. There are two members in each Ten persons P, Q, R, S, T, U, V, W, X and Z are there in
generation. a family of three generation. There are three married
Q is father-in-law of U. R is father of S. T is not married couple. S is the sister-in-law of R but not married to P. P
to U or R. T is not in 3rd generation. S is not married to is the son of V. W is the brother of T. Z is the father-in-
P. There is no single parent in the family. law of Q. U is the mother of X and married to the brother
19) How is P related to Q? of R. Q has only one sister, who is unmarried. X has no
a. Grandson sibling. Z has only one grandson and no girl child. T is
b. Father the youngest person of the family.
c. Son 22) What is the ratio of male members to female
d. Granddaughter members of the family respectively?
e. Cannot be determined a. 5:4
b. 1:1
Page 138 of 728

Subscribe the Xpress Video Course & Mock Test Package for Bank & Insurance Exams
If there are any suggestions/ errors in our PDFs Feel Free to contact us via this email: admin@exampundit.in
IBPS RRB PO Prelims – Ultra Practice Bundle PDF
c. 3:2 d. 5
d. 2:3 e. 4
e. none of these 26) How is P related to L?
23) How is S related to T? a. Daughter in Law
a. Mother b. Son in Law
b. Father c. Son
c. Uncle d. Daughter
d. Aunt e. None of the above
e. None of these 27) How is O related to K?
24) What is the relation of W with respect to P? a. Daughter in Law
a. Niece b. Son in Law
b. Nephew c. Son
c. Cousin d. Daughter
d. Brother e. None of the above
e. None of these Directions (28-30): Read the given information
Directions (25-27): Read the following information carefully and answer the questions given below.
carefully and answer the question that follows: There are eight members in a family. There are three
There are seven people from the same family. Also, there couples in the family. F is the son of A but A is not his
are only two married couple among them. O is the only father. H is the brother in law of F, who is married and
sister of P. P is the brother - in-law of M. T is daughter- has only one son. U is the grandchild of A. E has two
in-law of H. H is father of O. L who is a female member children and both of them are married. B is the sister -in -
is having two daughters of which only one is married. K law of C who is the mother of U. E is the father of C who
is the mother of P. is married to H. D is the grandson of E.
25) How many female members are there in the 28) How U related to the grandmother of D?
family? a) Grandson
a. 2 b) Granddaughter
b. 3 c) Son
c. 1 d) Daughter
Page 139 of 728

Subscribe the Xpress Video Course & Mock Test Package for Bank & Insurance Exams
If there are any suggestions/ errors in our PDFs Feel Free to contact us via this email: admin@exampundit.in
IBPS RRB PO Prelims – Ultra Practice Bundle PDF
e) Cannot be determined e. Cannot be determined.
29) What is the relation of H with the mother of F? 32) What is the relation of D with the husband of A?
a) Son a. Father
b) Daughter b. Father-In-law
c) Son-in-law c. Mother
d) Daughter-in-law d. Son
e) None of these e. Cannot be determined.
30) How is F related to U? Directions (33-35): Answer the questions based on the
a) Paternal Uncle information given below.
b) Maternal Uncle Eight persons A, B, C, D, E, F, G and H are in a family
c) Aunt of three generations. E is the only son of A. F is the son-
d) Daughter in-law of C, who is the husband of A. G is the only son
e) None of these of D. There are three married couples in the family. D is
Direction (31-32): Study the following information the not the wife of F. There is no single parent in the
carefully and answer the questions given below. family. H is a male and belongs to third generation.
A, B, C, D, E and F are members of a family. There are 33) How is A related to H?
two married couples. B is a psychologist and the father a. Grandmother
of E. F is the grandfather of C and is a manager. A is b. Son
daughter in law of D. D is the grandmother of E and is a c. Father
housewife. E and C are students. B is the son of D, but D d. Grandson
is not the father of B. There is one psychologist, one e. Can’t be determined
manager, one doctor, one housewife and two students in 34) How is B related to G?
the family. a. Niece
31) What is the profession of A? b. Nephew
a. Doctor c. Aunt
b. Student d. Uncle
c. Psychologist e. Sister-in-law
d. Housewife 35) How is H related to E?
Page 140 of 728

Subscribe the Xpress Video Course & Mock Test Package for Bank & Insurance Exams
If there are any suggestions/ errors in our PDFs Feel Free to contact us via this email: admin@exampundit.in
IBPS RRB PO Prelims – Ultra Practice Bundle PDF
a. Niece c. Cousin
b. Nephew d. Grandfather
c. Aunt e. None of these
d. Uncle Directions (39-42): The questions are based on the
e. Sister-in-law following information:
Direction (36- 38): Answer the questions based on the Three generations of people live together. There are 12
information given below. members in the family viz. M, N, O, P, Q, R, S, T, U, V,
There are nine persons in a family and there are three W and X who went out for a picnic.
generations in the family. R and Q are kids of T and S. V, who is the father in law
A and B are the only daughters- in- law of C, who is the of W, is the paternal uncle of R. T has a son and a
sister in law of D. D is the only sister of E. F and G are daughter. P is the husband of O. O is the daughter of Q.
the sons of H and I respectively. I is not married to B. M is the brother of N. R is married to U. Father of N is U
There is no single parent in the family. E has no brother. who has only one daughter. S is the father in law of U. W
36) How is E related to F? is the wife of X.
a. Brother in law 39) Four of the five are alike in a certain way and
b. Uncle hence form a group. Which one the following doesn’t
c. Cousin belong to the group?
d. Grandfather a. Q
e. None of these b. P
37) How is H related to A? c. O
a. Brother d. M
b. Sister e. N
c. Brother in law 40) How is U related to T?
d. Uncle a. Cousin
e. None of these b. Granddaughter
38) How is C related to G? c. Niece
a. Grandmother d. Son in law
b. Uncle e. Daughter in law
Page 141 of 728

Subscribe the Xpress Video Course & Mock Test Package for Bank & Insurance Exams
If there are any suggestions/ errors in our PDFs Feel Free to contact us via this email: admin@exampundit.in
IBPS RRB PO Prelims – Ultra Practice Bundle PDF
41) How is N related to S? b. Sister
a. Grandson c. Daughter – in – law
b. Daughter’s cousin d. Cousin
c. Granddaughter e. None of these
d. Son’s nephew. 45) In the expression A @ B $ C # D # E how is A
e. Daughter related to E?
42) How is O related to U? a. Cousin
a. Nephew b. Daughter- in - law
b. Niece c. Daughter
c. Daughter in law d. Sister
d. Cannot be determined e. None of these
e. None of these Directions (46- 47): Answer the questions based on
Directions (43-45): Study the following information the information given below.
carefully and answer the given questions. There are seven members, P, Q, R, S, T, U and V in a
P @ Q means P is mother of Q. family. There is no single parent. There are three
P $ Q means P is sister of Q. generations in a family.
P % Q means P is daughter of Q. S’s only brother’s mother’s mother’s only brother is Q.
P # Q means P is son of Q. U is the only son-in-law of T. Gender of P and V is
43) In expression K $ L @ M % N how is N related to same. V is unmarried. Gender of R and S is same.
K? 46) How is T related to Q’s niece?
a. Father or Mother a. Mother
b. Father b. Mother-in-law
c. Brother - in - law c. Father-in-law
d. None of these d. Brother
e. Cannot be determined e. None of these
44) In expression E $ F # L $ G @ H how E is related 47) How is R related to U?
to H? a. Husband
a. Daughter b. Wife
Page 142 of 728

Subscribe the Xpress Video Course & Mock Test Package for Bank & Insurance Exams
If there are any suggestions/ errors in our PDFs Feel Free to contact us via this email: admin@exampundit.in
IBPS RRB PO Prelims – Ultra Practice Bundle PDF
c. Sister c. G
d. Uncle d. B
e. None of these e. None of these.
Direction 48-50: Answer the questions given below 49) E is the ____ of C?
based on the information given below. a. Son
There are eight members in the family viz. A, B, C, D, E, b. Daughter
F, G and H and they are related to each other. c. Cousin
A is the mother of D, who is the father of G. B, who is d. Father
the grandfather of E, is the husband of A. D, who has e. Can’t be determined.
only two children, is a brother of C. A has only two 50) How D is related to F?
children and both of them are of same gender. F is the a. Son
paternal aunt of H who is the sister of G. C has a child. b. Daughter
48) Who is the father in law of F? c. Cousin
a. A d. Father
b. D e. Brother

7). Blood Relation Questions - Solutions with Explanation


Solutions 1-3
1) A is brother of C.
2) G is the husband of C.
3) B is the daughter of A.

4) J is the sister of B whose mother is H.


5) I is the father of H.
6) E, who is the son of D, is brother of H.
Page 143 of 728

Subscribe the Xpress Video Course & Mock Test Package for Bank & Insurance Exams
If there are any suggestions/ errors in our PDFs Feel Free to contact us via this email: admin@exampundit.in
IBPS RRB PO Prelims – Ultra Practice Bundle PDF
7) F, who has only one son and is married to X.
8) The mother - in - law of G is X.

4. d
5. e
6. c
Solution 7-9
1. a The person with 4 coins is the paternal uncle of D.
2. b The only sister of D has two coins more than the son of
3. d C.
Solutions 4-6 C is a female member.
1) Ramu has four children. Out of them, only one is The sibling of the person with 13 coins has one daughter,
married. so C must be the married sister in law of the person with
2) The married son of Ramu has a daughter named 4 coins.
Sakshi (implies Sakshi is the granddaughter of Ramu). Family Tree:
3) Sudha is grandmother of Sakshi (implies Ramu and
Sudha are husband and wife).
4) Sakshi’s mother is the wife of Shardul who has an
only sister named Ramya (implies Shardul has two
brothers and one sister).
5) Ravi is the younger brother of Shardul.
6) Teju is the wife of the brother of Ravi (implies Teju is
No female has 8 coins, so the husband of C has 8 coins.
the wife of only married Shardul).
Person with 16 coins is elder than F, who has 19 coins,
so C has 16 coins and F is the daughter of D.
Page 144 of 728

Subscribe the Xpress Video Course & Mock Test Package for Bank & Insurance Exams
If there are any suggestions/ errors in our PDFs Feel Free to contact us via this email: admin@exampundit.in
IBPS RRB PO Prelims – Ultra Practice Bundle PDF
Neither A nor E is married to C, so B is married to C.
E is younger than A, so E is the sister of D and A is the
brother of B.
The final family tree is given below:

10. b
11. d
12. e (Gender of G is unknown)

7. b Solution: 13-15

8. a From the given information

9. a 1. Q has only two married sons.

Solution 10-12 2. U is the sister in law of R, who is the father of V.

From the given information, 3. S is the mother of V.


Q has two married sons so one of them is surely R and

1. A is the father of E. there are no single parents so, P must be husband of Q

2. The sibling of D is the uncle of F. and T must be brother of R.

3. B has 2 children and both are of different gender.


4. The father-in-law of C is the grandfather of G.
5. No couple has more than 2 children
Based on given data, we can draw family tree

13.c
14.d
15. a
Page 145 of 728

Subscribe the Xpress Video Course & Mock Test Package for Bank & Insurance Exams
If there are any suggestions/ errors in our PDFs Feel Free to contact us via this email: admin@exampundit.in
IBPS RRB PO Prelims – Ultra Practice Bundle PDF
Solution: 16-18 5. S is not married to P.
From the given information
1. P is son of R. U is sister of P. V is daughter-in-law of
R, but not married to P.

2. Q is sister-in-law of S and mother of K. S is brother of 19. e (Gender of P is unknown)

U.K is grandson of L which means P and Q are married 20. a

couple. 21. c
Solution: 22-25
1. S is the sister-in-law of R but not married to P.
2. P is the son of V. W is the brother of T.
3. Z is the father-in-law of Q.

16.c
17. a
4. U is the mother of X and married to the brother of R.
18. a
5. Q has only one sister, who is unmarried. X has no
Solution: 19-21
sibling.
From the given information
6. Z has only one grandson and no girl child.
1. Q is father-in-law of U.
7. T is the youngest person of the family.
2. R is father of S.
3. T is not married to U or R.
4. T is not in 3rd generation.

Page 146 of 728

Subscribe the Xpress Video Course & Mock Test Package for Bank & Insurance Exams
If there are any suggestions/ errors in our PDFs Feel Free to contact us via this email: admin@exampundit.in
IBPS RRB PO Prelims – Ultra Practice Bundle PDF
2) H is the brother in law of F who is married and has
only one son
3) E is the father of C who is married to H.
That means, A and E is couple and their son and
daughter are F and C.

22. d
23. d
24. b
Solutions 25-27
1. O is the only sister of P. 4) B is the sister -in -law of C who is the mother of U
2. P is the brother - in-law of M. 5) E has two children and both of them are married.
3. T is daughter-in-law of H. H is father of O. That means that F is married to B who is the sister-in-law
4. L who is a female member is having two daughters of of C
which only one is married. 6) D is the grandson of E. It means D is the son of F.
5. K is the mother of P.

25. d
26. b
27. d
Solution 28-30
28. e (Gender of U is unknown)
1) F is the son of A but A is not his father that means A
29. c
is F’s mother.
Page 147 of 728

Subscribe the Xpress Video Course & Mock Test Package for Bank & Insurance Exams
If there are any suggestions/ errors in our PDFs Feel Free to contact us via this email: admin@exampundit.in
IBPS RRB PO Prelims – Ultra Practice Bundle PDF
30. b
Solutions 31-32
1. B is a psychologist and the father of E.
2. F is the grandfather of C and is a manager.
3. A is daughter in law of D. D is the grandmother of E
and is a housewife.
4. E and C are students. B is the son of D, but D is not
G is the only son of D, but D is not the wife of F, which
the father of B.
means D is the wife of E. H belongs to third generation
5. There is one psychologist, one manager, one doctor,
which means H is the son of F, because there is no single
one housewife and two students in the family
parent in the family.
The final arrangement is as follows;

31. a
32. c
33. a
Solutions 33-35
34. c
E is the only son of A.
35. b
F is the son-in-law of C, who is the husband of A.
Solution 36-38
A and B are the only daughter in law of C, who is the
sister in law of D, who is the only sister of E which
means E is brother of D. E has no brother.

Page 148 of 728

Subscribe the Xpress Video Course & Mock Test Package for Bank & Insurance Exams
If there are any suggestions/ errors in our PDFs Feel Free to contact us via this email: admin@exampundit.in
IBPS RRB PO Prelims – Ultra Practice Bundle PDF
F and G are the sons of H and I respectively which 7) M is the brother of N.
means F and G are married to A and B but I is not So, N is the female as U has only one daughter.
married to B.
Family tree:

8) S is the father in law of U.


This implies T is the wife of S.
36. d 9) V, who is the father in law of W is the paternal uncle
37. c of R.
38. a This implies V is the brother of S.
Solutions 39-42 10) W is the wife of X.
1) P is the husband of O. So, X is the son of V and husband of W.
2) O is the daughter of Q.
3) R and Q are the kids of T and S.

39. a
40. d
4) R is married to U. 41. c
5) Father of N is U who has only one daughter. 42. b
6) T has a son and a daughter. Solutions 43
This means R is the mother of N and daughter of T. So, Let us first decode the given symbols and then draw a
Q is the son of T. family tree.

Page 149 of 728

Subscribe the Xpress Video Course & Mock Test Package for Bank & Insurance Exams
If there are any suggestions/ errors in our PDFs Feel Free to contact us via this email: admin@exampundit.in
IBPS RRB PO Prelims – Ultra Practice Bundle PDF
P is
Symbol $ # @ %
Meaning sister Son Mother Daughter
Of Q
Expression K $ L @ M % N means that
K is sister of L. L is mother of M. M is daughter of N . Solutions 45
Now draw family tree using following notations: Let us first decode the given symbols and then draw a
family tree.

P is
Symbol $ # @ %
Meaning Sister Son Mother Daughter
Solutions 44 Of Q
Let us first decode the given symbols and then draw a Expression A @ B $ C # D # E means that
family tree. A is mother of B. B is sister of C. C is son of D. D is
P is son of E.
Symbol $ # @ % Now draw family tree using following notations:
Meaning sister Son Mother Daughter
Of Q
Expression E $ F # L $ G @ H means that
E is sister of F. F is son of L. L is sister of G. G is mother
of H.
Now draw family tree using following notations:
43. c
44. d
45. b
Solutions 46-47

Page 150 of 728

Subscribe the Xpress Video Course & Mock Test Package for Bank & Insurance Exams
If there are any suggestions/ errors in our PDFs Feel Free to contact us via this email: admin@exampundit.in
IBPS RRB PO Prelims – Ultra Practice Bundle PDF
1. S’s only brother’s mother’s mother’s only brother is 47. b
Q. Solutions 48-50
2. U is the only son-in-law of T. A is the mother of D who is father of G. B, who is the
3. Gender of P and V is same. grandfather of E is the husband of A. D, who has only
4. V is unmarried. Gender of R and S is same. two children, is a brother of C.
5. The final family tree diagram is as follows: The final arrangement is given as follows.

48. e
49. e (Gender of E is unknown)
46. a
50. e

8). Number Sequence Questions


Directions (1-5): Study the following information 2. If all the digits are arranged in descending order
carefully and answer the below questions- within the number then what is the difference of
second and fifth number from the left end?
465 897 364 899 215 456 a) 215
1. If all the numbers are arranged in descending b) 466
order from right to left, then which will be the second c) 400
digit of the third number from left end? d) 682
a) 6 e) None of these
b) 4
c) 5 3. If all the digits in the numbers are arranged in
d) 3 ascending order, what is the difference of third and
e).2 fourth number from the left end in the new
arrangement?
Page 151 of 728

Subscribe the Xpress Video Course & Mock Test Package for Bank & Insurance Exams
If there are any suggestions/ errors in our PDFs Feel Free to contact us via this email: admin@exampundit.in
IBPS RRB PO Prelims – Ultra Practice Bundle PDF
a) 445 by a digit which has a numerical value of more than
b) 535 four?
c) 553 a) None
d) 456 b) one
e) 510 c) Two
d) Three
4. If all the numbers are arranged in ascending order e) More than three
from right to left, then what will be the product of
second digit of fifth number and third digit of second 7. How many such 1’s is there in the above
number from the left end? arrangement each of which is immediately preceded
a) 56 by a perfect square?
b) 36 a) None
c) 84 b) One
d) 42 c) Two
e) None of these d) Three
e) More than three
5. If all the digits in the numbers are arranged in
ascending order from left to right and then all the 8. How many such 9’s is there in the above
numbers are arranged in ascending order from right arrangement each of which is immediately preceded
to left, then which of the following will be the sum of and followed by an odd digit?
second digit of fifth number and first digit of second a) None
number from the left end? b) One
a) 10 c) Two
b) 11 d) Three
c) 15 e) More than three
d) 13
e) None of these 9. Which of the following is third to the left of the
eighteenth digit from the left end of the above
Directions (6-10): Study the following information arrangement?
carefully and answer the below questions- a) 8
b) 3
7617924156492341258584831275267 c) 4
3953 d) 5
e) 1
6. How many such 7’s is there in the above
arrangement each of which is immediately followed

Page 152 of 728

Subscribe the Xpress Video Course & Mock Test Package for Bank & Insurance Exams
If there are any suggestions/ errors in our PDFs Feel Free to contact us via this email: admin@exampundit.in
IBPS RRB PO Prelims – Ultra Practice Bundle PDF
10. If all the even digits are deleted from the above b) 3.6
arrangement, which of the following will be ninth c) 0
from the right end of the arrangement? d) 1.5
a) 9 e) 1.6
b) 3
c) 1 14. If 1 is added to the first digit and 2 is subtracted
d) 5 from the last digit of each of the numbers then which
e) 7 of the following numbers will be the second highest
number?
Directions (11-15): Study the following information a) 386
carefully and answer the below questions- b) 685
c) 438
685 513 438 386 297 d) 297
11. If 2 is added to the first digit of each of the e) 513
numbers how many numbers thus formed will be
divisible by three? 15. If in each number the first and the second digits
a) None are interchanged then which will be the highest
b) One number?
c) Two a) 297
d) Three b) 513
e) None of these c) 438
d) 685
12. If all the digits in each of the numbers are e) 386
arranged in descending order within the number,
which of the following will be the highest number in Directions (16-20): Study the following information
the new arrangement of numbers? carefully and answer the below questions-
a) 685
b) 386 276 652 564 473 739
c) 297 16. If 2 is added to the middle digit of each number
d) 438 and then the first and the second digits are
e) None of these interchanged, which of the following will be the third
digit of the smallest number?
13. What will be the resultant number if the second a) 6
digit of the second lowest number is divided by the b) 4
third digit of the highest number? c) 3
a) 2.5 d) 2

Page 153 of 728

Subscribe the Xpress Video Course & Mock Test Package for Bank & Insurance Exams
If there are any suggestions/ errors in our PDFs Feel Free to contact us via this email: admin@exampundit.in
IBPS RRB PO Prelims – Ultra Practice Bundle PDF
e) 9 Directions (21-25): Study the following information
carefully and answer the below questions-
17. If in each number the first and the third digit are
interchanged which number will be the second largest 345 436 512 417 264
number? 21. If ‘1’ is added to each odd digit, ‘2’ is added to
a) 276 each even digit, which of the following number is the
b) 652 second largest?
c) 564 a) 345
d) 473 b) 512
e) 739 c) 436
d) 417
18. The sum of which of the following three-digit e) None of these
numbers is not an odd number?
a) 652 22. If first and third digits are interchanged then 5 is
b) 739 added to even number and 2 is subtracted from odd
c) 564 number, then what is the product of third digit of the
d) 473 highest number with second digit of the second lowest
e) 276 number?
a) 63
19. If in each number all the three digits are arranged b) 54
in ascending order within the number, which of the c) 72
following numbers will be the second smallest d) 18
number? e) None of these
a) 564
b) 652 23. If 2 is added to the digit which is below 5, 1 is
c) 276 subtracted from the digit which is above 5 (include 5
d) 739 also), how many digits appear twice within the
e) 473 number?
20. If in each number the second and the third digits a) 2
are interchanged, what will be the sum of the first b) 3
and the third digit of the largest number? c) 1
a) 9 d) None
b) 8 e) None of these
c) 10
d) 12 24. If first and third digits are interchanged, then 2 is
e) 15 added to 1st digit of each number, and 1 is subtracted

Page 154 of 728

Subscribe the Xpress Video Course & Mock Test Package for Bank & Insurance Exams
If there are any suggestions/ errors in our PDFs Feel Free to contact us via this email: admin@exampundit.in
IBPS RRB PO Prelims – Ultra Practice Bundle PDF
from second digit of each number, then what is the d) Six
sum of first digit of the highest number and second e) Five
digit of the third lowest number?
a) 8 28. How many 9s are there which is preceded by 6 but
b) 7 not followed by 7?
c) 12 a) Three
d) 11 b) Two
e) None of these c) One
d) More than Three
25. If in each number the second and the third digits e) None
are interchanged, what will be the sum of the first
and the third digit of the largest number? 29. How many numbers are there which are preceded
a) 5 by prime number & followed by perfect Square?
b) 6 a) Four
c) 7 b) Five
d) 8 c) Six
e) 9 d) Seven
e) None of These
Directions (26-30): Study the following information 30. How many odd numbers are there preceded by
carefully and answer the below questions- perfect cube?
a) One
9769724592724697865378976534284 b) Two
78456 c) Three
26. In the following series of number, find out how d) Four
many times 6 and 5 have appeared together? e) Five
a) Three
b) Two Directions (31-35): Study the following information
c) Four carefully and answer the below questions-
d) One
e) None 567 987 243 559 727
31. If the above series is arranged in descending order
27. In the following series of number, find out how then which among the following will be the third digit
many times 5 have appeared? from the right end of the second number from the left
a) Two end?
b) Three a) 3
c) Four b) 7

Page 155 of 728

Subscribe the Xpress Video Course & Mock Test Package for Bank & Insurance Exams
If there are any suggestions/ errors in our PDFs Feel Free to contact us via this email: admin@exampundit.in
IBPS RRB PO Prelims – Ultra Practice Bundle PDF
c) 9 c) 7
d) 5 d) 8
e) 2 e) 0

32. What is the difference between the third digit of Directions (36-40): Study the following information
the number which is second from the left end and the carefully and answer the below questions-
first digit of the number that is first from the right
end as given in the series? 625 389 458 876 268 932
a) 2 36. If the two-digit numbers are formed by taking the
b) 5 2nd and 3rd digit respectively of each of the given
c) 4 numbers and then the position of 2nd and 3rd digit of
d) 1 each number are interchanged. Then which of the
e) 0 following number will be the second highest number
in the revised form?
33. If ‘1’ is added to all the even digits and ‘2’ is a) 876
subtracted from all odd digits then how many 3’s can b) 389
be found in the above series? c) 458
a) 7 d) 268
b) 2 e) None of these
c) 5
d) 6 37. If these numbers are written with their digits in
e) 4 the reverse order, which number among them will be
third highest?
34. If all the even digits are removed then what is the a) 625
sum of first three digits from the right end and last b) 268
two digits from the left end as in the given series? c) 458
a) 35 d) 876
b) 27 e) None of these
c) 33
d) 38 38. If the first two digits of each number are
e) 42 interchanged, which among these numbers will be the
2nd lowest number?
35. What is the minimum difference between any two a) 625
numbers in the above given series? b) 932
a) 11 c) 458
b) 9 d) 268

Page 156 of 728

Subscribe the Xpress Video Course & Mock Test Package for Bank & Insurance Exams
If there are any suggestions/ errors in our PDFs Feel Free to contact us via this email: admin@exampundit.in
IBPS RRB PO Prelims – Ultra Practice Bundle PDF
e) None of these 42. How many such multiples of 2 are there in the
above sequence which is immediately preceded by
39. How many numbers in the given number series even number?
are divisible by 3? a) One
a) 0 b) Two
b) 1 c) Three
c) 2 d) Four
d) 3 e) Five
e) None of these
43. If all the odd numbers in the sequence are
40. If one is subtracted from the middle digit of each replaced by alphabets starting from A from the left,
number and then all the digits in each number are then which number is replaced by alphabet ‘G’?
arranged in ascending order from left to right end; a) 4
then which of the following number is second b) 9
smallest? c) 3
a) 458 d) 6
b) 625 e) 5
c) 932
d) 268 44. If every alternate number is dropped from the left
e) None of these end (1 is dropped), how many even numbers have
even numbers as both of their neighbours?
Directions (41-45): Study the following information a) None
carefully and answer the below questions- b) One
c) Two
5129387146832397125489354676 d) Three
Q 41. If all the odd numbers are dropped from the e) Four
above sequence, what is the sum of the numbers
present in the odd position of the sequence? 45. If all the even numbers are increased by 1 in the
a) 28 given sequence, then how many 3’s are there in the
b) 30 new sequence?
c) 24 a) Three
d) 36 b) Four
e) 32 c) Five
d) Six
e) Seven

Page 157 of 728

Subscribe the Xpress Video Course & Mock Test Package for Bank & Insurance Exams
If there are any suggestions/ errors in our PDFs Feel Free to contact us via this email: admin@exampundit.in
IBPS RRB PO Prelims – Ultra Practice Bundle PDF
Directions (46-50): Study the following information a) 12
carefully and answer the below questions- b) 8
c) 9
324 489 176 294 595 d) 10
46. If the entire digits in each number is added within e) None of these
itself. Then, what is the difference between highest
and lowest number? 49. If the third digit of each number is decreased by 1,
a) 15 then how many numbers are divisible by 3?
b) 12 a) 1
c) 20 b) 2
d) 18 c) None
e) None of these d) 3
e) 4
47. If the digits in each number are arranged in
ascending order within the number. Then, what is 50. Among the following numbers, how many
difference between highest and lowest number? numbers are divisible by 3?
a) 392 a) One
b) 362 b) Two
c) 374 c) Three
d) 382 d) More Than Three
e) None of these e) None of These
48. What is the product of highest digit of the second
highest number and lowest digit of the lowest
number?

8). Number Sequence Questions - Detailed Explanation with Answers

Answers (1-5): The given number series in question is- 2) After arranging digits in descending order within
465 897 364 899 215 456 number in second and fifth number from left end-
1) Arrange numbers in descending order from right to New number becomes 987 and 521
left- Difference between them- 987-521= 466
215 364 456 465 897 899 Answer: B (466)
Second digit of third number from left end is 5.
Answer: C (5) 3) After arranging digits in ascending order within
number in third and fourth number from left end-
Page 158 of 728

Subscribe the Xpress Video Course & Mock Test Package for Bank & Insurance Exams
If there are any suggestions/ errors in our PDFs Feel Free to contact us via this email: admin@exampundit.in
IBPS RRB PO Prelims – Ultra Practice Bundle PDF
New number becomes 899 and 346
Difference between them- 899-346= 553 Answer: C (two)
Answer: C (553)
8) 9’s which is immediately preceded and followed by an
4) Arrange the numbers in ascending order from right to odd number-
left-
899 897 465 456 364 215 7617924156492341258584831275267
Product of second digit of fifth number and third digit of 3953
second number- 6 * 7= 42.
Answer: D (42) Answer: B (one)

5) Firstly, arrange all the digits in the number in 9) Digit third to the left of the eighteenth digit from the
ascending order from left to right- left end is 4.
456 789 346 899 125 456 Answer: C (digit 4)
Then arrange all the numbers in ascending order from
right to left- 10) After eliminating all even digits from right end then
899 789 456 456 346 125 9th digit from right end is 3.
The sum of second digit of fifth number and first digit of Answer: B (digit 3)
second number- 4+7= 11
Answer: B (11) Answers (11-15): Given number series as follows-
685 513 438 386 297
Answers (6-10): Given number series are as follows-
7617924156492341258584831275267 11) Add 2 in first digit of every number one by one-
3953 New series becomes-
6) 7’s which is immediately followed by number greater 885 713 638 586 497
than 4- Only 885 is divisible by 3.
7617924156492341258584831275267
3953 Answer: B (one)

Answer: D (three) 12) Arrange all digits in ascending order within the
number, then new series becomes-
7) 1’s which is immediately preceded by a perfect 865 531 843 863 972
square- Highest number is 972 which is arrived by a given
number 297 in given series.
7617924156492341258584831275267
3953 Answer: C (297)

Page 159 of 728

Subscribe the Xpress Video Course & Mock Test Package for Bank & Insurance Exams
If there are any suggestions/ errors in our PDFs Feel Free to contact us via this email: admin@exampundit.in
IBPS RRB PO Prelims – Ultra Practice Bundle PDF

13) Second lowest number – 386 17) After interchanging first and third digit of every
Highest number- 685 number, new series as follows-
When second digit of the second lowest number is 672 256 465 374 937
divided by the third digit of the highest number- 8/5= 1.6 Second largest number is 672 which is obtained by 276.

Answer: E (1.6) Answer: A (276)

14) When 1 is added to the first digit and 2 is subtracted 18) First, add digits of every given number-
from the last digit of each of the number in given series, 276 - 2+7+6= 15
it becomes- 652 – 6+5+2 = 13
783 611 536 484 395 564 – 5+6+4 =15
Second highest number is 611 which is obtained by the 473 – 4+7+3 = 14
number 513. 739 – 7+3+9 = 19
The sum of the digits of number 473 is an even number
Answer: E (513) only.

15) After interchanging first and second digit of every Answer: D (473)
number, the new series becomes-
865 153 348 836 927 19) Arrange all digits in ascending order within the
Highest number will be 927 which is obtained by 297. number-
267 256 456 347 379
Answer: A (297) The second smallest number is 267 which is obtained by
276.
Answers (16-20): Given number series is-
276 652 564 473 739 Answer: C (276)

16) When 2 is added to the middle digit of every number, 20) When second and third digits are interchanged, then
the new series becomes- new series is-
296 672 584 493 759 267 625 546 437 793
Now, after interchanging first and second digit of every Largest number – 793
number- Sum of first and third digit is 7+3 = 10.
926 762 854 943 579
Third digit of smallest number is- 9 Answer: C (10)

Answer: E (9)

Page 160 of 728

Subscribe the Xpress Video Course & Mock Test Package for Bank & Insurance Exams
If there are any suggestions/ errors in our PDFs Feel Free to contact us via this email: admin@exampundit.in
IBPS RRB PO Prelims – Ultra Practice Bundle PDF
Answers (21-25): Given number series in question as 24) First interchange both first and third digit of every
follows- number, new series becomes-
345 436 512 417 264 543 634 215 714 462
After adding ‘2’ to every first digit and subtract ‘1’ to
21) After adding ‘1’ to odd digit and ‘2’ to even digit, second digit of every number-
new series becomes- 733 824 405 904 652
466 648 624 628 486 Highest number- 904
Second largest number is 628 which is obtained by the Third lowest number- 733
number 417. Product- 9+3 = 12.
Answer: C (12)
Answer: D (417)
25) After interchanging, second and third digit of every
22) Interchange the first and third digit of every number number, new series becomes-
of given series. New series as follows- 354 463 521 471 246
543 634 215 714 462 Highest number – 521
Now, add ‘5’ to even numbers and subtract ‘2’ from odd Add first and third digit- 5+1 = 6.
numbers- Answer: B (6)
543-2 = 541
634+5 = 639 Answers (26-30): Given number series in question as
215-2 = 213 follows-
714+5 = 719 9769724592724697865378976534284
462+5 = 467 78456
Highest number – 719
Second lowest number – 467 26) 9 7 6 9 7 2 4 5 9 2 7 2 4 6 9 7 8 6 5 3 7 8 9 7 6 5 3 4
The product of third digit of the highest number with 28478456
second digit of the second lowest number- 9 * 6= 54. Three time digit 6 and 5 come together.
Answer: B (54) Answer: A (three)

23) By adding, ‘2’ in digit which is less than 5 and 27) 9 7 6 9 7 2 4 5 9 2 7 2 4 6 9 7 8 6 5 3 7 8 9 7 6 5 3 4


subtract ‘1’ in every digit of number which is equal or 28478456
greater than 5, then new series becomes- Four times ‘5’ appeared in the given series.
564 655 434 636 456 Answer: C (four)
Three numbers are there in which digits come twice
within a number. 28) 9 7 6 9 7 2 4 5 9 2 7 2 4 6 9 7 8 6 5 3 7 8 9 7 6 5 3 4
Answer: B (3) 28478456

Page 161 of 728

Subscribe the Xpress Video Course & Mock Test Package for Bank & Insurance Exams
If there are any suggestions/ errors in our PDFs Feel Free to contact us via this email: admin@exampundit.in
IBPS RRB PO Prelims – Ultra Practice Bundle PDF
There is no 9s in the given series which is preceded by 6 Answer: C (five)
but not followed by 7.
Answer: E (none) 34) After eliminating even digits, first three digits from
right end are- 7+7+9= 23
29) 9 7 6 9 7 2 4 5 9 2 7 2 4 6 9 7 8 6 5 3 7 8 9 7 6 5 3 4 Last digits from left end- 5+7= 12
28478456 Total sum- 23+12= 35.
There are seven digits which is preceded by prime Answer: A (35)
number and followed by perfect square.
Answer:D (seven) 35) Given number series is-
567 987 243 559 727
30) 9 7 6 9 7 2 4 5 9 2 7 2 4 6 9 7 8 6 5 3 7 8 9 7 6 5 3 4 Here, we can clearly observe that minimal difference
28478456 must be between 567 and 559.
There is only one odd number which is preceded by Difference- 567-559= 8.
perfect cube. Answer: D (8)
Answer: A (one)
Answers (36-40): Given number series as follows-
Answers (31-35): Given number series as follows- 625 389 458 876 268 932
567 987 243 559 727
36) First take second and third digit of every number-
31) Arrange numbers in descending order from left end- 25 89 45 76 68 32
987 727 567 559 243 Now interchange their positions-
Now, third digit from right end of the number which is 52 98 54 67 86 23
second from the left end- number is- 727. Then the second largest number is 86 formed by given
Answer: B (7) number 268.
Answer: D (268)
32) 567 987 243 559 727
Third digit of the number which is second from the left- 37) Arrange number with reversing digits within the
7 number-
First digit of the number which is first from right end- 7 526 983 854 678 862 239
Difference- 7-7 = 0 Third highest number is 854 formed by 458.
Answer: E (zero) Answer: C (458)

33) First, add ‘1’ to even digits and subtract ‘2’ from odd 38) After interchanging first and second digit within the
digit- number-
375 795 351 337 535 265 839 548 786 628 392
There are five 3s in the obtained series. Second lowest number – 392 obtained by 932.

Page 162 of 728

Subscribe the Xpress Video Course & Mock Test Package for Bank & Insurance Exams
If there are any suggestions/ errors in our PDFs Feel Free to contact us via this email: admin@exampundit.in
IBPS RRB PO Prelims – Ultra Practice Bundle PDF
Answer: B (932)
44) After eliminating every alternative number, new
39) Only number 876 is divisible by 3. series becomes-
Answer: B (1) 5 2 3 7 4 8 2 9 1 5 8 3 4 7
Only 8 is the number which is immediately preceded and
40) When one is subtracted from middle digit of every followed by even number.
number- Answer: B (one)
615 379 448 668 258 229
Then arrange numbers in ascending order from left to 45) Here, we want to count number 3 only and it will be
right- formed when 1 is added to even number 2 only. So, total
156 379 448 668 258 229 number of 3 is- 7
Second smallest number- 229 obtained by 932. Answer: E (seven)
Answer: C (932)
Answers (46-50): Given number series as follows-
Answers (41-45) Given numbers series as follows- 324 489 176 294 595
5129387146832397125489354676
46) When entire digits of every number is added-
41) After removing every odd numbers- 3+2+4= 9
2 8 4 6 8 2 2 4 8 4 6 6 4+8+9= 21
Sum of the numbers present at odd position- 1+7+6= 14
(Satisfied with both ends i.e. right and left) 2+9+4= 15
2+4+8+2+8+6= 30 5+9+5= 19
6+4+4+2+6+8=30 Difference between highest and lowest number- 21-9 =
Answer: B (30) 12.
Answer: B (12)
42) 5 1 2 9 3 8 7 1 4 68 3 2 3 9 7 1 2 5 4 8 9 3 5 4 6 7 6
There are four numbers in the given series which is 47) Arrange digits in ascending order within the number-
multiple of 2 and preceded by even numbers. 234 489 167 249 559
Answer: D (four) Now difference between the highest and lowest number-
559-167= 392
43) 5 1 2 9 3 8 7 1 4 6 8 3 2 3 9 7 1 2 5 4 8 9 3 5 4 6 7 6 Answer: A (392)
Now every odd number is replaced by letter A from left
end, new series becomes- 48) Highest digit of second highest number (489) is 9
AB2CD8EF468G Smallest digit of lowest number (176) is 1
Letter G is 3 as per the series. Product- 1 * 9= 9.
Answer: C (3) Answer: C (9)

Page 163 of 728

Subscribe the Xpress Video Course & Mock Test Package for Bank & Insurance Exams
If there are any suggestions/ errors in our PDFs Feel Free to contact us via this email: admin@exampundit.in
IBPS RRB PO Prelims – Ultra Practice Bundle PDF
49) If ‘1’ is subtracted by the third digit of every Answer: A (one)
number- 50) Only three numbers in the given series- 324, 489 and
323 488 175 293 594 294 is divided by 3.
Now, only number 594 is divisible by 3. Answer: C (three)
9). Alphabet Series Questions
Directions (1-3): The following questions are based on E) Five
the five three letter words given below.
Directions (4-6): Answer these questions referring to
ALL HOW MAD NEW ANY the letter sequence given below:

1. If the positions of the first and third letters of all PSCRUVBLHVWDGYFXYBMODMQ


the words are interchanged, how many words will AM
form meaningful English words?
A) None 4. If the letters of the above given series are written in
B) One reverse order, which letter will be the fourth to the
C) Two left of eleventh letter from the right end?
D) Three A) F
E) Four B) G
C) H
2. If all the letters in each of the words are arranged D) L
alphabetically (within the word from left to right), E) None of These
how many words will remain unchanged?
A) One 5. What will come in place of question mark (?) in the
B) Two following series?
C) Three
D) Four SRC VLB VDW ?
E) None
A) YFX
3. If the first letter of all the words is changed to its B) YXF
next letter of the English alphabetical series, then how C) FYX
many words will have more than one vowel? D) FXY
A) One E) None of These
B) Two
C) Three 6. Which of the following is seventh to the right of the
D) Four seventeenth letter from the left end?
Page 164 of 728

Subscribe the Xpress Video Course & Mock Test Package for Bank & Insurance Exams
If there are any suggestions/ errors in our PDFs Feel Free to contact us via this email: admin@exampundit.in
IBPS RRB PO Prelims – Ultra Practice Bundle PDF
A) A D) D
B) B E) E
C) C

Directions (7-8): The following questions are based C) 10


upon the alphabetical series given below- D) 11
E) 12
BSMQUAMHJSDKRENZRSIKO
10. Which of the following is the sixth letter to the left
7. What will come in place of question mark (?) in the of the 11th letter from left end?
following series? A) A
BM AH DR ? B) B
A) SZ C) C
B) DK D) E
C) ZX E) D
D) ZS
E) None of These Directions (11-15): Study the following questions and
answering the questions referring to the word
8. Which of the following is the seventh to the right of sequence given below:
the third letter from the left end?
A) J NEXT ACID BACK COME DARK
B) S
C) D 11. If words are arranged according to the
D) K alphabetical series from right to left, then which word
E) None of These is third from the left end?
A) ACID
Directions (9-10): Answer these questions referring to B) BACK
the letter sequence given below: C) COME
D) NEXT
LHWUDIYXANPSDQTWC E) DARK

9. How many letters are there between first and 12. If each letter is arranged according to the
second last letter (from left to right) of the above alphabetical series within each word (from left to
series according to alphabetical order? right), then which is the fourth letter of the word,
A) 8 which is second from the left end?
B) 9 A) A

Page 165 of 728

Subscribe the Xpress Video Course & Mock Test Package for Bank & Insurance Exams
If there are any suggestions/ errors in our PDFs Feel Free to contact us via this email: admin@exampundit.in
IBPS RRB PO Prelims – Ultra Practice Bundle PDF
B) D
C) C 16. If each vowel is changed to the previous letter and
D) I each consonant is changed to the next letter, then how
E) None of These many meaningful words can be formed?
A) None
13. If each vowel in the word is changed to its B) One
previous letter and each consonant change to the next C) Two
letter according to the alphabetical series, then how D) Three
many words contain vowel? E) Four
A) One
B) Two 17. How many letters are there as per English
C) Three alphabetical order between the last letter of the
D) Four second word and first letter of the third word from
E) None of These the left end when all the words are arranged
alphabetically from left to right?
14. If each consonant in the word is changed to its A) Five
next letter according to the alphabetical series, then B) Four
how many meaningful words can be formed? C) Six
A) One D) Seven
B) Two E) Eight
C) Three
D) Four 18. If the first and third letters are interchanged, the
E) None of These second letter is changed to the next letter, then how
many words contain at least two vowels?
15. If 2nd and 4th letter of each word are A) Two
interchanged, then how many meaningful words can B) Three
be formed? C) Four
A) One D) One
B) Two E) None
C) Three
D) More Than Three Directions (19-20): Study the following information
E) None carefully and answer the questions given below:
Directions (16-18): Study the following information CROWN GRAND HEART NOVEL SHIFT
carefully and answer the questions given below: 19. If each consonant is replaced by next letter and
each vowel is replaced by its previous letter, then
BOY CAN GYM INK KEY which of the following word is second from the left

Page 166 of 728

Subscribe the Xpress Video Course & Mock Test Package for Bank & Insurance Exams
If there are any suggestions/ errors in our PDFs Feel Free to contact us via this email: admin@exampundit.in
IBPS RRB PO Prelims – Ultra Practice Bundle PDF
end when all the words are arranged in ascending dictionary (from left to right), then which of the word
order from left to right? is third from the left end?
A) HEART A) INPUT
B) NOVEL B) DRINK
C) GRAND C) ROBIN
D) CROWN D) THICK
E) SHIFT E) None of These

20. If the first and fourth letters are interchanged, 23. If all the consonants are replaced by previous
second and third letters are interchanged within the letter and all the vowels are replaced by next letter,
word and the fifth letter is replaced by its next letter, then how many words are having two or more same
then how many letters appear twice within the word? letter?
A) None A) Two
B) One B) One
C) Two C) Three
D) Three D) More Than Three
E) Four E) None of These

Directions (21-23): Study the series of five 5-letter Directions (24-28): Study the following information
words given below and answer the questions below. carefully and answer the below questions:

THINK INPUT DRINK ROBIN HAPPY OLD SHE NAP SAY USE TAN

21. If first letter of each word from left is replaced by 24. If all the 2nd letter of the words is changed to
‘E’, then how many meaningful words can be their next letter as in the alphabetical order then how
formed? many meaning full words are formed?
A) None A) None
B) One B) One
C) Two C) Two
D) Three D) Three
E) None of These E) More Than Three

22. If all the letters of the words are rearranged in 25. How many letters are there between 3rd and
alphabetical order from left to right and thus newly 3rdletter of the word which is from extreme left and
formed words are arranged as they appear in the extreme right end respectively after arranging all
words in alphabetical order (from left to right)?

Page 167 of 728

Subscribe the Xpress Video Course & Mock Test Package for Bank & Insurance Exams
If there are any suggestions/ errors in our PDFs Feel Free to contact us via this email: admin@exampundit.in
IBPS RRB PO Prelims – Ultra Practice Bundle PDF
A) 12 E) None
B) 13
C) 10 (Directions 29–32): The given questions are based on
D) 14 five, 4-letter words which are given below. Study the
E) 15 following information carefully and answer the
questions given below it.
26. All the letters of words are arranged in
alphabetical order within the word from left to right BOMB ROOF SOUL NICE MANY
end. What is the 1st letter of the 3rd word from right
end? 29. If ‘C’ is added before each given word then, how
A) B many 5-letter meaningful word can be formed?
B) C A) One
C) D B) Three
D) E C) Four
E) A D) None
E) Two
27.If all the letters in the words are arranged in the 30. If all the letters within given words are arranged
reverse alphabetical order (from right to left within in reverse alphabetical order (right to left) and thus
the word). After that all the words are arranged as in newly formed words are rearranged as they appear in
alphabetical order from left to right end, then which dictionary (from left to right) then, which of the
is the third letter of the first word from the right end? following word appears third from the left end?
A) B A) BOMB
B) C B) SOUL
C) D C) ROOF
D) A D) NICE
E) E E) MANY

28. If all the vowels in the words are changed to next 31. If all the letters of the words are written together
letter as in the alphabetical order and all the in an alphabetical order (from right to left) without
consonants are changed to previous letter as in any space then, how many letters are there in
alphabetical order. How many words contain at least alphabetical series between sixth letter from left end
one vowel after arrangement? and fifth letter from right end in the English alphabet
A) Two series?
B) One A) 5
C) Three B) 6
D) More Than Three C) 7

Page 168 of 728

Subscribe the Xpress Video Course & Mock Test Package for Bank & Insurance Exams
If there are any suggestions/ errors in our PDFs Feel Free to contact us via this email: admin@exampundit.in
IBPS RRB PO Prelims – Ultra Practice Bundle PDF
D) 11 C) Two
E) 8 D) Three
E) Four
32. If all the Vowels are changed to previous letter
while all the consonants are changed to next letter 35. If 2nd and 3rd letters are interchanged and first
within same word then, in how many newly formed letter is changed to second next letter, then how many
words having two or more same letter? meaningful English words can be formed?
A) One A) One
B) Two B) Two
C) Three C) Three
D) Four D) More Than Three
E) More Than Four E) None

Directions (33-35): Study the following information (Directions 36-40): Study the following information
carefully and answer the questions given below: carefully and answer the questions given below it.

ACT FED DIP HIM GEM CFD WVU TSR XYZ KLM

33. If each vowel is changed to previous letter and 36. If first and second letters of all the words are
each consonant is changed to next letter, then all the interchanged within themselves then how many
words are arranged according to alphabetical order meaningful words can be formed?
from left to right, then which of the following word is A) None
fourth from the left end? B) One
A) DIP C) Two
B) HIM D) Three
C) GEM E) Four
D) ACT
E) None of These 37. If all the consonants of each word are changed to
its previous letter then how many words will have
34. How many letters are there as per English only consonants?
alphabetical order between second letter of third A) One
word and second letter of fourth word from the left B) Two
end when all the words are arranged alphabetically C) Three
from left end? D) More Than Three
A) None E) None of These
B) One

Page 169 of 728

Subscribe the Xpress Video Course & Mock Test Package for Bank & Insurance Exams
If there are any suggestions/ errors in our PDFs Feel Free to contact us via this email: admin@exampundit.in
IBPS RRB PO Prelims – Ultra Practice Bundle PDF
38. If all the words are arranged in alphabetical order A) SIX
from right to left then which of the following will be B) ABS
second from the left end of the series? C) GOT
A) XYZ D) ONE
B) WVU E) WAX
C) TSR
D) KLM 42. If all the letters in each word are arranged in
E) CFD alphabetical order from left to right and then arrange
the words as in the English dictionary from right to
39. How many meaningful words can be formed with left. Which of the following will be exactly in the
the first and last letter of each word separately (in middle?
same order)? A) SIX
A) One B) GOT
B) Two C) ONE
C) Three D) WAX
D) More Than Three E) ABS
E) None
43. If all the vowels are changed to previous letter and
40. How many meaningful words can be formed by all the consonants are changed to next letter as in the
placing the vowels in the middle of the words English alphabetical order. Then how many words
(consider only the words having vowels)? does have repeated letter within the word?
A) One A) More Than Three
B) Two B) Three
C) None C) Two
D) Three D) One
E) None of These E) None

Direction (41-44): These questions are based on five 44. Which of the following letters is the middle letter
three-letter words given below. of the word which is third to the left of the word
which is at the right end?
ABS GOT SIX ONE WAX A) G
B) T
41. If the first and second letters in each word are C) A
interchanged, then which of the following is the D) O
second word from the left end, if they are arranged as E) S
in the English dictionary (from left to right)?

Page 170 of 728

Subscribe the Xpress Video Course & Mock Test Package for Bank & Insurance Exams
If there are any suggestions/ errors in our PDFs Feel Free to contact us via this email: admin@exampundit.in
IBPS RRB PO Prelims – Ultra Practice Bundle PDF
Direction (45-48): Read the following information D) A
carefully and answer the questions given below. E) None of These

WEB NET YOU TWO RAT 48. How many letters are there in between the middle
letter of the second word from the right end and the
45. If all the vowels are changed to second next letter first letter of the first word from the left end as in the
and all the consonants are changed to second English alphabet series?
previous letter as in the English alphabet series. Then
in how many words a letter appears twice? A) One
A) None B) Two
B) One C) Three
C) Two D) None
D) Three E) More Than Three
E) Four
49. How many such pairs of letters are there in the
46. If all the letters in each word are arranged as in word ‘REPRESENTATION’ as many letters between
the English alphabet series from right to left and then them as in the English alphabet series? (both forward
arrange the words as in the English dictionary from and backward)
left to right. Then which of the following word is A) One
second from the right end? B) Two
A) TWO C) Three
B) WEB D) None
C) NET E) More Than Three
D) YOU
E) RAT 50. How many such pairs of letters are there in the
word ‘AUTOMATION’ as many letters between
47. If first and the last letters in each word are them as in the English alphabet series? (both forward
interchanged and then arrange the words as in the and backward)
English dictionary from left to right. Then which of
the following letter will be third letter of the second A) One
word from the left end? B) Two
A) O C) Three
B) W D) More Than Three
C) T E) None of These

Page 171 of 728

Subscribe the Xpress Video Course & Mock Test Package for Bank & Insurance Exams
If there are any suggestions/ errors in our PDFs Feel Free to contact us via this email: admin@exampundit.in
IBPS RRB PO Prelims – Ultra Practice Bundle PDF
9). Alphabet Series Questions - Detailed Explanation with Answers
Answers (1-3): BLL IOW NAD OEW BNY
As per the newly formed series there are only two words
1) B which has more than one vowel and that word is: OEW
2) C and IOW
3) B So, answer is TWO
Answer: B (Two)
Solutions (1-3): The series given in this question is:
Answers (4-6):
ALL HOW MAD NEW ANY
4) A
1) In this question we have to change the positions of the 5) B
first and third letters of all the words are interchanged 6) A
and the new series will look like this:
Solutions (4-6): The series given in this question is:
LLA WOH DAM WEN YNA
PSCRUVBLHVWDGYFXYBMODMQ
As per the above new series the meaningful word formed AM
is DAM. So, answer isONE
Answer: B (ONE) 4) In this question we have to write the series in reverse
order and the new series will look like this:
2) In this question all the letters in each of the words are MAQMDOMBYXFYGDWVHLBVURC
arranged alphabetically (within the word) and the new SP
series will look like this:
As per the series, the eleventh letter from the right end is
ALL HOW ADM ENW ANY W and fourth letter to the left of this letter is F.
So, answer is F
As per the new series which is formed there are three Answer: A (F)
words which remained unchanged. So, answer is Three.
Answer: C (Three) 5) In this question we have to find the pattern between
the letters and the different pair as per given in the
3) In this question, we have to change the first letter of question and the pattern is:
the series with the next letter as per the English In the pair, third letter is between the first and the second
alphabetical series and the new series will look like this: letter as per the series and between the two adjacent pairs
one letter is left and then next letter is written
Page 172 of 728

Subscribe the Xpress Video Course & Mock Test Package for Bank & Insurance Exams
If there are any suggestions/ errors in our PDFs Feel Free to contact us via this email: admin@exampundit.in
IBPS RRB PO Prelims – Ultra Practice Bundle PDF
As per this pattern the next pair will be:YXF So, answer is S
So, answer isYXF Answer: B (S)
Answer: B (YXF) Answers (9-10):

6) In this question we have to find the letter which is 9) C


seventh to the right of the seventeenth letter from the left 10) E
end.
As per the series given in the question, the letter which is Solutions (9-10): The series given in this question is:
seventeenth letter from the left end is Y and the seventh
letter to the right of this letter is A LHWUDIYXANPSDQTWC
SO, answer is A
Answer: A (A) 9) In this question we have to find the number of letters
between first and second last letter of the above series as
Answers (7-8): per English alphabetical series
First letter of this series is L and the second last of this
7) D series is W and between them there are 10 letters
8) B So, answer is 10
Answer: C (10)
Solutions (7-8): The series given in this question is:
10) In this question, we have to find the sixth letter to the
BSMQUAMHJSDKRENZRSIKO left of the 11th letter from left end
As per the series the letter which is 11th from the left end
7) In this question, we have to find the pattern between is P and the sixth letter to the left of P is D
the letters and the pairs as well as per the given series So, answer is D
The pattern is there is a gap of one letter between the Answer: E (D)
letters of the pair and there is a gap of two letters
between the pairs and then the letter is written Answers (11-15):
As per this pattern, the next pair will be: ZS
So, answer is ZS 11) C
Answer: D (ZS) 12) D
13) C
8) In this question, we have to find the seventh to the 14) A
right of the third letter from the left end 15) E
As per the series given in the question, letter which is
third from the left end is M and the letter which is Solutions (11-15): The series given in this question is:
seventh to the right of this letter is S

Page 173 of 728

Subscribe the Xpress Video Course & Mock Test Package for Bank & Insurance Exams
If there are any suggestions/ errors in our PDFs Feel Free to contact us via this email: admin@exampundit.in
IBPS RRB PO Prelims – Ultra Practice Bundle PDF
NEXT ACID BACK COME DARK 14) In this question we have to change each consonant to
next letter and the new series will look like this:
11) In this question we have to arrange according to the OEYU ADIE CADL DONE EASL
alphabetical series from right to left and the new series
will look like this: As per the new series there is only one word which is
meaningful and framed and that word is DONE
NEXT DARK COME BACK ACID So, answer is DONE
Answer: A (One)
As per the new series the word which is third from the
left end is COME 15) In this question, we have to interchange 2nd and 4th
So, answer is COME letter of the word and the new series will look like this:
Answer: C (COME)
NTXE ADIC BKCA CEMO DKRA
12) In this question, we have to arrange each letter
alphabetically within each word, and the new series will As per the given series there are no meaningful words
look like this: formed after the operations
So, answer is NONE
ENTX ACDI ABCK CEMO ADKR Answer: E (None)

As per the new series word which is second from the left Answers (16-18):
end is ACDI and the fourth letter of this word is I
So, answer is I 16) A
Answer: D (I) 17) C
18) D
13) In this question, we have to change each vowel to its
previous letter and each consonant change to the next Solutions (16-18): The series given in this question is:
letter according to the alphabetical series and the new
series will look like this: BOY CAN GYM INK KEY

ODYU ZDHE CZDL DNND EZSL 16) In this question, we have to change each vowel is
changed to the previous letter and each consonant is
As per the given new series, the words which contain changed to the next letter and the new series will look
vowel are: ODYU, ZDHE and EZSL like this:
So, answer is ODYU ,ZDHE and EZSL
Answer: C (Three) CNZ DZO HZN HOL LDZ

Page 174 of 728

Subscribe the Xpress Video Course & Mock Test Package for Bank & Insurance Exams
If there are any suggestions/ errors in our PDFs Feel Free to contact us via this email: admin@exampundit.in
IBPS RRB PO Prelims – Ultra Practice Bundle PDF
As per the new series, there are no meaningful words
formed after the performing of the operations. 19) In this question we have to change each consonant to
So, answer is NONE the next letter and each vowel is replaced by its previous
Answer: A (None) letter and the new series will look like this:
DSNXO HSZOE IDZSU ONWDM TIHGU
17) In this question, we have to arrange the words
alphabetically and the new series will look like this: As per the new series, the word which is second from the
left end is HSZME and this word is obtained after
BOY CAN GYM INK KEY performing operations on GRAND
So, answer is GRAND
As per the new series the word which is second from the Answer: C (GRAND)
left end is CAN and the last letter of this word is N and
the first letter of the third word is G and the number of 20) In this question we have to interchange first and
letters between N and G is six fourth letters and 2nd and 3rd letters are also interchanged
So, answer is SIX and the fifth letter is also changed to the next letter and
Answer: C (Six) the new series will look like this:

18) In this question, we have to interchange first and the WORCO NARGE RAEHU EVONM FIHSU
third letter and second word is changed to the next letter
and the new series will look like this: As per the new series, there is only one word which in
which one letter is appeared twice and that word is
YPB NBC MZG KOI YFK WORCO
So, answer is WORCO
As per the new given series, there is only one word Answer: B (One)
which has at least two vowels and that word is: KOI
So, answer is KOI Answers (21-23):
Answer: D (One)
21) A
Answers (19-20): 22) B
23) C
19) C
20) B Solutions (21-23): The series given in this question is:

Solutions (19-20): The series given in this question is: THINK INPUT DRINK ROBIN HAPPY

CROWN GRAND HEART NOVEL SHIFT

Page 175 of 728

Subscribe the Xpress Video Course & Mock Test Package for Bank & Insurance Exams
If there are any suggestions/ errors in our PDFs Feel Free to contact us via this email: admin@exampundit.in
IBPS RRB PO Prelims – Ultra Practice Bundle PDF
21) In this question, we have to replace the first letter of So answer is SGJMJ, CQJMJ and GBOOX.
the word with E, the new series will look like this: Answer: C (Three)
EHINK ENPUT ERINK EOBIN EAPPY Answers (24-28):

As per the new series, there are no meaningful words 24) A


which are formed after performing the operations 25) C
So, answer is None 26) E
Answer: A (None) 27) D
28) B
22) In this question, we have to arrange all the letters are
arranged in alphabetical order from left to right within Solutions (24-28): The series given in this question is:
the word and the new series will look like this:
OLD SHE NAP SAY USE TAN
HIKNT INPTU DIKNR BINOR AHPPY
24) In this question, we have to change 2nd letter of each
Now, we have to arrange the new series in alphabetical word with the next letter and the new series will look like
order from left to right is: this:

AHPPY BINOR DIKNR HIKNT INPTU OMD SIE NBP SBY UTE TBN

As per the new series, word which is third from the left As per the new series there are no meaningful words are
end is DIKNR and this word is formed after performing framed after performing the operations.
operations on DRINK So, answer is NONE
So, answer is DRINK Answer: A (None)
Answer: B (DRINK)
25) In this question we have to arrange all the words in
23) In this question, we have to replace all the the alphabetical order and the new series will look like
consonants with the previous letters and all the vowels this:
with the next letters and the new series will look like
this: NAP OLD SAY SHE TAN USE

SGJMJ JMOVS CQJMJ QPAJM GBOOX As per the new series the word which is from the
extreme left is NAPand third letter of this word isP and
As per the new series, there are three words in which the word which is extreme right from this series
there are two or more same letters are there and they are: isUSEand the 3rdletter of this word isE and the number
SGJMJ, CQJMJ and GBOOX. of letters between P and E is 10

Page 176 of 728

Subscribe the Xpress Video Course & Mock Test Package for Bank & Insurance Exams
If there are any suggestions/ errors in our PDFs Feel Free to contact us via this email: admin@exampundit.in
IBPS RRB PO Prelims – Ultra Practice Bundle PDF
So, answer is 10
Answer: C (10) PKC RGF MBO RBX VRF SBM

26) In this question we have to arrange all the letters in As per the new series, there is only one word which has
alphabetical order within the word and the new series at least one vowel is MBO.
will look like this: So, answer is MBO
Answer: B (One)
DLO EHS ANP ASY ESU ANT
Answers (29-32):
rd
As per the new series the word which is 3 from the
right end is ASY and the first word of this word is A 29) D
So, answer is A 30) C
Answer: E (A) 31) E
32) E
27) In this question, firstly we have to arrange all the
letters within the word in reverse alphabetical order and Solutions (29-32): The series given in this question is:
the new series will look like this:
BOMB ROOF SOUL NICE MANY
OLD SHE PNA YSA USE TNA
29) In this we have to add C to every word and the new
Now, we have to arrange this new series in alphabetical series will look like this:
order from the left right and the new series will look like
this: CBOMB CROOF CSOUL CNICE CMANY

OLD PNA SHE TNA USE YSA As per the given new series there are no 5-letter
meaningful word which is framed
As per the new series word which is first from the right So, answer is NONE
end and that word is YSA and the third letter of this Answer: D (None)
word is A
So, answer is A 30) In this question, we have to arrange all the letters in
Answer: D (A) reverse alphabetical order within the word and the new
series will look like this:
28) In this question, we have to change each vowel is
changed to next letter and all the consonants is changed OMBB ROOF USOL NIEC YNMA
to previous letter as in alphabetical order and the new
series will look like this:

Page 177 of 728

Subscribe the Xpress Video Course & Mock Test Package for Bank & Insurance Exams
If there are any suggestions/ errors in our PDFs Feel Free to contact us via this email: admin@exampundit.in
IBPS RRB PO Prelims – Ultra Practice Bundle PDF
Now, we have to arrange the new series in the
alphabetical order and the new series will be like this: Solutions (33-35): The series given in this question is:

NIEC OMBB ROOF USOL YNMA ACT FED DIP HIM GEM
As per the new series, the word which is third from the
left end is ROOF 33) In this question we have to change each vowel to the
So, answer is ROOF previous letter and each consonant to the next letter and
Answer: C (ROOF) the new series will look like this:

31) In this question we have to arrange all the letters of ZDU GDE EHQ IHN HDN
the words in the alphabetical order and the new series
will look like this: As per the new series all the words are to be arranged in
alphabetical order and the new series will look like this:
ABBCEFILMMNNOOOORSUY
EHQ GDE HDN IHN ZDU
Now as per the series the letter which is sixth from the
left end is Fand the letter which is 5th from the right end As per the new series the word which is fourth from the
is Oand the number of letters between F and O is 8 left end is IHN and this word is framed after performing
So, answer is 8 operations on HIM.
Answer: E (8) So, answer is IHN
32) In this question, we have to change every consonant Answer: B (HIM)
to the next letter and each vowel to the previous letter
and after this new series will look like this: 34) In this question we have to arrange all the words in
alphabetical order and the new series will look like this:
CNNC SNNG TNTM OHDD NZOZ
As per the new series the words which has the same ACT DIP FED GEM HIM
letter areCNNC SNNG TNTM OHDD NZOZ
So, answer isCNNC SNNG TNTM OHDD NZOZ As per the new series third word is FED and the second
letter of this word is E and the word which is fourth from
Answer: E (all 5 words have 2 same letters) the left end is GEM and the second letter of GEM is E
and there are no letters between E and E
Answers (33-35): So, answer is NONE
Answer: A (None)
33) B
34) A
35) E

Page 178 of 728

Subscribe the Xpress Video Course & Mock Test Package for Bank & Insurance Exams
If there are any suggestions/ errors in our PDFs Feel Free to contact us via this email: admin@exampundit.in
IBPS RRB PO Prelims – Ultra Practice Bundle PDF
35) In this question we have to interchange the 2 nd the 3rd BEC VUU SRQ WXYJKL
letter of the words and first letter of the word is changed
to second next letter and they are as follows: As per the new series, there are three words which will
have only consonant and they are: SRQ, WXY and JKL
CTC HDE FPI JMI IME So, answer is Three
As per the new series there are no meaningful words Answer: C (Three)
framed 38) In this question we have to arrange all the words
So, answer is NONE from right to left in the alphabetical order and the new
Answer: E (None) series will look like this:

Answers (36-40): XYZ WVU TSR KLM CFD


36) A The word which second from the left end of the series as
37) C per the new series is WVU
38) B So, answer is WVU
39) E Answer: B (WVU)
40) C
39) In this question we have to take the first and the last
Solutions (36-40): The series given in this question is: letter of the words and they are as follows:

CFD WVU TSR XYZ KLM CD No Meaningful word


WU No Meaningful word
36) In this question we have to interchange the first and TR No Meaningful word
second letters within each word and the new series will XZ No Meaningful word
look like this after this operation: KM No Meaningful word

FCD VWU STR YXZ LKM As per the question there are no meaningful words can
be formed from first and the last letter of the words
As per the new series there are no meaningful words So, answer is None
which are framed Answer: E (None)
So, answer is None
Answer: A (None) 40) In this question we have to take that word which has
vowel and we have to put that vowel in the middle of the
37) In this question we have to change all the consonants word and it will look like this:
to the previous letter and the new series will look like
this: WUV

Page 179 of 728

Subscribe the Xpress Video Course & Mock Test Package for Bank & Insurance Exams
If there are any suggestions/ errors in our PDFs Feel Free to contact us via this email: admin@exampundit.in
IBPS RRB PO Prelims – Ultra Practice Bundle PDF
As per the series there are no meaningful word is framed Now, we have to arrange them in alphabetical order from
So, answer is None right to left and they will look like this:
Answer: C (None)
ISX GOT ENO AWX ABS
Answers (41-44):
As per the new series word which is in the middle is
41) B ENO and we get this word after performing operations
42) C on ONE
43) E So, answer is ENO
44) D Answer: C (One)

Solutions (41-44): The series given in this question is: 43) In this question we have to change all the vowels to
ABS GOT SIX ONE WAX the previous letter and all the consonants to the next
letter and the new series will look like this:
41) In this question we have to interchange the first and
the second letter of the word and the new series will look ZCT HNU THY NOD XZY
like this:
As per the new series there are no words which have
BAS OGT ISX NOE AWX repeated letters
So, answer is NONE
Now we have to arrange the new series in the Answer: E (NONE)
alphabetical order and the final series will look like this:
AWX BAS ISX NOE OGT 44) Word which is third word from the right end of the
word is GOT and the letter which is in the middle of the
As per the new series the word which is second from the word is O
left end is BAS and this word is framed after performing So, answer is O
operations on ABS Answer: D (O)
So, answer is BAS
Answer: B (ABS) Answers (44-48):

42) In this question we have to arrange all the letters 45) B


within the words as per the alphabetical order and they 46) A
will look like this: 47) C
48) D
ABS GOT ISX ENO AWX
Solutions (45-48): The series given in this question is:

Page 180 of 728

Subscribe the Xpress Video Course & Mock Test Package for Bank & Insurance Exams
If there are any suggestions/ errors in our PDFs Feel Free to contact us via this email: admin@exampundit.in
IBPS RRB PO Prelims – Ultra Practice Bundle PDF
Now, we have to arrange the above series in alphabetical
WEB NET YOU TWO RAT order and they will look like this:

45) In this question we have to change all the vowels to BEW OWT TAR TEN UOY
the second next letter and all the consonants are changed The word which second from the left end is OWT and
to second previous letter as in the English alphabet series the third letter of this word is T
and the new series will look like this: So, answer is T
Answer: C (T)
UGZ LGR WQW RUQ PCR
48) Word which is first from the left end is WEB and the
As per the new series there is only word (WQW) words first letter of this word is W and the word which is
in which a letter (W) appears twice second from the right end is TWO and the middle letter
So, answer is One of this word is W
Answer: B (One) So, there are no letters between W and W
So, answer is None
46) In this question we have to arrange all the letters Answer: D (None)
within the word as per the alphabetical order and the new
series will look like this: Solution 49) Word which is given in this question is:

WEB TNE YUO WTO TRA R E P R E S E N T A T I O N


Now, we have to arrange this series in alphabetical order
and they will look like this: In this question we have to find the number of pairs
TNE TRA WEB WTO YUO which has the same letter as per English alphabetical
order as same as in this spelling are: RP, EN, PS, RN
The word which is second from the right end is WTO and ON
and this word is framed after performing operations on So, answer is More Than Three
TWO Answer: E (More Than Three)
So, answer is TWO
Answer: A (TWO) Solution 50) Word which is given in this question is:

47) In this question firstly, we have to interchange first A U T O M A T I O N


and last letter of the word and they are as follows:
In this question we have to find the number of pairs
BEW TEN UOY OWT TAR which has the same letter as per English alphabetical
order as same as in this spelling are: UT and NO
So, answer is TWO

Page 181 of 728

Subscribe the Xpress Video Course & Mock Test Package for Bank & Insurance Exams
If there are any suggestions/ errors in our PDFs Feel Free to contact us via this email: admin@exampundit.in
IBPS RRB PO Prelims – Ultra Practice Bundle PDF
Answer: B (Two)

10). Coding Decoding Questions

(Direction 1 – 5): Study the following information D) Cannot Be Determined


carefully and answer the given questions. E) None of These
In a certain code language, 3. What could be the code of "Mangesh" in the given
language?
"Gagan Mangesh Suresh Trilok" is coded as "KZ LX A) AB
AB CZ" B) KZ
"Trilok Mangesh Akash Sanchit" is coded as "CZ LX C) LX
XC PC" D) CZ
"Sanchit Suresh Babita Candy" is coded as "PC AB IS E) Either C or D
VU"
"Gagan Candy Danny John" is coded as "KZ VU NS 4. 'AB LX VU' can be a code of which of the
LK" following?
1. What could be the code for ‘PC’? A) Suresh Magesh Candy
A) Gagan B) Suresh Trilok Candy
B) Mangesh C) Suresh Gagan Candy
C) Sanchit D) Either A or B
D) Suresh E) None of These
E) Danny
5. What can be the code of "Gagan" in the given
2. 'PC NA VU' can be a code of which of the language?
following? A) LX
A) Suresh Sanchit Candy B) KZ
B) Sanchit Sheri Candy C) PC
C) Sanchit Danny Candy D) NS

Page 182 of 728

Subscribe the Xpress Video Course & Mock Test Package for Bank & Insurance Exams
If there are any suggestions/ errors in our PDFs Feel Free to contact us via this email: admin@exampundit.in
IBPS RRB PO Prelims – Ultra Practice Bundle PDF
E) LK E) None of These

(Direction 6 – 10): Read the following information 8. What does "ZK" stands for?
carefully and answer the questions that follow: A) Rajasthan
In a certain code language, B) Andhra-Pradesh
“Kerala Karnataka Tamil-Nadu Andhra-Pradesh” is C) Tamil-Nadu
coded as “ZO ZK RA ZG” D) Either A or C
“Rajasthan Karnataka Tamil-Nadu Kerala” is coded as E) None of These
“ZK RA ZO ZH”
“Haryana Tamil-Nadu Punjab Gujarat” is coded as 9. Which of the following will be coded "ZM RS ZO"
“ZM RS ZT ZK” in the given language?
“Kerala Bihar Jharkhand Haryana” is coded as “RA A) Punjab Gujarat Karnataka
RN LQ ZT” B) Jharkhand Bihar Punjab
C) Gujarat Punjab Bihar
6. How will "Kerala Rajasthan Haryana" be coded in D) Tamil-Nadu Gujarat Rajasthan
the given language? E) Cannot Be Determined
A) RA ZH ZT
B) RA ZO ZH 10. Which of the following will be coded as "ZO" in
C) ZH RN ZK the given language?
D) LQ ZT ZM A) Karnataka
E) None of These B) Bihar
C) Jharkhand
7. How will "Haryana Tamil-Nadu Gujarat" will be D) Punjab
coded in the given language? E) Tamil-Nadu
A) Either B or C
B) ZH RN LR (Direction 11 – 15): Read the following information
C) RA ZH ZT carefully and answer the questions that follow:
D) RH ZO ZT In a certain code language,
Page 183 of 728

Subscribe the Xpress Video Course & Mock Test Package for Bank & Insurance Exams
If there are any suggestions/ errors in our PDFs Feel Free to contact us via this email: admin@exampundit.in
IBPS RRB PO Prelims – Ultra Practice Bundle PDF
"Indian Air Force Edition" is coded as "LA PA CA C) RO RE PA
KA" D) RO RE AR
"Indian Chief Marshal Air" is coded as "LA PA SH E) None of These
AR"
"Navy Air Current Edition" is coded as "PA RO CA 14. which of the following will be the code of
RE" "Force"?
"Navy Marshal Current Force" is coded as "RO SH A) KA
KA RE" B) PA
C) LA
11. Which of the following is the code of "Indian"? D) CA
A) LA E) None of These
B) PA 15. The code "AR" belongs to which of the following
C) KA words?
D) CA A) Indian
E) None of These B) Chief
C) Navy
12. Which of the following will be the code of D) Current
"Navy"? E) None of These
A) RO
B) RE Directions (16-20) Study the information below and
C) Either A or B answer the following question: –
D) SH In a certain code language
E) AR "International National Australia Canada" is coded as
"AC BC CC DC"
13. Which of the following will be the code of "Navy "Australia Romania Austria USA" is coded as "BC FC
Current Chief"? EC QC"
A) RO RE KA "International Romania Lenovo Dell" is coded as "CC
B) RO RE CA EC LO PS"
Page 184 of 728

Subscribe the Xpress Video Course & Mock Test Package for Bank & Insurance Exams
If there are any suggestions/ errors in our PDFs Feel Free to contact us via this email: admin@exampundit.in
IBPS RRB PO Prelims – Ultra Practice Bundle PDF
"Canada Lenovo Sudan Belgium" is coded as "DC PS A) PS BC DC
XY VU" B) PS BC XY
C) PS BC QX
16. What does the code "FC" stand for in the given D) Cannot Be Determined
coded language? E) None of These
A) Australia
B) Austria 20. Which of the following is the code of "Dell" in the
C) USA given coded language?
D) Either B or C A) CC
E) Cannot Be Determined B) EC
17. Which of the following may represent "Sudan C) PS
Lenskart" in the given code? D) XY
A) XY EC E) None of These
B) VU BX
C) VU CX Directions (21-25): Study the information below and
D) Cannot Be Determined answer the following question:
E) Either B or C In a certain code language
"Celebrate Million Perform Security" is coded as "CE
18. What is the code for "International" in the given MN PM SY"
coded language? "Perform Reserve Police Force" is coded as "PM RE
A) AC PE FE"
B) CC "Reserve Celebrate Category Quality" is coded as "RE
C) DC CE CY QY"
D) EC "Category Security Appreciate Create" is coded as "CY
E) None of These SY AE RT"

19. Which of the following may represent "Lenovo


Education Australia" in the given coded language?
Page 185 of 728

Subscribe the Xpress Video Course & Mock Test Package for Bank & Insurance Exams
If there are any suggestions/ errors in our PDFs Feel Free to contact us via this email: admin@exampundit.in
IBPS RRB PO Prelims – Ultra Practice Bundle PDF
21. If "Wing Police External" is coded as "WG PE A) Appreciate
EL" in the given coded language, then how will B) Security
"Celebrate Wing Guard" be coded as? C) Create
A) PM CE CY D) Either A or C
B) MN CE SY E) Cannot Be Determined
C) PM SY RE 25. What is the code of "Million" in the given coded
D) PM SY CE language?
E) CE WG GD A) CE
B) MN
22. Which of the following may represent the code of C) PM
"Police Reserve Category" in the given coded D) SY
language? E) None of These
A) PE RE CY
B) FE RE CY Directions (26-30) Study the information below and
C) Either A or B answer the following question:
D) PE RE SY In a certain code language,
E) None of These "Commonwealth Parliamentary Conference Author" is
coded as "CH PY CE AE"
23. Which of the following may represent the code of "Conference Aspirants Online Offline" is coded as "CE
"Perform Improve" in the given coded language? AS OE FL"
A) PM IE "Aspirants Parliamentary Personal Guidance" is coded
B) PM CE as "AS PY PL GE"
C) PM PE "Personal Validity Parliamentary Online" is coded as
D) PM AE "PL VY PY OE"
E) None of These
26. What is the code for "Offline" in the given coded
24. In the given coded language, what does the code language?
"AE" stands for? A) CE
Page 186 of 728

Subscribe the Xpress Video Course & Mock Test Package for Bank & Insurance Exams
If there are any suggestions/ errors in our PDFs Feel Free to contact us via this email: admin@exampundit.in
IBPS RRB PO Prelims – Ultra Practice Bundle PDF
B) AS
C) OE 30. What is the code for "Personal" in the given
D) FL coded language?
E) None of These A) CE
B) CH
27. In the given coded language, what does the code C) PL
"VY" stands for? D) PY
A) Personal E) None of These
B) Validity
C) Parliamentary Directions (31-35) Study the information below and
D) Guidance answer the following question:
E) Cannot Be Determined In a certain code language,
"Technology Summit Challenge Program" is coded as
28. What may be the code for "Supreme "TY ST CE PE"
Commonwealth" in the given coded language? "Technology Summit Japanese Prix" is coded as "TY
A) SE CH ST JE PX"
B) CH PY "Technology Summit Program Recently" is coded as
C) CH OE "TY ST PE RY"
D) CH GE "Technology Japanese Education Science" is coded as
E) CH VY "TY EN JE SE"
"Education Science Mayor World" is coded as "EN SE
29. What is the code for "Guidance" in the given MR WD"
coded language? 31. What may be the possible code for "Mayor" in
A) CH the given coded language?
B) PY A) MR
C) CE B) WD
D) AE C) Either A or B
E) None of These D) TY
Page 187 of 728

Subscribe the Xpress Video Course & Mock Test Package for Bank & Insurance Exams
If there are any suggestions/ errors in our PDFs Feel Free to contact us via this email: admin@exampundit.in
IBPS RRB PO Prelims – Ultra Practice Bundle PDF
E) ST A) Challenge
B) Program
32. What can be the possible code of "Education C) Education
Edition" in the given coded language? D) Science
A) EN SE E) None of These
B) EN MR Direction (36-40): Study the information below and
C) EN ST answer the following question:
D) EN EA In a certain code language,
E) EN TY "Australia USA Germany Spain" is coded as "ZOQ
ZGQ ZQQ ZHQ"
33. What will be the code of "Japanese" in the given "China Nepal Maldives Australia" is coded as "ZQQ
coded language? ZKQ ZIQ ZWQ"
A) EN "Myanmar USA Germany China" is coded as "ZWQ
B) SE ZGQ ZUQ ZOQ"
C) WD
D) MR 36. What is the code of "Spain"?
E) JE A) ZWQ
B) ZOJ
34. What will be the code of "Recently" in the given C) ZGQ
coded language? D) ZHQ
A) TY E) None of These
B) RY 37. "Australia China Myanmar" can be coded as
C) ST which of the following?
D) JE A) ZUQ ZQQ ZWQ
E) MR B) ZHQ ZQQ ZGQ
C) ZGQ ZWQ ZIQ
35. What is the code for ‘CE’ in the given code D) Cannot Be Determined
language? E) None of These
Page 188 of 728

Subscribe the Xpress Video Course & Mock Test Package for Bank & Insurance Exams
If there are any suggestions/ errors in our PDFs Feel Free to contact us via this email: admin@exampundit.in
IBPS RRB PO Prelims – Ultra Practice Bundle PDF
"B.Arch B.Pharm LLB LLM BBA" is coded as "CN
38. What is the code for "Nepal"? SN ZO ZH CL"
A) ZIQ "B.Sc B.Tech B.Arch" is coded as "CN VG OP"
B) ZKQ “BA B.Pharm BBA M.Tech B.Sc” is coded as “OP
C) Either A or B SN LX VA CL”
D) ZWQ
E) None of These 41. What does the code "ZO" stand for in the given
coded language?
39. Which of the following is the code for A) B.Arch
"Myanmar"? B) B.Pharm
A) ZUQ C) LLB
B) ZWQ D) LLM
C) ZQQ E) Either C or D
D) ZGQ
E) None of These 42. What is the code for "BBA"?
A) VA
40. What does "ZGQ" stand for? B) SN
A) Australia C) CL
B) Spain D) OP
C) Myanmar E) Either B or C
D) China 43. In the given coded language, which of the
E) None of These following means "LLB LLM BBA"?
Directions (41-45) Study the information below and A) ZO ZH ZN
answer the following question: B) CL VA OP
In a certain code language, C) CL ZH SN
"B.Com BA B.Tech B.Arch" is coded as "ZN LX VG D) Either A or B
CN" E) Cannot Be Determined

Page 189 of 728

Subscribe the Xpress Video Course & Mock Test Package for Bank & Insurance Exams
If there are any suggestions/ errors in our PDFs Feel Free to contact us via this email: admin@exampundit.in
IBPS RRB PO Prelims – Ultra Practice Bundle PDF
44. What does the code "LX" stand for? 46. What is the code of "Mahant"?
A) B.Pharm A) VB
B) LLB B) ZM
C) BA C) LA
D) LLM D) Either ZM or LA
E) None of These E) Either VB or ZM

45. What is the code "SN" in the given coded 47. What does "FH" stands for?
language? A) Mudit. B) Maan. C) Mahin. D) Makur
A) B.Pharm E) None of These
B) B.Com
C) BBA 48. What is the code for "Manas Maan Mirdul"?
D) Either A or C A) LNT ZI XRM
E) None of These B) XRM VB ORG
C) ZW ZI XRM
Directions (46-50) Study the information below and D) Cannot Be Determined
answer the following question: E) None of These

In a certain code language, 49. Which of the following "ORP ORG ZM" stand
"Maan Mahesh Mudit Mani" is coded as "ZT LNT for?
ORG FH" A) Manas Mukul Mansukh
"Mahin Makur Maan Magan Mahant" is coded as B) Mahin Maan Mahant
"ORP LA LNT VB ZM" C) Mani Magan Mahant
"Mani Makur Mansukh Manas" is coded as "LA ZI D) Mani Magan Mahin
ORG ZW" E) Either C or D
“Mansukh Mahesh Mridul Magan” is coded as “ZW
ZT XRM ORP” 50. Which of the following may represent "Modak
Mehul Mahesh Maan"?
Page 190 of 728

Subscribe the Xpress Video Course & Mock Test Package for Bank & Insurance Exams
If there are any suggestions/ errors in our PDFs Feel Free to contact us via this email: admin@exampundit.in
IBPS RRB PO Prelims – Ultra Practice Bundle PDF
A) ZT LA ORG ZW D) Cannot Be Determined
B) ZT ZY LNT VVM E) None of These
C) LNT ZY ORG ZI

10). Coding Decoding Questions - Detailed Explanation with Answers

Answers (1-5): Candy VU


1) C Danny/John LK/NS
2) B Babita Is
3) E Akash XC
4) D Trilok/Mangesh CZ/LX
5) B
Answers (6-10):
Solutions (1-5): In this type of question, we will look for 6) A
the common words between all the three sentences 7) E
Let’s just take an example: 8) C
First word is Gagan 9) A
We can see that Gagan word is in first and the last 10) A
sentences and now we will look for the common words
between the two sentences and the common word is Solutions (6-10): In this type of question, we will look
“KZ” for the common words between all the four sentences
Now we will decode all the words with the codes: Let’s just take an example:
Words Codes First word is Kerala
Gagan KZ We can see that Kerala word is in first, second and
Sanchit PC fourth sentences and now we will look for the common
Suresh AB words between the two sentences and the common word
is “RA”
Page 191 of 728

Subscribe the Xpress Video Course & Mock Test Package for Bank & Insurance Exams
If there are any suggestions/ errors in our PDFs Feel Free to contact us via this email: admin@exampundit.in
IBPS RRB PO Prelims – Ultra Practice Bundle PDF
Words Codes
Now we will decode all the words with the codes: Indian LA
Words Codes Air PA
Kerala RA Navy/Current RO/RE
Tamil-Nadu ZK Marshal SH
Karnataka ZO Force KA
Andhra-Pradesh ZG Chief AR
Rajasthan ZH Edition CA
Haryana ZT
Punjab/Gujarat ZM/RS Answers (16-20):
Bihar/Jharkhand RN/LQ 16) D
17) D
Answers (11-15): 18) B
11) A 19) C
12) C 20) E
13) D
14) A Solutions (16-20): In this type of question, we will look
15) B for the common words between all the three sentences
Let’s just take an example:
Solutions (11-15): In this type of question, we will look First word is Australia
for the common words between all the three sentences We can see that Australia word is in first two sentences
Let’s just take an example: and now we will look for the common words between
First word is Indian the two sentences and the common word is “BC”
We can see that Indian word is in first two sentences and Now we will decode all the words with the codes:
now we will look for the common words between the
two sentences and the common word is “LA” Words Codes
International CC
Now we will decode all the words with the codes:
Page 192 of 728

Subscribe the Xpress Video Course & Mock Test Package for Bank & Insurance Exams
If there are any suggestions/ errors in our PDFs Feel Free to contact us via this email: admin@exampundit.in
IBPS RRB PO Prelims – Ultra Practice Bundle PDF
Australia BC Celebrate CE
Romania EC Perform PM
Canada DC Security SY
Lenovo PS Million MN
National AC Reserve RE
Dell LO Category CY
Austria/USA FC/QC Quality QY
Sudan/Belgium XY/VU Police/ Force PE/FE
Appreciate/Create AE/RT
Answers (21-25): Answers (26-30):
21) E 26) D
22) C 27) B
23) A 28) A
24) D 29) E
25) B 30) C

Solutions (21-25): In this type of question, we will look Solutions (26-30):In this type of question, we will look
for the common words between all the three sentences for the common words between all the three sentences
Let’s just take an example: Let’s just take an example:
First word is Perform First word is Conference
We can see that Perform word is in first two sentences We can see that Conference word is in first two
and now we will look for the common words between sentences and now we will look for the common words
the two sentences and the common word is “PM” between the two sentences and the common word is
“CE”
Now we will decode all the words with the codes:
Now we will decode all the words with the codes:
Words Codes Words Codes

Page 193 of 728

Subscribe the Xpress Video Course & Mock Test Package for Bank & Insurance Exams
If there are any suggestions/ errors in our PDFs Feel Free to contact us via this email: admin@exampundit.in
IBPS RRB PO Prelims – Ultra Practice Bundle PDF
Conference CE Words Codes
Parliamentary PY Technology TY
Aspirants AS Summit ST
Personal PL Program PE
Commonwealth/Author CH/AE Challenge CE
Online OE Japanese JE
Offline FL Prix PX
Guidance GE Recently RY
Validity VY Education/Science EN/SE
Mayor/World MR/WD
Answers (31-35): Answers (36-40):
31) C 36) D
32) D 37) A
33) E 38) C
34) B 39) A
35) A 40) E

Solutions (31-35):In this type of question, we will look Solutions (36-40):In this type of question, we will look
for the common words between all the three sentences for the common words between all the three sentences
Let’s just take an example: Let’s just take an example:
First word is Technology First word is Australia
We can see that Technology word is in first four We can see that Australia word is in first two sentences
sentences and now we will look for the common words and now we will look for the common words between
between the two sentences and the common word is the two sentences and the common word is “ZQQ”
“TY”
Now we will decode all the words with the codes:
Now we will decode all the words with the codes: Words Codes

Page 194 of 728

Subscribe the Xpress Video Course & Mock Test Package for Bank & Insurance Exams
If there are any suggestions/ errors in our PDFs Feel Free to contact us via this email: admin@exampundit.in
IBPS RRB PO Prelims – Ultra Practice Bundle PDF
Australia ZQQ B.Sc OP
China ZWQ B.Com ZN
Nepal/Maldives ZKQ/ZIQ B.Pharm/BBA SN/CL
USA/Germany ZOQ/ZGQ LLB/LLM ZO/ZH
Spain ZHQ M.Tech/B.Sc OP/VA
Myanmar ZUQ
Answers (46-50):
Answers (41-45): 46) E
41) E 47) A
42) E 48) A
43) E 49) E
44) C 50) B
45) D
Solutions (46-50):In this type of question, we will look
Solutions (41-45):In this type of question, we will look for the common words between all the four sentences
for the common words between all the four sentences Let’s just take an example:
Let’s just take an example: First word is Maan
First word is B.Arch We can see that Maan word is in first two sentences and
We can see that B.Arch word is in first two sentences now we will look for the common words between the
and now we will look for the common words between two sentences and the common word is “LNT”
the two sentences and the common word is “CN”
Now we will decode all the words with the codes:
Now we will decode all the words with the codes: Words Codes
Words Codes Maan LNT
B.Arch CN Mani ORG
BA LX Makur LA
B.Tech VG Magan ORP

Page 195 of 728

Subscribe the Xpress Video Course & Mock Test Package for Bank & Insurance Exams
If there are any suggestions/ errors in our PDFs Feel Free to contact us via this email: admin@exampundit.in
IBPS RRB PO Prelims – Ultra Practice Bundle PDF
Mansukh ZW Manas ZI
Mahesh ZT Mudit FH
Mridul XRM Mahin/Mahant VB/ZM

11). Inequality Questions


Direction (1-5): In the given question, assuming the b. Only II is true
given statements to be true, find which of the
c. Either I or II is true
conclusions I and II given below the statements is/are
definitely true. d. Both I and II are true
Statements: e. Neither I nor II is true
D < J, D = B, J ≤ K, A ≤ B 3) Statements:
Conclusions: A = B < C; B > D; D > E
I. J = B Conclusions:
II. K > A I. D < C
a. If only conclusion I follows II. A > E
b. If only conclusion II follows a. Only conclusions I is true.
c. If either conclusion I or conclusion II follows b. Only conclusions II is true.
d. If neither conclusion I nor conclusion II follows c. Either conclusions I or II is true.
e. If both conclusions follow d. Neither conclusions I nor II is true.
2) Statements: P≤ Q > M; N ≥ Q > T; M < T e. Both conclusions I and II are true.
Conclusions: 4) Statements:
I. T > Q Q < W > R, W = T, T < S
II. N ≥ P Conclusions:
a. Only I is true I. R = T

Page 196 of 728

Subscribe the Xpress Video Course & Mock Test Package for Bank & Insurance Exams
If there are any suggestions/ errors in our PDFs Feel Free to contact us via this email: admin@exampundit.in
IBPS RRB PO Prelims – Ultra Practice Bundle PDF
II. S > Q a. Only II is true

a. Only conclusions I is true. b. Only I is true

b. Only conclusions II is true. c. Both I and II are true

c. Either conclusions I or II is true. d. None is true

d. Neither conclusions I nor II is true. e. Either I or II is true

e. Both conclusions I and II are true. 7) Statements: 1 < 2 < 3; 3 ≤ 4 > 5; 6 > 7 ≥ 1

5) Statements: A > B ≤ C; B > D > E; A < F ≤ G Conclusions:

Conclusions: I. 7 ≥ 3

I. A > E II. 7 < 3

II. F > D a. Only II is true

a. Only II is true b. Only I is true

b. Only I is true c. Both I and II are true

c. Both I and II are true d. None is true

d. None is true e. Either I or II is true

e. Either I or II is true 8) Statements: 10 > 9 = 8; 10 < 11 ≤ 12; 11 > 13 = 14

Direction (6-10): In each of the following question Conclusions:


assuming the given statements to be true, find which
I. 12 ≥ 14
of the conclusion among given conclusions is/ are
definitely true and then give your answers II. 13 > 9
accordingly.
a. Only II is true
6) Statements: P < Q ≥ R; R < S ≤ T; Q > U ≤ T
b. Only I is true
Conclusions:
c. Both I and II are true
I. P > T
d. None is true
II. Q ≥ S

Page 197 of 728

Subscribe the Xpress Video Course & Mock Test Package for Bank & Insurance Exams
If there are any suggestions/ errors in our PDFs Feel Free to contact us via this email: admin@exampundit.in
IBPS RRB PO Prelims – Ultra Practice Bundle PDF
e. Either I or II is true 11) Statements: A > E > C ; C = D ; E < F

9) Statements: P ≤ Q = R; R ≤ S > T; T = U > V Conclusions:

Conclusions: I. D < E

I. S > P II. C < A

II. S = P III. A > F

a. Only II is true a. Either II or III is correct.

b. Only I is true b. Only conclusion III is incorrect.

c. Both I and II are true c. Neither II nor III are correct.

d. None is true d. All conclusions are correct.

e. Either I or II is true e. Only conclusion II is correct.

10) Statements: 12) Statements: A > B = C; D < E = B; F > E

P ≤ Q > R ≥ S; T > R < X Conclusions:

Conclusions: I. F > C

I. X > S II. A < F

II. Q > S III. D > A

a. Only conclusion I follows. a. No conclusion is correct.

b. Both conclusions I & II follow. b. Only conclusion II is incorrect.

c. Only conclusion II follow. c. Neither I nor III are correct.

d. Either I or II follows. d. All conclusions are correct.

e. Neither conclusion I nor conclusion II follows. e. Only conclusion I is correct.

Direction (11-15): In the following question assuming 13) Statements: C > D > E; A > D; F < E < B
the given statements to be true, find which of the
option among the given options is/are definitely true Conclusions:
and then give your answers accordingly.
Page 198 of 728

Subscribe the Xpress Video Course & Mock Test Package for Bank & Insurance Exams
If there are any suggestions/ errors in our PDFs Feel Free to contact us via this email: admin@exampundit.in
IBPS RRB PO Prelims – Ultra Practice Bundle PDF
I. B > F I. I ≥ M

II. A > C II. K <O

III. D < B III. G>K

a. Conclusions I and III are correct. a. Only I follows

b. Only conclusion II is incorrect. b. Both II and III follows

c. Conclusion II is correct. c. Both I and II follow

d. All conclusions are correct. d. All follow

e. Only conclusion I is correct. e. Both I and III follow

14) Statements: Direction (16-20): In each of the following question


assuming the given statements to be true, find which
C ≥ D, D ≤ F, F > G, G < H
of the conclusion among given conclusions is/ are
Conclusions: definitely true and then give your answers
accordingly.
I. C > F
16) Statements: 50 < 60 ≤ 70; 60 > 40 > 30; 70 = 80 ≤
II. H > F 20

III. G < D Conclusions:


a. Only I follows I. 70 > 30

b. Only II follows II. 20 ≥ 60

c. Only III follows a. Only II is true

d. Both I and II follow b. Only I is true

e. None follows c. Both I and II are true

15) Statements: d. None is true

G < I, I ≥ K, K = M, M < O e. Either I or II is true

Conclusions: 17) Statements: 21 > 22 > 23; 22 < 24 ≤ 25; 23 ≥ 26 >


27
Page 199 of 728

Subscribe the Xpress Video Course & Mock Test Package for Bank & Insurance Exams
If there are any suggestions/ errors in our PDFs Feel Free to contact us via this email: admin@exampundit.in
IBPS RRB PO Prelims – Ultra Practice Bundle PDF
Conclusions: II. S > T

I. 23 ≥ 27 a. Only I is True

II. 21 > 26 b. Only II is True

a. Only II is true c. Either I or II is true

b. Only I is true d. Both conclusions I and II are True

c. Both I and II are true e. None is True

d. None is true 20) Statements: B ≤ L ≤ A > N = C > E; C ≥ V > R = S

e. Either I or II is true Conclusions:

18) Statement: I. N ≥ V

A ≤ B ≤ C = D > E > F; F > G II. E < V

Conclusions: a. Both conclusions I and II are true

I. D > G b. Either conclusion I or II is true

II. E = G c. Neither conclusion I nor II is true

a. Only I is True d. Only conclusion I is true

b. Only II is True e. Only conclusion II is true

c. Either I or II is true Directions (21-25): In the following question


assuming the given statements to be True, find which
d. Both conclusions I and II are True of the conclusion among given conclusions is/are
e. None is True definitely true and then give your answers
accordingly.
19) Statement:
21) Statements:
P = Q ≥ R = S; T > U ≥ V = S
D≥E>F=A≤U<L≥T=R
Conclusions:
Conclusions:
I. Q ≥ V
I. F < L

Page 200 of 728

Subscribe the Xpress Video Course & Mock Test Package for Bank & Insurance Exams
If there are any suggestions/ errors in our PDFs Feel Free to contact us via this email: admin@exampundit.in
IBPS RRB PO Prelims – Ultra Practice Bundle PDF
II. D > A b. Only II is true

a. None is True c. Either I or II true

b. Both I and II are True d. Neither I nor II is true

c. Only II is True e. Both I and II is true

d. Only I is True 24) Statement: X < C, W > D, G ≥ W, C = D

e. Either I and II is True Conclusion:

22) Statements: I. C ≤ G

J ≥ N ≤ T; T = S > R; K > V ≥ J II. W > X

Conclusions: a. None is true

I. V ≥ N b. Only II is true

II. K > J c. Both I and II are true

a. Only II is true d. Only I is true

b. Only I is true e. Either I or II is true

c. Either I or II is true 25) Statements:

d. Both I and II are true D = F; B > C; D < C; A > B

e. None of these Conclusions:

23) Statements: I. C ≥ F

Q ≤ R; S < T; P > Q; R > S II. B > F

Conclusions: a. Only I is true

I. S = Q b. Only II is true

II. T ≥ P c. Either I or II is true

a. Only I is true d. Neither I nor II is true

Page 201 of 728

Subscribe the Xpress Video Course & Mock Test Package for Bank & Insurance Exams
If there are any suggestions/ errors in our PDFs Feel Free to contact us via this email: admin@exampundit.in
IBPS RRB PO Prelims – Ultra Practice Bundle PDF
e. Both I and II is true e. Both conclusions I and II follow.

Directions (26-30): In the following question 28) Statements:


assuming the given statements to be True, find which
U ≥ X ≤ H; K ≤ X; V > Q = X
of the conclusion among given conclusions is/are
definitely true and then give your answers Conclusions:
accordingly.
I. V > U
26) Statements:
II. K ≤ H
O < P > R > N, N ≥ M, P < Q
a. Only conclusion I is true.
Conclusions:
b. Only conclusion II is true.
I. Q > R
c. Either conclusion I or II is true.
II. O < Q
d. Neither conclusion I nor II is true.
a. None is True
e. Both conclusion I and II are true.
b. Both I and II are True
29) Statements:
c. Only II is True
W < Q < R; R = T; T < S
d. Only I is True
Conclusions:
e. Either I and II is True
I. Q < T
27) Statement: M ≥ O ≥ G = R ≥ N = T
II. S > W
Conclusion
a. Only conclusions I is true.
I: M = T
b. Only conclusions II is true.
II: T ˂ M
c. Either conclusions I or II is true.
a. Only conclusion I follows.
d. Neither conclusions I nor II is true.
b. Only conclusion II follows.
e. Both conclusions I and II are true.
c. Either conclusion I or II follows.
30) Statement: T < Q ≤ W; M > P < Z; Z = R; M > Y
d. Neither conclusion I nor II follows.

Page 202 of 728

Subscribe the Xpress Video Course & Mock Test Package for Bank & Insurance Exams
If there are any suggestions/ errors in our PDFs Feel Free to contact us via this email: admin@exampundit.in
IBPS RRB PO Prelims – Ultra Practice Bundle PDF
Conclusion: Conclusions:

I. P < Q I. H < A

II. M < Q II. A < F

a. Only I is true a. Only conclusion I is true.

b. Only II is True b. Only conclusion II is true.

c. Either I and II is true c. Either conclusion I or II is true.

d. Neither I and II is true d. Neither conclusion I nor II is true.

e. Both I and II is true e. Both the conclusion I and II are true.

Direction (31-35): In these questions, a relationship 33) Statements: P ≤ Y; X ≥ Z ≤ Y; Z = P ≤ X


between different elements is shown in the
Conclusions:
statements. The statements are followed by two
conclusions. Give answer: I. Y ≥ Z
31. Statements: II. X = Z
S > A = N > D; A > L > E; M < L < O a. Only I is True
Conclusions: b. Only II is True
I) S > E c. Both I and II are True
II) A < O d. Either I and II is True
a. Both conclusions I and II are true e. None of these
b. Only conclusion II is true. 34) Statements:
c. Only conclusion I is true. L = O ≥ R, P > T, L > U
d. Either conclusion I or II is true. Conclusion:
e. Neither conclusion I nor II is true. I. U = R
32) Statements: II. L > R
B ≤ A; C < D > E; A = C; F ≥ H > B
Page 203 of 728

Subscribe the Xpress Video Course & Mock Test Package for Bank & Insurance Exams
If there are any suggestions/ errors in our PDFs Feel Free to contact us via this email: admin@exampundit.in
IBPS RRB PO Prelims – Ultra Practice Bundle PDF
a. None is True a. Only conclusion I follows.

b. Both I and II are True b. Only conclusion II follows.

c. Only II is True c. Either conclusion I or II follows.

d. Only I is True d. Neither conclusion I nor II follows.

e. Either I and II is True e. Both conclusions I and II follow.

35) Statements: X > T = S, R > S, T ≤ U 37) Statements:

Conclusions: A = C ≤ R; Q ≥ O ≥ J; J ≤ R

I. S < U Conclusions:

II. S > U I. R ≥ A

a. None is True II. J ≤ Q

b. Both I and II are True a. Only conclusion I follows.

c. Only II is True b. Only conclusion II follows.

d. Only I is True c. Either conclusion I or II follows.

e. Either I and II is True d. Neither conclusion I nor II follows.

Directions (36-40): In the following question e. Both conclusions I and II follow.


assuming the given statements to be true, find which
38) Statements:
of the conclusion among given conclusions is/are
definitely true/false and then give your answers A < B; B ≤ C; A > D; D = E
accordingly.
Conclusions:
36) Statements:
I. B < A
D > M > U ≥ E; N < U ≥ J
II. E < A
Conclusions:
III. C < B
I. D > N
a. None is true
II. E ≤ J

Page 204 of 728

Subscribe the Xpress Video Course & Mock Test Package for Bank & Insurance Exams
If there are any suggestions/ errors in our PDFs Feel Free to contact us via this email: admin@exampundit.in
IBPS RRB PO Prelims – Ultra Practice Bundle PDF
b. Only II is true d. Only conclusion II follows

c.. Only I and II is true e. Neither I nor II follows

d. Only II and III is true Direction (41-45): In the following question assuming
the given statements to be true, find which of the
e. Only III is true conclusion among given conclusions is / are definitely
39) Statements: true and then give your answers accordingly.

L > I = N > P; I > R > K; N < E < Z 41) Statements: T < M = J ≤ B; J < L; N ≥ M > C

Conclusions: Conclusions:

I) E > P I. T > B

II) R < L II. J < N

a. Conclusion I and II are true. III. J = N

b. Only conclusion II is true. a. Only conclusion I follows

c. Only conclusion I is true. b. Only conclusion II follows

d. Either conclusion I or II is true. c. Only conclusion III follows

e. Neither conclusion I nor II is true. d. Either conclusion II or conclusion III follows

40) Statement: e. None follows

X > Y ≤ Z = K ≠ L ≠ M = N; Z ≠ N 42) Statements:

Conclusion: A > T ≥ R; L ≥ M > N; T = N < S

I. N > Z Conclusions

II. Z > M I. A > R

a. Only conclusion I follows II. L = R

b. Both conclusions I and II follow III. S < M

c. Either I or II follows a. Only III is true

b. Only I is true
Page 205 of 728

Subscribe the Xpress Video Course & Mock Test Package for Bank & Insurance Exams
If there are any suggestions/ errors in our PDFs Feel Free to contact us via this email: admin@exampundit.in
IBPS RRB PO Prelims – Ultra Practice Bundle PDF
c. Both II and III are true d. Only II and III are true.

d. Both I and III are true e. Only III is true.

e. None is true 45) Statements: W ≥ N ≤ M; M ≤ H = I; W > X < Z

43) Statements: Conclusions:

S ≤ M < F ≥ G; M = T; L = G < M; W < Q I. Z < X

Conclusions: II. X > W

I. F > S III. N ≥ M

II. M = Q a. Only I is true.

III. M > L b. Only II is true.

a. Only III is true. c. Only III is true.

b. Only I and II are true. d. None is true.

c. Only I is true. e. All are true.

d. Only II is true. Direction (46-50): In the following question assuming


the given statements to be True, find which of the
e. Only I and III are true. conclusion among given conclusions is/are definitely
44) Statements: A > D < K; E > M > R; D > R true and then give your answers accordingly.

Conclusions: 46) Statements:

I. R < K X > H ≥ M = N, P > T < J, N > J

II. E > A Conclusions:

III. D > E I. T > H

a. Only I is true. II. P > M

b. Only II is true. III. T = H

c. Only I and II is true. a. Only II is True

b. Only I is True
Page 206 of 728

Subscribe the Xpress Video Course & Mock Test Package for Bank & Insurance Exams
If there are any suggestions/ errors in our PDFs Feel Free to contact us via this email: admin@exampundit.in
IBPS RRB PO Prelims – Ultra Practice Bundle PDF
c. Only I and III are True c. Only conclusion III and either conclusion I or II is
True
d. Only III is true
d. All conclusion is True
e. None is True
e. Either conclusion I or II is True
47) Statements:
49) Statements: D = E < F ≤ H = I; J > K = F ≥ L ≤ M; I
C > D; F ≥ D; F < G; G = M >P≥Q<S
Conclusions: Conclusions:
I. D < G I. I > M
II. M > D II. Q < L
III. M > F a. Only II is True.
a. None is true b. Only I is True.
b. Only I is true c. Both I and II are True.
c. Only II is true d. Neither I nor II is True.
d. Only III is true e. Either I or II is True.
e. All are true 50) Statements: Y < Z > X; W > D < R; Y > T = R; X >
48) Statement: W

A ≤ P > K ≤ D; M ≤ O = K > A; J > D Conclusions:

Conclusion: I. R < Z

I. M < D II. X > D

II. D = M III. T < W

III. A > D a. Only I is True

a. Both conclusions I and II are True b. Only II is True

b. Only conclusion III is True c. Only I and II are True

d. Only II and III are True


Page 207 of 728

Subscribe the Xpress Video Course & Mock Test Package for Bank & Insurance Exams
If there are any suggestions/ errors in our PDFs Feel Free to contact us via this email: admin@exampundit.in
IBPS RRB PO Prelims – Ultra Practice Bundle PDF
e. Only III and I are True

11). Inequality Questions - Solution and detailed Explanation


1. b Hence both the conclusions are true.
2. b Solution 4
3. e On combining: Q < W = T < S; R < W = T < S
4. b I. R = T → False as R > T
5. c II. S > Q → True as Q < W = T < S.
Solution 1 Hence only conclusion II is true.
Given: D < J, D = B, J ≤ K, A ≤ B Solution 5
On combining: A ≤ B = D < J ≤ K Given statement: A > B ≤ C; B > D > E; A < F ≤ G
Conclusions: On combining: G ≥ F > A > B > D > E
I. J = B → False Conclusion:
II. K > A → True I. A > E → True (as A > B > D > E → A > E)
Hence, option b is correct. II. F > D → True (as F > A > B > D → F > D)
Solution 2 Hence, both conclusion I and II are true.
Given statements: P ≤ Q> M; N ≥ Q > T; M < T Answers
On combining: P ≤ Q> M < T; N ≥ Q > T 6. d
Conclusions: 7. e
I. T > Q →False (as Q> M < T, the relation between T 8. d
and Q is not clear) 9. e
II. N ≥ P →True (as P ≤ Q ≤ N → N ≥ P) 10. b
Hence, only II is true. Solution 6
Solution 3 Given statement: P < Q ≥ R; R < S ≤ T; V > U ≤ T
On combining: D < B < C; A = B > D > E On combining: P < Q ≥ R < S ≤ T ≥ U < V
I. D < C is true as D < B < C. Conclusion:
II. A > E is true as A > D > E.
Page 208 of 728

Subscribe the Xpress Video Course & Mock Test Package for Bank & Insurance Exams
If there are any suggestions/ errors in our PDFs Feel Free to contact us via this email: admin@exampundit.in
IBPS RRB PO Prelims – Ultra Practice Bundle PDF
I. P > T → False (as P < Q ≥ R < S ≤ T) thus clear Hence, none is true is true.
relation between P and T cannot be determined as the Solution 9
symbols are in reverse order. Given statement: P ≤ Q = R; R ≤ S > T; T = U > V
II. Q ≥ S → False (as Q ≥ R < S) thus clear relation On combining: P ≤ Q = R ≤ S > T = U > V
between Q and S cannot be determined as the symbols Conclusion:
are in reverse order. I. S > P → False (as P ≤ Q = R ≤ S → P ≤ S)
Hence, none is true. II. S = P → False (as P ≤ Q = R ≤ S → P ≤ S)
Solution 7 So either S > P or S = P is true
Given statement: 1 < 2 < 3; 3 ≤ 4 > 5; 6 > 7 ≥ 1 Hence, either I or II follows.
On combining: 6 > 7 ≥ 1 < 2 < 3 ≤ 4 > 5 Solution 10
Conclusion: Given: P ≤ Q > R ≥ S; T > R < X
I. 7 ≥ 3 → False (as 7 ≥ 1 < 2 < 3) thus clear relation On combining: P ≤ Q > R < X; R ≥ S, R < T
between 7 and 3 cannot be determined I. X > S - True (R < X; R ≥ S)
II. 7 < 3 → False (as 7 ≥ 1 < 2 < 3) thus clear relation II. Q > S - True (Q > R ≥ S)
between 7 and 3 cannot be determined Hence, the conclusions I & II follows.
Therefore, conclusions I and II forms complementary Answers
pairs. All the three relations are given. 11. b
Hence, either I or II follows 12. e
Solution 8 13. e
Given statement: 10 > 9 = 8; 10 < 11 ≤ 12; 11 > 13 = 14 14. e
On combining: 12 ≥ 11 > 10 > 9 = 8; 11 > 13 = 14 15. c
Conclusion: Solution 11
I. 12 ≥ 14 → False (as 12 ≥ 11→ 11 > 13→ 13 = 14) ‘>’ Given statements: A > E > C ; C = D ; E < F
symbol is there between 12 and 14. Thus, the relation Conclusions:
between 12 and 14 is ’12 > 14’. I. D < E → True (as D = C < E < A → thus D < E)
II. 13 > 9 → False (as 13 < 11 > 10 > 9) thus clear II. C < A → True (as D = C < E < A → thus C < A)
relation between 13 and 9 cannot be determined as the III. A > F → False (as D = C < E < A; E < F → thus the
symbols are in reverse order. clear relation between A and F cannot be determined)
Page 209 of 728

Subscribe the Xpress Video Course & Mock Test Package for Bank & Insurance Exams
If there are any suggestions/ errors in our PDFs Feel Free to contact us via this email: admin@exampundit.in
IBPS RRB PO Prelims – Ultra Practice Bundle PDF
Hence, conclusion I and II are correct and conclusion III Conclusions:
is incorrect. I. C > F → False (as C ≥ D ≤ F → thus the clear relation
Solution 12 between C and F cannot be determined)
Given statements: A > B = C; D < E = B; F > E II. H > F → False (as F > G < H → thus the clear
Conclusions: relation between H and F cannot be determined)
I. F > C → True (as F > E = B = C → thus we can III. G < D → False (as D ≤ F > G → thus the clear
conclude that F > C) relation between G and D cannot be determined)
II. A < F → False (as A > B = C = E > D; F > E = B = C Hence, no conclusions follow.
→ F > B = C = E > D; A > B but the clear relation Solution 15
between A and F cannot be determined) Given statements: G < I, I ≥ K, K = M, M < O
III. D > A → False (as A > B = C = E > D → D < A; On combining: G < I ≥ K = M < O
thus, it is clear that the given conclusion is incorrect.) Conclusions:
Hence conclusion I is correct. I. I ≥ M → True (as I ≥ K = M → I ≥ M)
Solution 13 II. K < O → True (as K = M < O → K < O)
Given statements: C > D > E; A > D; F < E < B III. G > K → False (as G < I ≥ K → thus the clear
Conclusions: relation between G and K cannot be determined)
I. B > F → True (as A > D > E > F & B > E → B > E > Hence, both conclusions I and II follow.
F → B > F . So, this conclusion is correct.) Answers
II. A > C → False (as A > D > E > F & C > D; thus, no 16. c
clear relation between A and C could be inferred from 17. a
the given data.) 18. a
III. D < B → False (as A > D > E > F; C > D; B > E; 19. a
thus, clear relation between D and B cannot be 20. d
determined.) Solution 16
Hence, conclusion I is correct. Given statement: 50 < 60 ≤ 70; 60 > 40 > 30; 70 = 80 ≤
Solution 14 20
Given statements: C ≥ D, D ≤ F, F > G, G < H On combining: 50 < 60 ≤ 70 = 80 ≤ 20; 60 > 40 > 30
On combining: C ≥ D ≤ F > G < H Conclusion:
Page 210 of 728

Subscribe the Xpress Video Course & Mock Test Package for Bank & Insurance Exams
If there are any suggestions/ errors in our PDFs Feel Free to contact us via this email: admin@exampundit.in
IBPS RRB PO Prelims – Ultra Practice Bundle PDF
I. 70 > 30 → True (as 70 ≥ 60 → 60 > 40 → 40 > 30 → On Combining:
So, 70 > 30) P=Q≥R=S=V≤U<T
II. 20 ≥ 60 → True (as 20 ≥ 80 → 80 = 70 → 70 ≥ 60 → Conclusions:
So, 20 ≥ 60) I. Q ≥ V→ True (as Q ≥ R = S = V; therefore Q ≥ V)
Hence, both conclusions I and II are true. II. S > T→ False (as S = V ≤ U < T; therefore S < T)
Solution 17 Hence, only conclusion I is true.
Given statement: 21 > 22 > 23; 22 < 24 ≤ 25; 23 ≥ 26 > Solution 20
27 Given statements: B ≤ L ≤ A > N = C > E; C ≥ V > R =
On combining: 21 > 22 > 23 ≥ 26 > 27; 22 < 24 ≤ 25 S
Conclusion: On combining: B ≤ L ≤ A > N = C ≥ V > R = S
I. 23 ≥ 27 → False (as 23 ≥ 26 → 26 > 27) “>” symbol is Conclusions:
there between 23 and 27. Thus clear relation between 23 I. N ≥ V → True (as N = C ≥ V → N ≥ V)
and 27 is “23 > 27”. II. E < V → False (as C > E & C ≥ V → E < C ≥ V →
II. 21 > 26 → True (as 21 > 22 → 22 > 23 → 23 ≥ 26 → thus a clear relation between E and V cannot be
So, 21 > 26) determined)
Hence, only conclusion II is true. Therefore, only conclusion I is true.
Solution 18 Answers
Given: 21. b
A ≤ B ≤ C = D > E > F; F > G 22. d
On Combining: 23. d
A≤B≤C=D>E>F>G 24. b
Conclusions: 25. b
I. D > G → True (as D > E > F > G; therefore D > G) Solution 21
II. E = G → False (as E > F > G; therefore E > G) Given statements: D ≥ E > F = A ≤ U < L ≥ T = R
Hence, only conclusion I is true. Conclusions:
Solution 19 I. F < L → True (as D ≥ E > F = A ≤ U < L ≥ T = R)
Given: II. D > A → True (as D ≥ E > F = A ≤ U < L ≥ T = R)
P = Q ≥ R = S; T > U ≥ V = S Therefore, both conclusions follow.
Page 211 of 728

Subscribe the Xpress Video Course & Mock Test Package for Bank & Insurance Exams
If there are any suggestions/ errors in our PDFs Feel Free to contact us via this email: admin@exampundit.in
IBPS RRB PO Prelims – Ultra Practice Bundle PDF
Solution 22 II. B > F → True (as B > D = F, so B > F)
Statements: J ≥ N ≤ T; T = S > R; K > V ≥ J Hence, only II is true.
On combining: K > V ≥ J ≥ N ≤ T = S > R Answers
Conclusions: 26. b
I. V ≥ N → True (As V ≥ J ≥ N) 27. c
II. K > J → True (As K > V ≥ J) 28. b
Thus, Both I and II are true. 29. e
Solution 23 30. d
Given statements: Q ≤ R; S < T; P > Q; R > S Solution 26
On combining: P > Q ≤ R > S < T Given statements: O < P > R > N, N ≥ M, P < Q
Conclusions: Conclusions:
I. S = Q → False (as Q ≤ R > S, so a definite relation I. Q > R → True (as Q > P, O < P > R > N)
between S and Q cannot be determined) II. O < Q → True (as Q > P, O < P < Q)
II. T ≥ P → False (P > Q ≤ R > S < T, so a definite Therefore, both conclusions follow.
relation between T and P cannot be determined) Solution 27
Hence, neither I nor II is true. Given statement: M ≥ O≥ G = R ≥ N = T
Solution 24 I. M = T → False (as M ≥ O ≥ G = R ≥ N = T → M ≥ T).
Given Statements: X < C, W > D, G ≥ W, C = D II. T ˂ M → False (as M ≥ O ≥ G = R ≥ N = T → M ≥ T)
On combining: X < C = D < W ≤ G Here, conclusions I and II form a complementary pair.
Conclusions: So, either conclusion I or II follows.
I. C ≤ G → False (as C = D < W ≤ G → C < G) Solution 28
II. W > X → True (as X < C = D < W → X < W) Statements:
Hence, only II is true. U ≥ X ≤ H; K ≤ X; V > Q = X
Solution 25 Conclusions:
Given statements: D = F; B > C; D < C; A > B I. V > U ⇒ False (as V > Q = X and U ≥ X → V > X ≤ U
On combining: A > B > C > D = F ⇒ therefore clear relation between U and V cannot be
Conclusions: established)
I. C ≥ F → False (as C > D = F, so C > F)
Page 212 of 728

Subscribe the Xpress Video Course & Mock Test Package for Bank & Insurance Exams
If there are any suggestions/ errors in our PDFs Feel Free to contact us via this email: admin@exampundit.in
IBPS RRB PO Prelims – Ultra Practice Bundle PDF
II. K ≤ H ⇒ True (as K ≤ X and X ≤ H → K ≤ X ≤ H ⇒ I) S > E → True (as, S > A > L > E → S > E)
Therefore, K ≤ H) II) A < O → False (as, A > L < O → clear relation can
Hence, only conclusion II is true. not be determined between O and A)
Solution 29 Therefore, only conclusion I is true.
On combining: Q < R = T; W < R = T < S Solution 32
I. Q < T is true as Q < R = T Given Statements:
II. S > W is true as S > T = R > W B ≤ A; C < D > E; A = C; F ≥ H = B
Hence both the conclusions are true. ⇒F≥H>B≤A=C<D>E
Solution 30 Conclusions:
Given statements: T < Q ≤ W; M > P < Z; Z = R; M > Y I. H < A → H > B ≤ A → hence relationship between H
On combining: Y < M > P < Z = R, T < Q ≤ W and A cannot be determined.
Conclusions: II. A < F → F ≥ H > B ≤ A → F > B ≤ A → hence
I. P < Q → False (as relation between P and Q cannot be relationship between A and F cannot be determined.
established) Hence none of the conclusion follow.
II. M < Q → False (as relation between M and Q cannot Solution 33
be established) Given Statements: P ≤ Y; P ≤ Y; X ≥ Z ≤ Y; Z = P ≤ X
Therefore, neither conclusion I nor II is true. On combining: X ≥ Z = P ≤ Y
Answers Conclusions:
31. c I. Y ≥ Z → True (as Z = P ≤ Y)
32. d II. X = Z → False (as X ≥ Z)
33. a Therefore, only conclusion I follows.
34. a Solution 34
35. a Given Statement: L = O ≥ R, P > T, L > U On
Solution 31 combining: U < L = O ≥ R, P > T
Given statements: S > A = N > D; A > L > E; M < L < O I. U = R → False (as U < L = O ≥ R, clear relation
On combining: A > L < O; S > A > L > E; A = N > D; M between U and R cannot be determined)
<L II. L > R → False (L = O ≥ R, L ≥ R)
Conclusion: Hence neither conclusion I nor conclusion II is true.
Page 213 of 728

Subscribe the Xpress Video Course & Mock Test Package for Bank & Insurance Exams
If there are any suggestions/ errors in our PDFs Feel Free to contact us via this email: admin@exampundit.in
IBPS RRB PO Prelims – Ultra Practice Bundle PDF
Solution 35 A < B; B ≤ C; A > D; D = E
Given statements: X > T = S, R > S, T ≤ U On combining: E = D < A < B ≤ C,
On combining: X > T = S ≤ U, X > T = S < R Conclusions:
Conclusions: I. B < A → False (as A < B ≤ C → thus B < A is not
I. S < U → False (as S ≤ U) possible)
II. S > U → False (as S ≤ U) II. E < A → True (as E = D < A)
Hence, none of the conclusions is true. III. C < B → False (B ≤ C → thus C < B is not possible)
Answers Therefore, conclusion II is true.
36. a Solution 39
37. e Given statements:
38. b L > I = N > P; I > R > K; N < E < Z
39. a On combining:
40. c P < N < E < Z; L > I > R > K; I = N > P
Solution 36 Conclusions:
Given statements: D > M > U ≥ E; N < U ≥ J E > P → True (as, P < N < E < Z → E > P)
I. D > N → True (as D > M > U > N) R < L → True (as, L > I > R > K → R < L)
II. E ≤ J → False (as U ≥ E & U ≥ J → J ≤ U ≥ E → Therefore, both conclusion I and II are true.
there is no direct relationship between E and J) Solution 40
Here, only conclusion I follows. Given: X > Y ≤ Z = K ≠ L ≠ M = N; Z ≠ N
Solution 37 Conclusions:
Given statements: A = C ≤ R; Q ≥ O ≥ J; J ≤ R I. N > Z → False ( as Z = K ≠ L ≠ M = N; Z ≠ N, a clear
On combining: A = C ≤ R ≥ J ≤ O ≤ Q relation between N and Z cannot be determined)
Conclusion: II. Z > M → False (as Z = K ≠ L ≠ M = N; Z ≠ N, a clear
I. R ≥ A → True (as A = C ≤ R) relation between M and Z cannot be determined)
II. J ≤ Q → True (as J ≤ O ≤ Q) Here, Z ≠ N = M
Hence Both conclusion I and II follows. Therefore, conclusion I and II are complementary pairs
Solution 38 to each other.
Given statements: Hence either I or II follows.
Page 214 of 728

Subscribe the Xpress Video Course & Mock Test Package for Bank & Insurance Exams
If there are any suggestions/ errors in our PDFs Feel Free to contact us via this email: admin@exampundit.in
IBPS RRB PO Prelims – Ultra Practice Bundle PDF
Answers II. M = Q → False (as clear relationship between M and
41. d Q cannot be determined).
42. b III. M > L → True (as M > G = L).
43. e So, only I and III are true.
44. a Solution 44
45. d Given statements: A > D < K; E > M > R; D > R
Solution 41 On combining: A > D > R; E > M > R < D < A; K > D >
Given statements: T < M = J ≤ B; J < L; N ≥ M > C R
On combining: T < M = J ≤ B; T < M = J < L; N ≥ M > Conclusions:
C I. R < K → True, K > D > R therefore K > R.
Conclusions: II. E > A → Cannot say anything, E > M > R < D and A
I. T > B → False (as T < J and J ≤ B → T < B) > D therefore E > M > R < D < A and we cannot say
II. J < N → False (as J = M and M ≤ N → J ≤ N) anything about relationship between E and A.
III. J = N → False (as J = M and M ≤ N → J ≤ N) III. D > E → Cannot say anything, as we see that E > M
Here, conclusion II and III form complementary pair. > R < D, therefore, we cannot say anything about the
Hence, either conclusion II or conclusion III is true. relationship between E and D.
Solution 42 Hence, only I is true.
Given statement: A > T ≥ R; L ≥ M > N; T = N < S Solution 45
On combining: L ≥ M > N = T ≥ R; S > N; A > N Statements: W ≥ N ≤ M; M ≤ H = I; W > X < Z
I. A > R → True (A > T ≥ R; hence A > R) On combining: I = H ≥ M ≥ N ≤ W > X < Z
II. L = R → False (L ≥ M > N = T ≥ R, implies L > R) Conclusions:
III. S < M → False (M > N; S > N, relation between S I. Z < X → False (as X < Z)
and M can’t be determined) II. X > W → False (as W > X)
Only conclusion I is true. III. N ≥ M → False (as M ≥ N)
Solution 43 Thus, none is true.
On combining the given statements: S ≤ M < F ≥ G = L; Answers
S ≤ M = T; L = G < M; W < Q 46. e
I. F > S → True (as S ≤ M < F). 47. e
Page 215 of 728

Subscribe the Xpress Video Course & Mock Test Package for Bank & Insurance Exams
If there are any suggestions/ errors in our PDFs Feel Free to contact us via this email: admin@exampundit.in
IBPS RRB PO Prelims – Ultra Practice Bundle PDF
48. e III. A > D → False (As there is no relation between
49. d them)
50. c NOTE:
Solution 46 M and D are complementary pairs and both are false. So,
Given Statements: it is a case of either or.
X > H ≥ M = N, P > T < J, N > J Therefore, either conclusion I or II is True.
On Combining: Solution 49
X>H≥M=N>J>T<P Given statements: D = E < F ≤ H = I; J > K = F ≥ L ≤ M;
Conclusions: I>P≥Q<S
I. T > H → False (as H ≥ M = N > J > T → H > T) On combining: D = E < F = K ≤ H = I > P ≥ Q < S; J > K
II. P > M → False (as M = N > J > T < P → thus clear =F≥L≤M
relation between P and M cannot be determined) Conclusions:
III. T = H → False (as H ≥ M = N > J > T → H > T) I. I > M → False (as I = H ≥ K = F ≥ L and L ≤ M →
Hence, none of the conclusion is true. thus, a clear relationship between I and M cannot be
Solution 47 determined)
Given statements: C > D; F ≥ D; F < G; G = M II. Q < L → False (as Q ≤ P < I = H and H ≥ K = F ≥ L
On combining: C > D ≤ F < G = M → thus, a clear relationship between Q and L cannot be
Conclusions: determined)
I. D < G → True (as C > D ≤ F < G = M). Hence, both the conclusions are false.
II. M > D → True (as D ≤ F < G = M → D < M). Solution 50
III. M > F → True (as F < G = M → F < M). Given statements: Y < Z > X; W > D < R; Y > T = R; X
Therefore, all the given conclusions are true. >W
Solution 48 On combining: D < R = T < Y < Z > X > W > D
Given Statements: A ≤ P > K ≤ D; M ≤ O = K > A; J > D Conclusions:
On combining: A ≤ P > K = O ≤ D < J, M ≤ K = O > A I. R < Z → True (as R = T < Y < Z → R < Z)
Conclusions: II. X > D → True (as X > W > D → X > D)
I. M < D → False (As M ≤ D)
II. D = M → False (As M ≤ D)
Page 216 of 728

Subscribe the Xpress Video Course & Mock Test Package for Bank & Insurance Exams
If there are any suggestions/ errors in our PDFs Feel Free to contact us via this email: admin@exampundit.in
IBPS RRB PO Prelims – Ultra Practice Bundle PDF
III. T < W → False (as T < Y < Z > X > W → T < Z > determined)
W → thus clear relation between T and W cannot be Hence, only conclusion I and II are True.

12). Direction Questions


Directions (1-5): Study the following and answer the A) South-West
following questions: B) North-East
5 friends Prakash, Ramesh, Suresh, Tina and Uma are C) South-East
standing randomly. Ramesh is to the northeast of Uma. D) North-West
Tina is 5m to the east of Uma, who is 10m to the west of E) None of These
Prakash. Suresh is to the northwest of Tina and 6 m north 4. What is the shortest distance between Suresh and
of Uma. Tina is 6m to the south of Ramesh. Prakash?
1. What is the distance between Uma and Ramesh? A) 2√18*2 m
A) √61 m B) 2√17*2 m
B) √62 m C) 2√16*2 m
C) √63 m D) 2√15*2 m
D) √64 m E) 2√14*2 m
E) √65 m 5. In which direction is Prakash with respect to
2. What is the shortest distance between Ramesh and Suresh?
Prakash? A) Same direction of Prakash with respect to Ramesh
A) √58 m B) North-East
B) √59 m C) South-East
C) √60 m D) South-West
D) √61 m E) Both A and D
E) Cannot Be Determined Directions (6-8): Study the following and answer the
3. In which direction is Prakash with respect to following questions:
Ramesh?

Page 217 of 728

Subscribe the Xpress Video Course & Mock Test Package for Bank & Insurance Exams
If there are any suggestions/ errors in our PDFs Feel Free to contact us via this email: admin@exampundit.in
IBPS RRB PO Prelims – Ultra Practice Bundle PDF
Praveen starts walking from his house in north direction. 8. If point K is 30 meters to west of B then what is the
After walking 20 meters he reaches point B and after that shortest distance between K and D and what is the
he turns right and walks for 50 meters to reach point C. direction of Market with respect to the point which is
Then he turns left and walks for 40 meters and reaches at 10 meters south of PVR Mall.
point D. Then he again turns and walks in south-east A) 6√125, South-West
direction to reach market E which is in east of point C. B) 6√125, North-East
Again, he walks for 20 meters in south direction and C) 8√125, South-West
reaches at point F. Then he walks for 40 meters in west D) 8√125, North-East
and after reaching point G, which is south of point C. E) None of These
After that he turns left and walks for 50 meters to reach Direction (9-11): Answer the questions based on the
point H. Then he turns right and walks for 100 meters information given below:
and again turns right and walks for 10 meters to reach A is 7 m north of B. D is 10m south of C which is 4m
PVR Mall. east of B. G is 16m east of F and 4m north of H Which
6. What is the shortest distance between the PVR is to the south of D. I is 7m west of H and to the South-
Mall and Praveen’s house and what is the direction of west of G which is to the South-West of J. K is 10m
PVR Mall with respect to the Market? north of J. F is 15m south-west of E which is 4m west of
A) 10√51, South-West D. J is 12m east of D.
B) 10√41, South-West 9. What is the shortest distance between D and G and
C) 10√51, South-East also find the direction of A with respect to D?
D) 10√41, South-East A) 9m, South-East
E) Cannot Be Determined B) 9m, North-East
7. What is the direction of B with respect to H and C) 9m, South-West
find the shortest distance between them? D) 9m, North-West
A) South-East, 5√296 E) None of These
B) North-East, 5√296 10. What is the direction of F with respect to J and
C) North-West, 5√296 what is the shortest distance between J and the point
D) South-West, 5√296 which is 4m East of E?
E) Cannot Be Determined A) South-West, 12m
Page 218 of 728

Subscribe the Xpress Video Course & Mock Test Package for Bank & Insurance Exams
If there are any suggestions/ errors in our PDFs Feel Free to contact us via this email: admin@exampundit.in
IBPS RRB PO Prelims – Ultra Practice Bundle PDF
B) North-East, 16m travels for 3km and finally he turns right and covers 4km
C) South-East, 9m to reach the theatre.
D) North-West, 12m 12. What is the direction of Q with respect to the
E) None of These theatre and what is the shortest distance between
11. If L is 24 m east of K then find the shortest them?
distance between J and L and what is the direction of A) South-West, √160
L with respect to I? B) North-East, √170
A) 26m, South-West C) South-East, √180
B) 24m, North-West D) North-West, √190
C) 26m, North-East E) None of These
D) 24m, South-East 13. What is the distance between P and the one who
E) None of These lives in North-East direction to the theatre and find
Direction (12-15): Answer the questions based on the the direction of P with respect to the Theatre?
information given below: A) South-West, 13km
There are three friends P, Q and R. They make a plan for B) North-East, 15km
a movie and start their journey to the theatre from their C) South-East, 17km
home which is in different directions. D) North-West, 19km
P starts travelling to go to the theatre in east direction. E) None of These
After travelling 4km he turns right and travels for 5km. 14. What is the shortest distance between R and the
Then he turns left and after travelling 4km he turns right Theatre also find the shortest distance between R and
and travels 2km to reach the theatre. P?
Q starts travelling in south direction and after covering A) 2√5, √231
10km he turns right and covers 6km. Then he again turns B) 3√5, √251
right and travels for 3km after which he takes a left turn C) 4√5, √241
and covers 5km to reach the theatre. D) 5√5, √261
R starts travelling in west direction and travels for 7km. E) None of These
Then he turns right and travels for 5km then again turns 15. What is the shortest distance between R and Q
right and covers 7km. Then he takes a left turn and and what is the direction of R with respect to Q?
Page 219 of 728

Subscribe the Xpress Video Course & Mock Test Package for Bank & Insurance Exams
If there are any suggestions/ errors in our PDFs Feel Free to contact us via this email: admin@exampundit.in
IBPS RRB PO Prelims – Ultra Practice Bundle PDF
A) 5√18, South-West B) East, 3 m
B) 4√18, North-East C) South, 3 m
C) 3√18, South-East D) North, 3 m
D) 2√18, North-West E) None of These
E) None of These 19. P * Q # R & S # T, then T is in which direction
Directions (16 – 20): Read the following information with respect to Q?
carefully and answer the questions that follow: A) South-East
G # H means G is to the right of H at a distance of 3m. B) North-East
G $ H means G is to the left of H at a distance of 3m. C) South-West
G & H means G is to the north of H at a distance of 3m. D) North-West
G * H means G is to the south of H at a distance of 3m. E) None of These
16. P # Q * R # S, then S is in which direction with 20. A * B $ C $ D & E, Then B is in which direction
respect to P? with respect to D and at what distance?
A) South-East A) North, 3 m
B) North-West B) South, 3 m
C) North-East C) East, 3 m
D) South-West D) West, 2m
E) None of These E) West, 6 m
17. P * Q # R & S, then S is in which direction with 21. Ram starts walking in the South direction and
respect to P? after walking some distance he took a right turn
A) West followed by a right turn. After that she took two
B) North consecutive left turn, now he is walking in which
C) South direction?
D) East A) South
E) None of These B) North
18. P & Q $ R * S, then S is in which direction with C) East
respect to P and what is distance between P and S? D) West
A) West, 3 m E) None of These
Page 220 of 728

Subscribe the Xpress Video Course & Mock Test Package for Bank & Insurance Exams
If there are any suggestions/ errors in our PDFs Feel Free to contact us via this email: admin@exampundit.in
IBPS RRB PO Prelims – Ultra Practice Bundle PDF
22. Ramesh walks a distance of 10 km towards North, A) 50 m
then he turns to his left and walks 7 km. From here B) 40 m
he took a left turn and walks 8 km and stops at a C) 30 m
point A. Find the distance between the starting point D) 20 m
and A and A is in which direction with respect to E) None of These
starting point. 25. From his house, Lokesh went 25 km to the South.
A) √53 m, North East Then he turns east and covered 25 km. Then he
B) √53 m, South East turned north and covered 5 km. Finally turning to
C) √53 m, North West east, he covered 5 km. What is the shortest distance
D) √53 m, South East between his initial point and his ending point?
E) None of These A) 10 √10 m
23. Amisha travelled from a point A to point B in east B) 10 √11 m
direction at a distance of 60 meters. She turned to his C) 10 √12 m
left and walks 60 meters more, then again turned left D) 10 √13 m
and walks 70 meters. Finally, he turned to left and E) 10 √14 m
walks 60 meters. How far is she from the starting 26. Ramu walks 30 meters towards South. Then
point? turning to his left, he walks 50 meters. Then turning
A) 20 m to his right, he walks 40 meters. Again, he turns to his
B) 30 m right and walks 40 meters. How far is he from initial
C) 40 m position?
D) 10 m A) 50 √3 m
E) Cannot Be Determined B) 50 √4 m
24. Nitin walks 40 meter in the south direction, after C) 50 √5 m
that he took a left turn and walks 50 meters. After D) 50 √6 m
that he took a left turn and walks 40 meters more and E) 50 √2 m
finally he took a left turn and stop after walking 40 27. Rinni goes 40 km towards South from a fixed
meter. Find the distance of Nitin from the initial point, then after turning to her right she goes 20 km.
position? After this she goes 30 km after turning to her right.
Page 221 of 728

Subscribe the Xpress Video Course & Mock Test Package for Bank & Insurance Exams
If there are any suggestions/ errors in our PDFs Feel Free to contact us via this email: admin@exampundit.in
IBPS RRB PO Prelims – Ultra Practice Bundle PDF
How far and in what direction is she from her and walked 5 km more and finally turned left and
starting point? walked 10km. How many kilometres will he have to
A) 20 √5 km, North East walk to reach his office straight?
B) 20 √5 km, North West A) 11 km
C) 20 √5 km, South East B) 12 km
D) 20 √5 km, South West C) 13 km
E) None of These D) 14 km
28. Ajay starts walking from his office in east E) 15 km
direction and after walking 25 meter he took a right Directions (31-35): Study the following information
turn and walk 30 meters. Now he took a left turn and and answer the questions below:
walks 10 meters to reach the railway station. Find the There are 5 friends S, T, U, V and W standing randomly.
distance between office and railway station? T is to the northeast of W. V is 2km to the east of W,
A) 5√82 m who is 6km to the west of S. U is to the northwest of V
B) 5√83 m and in the line of WT. V is 4km the south of T.
C) 5√84 m 31. In which direction is U with respect to S?
D) 5√85 m A) South-West
E) 5√86 m B) South-East
29. Riddhi rides her car northwards, then turned left C) North-East
and rode 2 km and again turned left and rode 4 km. D) North-West
She found himself exactly 2 km west from the starting E) None of these
point. How far did she ride northwards initially? 32. In which direction is S with respect to T?
A) 4 km A) South-East
B) 3 km B) South-West
C) 2 km C) North-West
D) 1 km D) North-East
E) 5 km E) None of these
30. Rajan left office and walked 7 km northwards, 33. What is the distance between V and S?
turned right and walked for 10km and turned left A) 5km
Page 222 of 728

Subscribe the Xpress Video Course & Mock Test Package for Bank & Insurance Exams
If there are any suggestions/ errors in our PDFs Feel Free to contact us via this email: admin@exampundit.in
IBPS RRB PO Prelims – Ultra Practice Bundle PDF
B) 4km 37. What is the shortest distance between his initial
C) 3km point and his ending point?
D) 2km A) 3√26 km
E) None of these B) 4√26 km
34. What is the shortest distance between T and S? C) 5√26 km
A) 4√3 km D) 6√26 km
B) 4√2 km E) None of these
C) 4√4 km 38. Raju starts walking in north direction and walks
D) 4√5 km a distance of 9 meters. Now he tooks a right turn and
E) None of these walk 5 m. Again, he takes a right turn and walk 20m
35. What is the shortest distance between T and W? and reached a point Q. Find the distance between
A) 3√5 km starting point and Q and in which direction is the
B) 4√5 km person from the initial point.
C) 5√5 km A) √142 m, South-East
D) 2√5 km B) √143 m, South-East
E) None of these C) √144 m, South-East
Directions (36-37): Study the following information D) √145 m, South-East
and answer the questions below: E) √146 m, South-East
Kala walked 25km towards east, took a right turn and 39. Devika started from point A in North direction.
walked 20km. He again took a right turn and walked 20 After walking for 5 m she turned to her left and
km. He then took a left turn and went 5 km in that walked 6 m. Now she turned to her left and walked 4
direction and stopped. m after which she turned to her left again. Now she
36. Now he was facing which direction? walked 5 m and turned to her right and walked 15 m.
A) North Now finally she turned to her left and after walking
B) East for 8 m, she stopped at point B. What is the distance
C) West AB?
D) South A) 3√5 m
E) None of these B) 4√5 m
Page 223 of 728

Subscribe the Xpress Video Course & Mock Test Package for Bank & Insurance Exams
If there are any suggestions/ errors in our PDFs Feel Free to contact us via this email: admin@exampundit.in
IBPS RRB PO Prelims – Ultra Practice Bundle PDF
C) 5√5 m A) 9 km
D) 6√5 m B) 10 km
E) 7√5 m C) 11 km
40. Ridhima started from her office to park. She D) 12 km
started in west direction. After walking for 5 m she E) 13 km
turned to her left and walked 7 m, now she turned 43. If T is 6 km east of E. what is the shortest distance
right and walked 3 m. After this she turned to right between H and T?
walked 3 m. Now after turning to her right she A) 7 km
walked 13 m and reached park. Find the shortest B) 8 km
distance between her office and park? C) 9 km
A) √45 m D) 10 km
B) √44 m E) 11 km
C) √43 m Directions (44-46): Study the information and
D) √42 m answer the questions:
E) √41 m Naomi starts walking from her office in north direction
Directions (41-43): Study the information and answer and walks 7 km, then turn her right and walks 12 km and
the questions stop in a shop. After purchasing some items, she takes
A is 10 km north of B. E is 7 km south of F, which is 3 two consecutive left turns for 6 km and 20 km
km east of G. G is 9 km west of H.C is 4 km south of D. respectively and reached at point F. From point F she
D is 14 km west of E. I is 3 km west of A.B is 6 km east walks 4 km in south direction to reach her home.
of C. 44. What is the shortest distance between his home
41. In which direction is B with respect to F? and shop?
A) South-East A) 2√101 km
B) North-East B) 2√102 km
C) North-West C) 2√103 km
D) South-West D) 2√104 km
E) None of These E) 2√105 km
42. What is the shortest distance between E and A?
Page 224 of 728

Subscribe the Xpress Video Course & Mock Test Package for Bank & Insurance Exams
If there are any suggestions/ errors in our PDFs Feel Free to contact us via this email: admin@exampundit.in
IBPS RRB PO Prelims – Ultra Practice Bundle PDF
45. What is the direction of her office with respect to B) 6 km
her home? C) 7 km
A) South-West D) 8 km
B) North-West E) 9 km
C) South-East 48. What is the direction of T with respect to Q?
D) North-East A) South-East
E) None of These B) South-West
46. If Naomi walks 12 km west to his office and C) North-East
reached in Central Mall, then what is the shortest D) North-West
distance between Central Mall and her home? E) Cannot Be Determined
A) √94 km 49. What is the shortest distance between R and U?
B) √95 km A) 11 km
C) √96 km B) 12 km
D) √97 km C) 13 km
E) None of These D) 14 km
Directions (47-50): Study the information and answer E) 15 km
the questions: 50. What is the direction of P with respect to V?
P is 10 km west of Q. U is 12 km north of S. T is 20 km A) North-East
east of R. V is 17 km south-west of T and south of U. R B) North-West
is 8 km south of P. T is 15 km east of S. C) South-East
47. What is the shortest distance between S and V? D) South-West
A) 5 km E) Cannot Be Determined

12). Direction Questions - Detailed Explanation with Answers

Answers (1-5): 2) D
1) A 3) C

Page 225 of 728

Subscribe the Xpress Video Course & Mock Test Package for Bank & Insurance Exams
If there are any suggestions/ errors in our PDFs Feel Free to contact us via this email: admin@exampundit.in
IBPS RRB PO Prelims – Ultra Practice Bundle PDF
4) B Answer: D (√61 m)
5) E 3) Prakash is to the South East of Ramesh
Solutions (1-5): Answer: C (South-East)
1). As per the information given in the question, the 4) As per the information given in the question the
diagram will look like this: diagram will look like this:

(UR)2= (UT)2 + (RT)2 (SP)2= (SU)2 + (UP)2


(UR)2= (5)2 + (6)2 (SP)2= (6)2 + (10)2
(UR)2= 25+36 (SP)2= 36 + 100
(UR)2= 61 m (SP)2= 136 m
UR = √61 m SP = √136 m
Answer: A (√61 m) SP = √2 * 2 * 2 * 17 m
2) As per the information given in the question the SP = 2√17 * 2 m
diagram will look like this: Answer: B (2√ (17*2) m)
5) Prakash is in South East direction of Suresh and in the
same direction as Prakash is with respect to Ramesh
Answer: E (Both A and D)
Answers (6-8):
6) B
2 2 2
(RP) = (RT) + (TP) 7) C
2 2 2
(RP) = (6) + (5) 8) D
2
(RP) = 36 + 25 Solutions (6-8):
2
(RP) = 61 6) As per the information given in the question the
RP = √61 m diagram will look like this:
Page 226 of 728

Subscribe the Xpress Video Course & Mock Test Package for Bank & Insurance Exams
If there are any suggestions/ errors in our PDFs Feel Free to contact us via this email: admin@exampundit.in
IBPS RRB PO Prelims – Ultra Practice Bundle PDF

(JA)2 = (AO)2 + (JO)2


(JA)2 = (40)2 + (50)2 (BH)2 = (BO)2 + (OH)2
(JA)2 = 1600 + 2500 (BH)2 = (70)2 + (50)2
(JA)2 = 4100 (BH)2 = 4900 + 2500
JA = √4100 (BH)2 = 7400
JA = 10√41 BH = √7400
Direction of PVR Mall with respect to Market is South- BH = 5√296
West Direction of B with respect to H is North-West
So, answer will be 10√41and South-West So, answer will be North-West, 5√296
Answer: B (10√41, South-West) Answer: C (North-West, 5√296)
7) As per the information given in the question the 8) As per the information given in the question the
diagram will look like this: diagram will look like this:

Page 227 of 728

Subscribe the Xpress Video Course & Mock Test Package for Bank & Insurance Exams
If there are any suggestions/ errors in our PDFs Feel Free to contact us via this email: admin@exampundit.in
IBPS RRB PO Prelims – Ultra Practice Bundle PDF

(EF)2 = (EO)2 + (FO)2


2 2 2
(KD) = (KC) + (DC)
(15)2 = (EO)2 + (12)2
2 2 2
(KD) = (80) + (40)
225 = (EO)2 + 144
2
(KD) = 6400 + 1600
(EO)2 = 225-144
2
(KD) = 8000
(EO)2 = 81
KD = √8000
EO = 9 m
KD = 8√125
EO = DG = 9m
Direction of Marketwith respect to the point which is 10
Direction of A with respect to D is North-West
meters south of PVR Mall is North-East
So, answer will be 9m, North-West
So, answer will be North-East, 8√125
Answer: D (9m, North-West)
Answer:D (8√125, North-East)
10) As per the information given in the question the
Answers (9-11):
diagram will look like this:
9) D
10) A
11) C
Solutions (9-11):
9) As per the information given in the question the
diagram will look like this:

Page 228 of 728

Subscribe the Xpress Video Course & Mock Test Package for Bank & Insurance Exams
If there are any suggestions/ errors in our PDFs Feel Free to contact us via this email: admin@exampundit.in
IBPS RRB PO Prelims – Ultra Practice Bundle PDF
(JL)2 = 100 + 576
(JL)2 = 676
JL = √676
JL = 26 m
Direction of L with respect to I is North-East
So, answer is 26 m, North-East
Answer: C (26m, North-East)
Answers (12-15):
12) B
13) D
EO = 9 m (Calculated in the previous question)
14) C
EO = DG = 9 m
15) A
Point which is 4 m east of E is D
Solutions (12-15):
Thus, the distance between D and J is 12m.
12) As per the information given in the question the
Direction of F with respect to J is South-West
diagram will look like this:
So, answer is South-West
Answer: A (South-West, 12m)
11) As per the information given in the question, the
diagram will look like this:

(QO)2 = (OS)2 + (QS)2


(QO)2 = (11)2 + (7)2
(QO)2 = 121 + 49
(QO)2 = 170
QO = √170
2 2 2
(JL) = (KJ) + (KL) QO = √170 m
2 2 2
(JL) = (10) + (24) Direction of Q with respect to O is North-East

Page 229 of 728

Subscribe the Xpress Video Course & Mock Test Package for Bank & Insurance Exams
If there are any suggestions/ errors in our PDFs Feel Free to contact us via this email: admin@exampundit.in
IBPS RRB PO Prelims – Ultra Practice Bundle PDF
So, answer isNorth-East, √170
Answer: B (North-East, √170)
13) As per the information given in the question the
diagram will look like this:

(RP)2 = (US)2 + (PU)2


(RP)2 = (15)2 + (4)2
(RP)2 = 225 + 16
(RP)2 = 241
RP = √241
PQ = AB + PF + OC + DE
RP = √241km
PQ = 4 km + 4 km + 5 km + 6 km
(RO)2 = (RT)2 + (TO)2
PQ = 19 km
(RP)2 = (8)2 + (4)2
Direction of P with respect to Theatre is North-West
(RP)2 = 64 + 16
So, answer is North-West, 19 km
(RP)2 = 80
Answer: D (North-West, 19km)
RP = √80
14) As per the given information in the question, the
RP = 4√5 km
diagram will look like this:
So, answer will be √241 km and 4√5 km
Answer: C (4√5, √241)
15) As per the information given in the question, the
diagram will look like this:

Page 230 of 728

Subscribe the Xpress Video Course & Mock Test Package for Bank & Insurance Exams
If there are any suggestions/ errors in our PDFs Feel Free to contact us via this email: admin@exampundit.in
IBPS RRB PO Prelims – Ultra Practice Bundle PDF
As per the given information, the diagram will look like
this:

(RQ)2 = (MR)2 + (MQ = PQ – PM)2 As per the diagram S is in North-West of P


(RQ)2 = (15)2 + (15)2 Answer: B (North-West)
(RQ)2 = 225 + 225 17) First, we will decode the codes:
(RQ)2 = 450 P * Q: P is to the south of Q at a distance of 3m.
RQ = √450 Q # R: Q is to the right of R at a distance of 3m.
RQ = 5√18 km R & S: R is to the north of S at a distance of 3m.
Direction of R with respect to Q is South-West As per the given information, the diagram will look like
So, answer is5√18, South-West this:
Answer: A (5√18, South-West)
16) B
17) A
18) B
19) C
20) E
Solutions (16-20):
16) First we will decode the codes:
P # Q: P is to the right of Q at a distance of 3m.
As per the diagram S is in west of P
Q * R: Q is to the south of R at a distance of 3m.
Answer: A (West)
R # S: R is to the right of S at a distance of 3m.
18) First, we will decode the codes:
Page 231 of 728

Subscribe the Xpress Video Course & Mock Test Package for Bank & Insurance Exams
If there are any suggestions/ errors in our PDFs Feel Free to contact us via this email: admin@exampundit.in
IBPS RRB PO Prelims – Ultra Practice Bundle PDF
P & Q: P is to the north of Q at a distance of 3m. 20) First we will decode the codes:
Q $ R: Q is to the left of R at a distance of 3m. A * B: A is to the south of B at a distance of 3m.
R * S: R is to the south of S at a distance of 3m. B $ C: B is to the left of C at a distance of 3m.
As per the given information, the diagram will look like C $ D: C is to the left of D at a distance of 3m.
this: D & E: D is to the north of E at a distance of 3m.
As per the information, the diagram will look like this:

As per the given diagram, S is to the east of P at a


distance of 3 m. As per the diagram, B is west of D at a distance of 6m
Answer: B (East, 3m) Answer: E (West, 6m)
19) First, we will decode the codes: 21) A
P * Q: P is to the south of Q at a distance of 3m. 22) C
Q # R: Q is to the right of R at a distance of 3m. 23) D
R & S: R is to the north of S at a distance of 3m. 24) E
S # T: S is to the right of T at a distance of 3m. 25) D
As per the information, the diagram will look like this: Solution 21: As per the given information the diagram
will look like this:

As per the diagram, T is in South West of Q.


Answer: C (South-West)

Page 232 of 728

Subscribe the Xpress Video Course & Mock Test Package for Bank & Insurance Exams
If there are any suggestions/ errors in our PDFs Feel Free to contact us via this email: admin@exampundit.in
IBPS RRB PO Prelims – Ultra Practice Bundle PDF
Solution 23: As per the given information, the diagram
will look like this:

As per the diagram, he is walking in South direction


He is 10 m away from the starting point.
now.
Answer: D (10 m)
Answer: A (South)
Solution 24: As per the given information, the diagram
Solution 22: As per the given information the diagram
will look like this:
will look like this:

Nitin is 10 m away from the initial position.


Answer: E (None of These)
(AB)2 = (AE)2 + (EB)2 Solution 25: As per the given information, the diagram
(AB)2 = (7)2 + (2)2 will look like this:
(AB)2 = 49 + 4
(AB)2 = 53
AB = √53 km
Distance between A and B is √53 m and A is in North-
West of starting point.
Answer: C (√53 m, North West)

Page 233 of 728

Subscribe the Xpress Video Course & Mock Test Package for Bank & Insurance Exams
If there are any suggestions/ errors in our PDFs Feel Free to contact us via this email: admin@exampundit.in
IBPS RRB PO Prelims – Ultra Practice Bundle PDF
(AE)2 = (AF)2 + (FE)2
(AE)2 = (20)2 + (30)2
(AE)2 = 400 + 900
(AE)2 = 1300
AE = √1300
AE = 10 √ 13 km
Answer: D (10 √ 13 km)
Solution 26: As per the given information, the diagram
(AD)2 = (DE)2 + (AE)2
will look like this:
(AD)2 = (20)2 + (10)2
(AD)2 = 400 + 100
(AD)2 = 500
(AD) = √500
(AD) = 20√5 km
She is in South-West of the initial position.
(AE)2 = (AF)2 + (FE)2 Answer: D (20√5 km, South West)
(AE)2 = (70)2 + (10)2 Solution 28) As per the given information, the diagram
(AE)2 = 4900 + 100 will look like this:
(AE)2 = 5000
AE = √5000
AE = 50 √2 m
Answer: E (50 √2 m)
Solution 27: As per the given information the diagram
will look like this:

(AD)2 = (AE)2 + (ED)2


(AD)2 = (30)2 + (35)2
(AD)2 = 900 + 1225
(AD)2 = 2125

Page 234 of 728

Subscribe the Xpress Video Course & Mock Test Package for Bank & Insurance Exams
If there are any suggestions/ errors in our PDFs Feel Free to contact us via this email: admin@exampundit.in
IBPS RRB PO Prelims – Ultra Practice Bundle PDF
AD = 5√85 m 33) B
Answer: D (5√85 m) 34) B
Solution 29) As per the given information, the diagram 35) D
will look like this: Solutions (31-35):
31) As per the given information, the diagram will look
like this:

She rides 4 km initially


Answer: A (4 km)
U is in North-West of S
Solution 30) As per the given information, the diagram
Answer: D (North-West)
will look like this:
32) As per the given information, the diagram will look
like this:

He has to travel 12 km straight to reach his office.


Answer: B (12 km) S is in South-East of T
Answers (31-35): Answer: A (South-East)
31) D 33) As per the given information the diagram will look
32) A like this:
Page 235 of 728

Subscribe the Xpress Video Course & Mock Test Package for Bank & Insurance Exams
If there are any suggestions/ errors in our PDFs Feel Free to contact us via this email: admin@exampundit.in
IBPS RRB PO Prelims – Ultra Practice Bundle PDF

Distance between V and S is 4 km


Answer: B (4 km) (TW)2 = (TV)2 + (WV)2

34) As per the given information, the diagram will look (TW)2 = (4)2 + (2)2

like this: (TW)2 = 16 + 4


(TW)2 = 20
TW = 2√5 km
Answer: D (2√5 km)
Answers (36-37):
36) D
37) A
Solutions (36-37):
2 2 2
(TS) = (TV) + (VS) 36) As per the given information, the diagram will look
2 2 2
(TS) = (4) + (4) like this:
2
(TS) = 16 + 16
(TS)2 = 32
TS = 4√2 km
Answer: B (4√2 km)
35) As per the given information, the diagram will look
like this:

Page 236 of 728

Subscribe the Xpress Video Course & Mock Test Package for Bank & Insurance Exams
If there are any suggestions/ errors in our PDFs Feel Free to contact us via this email: admin@exampundit.in
IBPS RRB PO Prelims – Ultra Practice Bundle PDF

Kala will be facing South direction in the end


(NQ)2 = (NM)2 + (MQ)2
Answer: D (South)
(NQ)2 = (5)2 + (11)2
37) As per the given information, the diagram will look
(NQ)2 = 25 + 121
like this:
(NQ)2 = 146
NQ = √146 m
Raju is in South-East of his initial point
Answer: E (√146 m, South-East)
Solution 39) As per this information, the diagram will
look like this:

(AE)2 = (AF)2 + (FE)2


(AE)2 = (5)2 + (25)2
(AE)2 = 25 + 625
(AE)2 = 650
AE = 5√26 km
Answer: C (5√26 km)
Solution 38) As per the given information, the diagram (AB)2 = (AC)2 + (CB)2

will look like this: (AB)2 = (14)2 + (7)2


Page 237 of 728

Subscribe the Xpress Video Course & Mock Test Package for Bank & Insurance Exams
If there are any suggestions/ errors in our PDFs Feel Free to contact us via this email: admin@exampundit.in
IBPS RRB PO Prelims – Ultra Practice Bundle PDF
(AB)2 = 196 + 49
(AB)2 = 245
AB = √245 m
AB = 7√5 m
Answer: E (7√5 m)
Solution 40) As per the given information, the diagram
will look like this:
B is in South-West of F
Answer: D (South-West)
42) As per the given information, the diagram will look
like this:

(AF)2 = (AG)2 + (GF)2


(AF)2 = (4)2 + (5)2
(AF)2 = 16 + 25
(AF)2 = 41
(AE)2 = (AI)2 + (IE)2
AF = √41 m
(AF)2 = (6)2 + (8)2
Answer: E (√41 m)
(AF)2 = 36 + 64
Answers (41-43):
(AF)2 = 100
41) D
AF = 10 m
42) B
Answer: B (10 km)
43) A
43) As per the given information, the diagram will look
Solutions (41-43):
like this:
41) As per the given information, the diagram will look
like this:

Page 238 of 728

Subscribe the Xpress Video Course & Mock Test Package for Bank & Insurance Exams
If there are any suggestions/ errors in our PDFs Feel Free to contact us via this email: admin@exampundit.in
IBPS RRB PO Prelims – Ultra Practice Bundle PDF
GD = 2√101 km
Answer: A (2√101 km)
45) As per the given information, the diagram will look
like this:

Distance between H and T is 7 km


Answer: A (7 km)
Answers (44-46):
Direction of office is South-East of home
44) A
Answer: C (South East)
45) C
46) D
46) As per the given information, the diagram will look
Solutions (44-46):
like this:
44) As per the given information, the diagram will look
like this:

(GH)2 = (HI)2 + (GI)2

(GD)2 = (GH)2 + (HD)2 (GH)2 = (4)2 + (9)2

(GD)2 = (2)2 + (20)2 (GH)2 = 16 + 81

(GD)2 = 4 + 400 (GH)2 = 97

(GD)2 = 404 GH = √97 km

Page 239 of 728

Subscribe the Xpress Video Course & Mock Test Package for Bank & Insurance Exams
If there are any suggestions/ errors in our PDFs Feel Free to contact us via this email: admin@exampundit.in
IBPS RRB PO Prelims – Ultra Practice Bundle PDF
Answer: D (√97 km)
Answers (47-50):
47) D
48) A
49) C
50) B
Solutions (47-50):
Direction of T is South-East of Q
47) As per the given information, the diagram will look
Answer: A (South-East)
like this:
49) As per the given information, the diagram will look
like this:

(UR)2 = (US)2 + (RS)2


(UR)2 = (12)2 + (5)2
(TV)2 = (ST)2 + (SV)2
(UR)2 = 144 + 25
(17)2 = (15)2 + (SV)2 (UR)2= 169
2 2
(289) = 225 + (SV) UR = 13 km
(SV)2= 289 – 225 Answer: C (13 km)
2
(SV) = 64
50) As per the given information, the diagram will look
SV = 8 km like this:
Answer: D (8 km)
48) As per the given information the diagram will look
like this:

Page 240 of 728

Subscribe the Xpress Video Course & Mock Test Package for Bank & Insurance Exams
If there are any suggestions/ errors in our PDFs Feel Free to contact us via this email: admin@exampundit.in
IBPS RRB PO Prelims – Ultra Practice Bundle PDF
P is in North-West of Q
Answer: B (North-West)

13). Alpha Numeric Series


Directions (1-5): Study the following information b. B
carefully to answer the given questions:
c. W
F@53R$JPE1H%I84B6#AW2UGC*9&
ZNM©C d. A

e. None of these
1) How many such symbols are there in the above
arrangement each of which are immediately preceded 3) Which of the following is 10th to the left of the 18th
by an alphabet and followed by a number? element from the left end of the above arrangement?
a. Two a. J
b. Three b. E
c. None c. I
d. One d. P
e. More than three e. None of these
2) If all the numbers in the above arrangement are 4) Four of the following five are alike in a certain way
dropped, then which of the following will be eleventh based on their positions in the above arrangement
from right end? and so form a group. Which is the one that does not
a. U belong to that group?

Page 241 of 728

Subscribe the Xpress Video Course & Mock Test Package for Bank & Insurance Exams
If there are any suggestions/ errors in our PDFs Feel Free to contact us via this email: admin@exampundit.in
IBPS RRB PO Prelims – Ultra Practice Bundle PDF
a. 3 J $ a. E

b. E % H b. H
c. G
c. # 2 W
d. 6
d. Z © M e. $
e. U * 9 8) How many such consonants are there in the above
arrangement, each of which are immediately followed
5) How many such consonants are there in the above by a symbol and preceded by a vowel?
arrangement each of which is immediately preceded a. two
by a number but not immediately followed by a
symbol? b. one
a. Two c. three
b. One d. More the four
c. Three e. None of these
d. Five 9) Four of the following five elements are alike in a
certain way based on their positions. Find the one
e. None which does not belong to that group?
a. A6@
Directions (6-10): Study the following information b. @87
carefully and the questions given below.
c. EMQ
2@5KA7632@T8€V7%B3EG$MP©QT
%3UB65$# d. 5A3
6) How many such letters are there in the above e. ©TU
arrangement, each of which is immediately followed 10) If all the symbols are dropped then which among
by a letter but not immediately preceded by a letter? the following elements will be the fifteenth element
a. four from the right end?
b. three a. B
c. two b. G
d. more than four c. %
e. none of these d. 2
7) Which of the following elements will be twelfth to e. V
the right of the eighth element from the left end of the Directions (11-15): Study the following information
given arrangement? carefully to answer the given questions:
Page 242 of 728

Subscribe the Xpress Video Course & Mock Test Package for Bank & Insurance Exams
If there are any suggestions/ errors in our PDFs Feel Free to contact us via this email: admin@exampundit.in
IBPS RRB PO Prelims – Ultra Practice Bundle PDF
a. None
YX9MGP6I$A4LN@Z5#VC7E8B%T2 b. One
S3&W
c. Two
11) How many such numbers are there in the above
d. Three
arrangement each of which is immediately followed
by a vowel and immediately preceded by a e. More than three
consonant? 15) Which of the following is third to the left of the
a. One fifteenth element from the left end?
b. Two a. @
c. Three b. Z
d. More than three c. 7
e. None d. L
12) If the first and the last symbols in the above e. None of these
arrangement are dropped, then which of the Direction (16-20): Study the following arrangement of
following will be the twelfth from the left end? numbers, letters, and symbols carefully and answer
a. L the questions given below:
b. N
c. @ R@29TVAY5©#J1P8Q$E3*H%6W4I8
UZ
d. Z
16) Four of the following five are alike in a certain
e. None of these
way based on their positions in the above
13) Four of the following five are alike in a certain arrangement and so form a group. Which is the one
way based on their position in the above arrangement that does not belong to that group?
and hence from a group. Which is the one that does
a. JP©
not belong to that group?
b. E*Q
a. A 4 $
c. WI%
b. T B 2
d. 9V@
c. 9 Y M
e. 1#$
d. V 5 C
17) Which of the following is the fifth to the right of
e. P M 6
the nineteenth element from the right end?
14) How many such consonants are there in the above
a. P
arrangement each of which is immediately followed
by an even number and immediately preceded by a b. V
symbol?
Page 243 of 728

Subscribe the Xpress Video Course & Mock Test Package for Bank & Insurance Exams
If there are any suggestions/ errors in our PDFs Feel Free to contact us via this email: admin@exampundit.in
IBPS RRB PO Prelims – Ultra Practice Bundle PDF
c. W
d. 8 #6BG@IL73HA%©DFKE8JQ1*VTU2
$W
e. None of these.
21) How many such symbols are there in the above
18) How many such numbers are there in the above
arrangement each of which is immediately followed
arrangement, each of which is immediately preceded
by a consonant but not preceded by another symbol?
by a consonant and immediately followed by a
symbol? a. None
a. Three b. One
b. One c. Two
c. More than four d. Three
d. Four e. Four
e. Two 22) Four of the following five are alike in a certain
way based on the positions of their elements in the
19) If the positions of the last eighteen elements in the
above arrangement and so form a group. Which is
above arrangement are reversed, which of the
the one that does not belong to the group?
following will be the seventeenth from the left end?
a. IL@
a. E
b. AH%
b. P
c. KFE
c. W
d. QJ1
d. 6
e. V * T
e. None of these
23) If all the numbers are dropped from the above
20) How many such vowels are there in the above
arrangement which element will be fifth to the left of
arrangement, each of which is either immediately
eleventh from the left?
followed by a symbol or immediately preceded by a
symbol? a. I
a. One b. H
b. Two c. T
c. Three d. *
d. Four e. None of these
e. None 24) In the following question, select the related letters
from the given alternatives.
BG:L7::A%:?
Directions (21-25): Study the following information
carefully to answer the given questions: a. EK
Page 244 of 728

Subscribe the Xpress Video Course & Mock Test Package for Bank & Insurance Exams
If there are any suggestions/ errors in our PDFs Feel Free to contact us via this email: admin@exampundit.in
IBPS RRB PO Prelims – Ultra Practice Bundle PDF
b. DF a. I 9 E
c. KF b. 7 W 5
d. FK c. © 1 D
e. None of these d. Q A 3
25) How many such vowels are there in the above e. E K 4
arrangement each of which is immediately followed
28) If all the symbols are removed from the above
by a symbol and also immediately preceded by a
arrangement, which of the following will be the
consonant?
fourth to the left of the Twelfth from the left end?
a. None
a. H
b. One
b. 9
c. Two
c. B
d. Three
d. ⋆
e. Four
e. None of these
Directions (26-30): Study the following arrangement
29) If all the numbers are removed from the above
carefully and answer the questions given below:
arrangement, which of the following will be the sixth
to the right of the thirteenth from the right end?
MJ%4TEKI9#OA$Q38N5U7W⋆B@DF a. 5
©1Z6H
b. B
c. 7
26) What should come in place of the question mark
d. W
(?) in the following series based on the above
arrangement? e. None of these
J4E I#A Q85 7⋆@? 30) How many such symbols are there in the above
arrangement, each of which is immediately preceded
a. F16
by a number and immediately followed by a vowel?
b. D©Z a. None of these
c. F©6 b. One
d. F1Z
c. Two
e. None of these d. Three
27) Four of the following five are alike in a certain e. More than three
way based on their positions in the above
arrangement and so form a group. Which is the one
that does not belong to that group?
Page 245 of 728

Subscribe the Xpress Video Course & Mock Test Package for Bank & Insurance Exams
If there are any suggestions/ errors in our PDFs Feel Free to contact us via this email: admin@exampundit.in
IBPS RRB PO Prelims – Ultra Practice Bundle PDF
Directions (31-35): Study the following information a. P#4
carefully to answer the given questions: b. 6QP
@PT#34RSU%5BL6QP&9=KJBA#3C$
c. &=J
P*3NT
d. A3$
31) How many such symbols are there in the above
arrangement each of which are immediately preceded e. U5L
by a vowel and followed by an odd number? 35) How many such symbols are there in the above
a. None arrangement each of which are immediately preceded
by number and followed by letter?
b. Two
a. None
c. One
b. One
d. Three
c. Two
e. More than three
d. Four
32) If all the consonants in the above arrangement are
dropped, then which of the following will be the e. More than four
seventh from the left end?
a. 2 Direction (36-40): Study the following arrangement
b. $ carefully and answer the questions given.
c. # K12FI#6VJ9R#G713L$#59L&@N7KI
#
d. 5
36) How many symbols are there which are followed
e. &
by a number and preceded by a letter?
33) How many such letters are there in the above
a. 3
arrangement each of which is immediately followed
by a number and preceded by a symbol? b. 2
a. One c. 4
b. Two d. 0
c. Three e. 1
d. Four 37) How many numbers are there which are preceded
by a symbol and also succeeded by number?
e. None
a. 7
34) Four of following five are alike in a certain way
based on their positions in the above arrangement b. 2
and so form a group. Which is the one that does not c. 1
belong to that group?
d. 4
Page 246 of 728

Subscribe the Xpress Video Course & Mock Test Package for Bank & Insurance Exams
If there are any suggestions/ errors in our PDFs Feel Free to contact us via this email: admin@exampundit.in
IBPS RRB PO Prelims – Ultra Practice Bundle PDF
e. 3 41) Which element will be the eighth to the left of
nineteenth from left end in the above arrangement?
38) How many numbers are there which are preceded
by a letter and also succeeded by letter? a. %
a. 7 b. S
b. 2 c. L
c. 1 d. #
d. 4 e. None of these
e. 3 42) How many such consonants are there in the above
arrangement each of which are immediately preceded
39) How many such letters are there in the above
by a letter and followed by a number?
arrangement each of which is immediately followed
by a number and preceded by a symbol? a. None
a. One b. One
b. Two c. Two
c. Three d. Three
d. Four e. More than three
e. None
40) If all the numbers are removed from the above 43) If the order of the last 15 elements is reversed
arrangement, which of the following will be the fifth which of the following will be fifth to the right of
to the right of the eleventh from the right end? twelfth from the left end?
a. G a. Y
b. & b. K
c. 1 c. J
d. # d. #
e. None of these e. 3
44) What should come in the place of question mark
(?) in the following series based on the above
Direction (41-45): Study the following arrangement
arrangement?
carefully and answer the question given below:
5 @ E, © K %, ?, S Y $
5T@1EF©2KL%56BIM3*STY5$9GJ#
KA a. B M *
b. 5 B M
c. G # A
Page 247 of 728

Subscribe the Xpress Video Course & Mock Test Package for Bank & Insurance Exams
If there are any suggestions/ errors in our PDFs Feel Free to contact us via this email: admin@exampundit.in
IBPS RRB PO Prelims – Ultra Practice Bundle PDF
d. 6 I 3 d. ©
e. None of these e. None of these
45) If all the number in the above arrangement are 48) How many such symbols are there in the above
removed then which element will be fourteenth arrangement each of which is immediately preceded
element from left end? by a number but not immediately followed by a
consonant?
a. S
a. 4
b. Y
b. 1
c. $
c. 3
d. %
d. 5
e. T
e. 2
49) Four of the following five are alike in a certain
Direction (46-50): Study the following alphabet series
way based on their positions in the above
and answer the questions given below.
arrangement and so form a group. Which is the one
3ET@βHJ56Ω∑0RI4%DG56¥7HTLTK that does not belong to that group?
P8O1I3E©QZ52€#
a. 8 K P
46) How many numbers are there in the above
b. © 3 E
arrangement each of which is immediately preceded
by a consonant and followed by a vowel? c. 5 D G
a. 0 d. H @ β
b. 1 e. 6 5 J
c. 2 50) If all the symbols in the above arrangement are
removed then which element will be tenth element
d. 3
from left end?
e. 4
a. D
47) If all the symbols in the above arrangement are
b. 4
dropped, then which of the following will be the 16th
from the right end? c. I
a. L d. R
b. 7 e. G
c. H

Page 248 of 728

Subscribe the Xpress Video Course & Mock Test Package for Bank & Insurance Exams
If there are any suggestions/ errors in our PDFs Feel Free to contact us via this email: admin@exampundit.in
IBPS RRB PO Prelims – Ultra Practice Bundle PDF
13). Alpha Numeric Series - Answers with Explanation:

1. a Solution 3
2. c Given series:
3. d Left side F @ 5 3 R $ J P E 1 H % I 8 4 B 6 # A W 2 U
4. e G C * 9 & Z N M © C Right side
5. b As, left – Left = Left
Solution 1 18th from the left – 10th from the left = 8th from the left
Given series: Clearly, 8th from the left is ‘P’.
Left side F @ 5 3 R $ J P E 1 H % I 8 4 B 6 # A W 2 U Solution 4
G C * 9 & Z N M © C Right side
1) Symbols which are immediately preceded by an Given series:
alphabet and followed by a number: Left side F @ 5 3 R $ J P E 1 H % I 8 4 B 6 # A W 2 U
Required order is, Alphabet → Symbol → Number G C * 9 & Z N M © C Right side
F@53R$JPE1H%I84B6#AW2UGC*9& Here, the group is formed in which second element is
ZNM©C third to the right of the first element, and the third
element is just before the second element.
Hence, there are 2 symbols which are immediately
preceded by an alphabet and followed by a number: F @ 3J$→F@53R$JPE1H%I84B6#AW2UG
C*9&ZNM©C
5 and C * 9.
E%H→F@53R$JPE1H%I84B6#AW2U
Solution 2 GC*9&ZNM©C
Given series: #2W→F@53R$JPE1H%I84B6#AW2U
GC*9&ZNM©C
Left side F @ 5 3 R $ J P E 1 H % I 8 4 B 6 # A W 2 U
Z©M→F@53R$JPE1H%I84B6#AW2U
G C * 9 & Z N M © C Right side
GC*9&ZNM©C
1) If all the numbers are dropped: U*9→F@53R$JPE1H%I84B6#AW
2UGC*9&ZNM©C
Left side F @ R $ J P E H % I B # A W U G C * & Z N
Hence, U * 9 does not belong to the group.
M © C Right side
Solution 5
2) 11th element from the right end is: W
Given series:
Then, letter / number that is eleventh from right end is
‘W’.
Page 249 of 728

Subscribe the Xpress Video Course & Mock Test Package for Bank & Insurance Exams
If there are any suggestions/ errors in our PDFs Feel Free to contact us via this email: admin@exampundit.in
IBPS RRB PO Prelims – Ultra Practice Bundle PDF
Left side F @ 5 3 R $ J P E 1 H % I 8 4 B 6 # A W 2 U Left side: 2 @ 5 K A 7 6 3 2 @ T 8 € V 7 % B 3 E G $
G C * 9 & Z N M © C Right side M P © Q T % 3 U B 6 5 $ # Right
Consonants which are immediately preceded by a side
number but not immediately followed by a symbol:
Eighth element from the left end – 3
Required order is: Number → Consonant → Number /
Twelfth element to the right of eighth element from the
Alphabet left end (3) – G
F@53R$JPE1H%I84B6#AW2UGC*9
Therefore, “G” will be twelfth to the right of the eighth
&ZNM©C
element from the left end of the given arrangement
Hence, there is only one consonant which is immediately
Hence, “G” is the correct answer.
preceded by a number but not immediately followed by a
symbol: ‘4 B 6’ Solution 8
Given series: 2 @ 5 K A 7 6 3 2 @ T 8 € V 7 % B 3 E G
Answers: $MP©QT%3UB65$#
6. d Consonants which are immediately followed by a symbol
7. c and preceded by a vowel
8. b 2@5KA7632@T8€V7%B3EG$MP©QT
%3UB65$#
9. c
Therefore, there is only one consonant which is
10. e immediately followed by a symbol and preceded by a
Solution 6 vowel: EG$
Given series: 2 @ 5 K A 7 6 3 2 @ T 8 € V 7 % B 3 E G Hence, “one” is the correct answer.
$MP©QT%3UB65$# Solution 9
Letters, which are immediately followed by a letter but Given series: 2 @ 5 K A 7 6 3 2 @ T 8 € V 7 % B 3 E G
not immediately preceded by a letter $MP©QT%3UB65$#
2@5KA7632@T8€V7%B3EG$MP©QT The second element is two steps ahead of the first
%3UB65$# element and the third element is three steps ahead of the
second element.
Therefore, there are 5 letters. Which are immediately A6@ → A + 2 = 6, 6 + 3 = @
followed by a letter but not preceded by a letter: 5KA, @87 → @ + 2 = 8, 8 + 3 = 7
3EG, $MP, ©QT and 3UB
EMQ → E + 2 = $ ≠ M, M + 3 = Q
Hence, “more than four” is the correct answer.
5A3 → 5 + 2 = A, A + 3 = 3
Solution 7
©TU → © + 2 = T, T + 3 = U
Given series:
Therefore, only EMQ does not belong to the group.

Page 250 of 728

Subscribe the Xpress Video Course & Mock Test Package for Bank & Insurance Exams
If there are any suggestions/ errors in our PDFs Feel Free to contact us via this email: admin@exampundit.in
IBPS RRB PO Prelims – Ultra Practice Bundle PDF
Hence, “EMQ” is the correct answer. YX9MGP6IA4LN@Z5#VC7E8B%T2S
3W
Solution 10
12th character from the left end is N.
Given series: 2 @ 5 K A 7 6 3 2 @ T 8 € V 7 % B 3 E G
$MP©QT%3UB65$# Hence, the element that is twelfth from the left end is
‘N’.
If all the symbols are dropped then the series will be as
follows: Solution 13
Left end: 2 5 K A 7 6 3 2 T 8 V 7 B 3 E G M P Q T 3 U Given series:
B 6 5: Right end Left Side Y X 9 M G P 6 I $ A 4 L N @ Z 5 # V C 7 E 8
Fifteenth element from the right end – V B % T 2 S 3 & W Right Side
Answers: Here in the group, the first and the third elements are
neighbours of each other in the series. The second
11. b
element is placed before the first element in the series
12. b such that there is exactly one element between them.
13. a Hence, A 4 $ does not belong to the group.
14. b Solution 14
15. d Given series:
Solution 11 Left Side Y X 9 M G P 6 I $ A 4 L N @ Z 5 # V C 7 E 8
In the given series: B % T 2 S 3 & W Right Side

Left Side Y X 9 M G P 6 I $ A 4 L N @ Z 5 # V C 7 E 8 Consonants which are immediately followed by even


B % T 2 S 3 & W Right Side number and immediately preceded by a symbol:

Numbers which are immediately followed by a vowel YX9MGP6I$A4LN@Z5#VC7E8B%T


and immediately preceded by a consonant: 2S3&W

YX9MGP6I$A4LN@Z5#VC7E8B%T2 Hence, there is one consonant which is immediately


S3&W followed by even number and immediately preceded by a
symbol: % T 2.
Hence, there are two numbers which are immediately
followed by a vowel and immediately preceded by a Solution 15
consonant: P 6 I and C 7 E. Given series:
Solution 12 Left Side Y X 9 M G P 6 I $ A 4 L N @ Z 5 # V C 7 E 8
Given series: B % T 2 S 3 & W Right Side

Left Side Y X 9 M G P 6 I $ A 4 L N @ Z 5 # V C 7 E 8 1) Fifteenth element from the left end is: Z


B % T 2 S 3 & W Right Side YX9MGP6I$A4LN@Z5#VC7E8B%T2
If first and last symbols are dropped: ($ and &) S3&W
2) Third element to the left of the fifteenth element: L
Page 251 of 728

Subscribe the Xpress Video Course & Mock Test Package for Bank & Insurance Exams
If there are any suggestions/ errors in our PDFs Feel Free to contact us via this email: admin@exampundit.in
IBPS RRB PO Prelims – Ultra Practice Bundle PDF
YX9MGP6I$A4LN@Z5#VC7E8B%T2 R@29TVAY5©#J1P8Q$E3*H%6W4I8
S3&W UZ
Hence, L is third to the left of the fifteenth element from Hence, the answer is none of these.
the left end. Note: Right End– Right → 19th – 5th = 14th from right
Answers: i.e. Q.
16. e Solution 18
17. e Given series:
18. b Left Side R @ 2 9 T V A Y 5 © # J 1 P 8 Q $ E 3 * H %
6 W 4 I 8 U Z Right Side
19. c
Numbers which are immediately preceded by a
20. a
consonant and immediately followed by a
Solution 16 symbol: Consonant → Number → Symbol
Given series: R@29TVAY5©#J1P8Q$E3*H%6W4I8
Left Side R @ 2 9 T V A Y 5 © # J 1 P 8 Q $ E 3 * H % UZ
6 W 4 I 8 U Z Right Side E is a vowel hence 'E 3 * ' is not counted.
Pattern followed is: (First Term ± 2 = Second Term); Clearly only one number is there i.e. Y 5 ©
(Second Term ± 4 = Third Term)
Solution 19
1) JP© → J + 2 = P; P – 4 = ©
Given series:
2) EQ* → E +2 = *; * - 4 = Q
Left Side R @ 2 9 T V A Y 5 © # J 1 P 8 Q $ E 3 * H %
3) WI% → W + 2 = I; I – 4 = % 6 W 4 I 8 U Z Right Side
4) 9V@ → 9 + 2 = V; V – 4 = @ 1) Positions of the last eighteen elements in the above
5) 1#$ → 1 – 2 = #; # + 6 = $ arrangement are reversed:

Here, all except option 5 follow the pattern. R@29TVAY5©#ZU8I4W6%H*3E$Q8


P1J
Hence, 1#$ does not belong to the given group.
2) The seventeenth element from the left end:
Solution 17
R@29TVAY5©#ZU8I4W6%H*3E$Q8
Given series: P1J
Left Side R @ 2 9 T V A Y 5 © # J 1 P 8 Q $ E 3 * H % Hence, the seventeenth element from left in this
6 W 4 I 8 U Z Right Side arrangement can be easily seen as W.
1) The nineteenth element from the right is #. Solution 20
2) The fifth element to the right of the nineteenth Given Series:
element from the right end; I.e. Fifth element to the right
of #: Q Left Side R @ 2 9 T V A Y 5 © # J 1 P 8 Q $ E 3 * H %
6 W 4 I 8 U Z Right Side
Page 252 of 728

Subscribe the Xpress Video Course & Mock Test Package for Bank & Insurance Exams
If there are any suggestions/ errors in our PDFs Feel Free to contact us via this email: admin@exampundit.in
IBPS RRB PO Prelims – Ultra Practice Bundle PDF
Vowels which are immediately followed by a symbol or Left Side # 6 B G @ I L 7 3 H A % © D F K E 8 J Q
immediately preceded by a symbol: Symbol → Vowel 1 * V T U 2 $ W Right Side
OR Vowel → Symbol Hence, I L @ does not belong to the group.
R@29TVAY5©#J1P8Q$E3*H%6W4I8 Solution 23
UZ
Given series:
Hence, only one vowel is there which is immediately
Left Side # 6 B G @ I L 7 3 H A % © D F K E 8 J Q 1 *
preceded by symbol.
V T U 2 $ W Right Side
Answers:
1) If all the numbers are dropped:
21. c
#BG@ILHA%©DFKEJQ*VTU$W
22. a
2) Fifth to the left of eleventh from the left.
23. e
As, Left – Left = Left
24. d
11th from the left – 5th from the left = 6th from the left
25. b
Solution 21 Clearly, 6th from the left is L.

Given series: Then, letter/ Symbol that is fifth to the left of eleventh
from the left is ‘L’.
Left Side # 6 B G @ I L 7 3 H A % © D F K E 8 J Q 1 * Solution 24
V T U 2 $ W Right Side
Given series:
1) Symbols which are immediately followed by a
consonant but not preceded by another symbol: Left Side # 6 B G @ I L 7 3 H A % © D F K E 8 J Q 1 *
V T U 2 $ W Right Side
#6BG@IL73HA%©DFKE8JQ 1*VTU2
$W BG:L7::A%:?
Hence, there is 2 Symbols which is immediately Clearly, the logic behind this is; the elements are
followed by a consonant but not preceded by another neighbors and there are 2 spaces between two groups of
symbol: neighbors.
1 * V, 2 $ W Hence, the next term is FK.
Solution 25
Solution 22 Given series:
Given series: Left Side # 6 B G @ I L 7 3 H A % © D F K E 8 J Q 1 *
Left Side # 6 B G @ I L 7 3 H A % © D F K E 8 J Q 1 * V T U 2 $ W Right Side
V T U 2 $ W Right Side 1) Vowels which are immediately followed by a symbol
Here the group is formed in which second element is to and also immediately preceded by a consonant:
the immediate left of the first element, and the third #6BG@IL73HA%©DFKE8JQ1*VTU2
element is immediate right of the first element. $W

Page 253 of 728

Subscribe the Xpress Video Course & Mock Test Package for Bank & Insurance Exams
If there are any suggestions/ errors in our PDFs Feel Free to contact us via this email: admin@exampundit.in
IBPS RRB PO Prelims – Ultra Practice Bundle PDF
Hence, there is 1 Vowel which is immediately followed 2) 7 W 5: 7 and W are consecutive, 5 = ‘7’ - 2 unit.
by a symbol and also immediately preceded by a
3) © 1 D: © and 1 are consecutive, D = ‘©’ - 2 unit.
consonant:
4) Q A 3: Q and A are not consecutive.
HA%
5) E K 4: E and K are consecutive, 4 = ‘E’ - 2 unit.
The pattern followed is first two elements are
Answers:
consecutive to each other, third element is - 2 of
26. a 1st element. Hence, (Q A 3) is odd-one out.
27. d Solution 28
28. b Given Series: M J % 4 T E K I 9 # O A $ Q 3 8 N 5 U 7
29. b W⋆B@DF©1Z6H

30. b Removing symbols:


(Left End) M J 4 T E K I 9 O A Q 3 8 N 5 U 7 W B D F
Solution 26
1 Z 6 H (Right End)
Given Series: M J % 4 T E K I 9 # O A $ Q 3 8 N 5 U 7
W⋆B@DF©1Z6H 12th from left end = 3

Question series: 4th to left of ‘3’ = 9.


Hence answer is 9.
J4E I#A Q85 7⋆@ ?
Here, ‘J4E’ have J, J + 2 unit = ‘4’, ‘4’ + 2 unit = E
Solution 29
E + 2 unit = I, I + 2 unit = #, # + 2 unit = A ⇒ I#A is
2nd element Given Series: M J % 4 T E K I 9 # O A $ Q 3 8 N 5 U 7
W⋆B@DF©1Z6H
A + 2 unit = Q, Q + 2 unit = ‘8’, ‘8’ + 2 unit = ‘5’ ⇒
Q85 is 3rd element Removing numbers:
‘5’ + 2 unit = 7, ‘7’ + 2 unit = ⋆, ⋆ + 2 unit = @ ⇒ 7⋆@ (Left End) M J % T E K I # O A $ Q N U W ⋆ B @ D F
is 4th element © Z H (Right End)
For 5th element: @ + 2 unit = F, F + 2 unit = ‘1’, ‘1’ + 2 ⇒ 13th from the right end = $
unit = ‘6’.
⇒ 6th to right of ‘$’ = B
So, F16 is the next term.
Hence answer is ‘B’.
Solution 30
Solution 27
Given Series: M J % 4 T E K I 9 # O A $ Q 3 8 N 5 U 7
Given Series: M J % 4 T E K I 9 # O A $ Q 3 8 N 5 U 7 W⋆B@DF©1Z6H
W⋆B@DF©1Z6H
Pattern to look for: Number → Symbols → Vowel
1) I 9 E: I and 9 are consecutive, E = ‘I’ - 2 unit.
Such Symbols: #.
Page 254 of 728

Subscribe the Xpress Video Course & Mock Test Package for Bank & Insurance Exams
If there are any suggestions/ errors in our PDFs Feel Free to contact us via this email: admin@exampundit.in
IBPS RRB PO Prelims – Ultra Practice Bundle PDF
Hence only one such symbol is there. Left Side @ P T # 3 4 R S U % 5 B L 6 Q P & 9 = K J B
A # 3 C $ P * 3 N T Right Side
There are no such pattern exists in the above
Answers:
arrangement.
31. b
Hence answer should be none.
32. d
Solution 34
33. e
Given series:
34. b
Left Side @ P T # 3 4 R S U % 5 B L 6 Q P & 9 = K J B
35. b A # 3 C $ P * 3 N T Right Side
Solution 31 Here the group is formed in which second element is
Given series: second to the right of the first element and third element
is the second to the right of the second element. While in
Left Side @ P T # 3 4 R S U % 5 B L 6 Q P & 9 = K J B 6QP, each element is next to its previous element.
A # 3 C $ P * 3 N T Right Side
Hence, 6QP does not belong to the group.
1) Symbols which are immediately preceded by a vowel
and followed by an odd number:
Solution 35
@PT#34RSU%5BL6QP&9=KJBA#3C
$P*3NT Given series:
Hence, there are 2 symbols which is immediately @PT#34RSU%5BL6QP&9=KJBA#3C$
preceded by a vowel and followed by an odd number: U P*3NT
% 5 and A # 3. Hence, there is only one symbol which is immediately
preceded by a number and followed by a letter.

Solution 32 Answers:

Given series: 36. e

Left Side @ P T # 3 4 R S U % 5 B L 6 Q P & 9 = K J B 37. c


A # 3 C $ P * 3 N T Right Side 38. b
1) If all the consonants are dropped: 39. b
@#34U%56&9=A#3$*3 40. b
2) 7th element from the left end is 5. Then, symbol/ Solution 36
number that is seventh from left end is ‘5’.
Given series:
Solution 33
K12FI#6VJ9R#G713L$#59L&@N7KI
Given series: #

Page 255 of 728

Subscribe the Xpress Video Course & Mock Test Package for Bank & Insurance Exams
If there are any suggestions/ errors in our PDFs Feel Free to contact us via this email: admin@exampundit.in
IBPS RRB PO Prelims – Ultra Practice Bundle PDF
After finding such symbols which are followed by a K12FI#6VJ9R#G713L$#59L&@N7KI
number and preceded by a letter, we get #
K12FI#6VJ9R#G713L$#59L&@N7KI If all the numbers are removed from the above
# arrangement
So, there is 1 such symbol.
KFI#VJR#GL$#L&@NKI#
Solution 37
S0, 11th right- 5th right= 6th right = &
Given series:
K12FI#6VJ9R#G713L$#59L&@N7KI
# Answers:

After finding such numbers which are preceded by a 41. a


symbol and also succeeded by number. 42. c
K12FI#6VJ9R#G713L$#59L&@N7KI 43. d
#
44. d
So, there are 1such numbers.
45. e
Solution 38
Solution 41
Given series:
Given series:
K12FI#6VJ9R#G713L$#59L&@N7KI
(Left Side) 5 T @ 1 E F © 2 K L % 5 6 B I M 3 * S T Y
#
5 $ 9 G J # K A (Right Side)
So, numbers are there which are preceded by a letter and
As, Left - Left = Left
also succeeded by letter is 2
19th from the Left - 8th from the Left = 11th from the
K12FI#6VJ9R#G713L$#59L&@N7KI
Left
#
Clearly, 11th from the Left is %.
Solution 39
Solution 42
Given series:
Given series:
K12FI#6VJ9R#G713L$#59L&@N7KI
# (Left Side) 5 T @ 1 E F © 2 K L % 5 6 B I M 3 * S T Y
5 $ 9 G J # K A (Right Side)
Combination required is (Symbol number letter)
Required pattern: Letter → Consonant → Number
#G7, @N7
5T@1EF©2KL%56BIM3*STY5$9GJ#
Hence, two pair exist.
KA
Solution 40 (REDO) Hence, there are two consonants which are immediately
Given series: preceded by a letter and followed by a number: I M 3
and T Y 5.
Page 256 of 728

Subscribe the Xpress Video Course & Mock Test Package for Bank & Insurance Exams
If there are any suggestions/ errors in our PDFs Feel Free to contact us via this email: admin@exampundit.in
IBPS RRB PO Prelims – Ultra Practice Bundle PDF
Hence, answer is T.
Solution 43
Given series: Answers:
(Left Side) 5 T @ 1 E F © 2 K L % 5 6 B I M 3 * S T Y 46. b
5 $ 9 G J # K A(Right Side) 47. c
1) If the order of the last 15 elements is reversed: 48. c
5T@1EF©2KL%56BAK#JG9$5YTS*3 49. e
MI
50. c
2) 12th from the left + 5th to the right of 12th from the left
Solution 46
= 17th from the left.
Given series:
Clearly, 17th from the left is #.
Left Side 3 E T @ β H J 5 6 Ω ∑ 0 R I 4 % D G 5 6 ¥ 7
Solution 44 H T L T K P 8 O 1 I 3 E © Q Z 5 2 € # Right Side
Given series: 1) Numbers which are immediately preceded by a
(Left Side) 5 T @ 1 E F © 2 K L % 5 6 B I M 3 * S T Y consonant and followed by a vowel are:
5 $ 9 G J # K A (Right Side) 3ET@βHJ56Ω∑0RI4%DG56¥7HTLT
Here, the sequence is formed by leaving one element: KP8O1I3E©QZ52€#

5 @ E, © K %, 6 I 3, S Y $ Hence, there is only ‘one’ number in the above


arrangement which is immediately preceded by a
Here, according to the positions of the elements in the consonant and followed by a vowel.
series:
Solution 47
E + 2 = ©, % + 2 = 6, 3 + 2 = S
Given series:
According to the pattern followed, in place of '?' Left Side 3 E T @ β H J 5 6 Ω ∑ 0 R I 4 % D G 5 6 ¥ 7
(question mark) there should be 6 I 3. H T L T K P 8 O 1 I 3 E © Q Z 5 2 € # Right Side
1) All the symbols in the above arrangement are
Solution 45 dropped:
Given series: 3ETHJ560RI4DG567HTLTKP8O1I3E
QZ52
(Left Side) 5 T @ 1 E F © 2 K L % 5 6 B I M 3 * S T Y
5 $ 9 G J # K A (Right Side) 2) Letter/Number which is 16th from the right end:
1) If all the numbers are removed: 3ETHJ560RI4DG567HTLTKP8O1I3E
QZ52
T@EF©KL%BIM*STY$GJ#KA
Hence, ‘H’ is 16th element from the right end.
2) 14th element from the left end is T.
Solution 48
Page 257 of 728

Subscribe the Xpress Video Course & Mock Test Package for Bank & Insurance Exams
If there are any suggestions/ errors in our PDFs Feel Free to contact us via this email: admin@exampundit.in
IBPS RRB PO Prelims – Ultra Practice Bundle PDF
Given series: 1) 8KP → 8 - 2 = K; K + 1 = P
Left Side 3 E T @ β H J 5 6 Ω ∑ 0 R I 4 % D G 5 6 ¥ 7 2) © 3 E → © - 2 = 3; 3 + 1 = E
H T L T K P 8 O 1 I 3 E © Q Z 5 2 € # Right Side
3) 5 D G → 5 - 2 = D; D + 1 = G
1) Symbols which are immediately preceded by a
4) H @ β → H - 2 = @; @ + 1 = β
number but not immediately followed by a consonant i.e.
Number → Symbol → (Except consonant) 5) 6 5 J → 6 - 1 = 5; 5 - 1 = J
3ET@βHJ56Ω∑0RI4%DG56¥7HTLTK Hence, ‘6 5 J’ does not belong to the given group.
P8O1I3E©QZ52€# Solution 50
Hence, three such symbols are there in the above Left Side 3 E T @ β H J 5 6 Ω ∑ 0 R I 4 % D G 5 6 ¥ 7
arrangement each of which is immediately preceded by a H T L T K P 8 O 1 I 3 E © Q Z 5 2 € # Right Side
number but not immediately followed by a consonant.
After removing the symbols from the series
Solution 49
3ETHJ560RI4DG567HTLTKP8O1I3E
Given series: QZ52
Left Side 3 E T @ β H J 5 6 Ω ∑ 0 R I 4 % D G 5 6 ¥ 7 Element which is 10th from the left end is I.
H T L T K P 8 O 1 I 3 E © Q Z 5 2 € # Right Side

14). Reasoning Data Sufficiency


Direction: In the following question, a given questions b) Only I and II are sufficient
is followed by information in three statements. You
c) Only II and III are sufficient4
have to decide the data in which statement (s) is
sufficient to answer the question and mark your d) Only I and III are sufficient
answer accordingly.
e) Question cannot be answered even with the
1) Among the seven students A, B, C, D, E, F and G, information in all three statements.
who scored 2nd highest in quant test?
2. Direction: In the following question, a given
I: A scored more than only C. Two students scored more questions is followed by information in three
than G, who scored more than A and B. statements. You have to decide the data in which
II: E scored less than D, who scored more than B. statement (s) is sufficient to answer the question and
mark your answer accordingly.
III. C scored less than D, who does not score the highest.
In which month of the year 2019 did Karan go to Delhi
a) All the statements are required for vacation?

Page 258 of 728

Subscribe the Xpress Video Course & Mock Test Package for Bank & Insurance Exams
If there are any suggestions/ errors in our PDFs Feel Free to contact us via this email: admin@exampundit.in
IBPS RRB PO Prelims – Ultra Practice Bundle PDF
I: Karan’s son remembers that he went after 20th July e) Question cannot be answered even with the
2019 information in all three statements.

II: Karan remembers that he went before 10th August 4) Direction: In the following question, a given
2019. questions is followed by information in three
statements. You have to decide the data in which
III: Rohan, a friend of Karan, remembers that he went to
statement (s) is sufficient to answer the question and
Delhi either in 7th or 8th month of year 2019.
mark your answer accordingly.
a) All the statements are required In a four membered family, how A is related to C?
b) Only I and II are sufficient I. A is father of D who is a husband of E.
c) Only II and III are sufficient II. C is daughter of E, who is daughter in law of A
d) Only I and III are sufficient III. C is child of D. D is son of A.
e) Question cannot be answered even with the a) All the statements are required
information in all three statements.
b) Only I and II are sufficient
3. Direction: In the following question, a given
questions is followed by information in three c) Only II and III are sufficient
statements. You have to decide the data in which
d) Any two statements are sufficient
statement (s) is sufficient to answer the question and
mark your answer accordingly. e) Question cannot be answered even with the
How many students are there between R and A? information in all three statements.

I: R is 7th from the left end. A is 17th from the right end. 5) Direction: In the following question, a given
questions is followed by information in three
II: R is 6 places away from V, who is 20th from the left statements. You have to decide the data in which
end. statement (s) is sufficient to answer the question and
mark your answer accordingly.
III: There are total 40 students in the class.
There are six persons sitting in circular table. All are
a) All the statements are required facing towards center. Who among the following sits 2nd
b) Only I and II are sufficient to the left of P?

c) Only II and III are sufficient I. P sits immediate right of R who faces S.

d) Only I and III are sufficient II. P faces Q who is an immediate neighbor of S.

Page 259 of 728

Subscribe the Xpress Video Course & Mock Test Package for Bank & Insurance Exams
If there are any suggestions/ errors in our PDFs Feel Free to contact us via this email: admin@exampundit.in
IBPS RRB PO Prelims – Ultra Practice Bundle PDF
III. S is not an immediate neighbor of P. R sits 7) Direction: In the following question, a given
immediate right of S. questions is followed by information in three
statements. You have to decide the data in which
a) All the statements are required
statement (s) is sufficient to answer the question and
b) Only I and II are sufficient mark your answer accordingly.

c) Only II and III are sufficient In a certain code language which of the following is the
code for “trunk”?
d) Only I and III are sufficient
Statements:
e) Question cannot be answered even with the
information in all three statements. I. “trunk call given” is coded as “ckgsnc”

II. “given new behalf” is coded as “pknctp”

6) Direction: In the following question, a given III. “call our given” is coded as “gsncmj”
questions is followed by information in three a) All the statements are required
statements. You have to decide the data in which
statement (s) is sufficient to answer the question and b) Only I and II are sufficient
mark your answer accordingly.
c) Only II and III are sufficient
Who is the tallest among five persons i.e. P, Q, R, S and
T? d) Only I and III are sufficient

Statements: e) Question cannot be answered even with the


information in all three statements.
I. S is taller than only one person.
8) Direction: In the following question, a given
II. Q is not the tallest. questions is followed by information in three
statements. You have to decide the data in which
III. R is taller than T but not taller than P. statement (s) is sufficient to answer the question and
a) All the statements are required mark your answer accordingly.

b) Only I and II are sufficient What is the direction of A with respect to B?

c) Only II and III are sufficient I. A is in North of C who is south-west of B.

d) Only I and III are sufficient II. D is in East of C.

e) Question cannot be answered even with the II. A is in North-west of D who is south of B.
information in all three statements.
Page 260 of 728

Subscribe the Xpress Video Course & Mock Test Package for Bank & Insurance Exams
If there are any suggestions/ errors in our PDFs Feel Free to contact us via this email: admin@exampundit.in
IBPS RRB PO Prelims – Ultra Practice Bundle PDF
a) All the statements are required 10) Direction: In the following question, a given
questions is followed by information in three
b) Only I and II are sufficient
statements. You have to decide the data in which
c) Only II and III are sufficient statement (s) is sufficient to answer the question and
mark your answer accordingly.
d) Only I and III are sufficient
Certain number of Persons sitting in the row and facing
e) Question cannot be answered even with the north. How many persons are sitting between R and K?
information in all three statements.
I: R is 6th from the right end of the row.
9) Direction: In the following question, a given
questions is followed by information in three II. R sits 5th to the right of J.
statements. You have to decide the data in which III: J is 6th from the left end of the row and also 7th to
statement (s) is sufficient to answer the question and the left of K.
mark your answer accordingly.
a) All the statements are required
Six boxes, P through U are placed one above another,
where the bottommost position is 1 and the position b) Only I and II are sufficient
above it is 2 and so on. Which box is placed at the
bottommost position? c) Only II and III are sufficient

d) Only I and III are sufficient


I. Two boxes are placed between S and R, which is
placed immediately below of P. e) Question cannot be answered even with the
information in all three statements.
II. T is placed above S, but not adjacent to R.

III. Two boxes are placed between P and Q. U is placed 11) The question given below consists of two
somewhere below Q. statements numbered I and II below them. You have
to decide whether the data provided in the statements
a) All the statements are required are sufficient to answer the question. Read all the
statements and give answer:
b) Only I and II are sufficient
Six persons J, K, L, M, N and O have different heights.
c) Only II and III are sufficient Who is the 4th shortest person?
d) Only I and III are sufficient Statement I: Two persons are taller than J. L is not the
e) Question cannot be answered even with the tallest person. Neither N nor L is shorter than J. M is
information in all three statements. shorter than K but not the shortest.

Page 261 of 728

Subscribe the Xpress Video Course & Mock Test Package for Bank & Insurance Exams
If there are any suggestions/ errors in our PDFs Feel Free to contact us via this email: admin@exampundit.in
IBPS RRB PO Prelims – Ultra Practice Bundle PDF
Statement II: J is taller than M. L is not shorter than K. N b. The data in statement II alone are sufficient to answer
is not shorter than O. the question, while the data in statement I alone are not
sufficient to answer the question
a. If the data given in statement II alone is sufficient to
answer the question. c. The data either in statement I alone or in statement II
alone are sufficient to answer the question
b. If the data given in statement I alone is sufficient to
answer the question. d. The data given in both statements I and II together are
not sufficient to answer the question
c. If the data given in either statement I or statement II
alone is sufficient to answer the question. e. The data in both statements I and II together are
necessary to answer the question.
d. If the data given in both statement I and statement II
together are sufficient to answer the question. 13) The question consists of two statements numbered
I and II given below it. You have to decide whether
e. If the data given in both statement I and statement II the data provided in the statements are sufficient to
together are not sufficient to answer the question.
answer the question.
12) The question consists of two statements numbered Six persons, A1, A2, A3, A4, A5 and A6 are sitting at
I and II given below it. You have to decide whether some distance from each other. What is the shortest
the data provided in the statements are sufficient to distance between A3 and A6?
answer the question.
Statement I: A1 is sitting 19m west to A2, who is sitting
Six persons, Abhi, Bob, Diana, Ella, Geet and Henry are south west to A3. A5 is sitting south east to A3.
sitting around a circular table. All of them are facing the
centre. Who is sitting opposite to Ella? Statement II: A6 is sitting 22m south to A4, who is
sitting 6m west to A5.
Statement I: Abhi is sitting 2nd to the right of Diana,
who is sitting 2nd to the left of Bob, who is sitting a. The data in statement I alone are sufficient to answer
adjacent to Geet. Henry is not sitting adjacent to Abhi. the question, while the data in statement II alone are not
sufficient to answer the question
Statement II: Diana is sitting opposite to Geet, who is
sitting 2nd to the left of Henry. b. The data in statement II alone are sufficient to answer
the question, while the data in statement I alone are not
a. The data in statement I alone are sufficient to answer sufficient to answer the question
the question, while the data in statement II alone are not
sufficient to answer the question c. The data either in statement I alone or in statement II
alone are sufficient to answer the question

Page 262 of 728

Subscribe the Xpress Video Course & Mock Test Package for Bank & Insurance Exams
If there are any suggestions/ errors in our PDFs Feel Free to contact us via this email: admin@exampundit.in
IBPS RRB PO Prelims – Ultra Practice Bundle PDF
d. The data given in both statements I and II together are statements are sufficient to answer the question. Read
not sufficient to answer the question all the statements and give answer:

e. The data in both statements I and II together are In the given coded language, how is the word ‘come’
necessary to answer the question. coded?

14) The question given below consists of two Statement I: ‘you come forward and’ is coded as ‘bk, dk,
statements numbered I and II below them. You have rk, sk’ and ‘forward always win’ is coded as ‘dk, ck, ak’
to decide whether the data provided in the statements
Statement II: ‘come and went’ is coded as ‘nk, bk, sk’
are sufficient to answer the question. Read all the
and ‘he went forward’ is coded as ‘dk, lk, nk’.
statements and give answer:

Six persons, A, B, C, D, E and F are of different heights. Statement III: ‘went with you’ is coded as ‘zk, nk, rk’
and ‘you and come’ is coded as ‘bk, rk, sk’.
Who is the 3rd shortest person?

Statement I: A and C are shorter than D. F is not taller a. Data given in both statements I and III are together
than E. B is taller than E but not the tallest. sufficient to answer.

Statement II: B is taller than A and is the 2nd tallest b. Data given in both statements II and III are together
person. F is shorter than D. C is not the tallest person. E sufficient to answer.
is shorter than A and C. E is not the shortest. c. Data given in both statements I and II are together
a. If the data given in statement II alone is sufficient to sufficient to answer.
answer the question. d. Data given in all three statements I, II and III are
b. If the data given in statement, I alone is sufficient to together sufficient to answer.
answer the question. e. Data given in all three statements I, II and III are
c. If the data given in either statement I or statement II together not sufficient to answer.
alone is sufficient to answer the question. 16) The question given below consists of three
d. If the data given in both statement I and statement II statements numbered I, II and III below them. You
together are sufficient to answer the question. have to decide whether the data provided in the
statements are sufficient to answer the question. Read
e. If the data given in both statement I and statement II all the statements and give answer:
together are not sufficient to answer the question.
Six persons A, B, C, D, E and F have different heights.
15) The question given below consists of three How many persons are taller than D?
statements numbered I, II and III below them. You
Statement I: B is taller than A and F. D is shorter than E
have to decide whether the data provided in the

Page 263 of 728

Subscribe the Xpress Video Course & Mock Test Package for Bank & Insurance Exams
If there are any suggestions/ errors in our PDFs Feel Free to contact us via this email: admin@exampundit.in
IBPS RRB PO Prelims – Ultra Practice Bundle PDF
Statement II: C is taller than E. F is taller than D but a. The data in statement I alone are sufficient to answer
shorter than B. the question, while the data in statement II alone are not
sufficient to answer the question
Statement III: B is not the tallest. E is not taller than A. F
is shorter than two persons. A is not the shortest. b. The data in statement II alone are sufficient to answer
the question, while the data in statement I alone are not
a. Data given in both statements I and III are together sufficient to answer the question
sufficient to answer.
c. The data either in statement I alone or in statement II
b. Data given in both statements II and III are together alone are sufficient to answer the question
sufficient to answer.
d. The data given in both statements I and II together are
c. Data given in both statements I and II are together not sufficient to answer the question
sufficient to
e. The data in both statements I and II together are
d. Data given in all three statements I, II and III are necessary to answer the question.
together sufficient to answer.
18) The question given below consists of two
e. Data given in all three statements I, II and III are statements numbered I and II below them. You have
together not sufficient to answer. to decide whether the data provided in the statements
17) The question consists of two statements numbered are sufficient to answer the question. Read all the
I and II given below it. You have to decide whether statements and give answer:
the data provided in the statements are sufficient to Six persons Maahi, Maya, Manoj, Manish, Mehak and
answer the question. Minu have different heights. Who is the 3rd shortest
Six boxes, P, Q, R, S, T and U are placed one above person?
another, where the bottommost position is 1 and the Statement I: Two persons are taller than Maahi. Manoj is
position above it is 2 and so on. Which box is placed at not the tallest person. Neither Mehak nor Manoj is
the bottommost position? shorter than Maahi. Manish is shorter than Maya but not
I. R is placed 2nd to the above of P, which is placed just the shortest.
above of U. One box is placed between Q and T. S is Statement II: Maahi is taller than Manish. Mehak is not
placed below T. shorter than Minu. Manoj is not shorter than Maya.
II. Two boxes are placed between Q and P. One box is a. If the data given in statement II alone is sufficient to
placed between R and P. answer the question.

b. If the data given in statement, I alone is sufficient to


answer the question.
Page 264 of 728

Subscribe the Xpress Video Course & Mock Test Package for Bank & Insurance Exams
If there are any suggestions/ errors in our PDFs Feel Free to contact us via this email: admin@exampundit.in
IBPS RRB PO Prelims – Ultra Practice Bundle PDF
c. If the data given in either statement I or statement II e. If the data given in both statement I and statement II
alone is sufficient to answer the question. together are not sufficient to answer the question.

d. If the data given in both statement I and statement II 20) The question given below consists of two
together are sufficient to answer the question. statements numbered I and II given below it. You
have to decide whether the data provided in the
e. If the data given in both statement I and statement II
statements are sufficient to answer the question. Read
together are not sufficient to answer the question.
all the statements and give answer.
19) The question given below consists of two In the given coded language, how is the word ‘numbers’
statements numbered I and II below them. You have coded?
to decide whether the data provided in the statements
are sufficient to answer the question. Read all the Statement I:‘decoding followed by coding’ is coded as
statements and give answer: ‘zx, ar, cp, lt’ and ‘coded numbers and digits’ is coded as
‘mk, kr, dn, st’.
Eight persons S, T, U, V, W, X, Y and Z live on 8 storey
building. The bottommost floor is numbered 1 and the Statement II: ‘coding and decoding numbers’ is coded as
topmost floor is numbered 8. Who lives floor 5? ‘lt, mk, ar, st’ and ‘digits with numbers coded’ is coded
as ‘dn, zx, mk, kr’.
Statement I: S lives three floors above W. Z lives three
floors below W. Y lives on 3rd floor. V lives a. Data given in statement I alone is sufficient to answer
immediately above U. T lives above X. the question.

Statement II: T lives two floors above V. Two persons b. Data given in statement II alone is sufficient to answer
live between V and Y, who lives below floor 4. There is the question.
one floor between Y and U, who doesn’t live on floor 1.
c. Data given in either statement I or statement II alone is
X lives immediately above Z. W lives below S.
sufficient to answer the question.
a. If the data given in statement, I alone is sufficient to
d. Data given in both statement I and statement II
answer the question.
together are sufficient to answer the question.
b. If the data given in statement II alone is sufficient to
e. Data given in both statement I and statement II
answer the question.
together are not sufficient to answer the question.
c. If the data given in either statement I or statement II
alone is sufficient to answer the question. 21) The question given below consists of two
statements numbered I and II given below it. You
d. If the data given in both statement I and statement II have to decide whether the data provided in the
together are sufficient to answer the question. statements are sufficient to answer the question. Read
all the statements and give answer.
Page 265 of 728

Subscribe the Xpress Video Course & Mock Test Package for Bank & Insurance Exams
If there are any suggestions/ errors in our PDFs Feel Free to contact us via this email: admin@exampundit.in
IBPS RRB PO Prelims – Ultra Practice Bundle PDF
Seven persons A, B, C, D, E, F and G sit around a Statement II: Three persons have lesser marks than R.
circular table facing towards the centre. Who sits Neither Q nor U has fewer marks than R. P has more
immediate right of F? marks than S and T.

Statement I: G sits 2nd to the right of F. Two persons sit


between C and E. D sits adjacent to E and to the
a. Data given in statement I alone is sufficient to answer
immediate right of B.
the question.
Statement II: B sits 4th to the right of F. G sits 2nd to the
left of B. D sits to the immediate left of E. C doesn’t sit b. Data given in statement II alone is sufficient to answer
the question.
beside F.

a. Data given in statement I alone is sufficient to answer c. Data given in either statement I or statement II alone is
sufficient to answer the question.
the question.

b. Data given in statement II alone is sufficient to answer d. Data given in both statement I and statement II
the question. together are sufficient to answer the question.

c. Data given in either statement I or statement II alone is e. Data given in both statement I and statement II
sufficient to answer the question. together are not sufficient to answer the question.

d. Data given in both statement I and statement II 23) The question given below consists of three
together are sufficient to answer the question. statements numbered I, II and III below them. You
have to decide whether the data provided in the
e. Data given in both statement I and statement II statements are sufficient to answer the question. Read
together are not sufficient to answer the question. all the statements and give answer:

22) The question given below consists of two Seven persons A, B, C, E, F, G and H sit in a straight
statements numbered I and II given below it. You row facing north direction. Who sits 2nd from the left
have to decide whether the data provided in the end?
statements are sufficient to answer the question. Read
Statement I: C sits 2nd to the left of F. B sits 2nd to the
all the statements and give answer.
left of A, who sits to the immediate left of E. Only A and
Six persons P, Q, R, S, T and U have different marks in a E sit to the right of F.
class test. Who has 2nd highest marks?
Statement II: Four persons sit between G and A, who sits
Statement I: R has more marks than S and T. At least two 2nd from the right end. H sits 3rd to the left of F, who
persons have more marks than R. P has 3rd lowest sits adjacent to A. C sits to the immediate left of B.
marks. Q doesn’t have the highest marks. S doesn’t have
the lowest marks.
Page 266 of 728

Subscribe the Xpress Video Course & Mock Test Package for Bank & Insurance Exams
If there are any suggestions/ errors in our PDFs Feel Free to contact us via this email: admin@exampundit.in
IBPS RRB PO Prelims – Ultra Practice Bundle PDF
Statement III: G sits to the left of B. A sit to the right of a. Data given in both statements II and III together or
C. E sits at one of the extreme ends. only statement I alone is sufficient.

a. Data given in both statements I and II together or b. Data given in any two statements together is sufficient.
statements II and III together is sufficient.
c. Data given in either only statement I alone or
b. Data given in either only statement I alone or both statement II alone is sufficient.
statements II and III together is sufficient.
d. Data given in either only statement II alone or
c. Data given in either only statement I alone or statement III alone is sufficient.
statement II alone is sufficient.
e. Data given in either only statement III alone or both
d. Data given in either only statement II alone or statements I and II together is sufficient.
statement III alone is sufficient.
25) The question given below consists of two
e. Data given in either only statement III alone or both statements numbered I and II given below it. You
statements I and II together is sufficient. have to decide whether the data provided in the
statements are sufficient to answer the question. Read
24) The question given below consists of three all the statements and give answer.
statements numbered I, II and III below them. You
have to decide whether the data provided in the What is the code of ‘good morning’ in the certain code of
statements are sufficient to answer the question. Read language?
all the statements and give answer:
Statement I: In a certain code of language ‘all said good
Seven persons P, Q, R, S, T, U and V live on different morning’ is coded as ‘md vg db op’, ‘all nights are good’
floors of a seven storey building. The bottommost floor is coded as ‘ghdb op wt’
is numbered 1 and the topmost floor is numbered 7. Who
lives on floor 5? Statement II: In a certain code of language, ‘we said
good morning’ is coded as ‘md fgdb vg’
Statement I: P lives two floors above Q. Two persons
live between Q and V, who lives on floor 1. T lives a. Data in statement I alone is sufficient to answer the
immediately above U. question.

Statement II: P lives above R. One person lives between b. Data in either both statement I and statement II alone
R and T. V lives below T. Q lives above V but not sufficient to answer the question
immediately above V. c. Data in statement II alone is sufficient to answer the
Statement III: T lives two floors above V. At least three question.
persons live above T. Two persons live between T and P. d. Data in both statements I and statement II together is
R lives immediately above Q. S lives above R. sufficient to answer the question.
Page 267 of 728

Subscribe the Xpress Video Course & Mock Test Package for Bank & Insurance Exams
If there are any suggestions/ errors in our PDFs Feel Free to contact us via this email: admin@exampundit.in
IBPS RRB PO Prelims – Ultra Practice Bundle PDF
e. Data in statement I and II together is not sufficient to Statement I: Shikha devotes maximum time watching
answer the question. Cricket.

26) The question given below consists of two Statement II: Shikha does not like Football but her
statements numbered I and II given below it. You brother Kunal likes Hockey.
have to decide whether the data provided in the
Statement III: Shikha played Badminton in her school
statements are sufficient to answer the question. Read
and won a Gold medal.
all the statements and give answer.

How many persons are sitting in the row (assuming all a. Data in statement I alone is sufficient to answer the
question.
are facing north)?

Statement I: Sanju sits 5th from the right end of the row. b. Data in either both statement I and statement II or
statement III alone is sufficient to answer the question.
Rahul sits 11th from the left end of the row.

Statement II: Tarun sits 6th from the left end. Rahul sits c. Data in none of the statements alone or together
3rd to the left of the one who is 7th from the right end. sufficient to give the answer.

a. Data in statement I alone is sufficient to answer the d. Data in either statement I alone or statement III alone
question. is sufficient to answer the question.

b. Data in either both statement I and statement II alone e. Data in statement I and II are sufficient to answer the
sufficient to answer the question question, while the data in statement III are not sufficient
to answer the question.
c. Data in statement II alone is sufficient to answer the
question. 28) The question given below consists of three
statements numbered I, II and III given below it. You
d. Data in both statement I and statement II together is have to decide whether the data provided in the
sufficient to answer the question. statements are sufficient to answer the question. Read
all the statements and give answer.
e. Data in statement I and II together not sufficient to
answer the question. Seven persons A, B, C, D, E, F and G have an exam on
seven different days starting from Monday to Sunday in
27) The question given below consists of three the same week. No two persons have exam on same day.
statements numbered I, II and III given below it. You Who among these has exam on Friday?
have to decide whether the data provided in the
statements are sufficient to answer the question. Read Statement I: Only two persons have exam between the
all the statements and give answer. exams of A and B, whose exam is not on Monday. Only
one person has the exam between A and C.
What is the favorite sport of Shikha?

Page 268 of 728

Subscribe the Xpress Video Course & Mock Test Package for Bank & Insurance Exams
If there are any suggestions/ errors in our PDFs Feel Free to contact us via this email: admin@exampundit.in
IBPS RRB PO Prelims – Ultra Practice Bundle PDF
Statement II: F’s exam is neither on Thursday nor on a. The data in statement I alone are sufficient to answer
Saturday but F has exam two days before the exam of G. the question, while the data in statement II alone are not
sufficient to answer the question
Statement III: A’s exam is on Monday and G’s exam is
on Saturday. Only one exam is held between the exams b. The data in statement II alone are sufficient to answer
of C and D. the question, while the data in statement I alone are not
sufficient to answer the question
a. Data in statement I alone is sufficient to answer the
question. c. The data either in statement I alone or in statement II
alone are sufficient to answer the question
b. Data in either both statement I and statement II or
statement III alone is sufficient to answer the question. d. The data given in both statements I and II together are
not sufficient to answer the question
c. Data in none of the statement is alone or together
sufficient to give the answer. e. The data in both statements I and II together are
necessary to answer the question
d. Data in either statement I alone or statement III alone
is sufficient to answer the question. 30) The question given below consists of two
statements numbered I and II given below it. You
e. Data in statement I and III are sufficient to answer the have to decide which of the statements are sufficient
question, while the data in statement II are not sufficient to answer the question. Read all the statements and
to answer the question.
give answer.
29) The question given below consists of two Among the given persons J, K, L, W, X, Y and Z, who is
statements numbered I and II given below it. You the shortest?
have to decide which of the statements are sufficient
to answer the question. Read all the statements and Statement I: Y is taller than W and K. X is shorter than
give answer. only Z. Equal number of persons are taller and shorter
than W.
Six boxes A, B, C, D, E and F are kept one above one
another such that bottommost box is numbered as 1 Statement II: K is not the shortest. L is taller than J.
while the topmost box is numbered as 6. Which box is
a. The data in statement I alone are sufficient to answer
just below box C?
the question, while the data in statement II alone are not
I. Box A is just above box B, which is an odd number sufficient to answer the question
box. Box D is three boxes above box F but not the
b. The data in statement II alone are sufficient to answer
topmost box. Box E is just above box C.
the question, while the data in statement I alone are not
II. Box A is three boxes above box E. Box B is above sufficient to answer the question
box E. Box F is below box E.
Page 269 of 728

Subscribe the Xpress Video Course & Mock Test Package for Bank & Insurance Exams
If there are any suggestions/ errors in our PDFs Feel Free to contact us via this email: admin@exampundit.in
IBPS RRB PO Prelims – Ultra Practice Bundle PDF
c. The data either in statement I alone or in statement II c. Data in either statement II or statement III alone is
alone are sufficient to answer the question sufficient to answer the question.

d. The data given in both statements I and II together are d. Data in either statement II or statement I alone is
not sufficient to answer the question sufficient to answer the question.

e. The data in both statements I and II together are e. Data in both statement I and II together is sufficient to
necessary to answer the question answer the question.

31) The question given below consists of two 32) The question given below consists of two
statements numbered I and II given below it. You statements numbered I and II given below it. You
have to decide which of the statements are sufficient have to decide whether the data provided in the
to answer the question. Read all the statements and statements are sufficient to answer the question. Read
give answer. all the statements and give answer.

The question given below consists of three statements The question given below consists of three statements
numbered I, II and III given below it. You have to decide numbered I, II and III given below it. You have to
whether the data provided in the statements are sufficient decide whether the data provided in the statements
to answer the question. Read all the statements and give are sufficient to answer the question. Read all the
answer. statements and give answer.

In the given coded language, how is the word ‘sufficient’ Seven persons A, B, C, D, E, F and G live on different
coded? floors of a seven storey building, where the bottommost
floor is 1 and the floor above it is 2 and so on. Who live
Statement I: ‘he is efficient’ is coded as ‘ubalbahba’ and on the 2nd floor?
‘he gets sufficient marks’ is coded as ‘pbalbasbatba’.
Statement I: Three persons live between D and C, who
Statement II: ‘this amount is sufficient’ is coded as lives just below F. Two persons live between F and B.
‘ubatbarbabba’ and ‘enough is here’ is coded as
‘ubajbakba’. Statement II: A lives above F. E doesn’t live below D. C
lives above B.
Statement III: ‘all different marks’ is coded as
‘pbawbayba’ and ‘he is good’ is coded as ‘ubalbaoba’. Statement III: E doesn’t live just below C. At least 2
persons live between B and A.
a. Data in either statement III alone or both statement I
and statement II together are sufficient to answer the a. Data in all statement I,statement II and statement III
question. together are sufficient to answer the question.

b. Data in either statement I alone or both statement I and b. Data in either statement I alone or both statement I and
statement II together is sufficient to answer the question. statement II together is sufficient to answer the question.
Page 270 of 728

Subscribe the Xpress Video Course & Mock Test Package for Bank & Insurance Exams
If there are any suggestions/ errors in our PDFs Feel Free to contact us via this email: admin@exampundit.in
IBPS RRB PO Prelims – Ultra Practice Bundle PDF
c. Data in either statement II or statement III alone is e. The data given in both statements I and II together are
sufficient to answer the question. not sufficient to answer the question.

d. Data in either statement II or statement I alone is 34) The question given below consists of two
sufficient to answer the question. statements numbered I and II given below it. You
have to decide whether the data provided in the
e. Data in both statement I and II together is sufficient to
statements are sufficient to answer the question. Read
answer the question.
all the statements and give answer.
33) The question given below consists of two Six persons P, Q, R, S, T and U attend the class in a
statements numbered I and II given below it. You week starting from Monday and ends on Saturday. Who
have to decide whether the data provided in the attends the class immediately after U?
statements are sufficient to answer the question. Read
all the statements and give answer. Statement I: R attends the class before Wednesday.
Number of persons who attend the class before Q is same
Six persons namely, A, B, C, D, E and F are sitting as the number of persons attending after S. T attends the
around a circular table facing the centre. Who is second class after P. P attends the class before U.
to the left of B?
Statement II: Two persons attend the class between S and
Statement I: D sits opposite to E and is the immediate Q. R attends the class before Q. As many persons attends
neighbor of F and A. C is third to the left of A. the class before P as after U, who attends the class on
Statement II: E is immediate neighbor of B and C. F is Thursday.
second to the left of A. a. The data in statement I alone are sufficient to answer
a. The data in statement both I and II together are the question, while the data in statement II alone are not
necessary to give the answer the question. sufficient to answer the question

b. The data in statement I alone are sufficient to answer b. The data in statement II alone are sufficient to answer
the question, while the data in statement II alone are not the question, while the data in statement I alone are not
sufficient to answer the question sufficient to answer the question

c. The data in statement II alone are sufficient to answer c. The data either in statement I alone or in statement II
the question, while the data in statement I alone are not alone are sufficient to answer the question
sufficient to answer the question d. The data given in both statements I and II together are
d. The data either in statement I alone or in statement II not sufficient to answer the question
alone are sufficient to answer the question e. The data in both statements I and II together are
necessary to answer the question

Page 271 of 728

Subscribe the Xpress Video Course & Mock Test Package for Bank & Insurance Exams
If there are any suggestions/ errors in our PDFs Feel Free to contact us via this email: admin@exampundit.in
IBPS RRB PO Prelims – Ultra Practice Bundle PDF
35) The question given below consists of two statements are sufficient to answer the question. Read
statements numbered I and II given below it. You all the statements and give answer.
have to decide whether the data provided in the
On which date and month of the year 2019 did Raj visit
statements are sufficient to answer the question. Read
Mumbai?
all the statements and give answer.
Statement I: Raj remembers that he visited on the 2nd
Seven boxes Red, Green, Blue, Orange, Yellow, White
Friday of a month having 31 days in first half of the year.
and Silver are arranged one above another. Which box is
kept third from the top? Statement II: Raj’ sister remembers that January 1st is
Friday and Raj visited to the Mumbai in a month which
Statement I: Only one box is kept between yellow color
is between two months having 30 days.
box and silver color box. Three boxes are kept between
blue color box and silver color box. Blue box is kept a. The data in statement I alone are sufficient to answer
above the Silver color box. Green color box is kept the question, while the data in statement II alone are not
immediately below the blue color box. sufficient to answer the question
Statement II: Only Red color box is kept above the Blue b. The data in statement II alone are sufficient to answer
color box. No box is kept below the Orange color box. the question, while the data in statement I alone are not
sufficient to answer the question
a. The data in statement I alone are sufficient to answer
the question, while the data in statement II alone are not c. The data either in statement I alone or in statement II
sufficient to answer the question alone are sufficient to answer the question
b. The data in statement II alone are sufficient to answer d. The data given in both statements I and II together are
the question, while the data in statement I alone are not not sufficient to answer the question
sufficient to answer the question
e. The data in both statements I and II together are
c. The data either in statement I alone or in statement II necessary to answer the question
alone are sufficient to answer the question
37) The question given below consists of two
d. The data given in both statements I and II together are statements numbered I and II below them. You have
not sufficient to answer the question to decide whether the data provided in the statements
e. The data in both statements I and II together are are sufficient to answer the question. Read all the
necessary to answer the question statements and give answer:

Six persons P, Q, R, S, T and U are sitting in a row


36) The question given below consists of two
facing north. Who sits to the immediate right of R?
statements numbered I and II given below it. You
have to decide whether the data provided in the

Page 272 of 728

Subscribe the Xpress Video Course & Mock Test Package for Bank & Insurance Exams
If there are any suggestions/ errors in our PDFs Feel Free to contact us via this email: admin@exampundit.in
IBPS RRB PO Prelims – Ultra Practice Bundle PDF
Statement I: P sits second to the left of Q and neither sits b. If the data given in statement II alone is sufficient to
at extreme end. T sits to the immediate left of S, who is answer the question.
not adjacent to P. R and Q are not adjacent to each other.
c. If the data given in either statement I or statement II
Statement II: P sits third to the left of T. R sits second to alone is sufficient to answer the question.
the left of U. Q does not sit at extreme end.
d. If the data given in both statement I and statement II
a. If the data given in statement, I alone is sufficient to are sufficient to answer the question.
answer the question.
e. If the data given in both statement I and statement II
b. If the data given in statement II alone is sufficient to are not sufficient to answer the question.
answer the question.
39) The question given below consists of two
c. If the data given in either statement I or statement II statements numbered I and II given below it. You
alone is sufficient to answer the question. have to decide whether the data provided in the
statements are sufficient to answer the question. Read
d. If the data given in both statement I and statement II
both the statements and give answer.
are sufficient to answer the question.
There are only six members (A, B, C, D, E and F) in the
e. If the data given in both statement I and statement II family. Only married couples have children. How is B
are not sufficient to answer the question. related to F?
38) The question given below consists of two Statement I: C’s only sister’s only brother’s wife’s
statements numbered I and II below them. You have mother-in-law is B. A is the only son of E.
to decide whether the data provided in the statements
are sufficient to answer the question. Read all the Statement II: F’s husband’s mother’s daughter is sister of
statements and give answer: D. E is father of C’s brother.

Six persons A, B, C, D, E and F have different heights. a. The data in statement I alone are sufficient to answer
Who among the following is the tallest? the question, while the data in statement II alone are not
sufficient to answer the question
Statement I: A is taller than at least three persons. B is
shorter than F. Two persons are shorter than D. b. The data in statement II alone are sufficient to answer
the question, while the data in statement I alone are not
Statement II: The number of persons taller than B and sufficient to answer the question
shorter than E is same. C is the shortest. Two persons are
taller than A. F is taller than D. c. The data either in statement I alone or in statement II
alone are sufficient to answer the question
a. If the data given in statement, I alone is sufficient to
answer the question.

Page 273 of 728

Subscribe the Xpress Video Course & Mock Test Package for Bank & Insurance Exams
If there are any suggestions/ errors in our PDFs Feel Free to contact us via this email: admin@exampundit.in
IBPS RRB PO Prelims – Ultra Practice Bundle PDF
d. The data given in both statements I and II together are statements are sufficient to answer the question. Read
not sufficient to answer the question all the statements and give answer.

e. The data in both statements I and II together are The question given below consists of three statements
necessary to answer the question. numbered I, II and III given below it. You have to decide
whether the data provided in the statements are sufficient
40) The question given below consists of two to answer the question. Read all the statements and give
statements numbered I and II given below it. You answer.
have to decide whether the data provided in the
statements are sufficient to answer the question. Read In the given coded language, how is the word ‘pollution’
all the statements and give answer. coded?

6 persons (A, B, C, D, E and F) have different heights. Statement I: ‘pollution in major area’ is coded as ‘ac, mt,
How many persons are shorter than A? bc, rc’ and ‘area in polluted region’ is coded as ‘at, bt,
bc, mt’.
Statement I: Only F is taller than B. E is the shortest
person. C is shorter than A but taller than D. Statement II: ‘now Delhi more polluted’ is coded as ‘ct,
kc, at, kt’ and ‘major region are affected’ is coded as ‘bt,
Statement II: A is taller than B but shorter than F. E is ac, dt, pc’.
shorter than D. C is the not tallest.
Statement III: ‘affected parts of area’ is coded as ‘lc, bc,
a. If the data given in statement, I alone is sufficient to rc, dt’ and ‘pollution more in Delhi’ is coded as ‘kc, rc,
answer the question. kt, mt’.
b. If the data given in statement II alone is sufficient to a. Data in both statement I and statement III are
answer the question. sufficient to answer the question.
c. If the data given in either statement I or statement II b. Data in either both statement I and statement II or both
alone is sufficient to answer the question. statement I and statement III are sufficient to answer the
d. If the data given in both statement I and statement II question.
are sufficient to answer the question. c. Data in only statement II or statement I is sufficient to
e. If the data given in both statement I and statement II answer the question.
are not sufficient to answer the question. d. Data in only statement III is sufficient to answer the
41) The question given below consists of two question.
statements numbered I and II given below it. You e. None of the statements to answer the question.
have to decide whether the data provided in the

Page 274 of 728

Subscribe the Xpress Video Course & Mock Test Package for Bank & Insurance Exams
If there are any suggestions/ errors in our PDFs Feel Free to contact us via this email: admin@exampundit.in
IBPS RRB PO Prelims – Ultra Practice Bundle PDF
42) The question given below consists of two and the topmost floor is numbered 6. Who lives on floor
statements numbered I and II below them. You have 6?
to decide whether the data provided in the statements
Statement I: P lives two floors above U, who lives three
are sufficient to answer the question. Read all the
floors below T. Q lives immediately above R. S doesn’t
statements and give answer:
live on floor 2.
In the given coded language, how is the word “rules”
Statement II: Q lives two floors below S, who lives on
coded?
floor 4. T lives immediately above P. U lives above R.
Statement I: ‘always followed the rules’ is coded as ‘lt,
a. Data given in statement I alone is sufficient to answer
rk, kc, rm’ and ‘rules are followed here’ is coded as ‘pu,
the question.
rk, fr, kc’.
b. Data given in statement II alone is sufficient to answer
Statement II: ‘here are two rules’ is coded as ‘rk, fr, tv,
the question.
pu’ and ‘you always rules are’ is coded as ‘nm, rk, rm,
pu’. c. Data given in either statement I or statement II alone is
sufficient to answer the question.
a. Data given in statement I alone is sufficient to answer
the question. d. Data given in both statement I and statement II
b. Data given in statement II alone is sufficient to answer together are sufficient to answer the question.
the question. e. Data given in both statement I and statement II
c. Data given in either statement I or statement II alone is together are not sufficient to answer the question.
sufficient to answer the question. 44) The questions given below consist of two
d. Data given in both statement I and statement II statements numbered I and II below them. You have
together are sufficient to answer the question. to decide whether the data provided in the statements
are sufficient to answer the question. Read all the
e. Data given in both statement I and statement II statements and give answer:
together are not sufficient to answer the question.
There are 5 persons A, B, C, D and E, who have different
43) The question given below consists of two heights. No two persons have same height. How many
statements numbered I and II given below it. You persons are taller than B?
have to decide whether the data provided in the
Statement I: A is taller than D, who is the 2nd shortest
statements are sufficient to answer the question. Read
person. E is not taller than D. Neither B nor C is taller
all the statements and give answer.
than A.
Six persons P, Q, R, S, T and U live on different floors of
a 6 storey building. The bottommost floor is numbered 1
Page 275 of 728

Subscribe the Xpress Video Course & Mock Test Package for Bank & Insurance Exams
If there are any suggestions/ errors in our PDFs Feel Free to contact us via this email: admin@exampundit.in
IBPS RRB PO Prelims – Ultra Practice Bundle PDF
Statement II: E is shorter than C. A is the tallest person. b. The data in statement II alone are sufficient to answer
B is taller than C. D is shorter than B but not the shortest. the question, while the data in statement I alone are not
sufficient to answer the question
a. If the data given in statement, I alone is sufficient to
answer the question. c. The data either in statement I alone or in statement II
alone are sufficient to answer the question
b. If the data given in statement II alone is sufficient to
answer the question. d. The data given in both statements I and II together are
not sufficient to answer the question
c. If the data given in either statement I or statement II
alone is sufficient to answer the question. e. The data in both statements I and II together are
necessary to answer the question.
d. If the data given in both statement I and statement II
are sufficient to answer the question. 46) The question consists of two statements numbered
I and II given below it. You have to decide whether
e. If the data given in both statement I and statement II the data provided in the statements are sufficient to
are not sufficient to answer the question.
answer the question.
45) The question given below consists of two Six boxes, S, T, U, V, W and X are placed one above
statements numbered I and II given below it. You another such that the bottommost position is 1 and the
have to decide whether the data provided in the position above it is 2 and so on. Which among the
statements are sufficient to answer the question. Read following is placed at the topmost position?
both the statements and give answer.
Statement I: S is placed 2nd to the above of V, which is
What is the direction of village C with respect to F? placed just above T. W, which is not at the topmost
Statement I: Village A is exactly between village C and position, is placed 2nd to the above of X.
village E. Village F is in north-east direction of village E. Statement II: X is placed 2nd to the above of T. Two
Village A is in west direction of village D and village F. boxes are placed between W and V.
C is in north of A.
a. The data in statement I alone are sufficient to answer
Statement II: Village F is exactly between village A and the question, while the data in statement II alone are not
village D. Village B is in south-east direction of village sufficient to answer the question
A. Village E is in south of village C and in west of
village B. b. The data in statement II alone are sufficient to answer
the question, while the data in statement I alone are not
a. The data in statement I alone are sufficient to answer sufficient to answer the question
the question, while the data in statement II alone are not
sufficient to answer the question c. The data either in statement I alone or in statement II
alone are sufficient to answer the question
Page 276 of 728

Subscribe the Xpress Video Course & Mock Test Package for Bank & Insurance Exams
If there are any suggestions/ errors in our PDFs Feel Free to contact us via this email: admin@exampundit.in
IBPS RRB PO Prelims – Ultra Practice Bundle PDF
d. The data given in both statements I and II together are 48) The question given below consists of two
not sufficient to answer the question statements numbered I and II below them. You have
to decide whether the data provided in the statements
e. The data in both statements I and II together are
are sufficient to answer the question. Read all the
necessary to answer the question.
statements and give answer:
47) The question given below consists of two In the given coded language, how is the word ‘loud’
statements numbered I and II given below it. You coded?
have to decide whether the data provided in the
statements are sufficient to answer the question. Read Statement I: In a certain language, ‘does that matter’ is
all the statements and give answer. coded as ‘di ci ni’ and ‘matter loud clear’ is coded as ‘ni
pi qi’.
Six persons A, B, C, D, E and F are sitting around the
circular table facing towards the center. Who sits to the Statement II: In a certain language, ‘loud part apart’ is
immediate right of F? coded as ‘qi ei fi’ and ‘trans apart slice’ is coded as ‘fi gi
mi’
I. B sits opposite to F, who sits to the immediate right of
E. D sits to the immediate right of A. a. If the data given in statement, I alone is sufficient to
answer the question.
II. D sits opposite to E, who sits adjacent to F. B sits to
the immediate left of C. B is not opposite to A. b. If the data given in statement II alone is sufficient to
answer the question.
a. The data in statement I alone are sufficient to answer
the question, while the data in statement II alone are not c. If the data given in either statement I or statement II
sufficient to answer the question alone is sufficient to answer the question.

b. The data in statement II alone are sufficient to answer d. If the data given in both statement I and statement II
the question, while the data in statement I alone are not are sufficient to answer the question.
sufficient to answer the question
e. If the data given in both statement I and statement II
c. The data either in statement I alone or in statement II are not sufficient to answer the question.
alone are sufficient to answer the question
49) The question given below consists of two
d. The data given in both statements I and II together are statements numbered I and II below them. You have
not sufficient to answer the question to decide whether the data provided in the statements
are sufficient to answer the question. Read all the
e. The data in both statements I and II together are
statements and give answer:
necessary to answer the question
There are six persons A, C, D, E, F and G, who have
different heights. Who is the shortest person?
Page 277 of 728

Subscribe the Xpress Video Course & Mock Test Package for Bank & Insurance Exams
If there are any suggestions/ errors in our PDFs Feel Free to contact us via this email: admin@exampundit.in
IBPS RRB PO Prelims – Ultra Practice Bundle PDF
Statement I: D is taller than E and F. A is shorter than G. Six persons U, V, W, X, Y and Z are sitting in a row
C is 3rd tallest. facing north. Who sits to the immediate right of X?

Statement II: D is 2nd tallest. C is taller than E and F but Statement I: Two persons sit between V and Z. Y sits to
not the tallest. Neither E nor F is the shortest. G is taller the immediate right of Z. V does not sit at any end. X sits
than A. adjacent to Z but not at any end.

a. If the data given in statement I alone is sufficient to Statement II: U sits second to the left of Z. Only two
answer the question. persons sit to the left of U.

b. If the data given in statement II alone is sufficient to a. If the data given in statement, I alone is sufficient to
answer the question. answer the question.

c. If the data given in either statement I or statement II b. If the data given in statement II alone is sufficient to
alone is sufficient to answer the question. answer the question.

d. If the data given in both statement I and statement II c. If the data given in either statement I or statement II
are sufficient to answer the question. alone is sufficient to answer the question.

e. If the data given in both statement I and statement II d. If the data given in both statement I and statement II
are not sufficient to answer the question. are sufficient to answer the question.

50) The question given below consists of two e. If the data given in both statement I and statement II
statements numbered I and II below them. You have are not sufficient to answer the question.
to decide whether the data provided in the statements
are sufficient to answer the question. Read all the
statements and give answer:

14). Reasoning Data Sufficiency - Solution with Explanation

Answer: A scored more than only C. Two students scored more


1. a than G, who scored more than A and B.
Solution 1 ? > ? > G > ?> ? > A > C
From statement I and II: E scored less than D, who scored more than B.

Page 278 of 728

Subscribe the Xpress Video Course & Mock Test Package for Bank & Insurance Exams
If there are any suggestions/ errors in our PDFs Feel Free to contact us via this email: admin@exampundit.in
IBPS RRB PO Prelims – Ultra Practice Bundle PDF
So, no proper information is given. From I and III,
From statement II and III: Karan’s son remembers that he went after 20th July 2019
E scored less than D, who scored more than B. Rohan, a friend of Karan, remembers that he went to
C scored less than D, who does not score the highest. Delhi either in 7th or 8th month of year 2019.
So, no proper information is given. So, from I and III Karan either went in July or August.
From both I and III: Combining all statements:
A scored more than only C. Two students scored more Karan can either went in July or August hence Question
than G, who scored more than A and B. cannot be answered even with the information in all three
C scored less than D, who does not score the highest. statements.
? > ? > G > ?> ? > A > C Answer.
So, no proper information is given. 3. d
From I, II and III: Solution: 3
F > D > G > E/B > B/E > A > C From statement I and II:
So, from I, II and III It is clear that D, scored the 2nd R is 7th from the left end. A is 17th from the right end.
highest. R is 6 places away from V, who is 20th from the left end.
Number of students between R and A cannot be found
Answer: without the total number of students
2. e So, from I and II we cannot get the answer.
Solution 2 From statement II and III:
From statement I and II: R is 6 places away from V, who is 20th from the left end.
Karan’s son remembers that he went after 20th July. There are total 40 students in the class.
Karan remembers that he went before 10th August 2019. So, no proper information is given as nothing mentioned
So, from I and II Karan either went in July or August. about A
From statement II and III: From I and III
Rohan, friend of Karan remembers that he went Delhi in R is 7th from the left end. A is 17th from the right end.
7th or 8th month of year 2019 There are total 40 students in the class.
Karan remembers that he went before 10th August 2019.
So, from II and III Karan either went in July or August.
Page 279 of 728

Subscribe the Xpress Video Course & Mock Test Package for Bank & Insurance Exams
If there are any suggestions/ errors in our PDFs Feel Free to contact us via this email: admin@exampundit.in
IBPS RRB PO Prelims – Ultra Practice Bundle PDF

Hence, Only I and III are sufficient


Answer Hence, Only I and II are sufficient

4. b Answer

Solution: 4 5. c

From I and II: Solution: 5

A is father of D who is a husband of E. From statement I and II


P sits immediate right of R who faces S.
P faces Q who is an immediate neighbor of S.
Hence, we don’t know who sits second to the left of P.

C is daughter of E, who is daughter in law of A

From statement II and III


II. P faces Q who is an immediate neighbor of S.
III. S is not an immediate neighbor of P. R sits
Combining I and II immediate right of S.

Page 280 of 728

Subscribe the Xpress Video Course & Mock Test Package for Bank & Insurance Exams
If there are any suggestions/ errors in our PDFs Feel Free to contact us via this email: admin@exampundit.in
IBPS RRB PO Prelims – Ultra Practice Bundle PDF
7. d
Solution: 7
From I and II

Hence, Only II and III are sufficient Hence, I and II are not sufficient

Answer From II and III

6. a
Solution 6
From Statement I and II
I. S is taller than only one person.
From I and III
II. Q is not the tallest.
>>> S >
From Statement I and III
S is taller than only one person.
Hence, Trunk is coded as ck
>>> S >
Therefore, Only I and III are sufficient
R is taller than T but not taller than P.
Answer
P>R>T
8. e
From Statement II and III
Solution 8
Q is not the tallest.
From I and II
R is taller than T but not taller than P.
I. A is in North of C who is south-west of B.
P>R>T
II. D is in East of C.
Combining all the statements,
P >R > T > S > Q
Hence, all the statements are required

Answer
Page 281 of 728

Subscribe the Xpress Video Course & Mock Test Package for Bank & Insurance Exams
If there are any suggestions/ errors in our PDFs Feel Free to contact us via this email: admin@exampundit.in
IBPS RRB PO Prelims – Ultra Practice Bundle PDF
T is placed above S, but not adjacent to R.
From II and III Positions Boxes Boxes Boxes Boxes
D is in East of C.
6 T P
A is in North-west of D who is south of B.
5 T P S R

4 S R

3 P T

2 P T R S
From I and III
A is in North of C who is south-west of B 1 R S

A is in North-west of D who is south of B.


Hence, I and II are not sufficient
From II and III
T is placed above S, but not adjacent to R.
Two boxes are placed between P and Q. U is placed
somewhere below Q.
As position of B is not defined, hence, question cannot Positions Boxes Boxes Boxes Boxes Boxes
be answered even with the information in all three
6 P Q
statements.
5 P Q

4 Q
Answer
9. a 3 Q P
Solution 9
2 Q P
From I and II
Two boxes are placed between S and R, which is placed 1 P

immediately below of P.
Page 282 of 728

Subscribe the Xpress Video Course & Mock Test Package for Bank & Insurance Exams
If there are any suggestions/ errors in our PDFs Feel Free to contact us via this email: admin@exampundit.in
IBPS RRB PO Prelims – Ultra Practice Bundle PDF
Hence, II and III are not sufficient 4 S
From I and III
3 U
Two boxes are placed between P and Q. U is placed
somewhere below Q. 2 P
Two boxes are placed between S and R, which is placed
1 R
immediately below of P.

Positions Boxes Boxes Boxes


Answer 10
6 P T Q 10. c
Solution
5 R Q S
From I and II
4 T S U/T R is 6th from the right end of the row.

3 Q U P R sits 5th to the right of J.

2 S P R

1 U R U/T Hence, I and II are not sufficient


From II and III

Combining all the given statements R sits 5th to the right of J.

Two boxes are placed between S and R, which is placed J is 6th from the left end of the row and also 7th to the

immediately below of P. left of K.

T is placed above S, but not adjacent to R.


Two boxes are placed between P and Q. U is placed
somewhere below Q.
Hence, one person sits between R and K.
Positions Boxes Therefore, only II and III are sufficient

6 T Answer
11. b
5 Q
Solution 11

Page 283 of 728

Subscribe the Xpress Video Course & Mock Test Package for Bank & Insurance Exams
If there are any suggestions/ errors in our PDFs Feel Free to contact us via this email: admin@exampundit.in
IBPS RRB PO Prelims – Ultra Practice Bundle PDF
From statement I alone, as Two persons are taller than J. From statement II alone, insufficient information is
L is not the tallest person. Neither N nor L is shorter than given.
J. M is shorter than K but not the shortest, so So, statement II alone is not sufficient.
N>L>J>K>M>O Answer
Clearly, J is the 4th shortest person. 13. d
From statement II alone, as J is taller than M. L is not Solution 13
shorter than K. N is not shorter than O, no arrangement From statement I alone, A1 is sitting 19m west to A2,
can be formed. who is sitting south west to A3. A5 is sitting south east to
Data given in statement I alone is sufficient to answer the A3.
question. But the distance between A3 and A6 cannot be
Answer: determined.
12. a So, Statement I alone is not sufficient.
Solution 12 From statement II alone, A6 is sitting 22m south to A4,
From statement I alone, Abhi is sitting 2nd to the right of who is sitting 6m west to A5. But the distance between
Diana, who is sitting 2nd to the right of Bob, who is A3 and A6 cannot be determined.
sitting adjacent to Geet. Henry is not sitting adjacent to So, statement II alone is not sufficient.
Abhi. On combining both statements we will not get the answer
The final arrangement is as follows: of the given question.
Answer
14. e
Solution 14
From statement I alone, as A and C are shorter than D. F
is not taller than E. B is taller than E but not the tallest,
so the given information is not sufficient
From statement II alone, as B is taller than A and the 2nd
tallest person. F is shorter than D. C is not the tallest
Bob is sitting opposite to Ella.
person. E is shorter than A and C. E is not the shortest,
So, Statement I alone is sufficient.
so
Page 284 of 728

Subscribe the Xpress Video Course & Mock Test Package for Bank & Insurance Exams
If there are any suggestions/ errors in our PDFs Feel Free to contact us via this email: admin@exampundit.in
IBPS RRB PO Prelims – Ultra Practice Bundle PDF
D > B > A/C > C/A > E > F On combining both statement I and statement III,
Either C or A is 3rd shortest person. C>B>F>A>E>D
Data given in both statements I and statement II are not Clearly, five persons are taller than D.
sufficient to answer the question. Data given in both statements I and III are together
Answer sufficient to answer.
15. e Answer
Solution 15 17. a
From statement I alone, the word ‘come’ is coded as Solution 17
either ‘bk’ or ‘rk’ or ‘sk’. From statement I alone, R is placed 2 nd to the above of P,
From statement II alone, the word ‘come’ is coded as which is placed just above of U. One box is placed
either ‘bk’ or ‘sk’. between Q and T. S is placed below T.
From statement III alone, the word ‘come’ is coded as The final arrangement is as follows:
either ‘bk’ or ‘sk’. Position Boxes
On combining any of the two statements, the word
6 Q/T
‘come’ is coded as either ‘bk’ or ‘sk’
Even on combining all the three statements, the word 5 R
‘come’ is coded as either ‘bk’ or ‘sk’
4 T/Q
Data given in all three statements I, II and III are
together not sufficient to answer. 3 P
Answer
2 U
16. a
Solution 16 1 S

From statement I alone, no proper arrangement is S is placed at the bottommost position.


possible. So, statement I alone is sufficient.
From statement II alone, no proper arrangement is From statement II alone, insufficient information is
possible. given.
From statement III alone, no proper arrangement is So, statement II alone is not sufficient.
possible. Answer
Page 285 of 728

Subscribe the Xpress Video Course & Mock Test Package for Bank & Insurance Exams
If there are any suggestions/ errors in our PDFs Feel Free to contact us via this email: admin@exampundit.in
IBPS RRB PO Prelims – Ultra Practice Bundle PDF
18. b 2 Z X
Solution 18
1 X Z
From statement I alone, as two persons are taller than
Maahi. Manoj is not the tallest person. Neither Mehak Either W or U lives on floor 5.
nor Manoj is shorter than Maahi. Manish is shorter than From statement II alone, as T lives two floors above V.
Maya but not the shortest, so Two persons live between V and Y, who lives below
Mehak>Manoj>Maahi> Maya > Manish >Minu floor 4. There is one floor between Y and U, who doesn’t
Clearly, Maya is the 3rd shortest person. live on floor 1. X lives immediately above Z. W lives
From statement II alone, as Maahi is taller than Manish. below S, so
Manoj is not shorter than Maya. Mehak is not shorter 8 X T
than Minu, no arrangement can be formed.
7 Z S
Data given in statement I alone is sufficient to answer the
question. 6 T V
Answer
5 S U
19. d
Solution 19 4 V W
From statement I, as S lives three floors above W. Z lives
3 U Y
three floor below W. Y lives on 3rd floor. V lives
immediately above U. T lives above X, so 2 W X

8 S T 1 Y Z

7 V S
Either S or U lives on floor 5.
6 U V
On combining statement I and statement II, U lives on
5 W U floor 5.
Answer
4 T W
20. b
3 Y Y Solution 20

Page 286 of 728

Subscribe the Xpress Video Course & Mock Test Package for Bank & Insurance Exams
If there are any suggestions/ errors in our PDFs Feel Free to contact us via this email: admin@exampundit.in
IBPS RRB PO Prelims – Ultra Practice Bundle PDF
From statement I alone, ‘numbers’ is coded as either Data given in statement II alone is sufficient to answer
‘mk’ or ‘kr’ or ‘dn’ or ‘st’. the question.
From statement II alone, ‘numbers’ is coded as ‘mk’. Answer
Data given in statement II alone is sufficient to answer 22. a
the question. Solution 22
Answer From statement I alone, as R has more marks than S and
21. b T. At least two persons have more marks than R. P has
Solution 21 3rd lowest marks. Q doesn’t have the highest marks. S
From statement I alone, as G sits 2nd to the right of F. doesn’t have the lowest marks, so
Two persons sit between C and E. D sits adjacent to E U>Q>R>P>S>T
and immediate right of B, so Clearly, Q has 2nd highest marks.
From statement II alone, as three persons have lesser
marks than R. Neither Q nor U has less marks than R. P
has more marks than S and T, so
U/Q > Q/U > R > P > S/T > T/S
Either Q or U has 2nd highest marks.
Data given in statement I alone is sufficient to answer the
Either A or C sits to the immediate right of F. question.
th
From statement II alone, as B sits 4 to the right of F. G Answer
nd
sits 2 to the left of B. D sits immediate left of E. C 23. a
doesn’t sit beside F, so Solution 23

From statement I, as C sits 2nd to the left of F. B sits


2nd to the left of A, who sits immediate left of E. Only A
and E sit to the right of F, so

G/H H/G C B F A E

A sits to the immediate right of F.

Page 287 of 728

Subscribe the Xpress Video Course & Mock Test Package for Bank & Insurance Exams
If there are any suggestions/ errors in our PDFs Feel Free to contact us via this email: admin@exampundit.in
IBPS RRB PO Prelims – Ultra Practice Bundle PDF
From statement II alone, Four persons sit between G and 6 P
nd rd
A, who sits 2 from the right end. H sits 3 to the left of
5 R/S
F, who sits adjacent to A. C sits immediate left of B, so

G C B H E A F 4 Q

Or 3 T

G H C B F A E 2 U
From statement III alone, as G sits to the left of B. A sits
1 V
to the right of C. E sits at one of the extreme ends, so no
From statement II alone, as P lives above R. One person
proper arrangement is possible.
lives between R and T. V lives below T. Q lives above V
On combining both statement I and statement II,
but not immediately above, so no proper arrangement is
G H C B F A E
possible.
nd
H sits 2 from the left end. From statement III alone, as T lives two floors above V.
On combining statements II and III, At least three persons lives above T. Two persons lives
G H C B F A E between T and P. R lives immediately above Q. S lives

Data given in both statements I and II together is above R, so

sufficient. 7 S S
And Data given in both statements II and III together is
6 R P
sufficient.
Answer 5 Q R

24. b 4 T Q
Solution 24
From statement I, as P lives two floors above Q. Two 3 U T

persons lives between Q and V, who lives on floor 1. T 2 V U


lives immediately above U, so
1 P V
7 S/R
On combining both statement I and statement II,

Page 288 of 728

Subscribe the Xpress Video Course & Mock Test Package for Bank & Insurance Exams
If there are any suggestions/ errors in our PDFs Feel Free to contact us via this email: admin@exampundit.in
IBPS RRB PO Prelims – Ultra Practice Bundle PDF
7 S 5 R

6 P 4 Q

5 R 3 T

4 Q 2 U

3 T 1 V

2 U Data given in both statements I and III together or both


statements I and II together or both II and III is
1 V
sufficient.
Clearly, P lives on floor 5.
On combining statement, I and statement III, Answer
7 S 25. e
Solution 25
6 P
From statement I:
5 R ‘all said good morning’ is coded as ‘md vg db op’, ‘all
nights are good’ is coded as ‘ghdb op wt’
4 Q
So, from I we do not get the exact code of any word such
3 T as, said/morning can be coded as vg/md and all/good can

2 U be coded as op/db.
From statement II:
1 V
‘we said good morning’ is coded as ‘md fgdb vg’
Clearly, P lives on floor 5. No proper information is given.
On combining statements II and III, From both I and II:

7 S By merging both the statement I and II we get the code


of only one word i.e. Good – db.
6 P
Answer
26. d
Page 289 of 728

Subscribe the Xpress Video Course & Mock Test Package for Bank & Insurance Exams
If there are any suggestions/ errors in our PDFs Feel Free to contact us via this email: admin@exampundit.in
IBPS RRB PO Prelims – Ultra Practice Bundle PDF
Solution 26 28. e
From statement I: No proper information is given. Solution 28
From statement II: No proper information is given. From statement I: Only two persons have exam between
From I and II together, it is clear that there are 20 the exams of A and B, whose exam is not on Monday.
persons sitting in the row. Only one person has the exam between A and C.
No proper information is given.
From statement II: F’s exam is neither on Thursday nor
on Saturday but F has exam on two days before the exam
Answer of G.
27. c No proper information is given.
Solution 27 From statement III: A’s exam is on Monday and G’s
From I: Shikha devotes maximum time watching exam is on Saturday. Only one exam is held between the
Cricket. exams of C and D.
It is clear from this statement that Shikha devotes
Monday Person
maximum time watching Cricket but this does not mean
that her favorite sport is Cricket. Monday A
From II: Shikha does not like Football but her brother Tuesday
Kunal likes Hockey.
No proper information is given. Wednesday

From III: Shikha played Badminton in her school and Thursday


won a Gold medal.
Friday
It is clear from this statement that she played Badminton
in her and won a Gold medal but this does not mean that Saturday G
Badminton is her favorite sport.
Sunday
Hence, favorite sports of Shikha cannot be determined as
From I and III:
information provided by these statements is insufficient
to give the answer. Monday Person
Answer:

Page 290 of 728

Subscribe the Xpress Video Course & Mock Test Package for Bank & Insurance Exams
If there are any suggestions/ errors in our PDFs Feel Free to contact us via this email: admin@exampundit.in
IBPS RRB PO Prelims – Ultra Practice Bundle PDF
Monday A 1 F

Tuesday So, box F is just below box C.


So, statement I alone is sufficient.
Wednesday C
From statement II alone, Data is insufficient.
Thursday B So, statement II alone is not sufficient.

Friday D
Answer:
Saturday G 30. e
Solution 30
Sunday
From statement I and II alone, we cannot determine the
So, from I and III we get that D has the exam on Friday.
shortest person.
Answer:
By combining statement, I and II,
29. a
Now, Y is taller than W and K. X is shorter than only Z.
Solution 29
Equal number of persons are taller and shorter than W. K
From statement I alone,
is not the shortest. L is taller than J. So, we have,
Box A is just above box B, which is an odd number box.
Z > X > Y > W > K/L > L/K > J
Box D is three boxes above box F but not the topmost
So, J is the shortest.
box. Box E is just above box C. We have,
Answer:
Box number Boxes 31. e

6 A Solution 31
From statement I:
5 B

4 D

3 E
From statement II:
2 C

Page 291 of 728

Subscribe the Xpress Video Course & Mock Test Package for Bank & Insurance Exams
If there are any suggestions/ errors in our PDFs Feel Free to contact us via this email: admin@exampundit.in
IBPS RRB PO Prelims – Ultra Practice Bundle PDF
From statement III: E lives on the 2nd floor.
Answer:
33. a
Solution 33
We cannot get the code of sufficient from any of the From statement I: D sits opposite to E and is the
statements alone. immediate neighbor of F and A. C is third to the left of
On combining both statement I and statement II, A.
‘sufficient’ is coded as ‘tba’ Case I: When A sits to the immediate left of D.
Answer:
32. a
Solution 32
From statement I alone, we get no proper arrangement.
From statement II alone, we cannot get any proper
arrangement.
From statement III alone, data is not sufficient.
On combining statement, I, II and statement III, we get, Case II: When A sits to the immediate right of D.
Floors Persons

7 A

6 F

5 C

4 G

3 B
From statement II: E is immediate neighbor of B and C.
2 E
F is second to the left of A.
1 D So, no proper information is given.
From statements I and II:
Page 292 of 728

Subscribe the Xpress Video Course & Mock Test Package for Bank & Insurance Exams
If there are any suggestions/ errors in our PDFs Feel Free to contact us via this email: admin@exampundit.in
IBPS RRB PO Prelims – Ultra Practice Bundle PDF
Thursday U

Friday S/Q

Saturday

After merging I and II, answer can’t be obtained from


merging both I and II.
Answer:
So, D sits to the 2nd to the left of B.
35. e
Answer:
Solution 35
34. d
From statement I:
Solution 34
Only one box is kept between yellow color box and
From statement I:
silver color box. Three boxes are kept between blue color
R attends the class before Wednesday. Number of
box and silver color box. Blue box is kept above the
persons who attend the class before Q is same as the
Silver color box. Green color box is kept immediately
number of persons attend after S. T attends the class after
below the blue color box.
P. P attends the class before U.
Case I: When Yellow color box is placed above the
So, no proper information is given which means this
Silver color box.
statement is invalid.
From statement II: Blue
Two persons attend the class between S and Q. R attends
Green
the class before Q. As many persons attend the class
before P as after U, who attends the class on Thursday. Yellow

Day Person

Monday Silver

Tuesday S/Q Case II: When Yellow color box is placed below the
Silver color box.
Wednesday P
Blue

Page 293 of 728

Subscribe the Xpress Video Course & Mock Test Package for Bank & Insurance Exams
If there are any suggestions/ errors in our PDFs Feel Free to contact us via this email: admin@exampundit.in
IBPS RRB PO Prelims – Ultra Practice Bundle PDF
Green So, after merging both the statement we get, that Green
color box is placed 3rd from the top.
Answer:
36. e
Solution 36
Silver
From statement I:
Raj remembers that he visited on the 2nd Friday of a
month having 31 days in first half of the year.
Yellow
So, from the above information it is clear that Raj visited
So, there is no proper information is given regarding
on 2nd Friday of January or March or May. So, we cannot
which box is placed third from the top.
answer the question using this statement.
From statement II:
From statement II:
Only Red color box is kept above the Blue color box. No
Raj’ sister remembers that January 1 st is Friday and Raj
box is kept below the Orange color box.
visited to the Mumbai in a month which is between two
No proper information is given.
months having 30 days.
From both I and II:
No proper information is given.
Box From both I and II:
So, out of January, March and May, May is the only
Red
month in between the two months having 30 days (April
Blue and June)

Green So, Raj must have visited in the month of May and it is
also given that January 1 st is Friday, so the date can also
Yellow
be known.
White
Answer:
Silver
37. a
Orange Solution 37

Page 294 of 728

Subscribe the Xpress Video Course & Mock Test Package for Bank & Insurance Exams
If there are any suggestions/ errors in our PDFs Feel Free to contact us via this email: admin@exampundit.in
IBPS RRB PO Prelims – Ultra Practice Bundle PDF
From statement I, P sits second to the left of Q and C’s only sister’s only brother’s wife’s mother-in-law is
neither sits at extreme end. T sits immediate left of S, B, so, B is mother of C. A is only son of E, so, C must be
who is not adjacent to P. R and Q are not adjacent to sister of A. F’s husband’s mother’s daughter is sister of
each other. So, we have, D, so, F is wife of A. C and D are sisters of A.

R P U Q T S Then final relation is shown below:

So, P sits to the immediate right of R.


From statement II alone, we cannot determine the person
sitting immediate right of R.

B is mother-in-law of F.
Answer:
Therefore, we get the answer from both statements I and
38. b
II together.
Solution 38
Answer:
From statement I, A is taller than at least three persons. B
40. a
is shorter than F. Two persons are shorter than D.
Solution 40
So, we cannot determine the tallest person.
From statement I alone, as only F is taller than B, who is
From statement II, the number of persons taller than B
taller than A and D. E is the shortest person. C is shorter
and shorter than E is same. C is the shortest. Two
than A but taller than D, so
persons are taller than A. F is taller than D. We have,
F>B>A>C>D>E
F > B/E > A > D > E/B > C
(Tallest) (Shortest)
Therefore, F is the tallest.
Clearly, 3 persons are shorter than A.
Answer:
From statement II alone, as A is taller than B but shorter
39. e
than F. E is shorter than D. C is the not tallest, so proper
Solution 39
arrangement cannot be formed.
From statement I and statement II alone, we cannot
Data given in statement I alone is sufficient to answer the
answer the question. On combining both statements, we
question.
get,
Answer:
41. b
Page 295 of 728

Subscribe the Xpress Video Course & Mock Test Package for Bank & Insurance Exams
If there are any suggestions/ errors in our PDFs Feel Free to contact us via this email: admin@exampundit.in
IBPS RRB PO Prelims – Ultra Practice Bundle PDF
Solution 41 From statement I alone, as P lives two floors above U,
From statement I alone, ‘pollution’ is coded as either ‘rc’ who lives three floors below T. Q lives immediately
or ‘ac’. above R. S doesn’t live on floor 2, so
From statement II alone, the code for ‘pollution’ can’t be 6 T
known.
5 P
From statement III alone, ‘pollution’ is coded as either
‘kc’ or ‘rc’ or ‘kt’ or ‘mt’. 4 S
On combining statement, I and statement II, ‘pollution’
3 U
is be coded as ‘rc’.
On combining statement, I and statement III, ‘pollution’ 2 Q

is be coded as ‘rc’. 1 R
Both statement I and statement II or both statement I and
T lives on floor 6.
statement III is sufficient to answer the question.
From statement II alone, as Q lives two floors below S,
Answer:
who lives on floor 4. T lives immediately above P. U
42. d
lives above R, so
Solution 42
6 T
From statement I alone, the word “rules” is coded as
either “rk” or “kc” 5 P
From statement II alone, the word “rules” is coded as
4 S
either “rk” or “pu”.
3 U
On combining statement, I and statement II, the word
“rules” is coded as “rk”. 2 Q
Data given in both statement I and statement II together
1 R
are sufficient to answer the question.
T lives on floor 6.
Answer:
Data given in either statement I or statement II alone is
43. c
sufficient to answer the question.
Solution 43
Answer:

Page 296 of 728

Subscribe the Xpress Video Course & Mock Test Package for Bank & Insurance Exams
If there are any suggestions/ errors in our PDFs Feel Free to contact us via this email: admin@exampundit.in
IBPS RRB PO Prelims – Ultra Practice Bundle PDF
44. b Answer:
Solution 44 46. a
From statement I alone, as A is taller than D, who is the Solution 46
2nd shortest person. E is not taller than D. Neither B nor From statement I alone, S is placed 2nd to the above of V,
C is taller than A, which is placed just above T. W, which is not at the
so they can be arranged asA> B/C > C/B > D > E topmost position is placed 2nd to the above of X.
Either one or two persons are taller than B. The final arrangement is as follows:
From statement II alone, E is shorter than C. A is the Positions Boxes
tallest person. B is taller than C, D is shorter than B but
6 U
not the shortest.
So, A > B > C/D > D/C > E 5 W
Only one person is taller than B.
4 S
Statement II alone is sufficient to answer the question.
Answer: 3 X

45. a 2 V
Solution 45
1 T
From statement I alone,
we get, Only statement I is sufficient to answer the question.
U is placed at the topmost position.
No information is given in statement-II.
Answer:
47. c
Solution 47
Now, from statement I alone,
So, village C is in north-west direction of village F. As, B sits opposite to F, who sits to the immediate right
From statement II alone, we cannot determine the of E. D sits to the immediate right of A. So, we have,
position of C with respect to F.
Therefore, we get the answer from statement I alone.

Page 297 of 728

Subscribe the Xpress Video Course & Mock Test Package for Bank & Insurance Exams
If there are any suggestions/ errors in our PDFs Feel Free to contact us via this email: admin@exampundit.in
IBPS RRB PO Prelims – Ultra Practice Bundle PDF
48. d
Solution 48
From statement I alone,
The code of ‘loud’ is either ‘qi’ or ‘pi’
From statement II alone,
The code of ‘loud’ is either ‘qi’ or ‘ei’
Therefore, by combining statements I and II, we get,
The code of ‘loud’ is ‘qi’.
Answer:

So, A sits immediate right of F. 49. b

From statement II alone, Solution 49


From statement I alone, we cannot determine the shortest
As, D sits opposite to E, who sits adjacent to F. B sits to
person.
the immediate left of C. B is not opposite to A. So, we
From statement II, D is 2nd tallest. C is taller than E and
have,
F but not the tallest. Neither E nor F is the shortest. G is
taller than A. we get,
G > D > C > E/F > F/E > A
Therefore, A is the shortest.
Answer:
50. a
Solution 50
From statement I, Two persons sit between V and Z. Y
sits to the immediate right of Z. V does not sit at any end.
X sits adjacent to Z but not at any end. So, we have,
So, A sits to the immediate right of F.
So, Z sits to the immediate right of X.
Therefore, statement I and statement II alone are
sufficient.
Answer:

Page 298 of 728

Subscribe the Xpress Video Course & Mock Test Package for Bank & Insurance Exams
If there are any suggestions/ errors in our PDFs Feel Free to contact us via this email: admin@exampundit.in
IBPS RRB PO Prelims – Ultra Practice Bundle PDF
From statement II, we cannot determine the person sitting to the immediate right of X.

15). Days Based Puzzle


Directions (1-4): Read the following information carefully and answer the questions given below it:

Seven persons bought different things one after the other in a week starting from Monday to Sunday. Only one person
bought things between P and R. Q and R didn’t buy things on consecutive days. Only one person bought things
between T and U. Two persons bought things between P and Q. U bought things immediately before V. Number of
persons bought things before S is as same as the number of persons bought things after Q. Not more than two persons
bought things between S and T.
1). S bought things on which among the following 3). In a certain way, S is related to T, U is related to Q
day? and in the same way, P is related to ____ of the
a) Monday following?
b) Tuesday a) R
c) Wednesday b) The one who bought things on Thursday
d) Thursday c) V
e) None of the above d) The one who bought things on Sunday
2). How many persons bought things between T and e) None of the above
the one, who bought things immediately before Q? 4). Which among the following combination is
a) One definitely TRUE?
b) Two a) S-Sunday
c) Three b) T-Monday
d) Four c) P-Wednesday
e) None of the above d) U-Thursday
e) None of the above
Directions (5-7): Read the following information carefully and answer the questions given below it

Page 299 of 728

Subscribe the Xpress Video Course & Mock Test Package for Bank & Insurance Exams
If there are any suggestions/ errors in our PDFs Feel Free to contact us via this email: admin@exampundit.in
IBPS RRB PO Prelims – Ultra Practice Bundle PDF
Seven events held in a week starting from Monday to Sunday. Three events are held between Event A and Event C
such that Event A is held before Event C. Event E is held before Event D and more than three events are held in
between them. Event B is held exactly between Event A and Event C. Event F and Event C are not held on
consecutive days. Event G and Event A are not held on consecutive days.
5). Which among the following event is held d) Four
immediately after Event G? e) None
a) Event E 7). Which among the following events held on
b) Event C consecutive days?
c) Event F i. Event C and the event held on Thursday
d) Event B ii. Event A and Event F
e) None of the above iii. Event B and the event held on Saturday
6). If all events are held in English alphabetical order a) Only I
from Monday to Sunday, how many events remain on b) Both I and II
the same day as before? c) Only II
a) One d) Both II and III
b) Two e) None of the above
c) Three
Directions (8-12): Read the following information carefully and answer the questions given below it:
Six professors attended invigilation duty on different days of the week starting from Monday to Saturday. Each of
them belongs to different departments such that no two persons belong to the same department.
P attended the invigilation duty immediately before the one, who belongs to the CSE department. Q attended the
invigilation duty before T. R, who doesn’t belong to the Civil department, attended the invigilation duty three days
before S. The one, who belongs to the Civil department attended invigilation duty second to before the one, who
belongs to the EEE department but neither of them attended on Monday or Saturday. The number of persons who
attended the invigilation duty after S is as same as the number of persons who attended the invigilation duty before U.
Only one person attended the invigilation duty between U and the one, who belongs to the ECE department. The one,
who belongs to the IT department, attended the invigilation duty immediately before the one, who belongs to the
Mechanical department.
Page 300 of 728

Subscribe the Xpress Video Course & Mock Test Package for Bank & Insurance Exams
If there are any suggestions/ errors in our PDFs Feel Free to contact us via this email: admin@exampundit.in
IBPS RRB PO Prelims – Ultra Practice Bundle PDF
8). Who among the following belongs to the CSE b) The one who belongs to the EEE department
department? c) The one who belongs to the IT department
a) R d) The one who belongs to the CSE department
b) Q e) None of the above
c) T 11). U and the one, who belongs to the Mechanical
d) U department, attended the invigilation duty on ____
e) None of the above and ___ days respectively?
9). How many persons attended the invigilation duty a) Monday and Saturday
between Q and the, one who belongs to the ECE b) Friday and Wednesday
department? c) Thursday and Tuesday
a) One d) Wednesday and Saturday
b) Two e) None of the above
c) Three 12). Which among the following combination is true?
d) Four a) U-Civil-Wednesday
e) None of the above b) S-Mechanical-Saturday
10). Who among the following attended the c) R-ECE-Wednesday
invigilation duty on Friday? d) Q-IT-Friday
a) The one who belongs to the ECE department e) None of the above
Directions (13-17): Read the following information carefully and answer the questions given below it:
Seven packages were arrived at an Amazon facility-Mumbai on different days of the week starting from Monday to
Sunday. Each package contains different items such that no two packages have the same item.
Three packages arrived between Package F and the package which contains the Printer such that Package F arrived
before the package which contains Printer. Two packages arrived between Package G and the package which contains
a Calculator. Two packages arrived between Package A and Package B, but neither of them contains a Calculator or
Printer. Package A arrived after Package B. As many packages arrived before Package F is as same as many packages
arrived after Package G. Package C arrived after package D and more than two packages were arrived in between
them. The package which contains Data cable arrived after the Package which contains Pendrive. Package E contains

Page 301 of 728

Subscribe the Xpress Video Course & Mock Test Package for Bank & Insurance Exams
If there are any suggestions/ errors in our PDFs Feel Free to contact us via this email: admin@exampundit.in
IBPS RRB PO Prelims – Ultra Practice Bundle PDF
Mouse, which was arrived immediately after the package which contains Keyboard. The package which contains
Headset arrived immediately before the Package which contains Pendrive.
13). The package which contains Data cable was d) Package F
arrived on which of the following day? e) None of the above
a) Tuesday 16). Which among the following statement is True?
b) Sunday a) Packages which contains Keyboard and
c) Saturday Pendrive arrived in consecutive days
d) Thursday b) Only one package arrived between Package D
e) None of the above and Package A
14). How many packages was/were arrived between c) Package F contains Headset and was arrived
Package D and the package which contains a second to before Package G
Headset? d) Neither Package B nor Package C contains Data
a) One cable
b) Two e) None of the statement is True
c) Three 17). Package C was arrived _____ days after the
d) Four package which contains the keyboard?
e) None of the above a) 3 days
15). Which among the following package contains Pen b) 2 days
drive? c) 4 days
a) Package A d) 1 day
b) The package which was arrived on Sunday e) None of the above
c) The package which was arrived on Thursday
Directions (18-22): Read the following information carefully and answer the questions given below it:
Seven persons attended the interviews starting from Monday to Sunday. Each of them has a different weight in kg
among 52kg, 56kg, 61kg, 63kg, 67kg, 71kg, and 76kg, but not necessarily in the same order.
Four persons attended the interview between the heaviest person and S, who attended the interview before the
heaviest person. U is heavier than R but lighter than V. Number persons attended the interview after U is as same as
the number of persons attended the interview before the third lightest person. Only one person attended the interview
Page 302 of 728

Subscribe the Xpress Video Course & Mock Test Package for Bank & Insurance Exams
If there are any suggestions/ errors in our PDFs Feel Free to contact us via this email: admin@exampundit.in
IBPS RRB PO Prelims – Ultra Practice Bundle PDF
between S and the second lightest person. T attended the interview third to before the lightest person. Two persons
attended the interview between the third heaviest person and the person, who is lighter than only three persons. The
number of persons who attended the interview between S and T is as same as the number of persons who attended the
interview between T and Q. P attended the interview immediately before the third heaviest person.
18). What is the weight of the person who attended b) 9kg
the interview on Monday? c) 7kg
a) 61kg d) 4kg
b) 71kg e) Cannot be determined
c) 63kg 21). In a certain way, V is related to 63kg, R is related
d) Either A or B to 56kg and in the same way ___ is related to 71kg?
e) None of the above a) S
19). How many persons are heavier than the one, who b) Q
attended the interview on Wednesday? c) U
a) Five d) T
b) Four e) P
c) One 22). On which among the following day the one, who
d) Three is 6kg heavier than T, attended the interview?
e) None of the above a) Monday
20). What is the difference between the weights of the b) Tuesday
persons who attended the interview immediately c) Saturday
before and after R? d) Sunday
a) 13kg e) None of the above
Directions (23-27): Read the following information carefully and answer the questions given below it:
Seven workshops were held in a different city on different days of the week starting from Monday to Sunday. Each
workshop was attended by the different number of persons among 51, 57, 59, 63, 67, 72, and 77 but not necessarily in
the same order.
The number of persons attended the workshop in City R is 20 more than the number of persons attended the
workshop in City A. Three workshops were held between the workshop, which is held in City M and City A. The
Page 303 of 728

Subscribe the Xpress Video Course & Mock Test Package for Bank & Insurance Exams
If there are any suggestions/ errors in our PDFs Feel Free to contact us via this email: admin@exampundit.in
IBPS RRB PO Prelims – Ultra Practice Bundle PDF
workshop held in City H and the workshop held in City R is held in consecutive days. The sum of the number of
persons attended the workshop held in City H and City G is 131. The workshop held in City A was held after the
workshop held in City M. The workshop which was attended by 63 persons was held just before or just after the
workshop held in City H. Four workshops were held between the workshop, which is held in City P and the
workshop which was attended by 57 persons. The workshop, which is held in City P, is not on Monday. The
workshop held in City S was attended by 51 persons. The workshop held in City G and the workshop held in City A
is held in consecutive days. Only one workshop was held between the workshop, which is held in City P and City H.
Number of persons attended the workshop in City H more than the number of persons attended the workshop in City
G.
23). On which among the following the workshop c) 63 persons
held, which was attended by 67 persons? d) 72 persons
a) Monday e) None of the above
b) Wednesday 26). Four of the following five are alike in a certain
c) Thursday way and hence form a group. Which of the following
d) Tuesday does not belong to the group?
e) None of the above a) City H-Thursday
24). How many workshops were held between the b) City R-Friday
workshop held in City H and City S? c) City A-Sunday
a) One d) City S-Monday
b) Two e) City G-Saturday
c) Three 27). What is the sum of the total number of the
d) Four persons attended the workshop in City P and City M?
e) None of the above a) 136 persons
25). How many persons have attended the workshop, b) 135 persons
which was held on Thursday? c) 130 persons
a) 51 persons d) 124 persons
b) 67 persons e) None of the above
Directions (28-31): Read the following information carefully and answer the questions given below it:
Page 304 of 728

Subscribe the Xpress Video Course & Mock Test Package for Bank & Insurance Exams
If there are any suggestions/ errors in our PDFs Feel Free to contact us via this email: admin@exampundit.in
IBPS RRB PO Prelims – Ultra Practice Bundle PDF
Seven teachers taught different subjects in a week starting from Monday to Sunday such that one teacher taught only
one subject in a day.
The mathematics teacher taught on Friday. Either T or V taught on Wednesday and one among them taught Zoology.
Q taught three days after the one, who taught English. Either Q or S taught on Thursday and neither of them taught
after T. P taught the subject before V. Neither V nor P taught English. U taught immediately after the one, who taught
Tamil. R and the one, who taught Tamil, taught on Tuesday and Saturday respectively. Neither U nor P taught
Chemistry. The one, who taught Botany taught after the one, who taught Physics.
28). Who among the following taught Botany? 30). Which among the following subject was taught
a) Q immediately before the one, who taught Chemistry?
b) T a) Physics
c) U b) English
d) R c) Zoology
e) None of the above d) Botany
29). How many teachers taught subjects between the e) None of the above
one, who taught Physics and Q? 31). On which among the following day the English
a) One subject was taught by the teacher?
b) Three a) Tuesday
c) Two b) Wednesday
d) Four c) Thursday
e) None of the above d) Sunday
e) None of the above
Directions (32-35): Read the following information carefully and answer the questions given below it:
Team India played seven cricket matches (50 over) against different teams (P to V) in a week starting from Monday
to Sunday, but not necessarily in the same order. Indian team scored different runs against each team such as 75, 125,
200, 250, 325, 350, and 425, but not necessarily in the same order.
Indian team scored 200 runs immediately after the match played against P. India played a match against P on
Tuesday or Saturday. The number of matches played before playing against P is same as after playing against Q.
Indian team scored 325 runs against the match, which is played adjacent to the matches, where India played against V
Page 305 of 728

Subscribe the Xpress Video Course & Mock Test Package for Bank & Insurance Exams
If there are any suggestions/ errors in our PDFs Feel Free to contact us via this email: admin@exampundit.in
IBPS RRB PO Prelims – Ultra Practice Bundle PDF
and Q. Two matches were played between the matches, where Indian team scored 125 and 325 runs. India played a
match against T on Sunday. India doesn’t score 125 runs in a match, which is played on Saturday. Indian team
scored the highest runs immediately before the match played against U. Indian team scored 250 runs in a match
against either U or T. India doesn’t score 200 or 350 runs in a match against R.
32). Indian team scored how many runs in a match 34). Indian team played a match against S on which
played against Q? among the following day?
a) 75 runs a) Monday
b) 350 runs b) Wednesday
c) 125 runs c) Thursday
d) 425 runs d) Friday
e) None of the above e) None of the above
33). How many matches were played between the 35). What is the difference between the runs scored
match, where the Indian team scored 75 runs and the by the Indian team in matches played against R and
match played against U? V?
a) Four a) 150 runs
b) Three b) 100 runs
c) Two c) 350 runs
d) One d) 200 runs
e) None of the above e) None of the above
Directions (36-40): Read the following information carefully and answer the questions given below it:
Seven COVID-19 patients were admitted to a hospital on different days of the week starting from Monday to Sunday.
Each of them likes different colors among Red, Pink, Yellow, Purple, Green, Orange, and Black but not necessarily
in the same order.
P was admitted to a hospital before S. Only one person was admitted to a hospital between U and the one, who likes
Green. More than three persons were admitted to a hospital between the one, who likes Yellow and Purple color.
Only two persons were admitted to a hospital after R. The one, who likes Yellow, was admitted to a hospital
immediately before the one, who likes Pink. Q was admitted to a hospital immediately before U. No one was
admitted to a hospital between R and V. Q wasn’t admitted to a hospital on Monday. Not less than three persons were
Page 306 of 728

Subscribe the Xpress Video Course & Mock Test Package for Bank & Insurance Exams
If there are any suggestions/ errors in our PDFs Feel Free to contact us via this email: admin@exampundit.in
IBPS RRB PO Prelims – Ultra Practice Bundle PDF
admitted to a hospital before the one who likes Orange. Four persons were admitted to a hospital between T and the
one, who likes Purple. The one, who likes Orange, was admitted in a hospital before the one, who likes Black but
after the one, who likes Red. Two persons were admitted to a hospital between V and the one, who likes Pink.
36). Which among the following color was liked by a) Monday
the one, who was admitted to a hospital on Monday? b) Friday
a) Red c) Sunday
b) Purple d) Saturday
c) Green e) None of the above
d) Yellow 39). Who was admitted to a hospital three days before
e) None of the above S?
37). Who among the following likes Orange and a) V
Black color in the same order? b) The one who likes Black
a) S and the one, who was admitted in a hospital on c) Q
Wednesday d) The one who likes Pink
b) Q and V e) None of the above
c) U and R 40). Which among the following combination is false?
d) T and the one, who was admitted in a hospital on a) P-Purple-Monday
Friday b) Q-Pink-Tuesday
e) None of the above c) V-Black-Thursday
38). The one, who likes Green, was admitted in a d) R-Green-Friday
hospital on which among the following day? e) All the above
Directions (41-45): Read the following information carefully and answer the questions given below it:
Eight different flights were departed from Chennai-Mumbai on four different days in a week (Thursday, Friday,
Saturday, and Sunday) at two different slots i.e. 10.30 AM and 3 PM. On each day only two flights were departed at
different slots.
Flight B was departed immediately after the day of Flight A’s departure, but at different slots. Flight C and Flight E
were departed at the same slot and not more than one flight is departed in between them. Flight G was departed on

Page 307 of 728

Subscribe the Xpress Video Course & Mock Test Package for Bank & Insurance Exams
If there are any suggestions/ errors in our PDFs Feel Free to contact us via this email: admin@exampundit.in
IBPS RRB PO Prelims – Ultra Practice Bundle PDF
Saturday. Flight D was departed immediately after Flight E’s departure on the same day. Only two flights were
departed between Flight C and Flight H. Flight F was departed at 10.30 AM on Friday.
41). Which among the following flight is departed at c) Two
10.30 AM? d) Three
a) Flight H e) More than three
b) Flight D 44). Which among the following combination is true?
c) Flight G a) Flight E-Thursday-10.30 AM
d) Flight C b) Flight C-Sunday-10.30 AM
e) All the above c) Flight H-Thursday-3 PM
42). Flight A was departed on which among the d) Flight G-Saturday-10.30 AM
following day? e) None of the above
a) Thursday 45). Four of the following five are alike in a certain
b) Friday way and hence form a group. Which of the following
c) Saturday does not belong to the group?
d) Sunday a) Flight H
e) None of the above b) Flight B
43). How many flights departed between Flight B and c) Flight G
Flight C? d) Flight C
a) None e) Flight D
b) One
Directions (46-50): Read the following information carefully and answer the questions given below it:
Eight persons have purchased things from Amazon on different days i.e. Monday to next Monday. Each of them was
purchased for different amounts such as Rs.150, Rs.200, Rs.350, Rs.400, Rs.500, Rs.600, Rs.800, and Rs.1600, but
not necessarily in the same order.
Two persons have purchased things between B and C. A was purchased thing for either Rs.150 or Rs.350 and only
one person was purchased things between A and C. No one has purchased things before B. A was purchased things
before C. E was purchased things for the second-highest amount. Not more than four persons have purchased things
before H. Difference between the amounts spent by C and G is Rs.50. No one has purchased things between H and F,
Page 308 of 728

Subscribe the Xpress Video Course & Mock Test Package for Bank & Insurance Exams
If there are any suggestions/ errors in our PDFs Feel Free to contact us via this email: admin@exampundit.in
IBPS RRB PO Prelims – Ultra Practice Bundle PDF
who purchased two days before G. F was purchased things for the highest amount. B doesn’t purchase things for
Rs.500. C has purchased things for more amount than G. D was purchased things before F. B was purchased things
for Rs.100 more than D. H doesn’t purchase things for Rs.200.
46). Who among the following purchased things for e) None of the above
Rs.400? 49). Who among the following purchased things on
a) H Thursday?
b) C a) The one who purchased things for Rs.400
c) D b) E
d) B c) F
e) None of the above d) The one who purchased things for Rs.200
47). How many persons purchased things between G e) None of the above
and E? 50). Which among the following statement is True?
a) None I. More than three persons purchased things after
b) One E
c) Two II. The sum of the amounts purchased by C and H
d) Three together is equal to B’s purchased amount
e) More than three III. D and the one who purchased for Rs.400 have
48). What is the difference between the amounts purchased things in consecutive days
purchased by H and the one, who purchased on a) Only I
Sunday? b) Only II
a) Rs.600 c) Both I and II
b) Rs.100 d) Both II and III
c) Rs.400 e) None of the above
d) Rs.250

15). Days Based Puzzle - Solution and Detailed Explanation


SOLUTION (1-4):
Page 309 of 728

Subscribe the Xpress Video Course & Mock Test Package for Bank & Insurance Exams
If there are any suggestions/ errors in our PDFs Feel Free to contact us via this email: admin@exampundit.in
IBPS RRB PO Prelims – Ultra Practice Bundle PDF
 Seven persons bought different things one after the other in a week starting from Monday to Sunday.

Days Persons
Monday
Tuesday
Wednesday
Thursday
Friday
Saturday
Sunday
 Two persons bought things between P and Q.
 Only one person bought things between P and R.
 Q and R didn’t buy things on consecutive days.

R Q
R
P
P
R
Q R
From the above table, we get four possible cases as shown below,

Case-1 Case-2 Case-3 Case-4


Days
Persons Persons Persons Persons
Monday R Q
Tuesday R R Q
Wednesday P R
Thursday P P
Friday R P
Saturday Q R R
Sunday Q R

 U bought things immediately before V.


 Only one person bought things between T and U.

Case-3 gets eliminated since only one person bought things between T and U is not possible.

Case-3
Case-1 Case-2 Case-4
Days Eliminated
Persons Persons Persons Persons
Page 310 of 728

Subscribe the Xpress Video Course & Mock Test Package for Bank & Insurance Exams
If there are any suggestions/ errors in our PDFs Feel Free to contact us via this email: admin@exampundit.in
IBPS RRB PO Prelims – Ultra Practice Bundle PDF
Monday R Q T
Tuesday T R U Q
Wednesday P T V U
Thursday U P P V
Friday V U P
Saturday Q V R
Sunday Q R

 The number of persons bought things before S is as same as the number of persons bought things after Q.
 Not more than two persons bought things between S and T.

Case-1 gets eliminated since we can’t satisfy the 1st condition. Case-4 gets eliminated since we can’t satisfy the
2nd condition.

Case-1 Case-4
Case-2
Days Eliminated Eliminated
Persons Persons Persons
Monday R S T
Tuesday T R Q
Wednesday P T U
Thursday U P V
Friday V U P
Saturday Q V S
Sunday S Q R

FINAL ARRANGEMENT

Case-2
Days
Persons
Monday S
Tuesday R
Wednesday T
Thursday P
Friday U
Saturday V
Sunday Q
1. A) Monday

2. B) Two
Page 311 of 728

Subscribe the Xpress Video Course & Mock Test Package for Bank & Insurance Exams
If there are any suggestions/ errors in our PDFs Feel Free to contact us via this email: admin@exampundit.in
IBPS RRB PO Prelims – Ultra Practice Bundle PDF
3. C) V

4. E) None of the above

SOLUTION (5-7)

 Seven events held in a week starting from Monday to Sunday.

Days Events
Monday
Tuesday
Wednesday
Thursday
Friday
Saturday
Sunday

 Three events are held between Event A and Event C such that Event A is held before Event C.
 Event B is held exactly between Event A and Event C.

Case-1 Case-2 Case-3


Days
Events Events Events
Monday A
Tuesday A
Wednesday B A
Thursday B
Friday C B
Saturday C
Sunday C
 Event E is held before Event D and more than three events are held in between them.

Case-1 Case-2 Case-3


Days
Events Events Events
Monday A E E
Tuesday E A
Wednesday B A
Thursday B
Friday C B
Saturday C D
Sunday D D C

Page 312 of 728

Subscribe the Xpress Video Course & Mock Test Package for Bank & Insurance Exams
If there are any suggestions/ errors in our PDFs Feel Free to contact us via this email: admin@exampundit.in
IBPS RRB PO Prelims – Ultra Practice Bundle PDF
 Event F and Event C are not held on consecutive days.
 Event G and Event A are not held on consecutive days.

Case-1 gets eliminated since we can’t place Event F. Case-3 gets eliminated since we can’t place Event G.

Case-1 Case-3
Case-2
Days Eliminated Eliminated
Events Events Events
Monday A E E
Tuesday E A G
Wednesday B F A
Thursday F B G
Friday C G B
Saturday F C D
Sunday D D C

FINAL ARRANGEMENT

Case-2
Days
Events
Monday E
Tuesday A
Wednesday F
Thursday B
Friday G
Saturday C
Sunday D
5. B) Event C

6. E) None

7. C) Only II

SOLUTION (8-12)

 Six professors attended invigilation duty on different days of the week starting from Monday to Saturday.
 Each of them belongs to different departments such that no two persons belong to the same department.

Days Professors Department


Monday
Tuesday
Page 313 of 728

Subscribe the Xpress Video Course & Mock Test Package for Bank & Insurance Exams
If there are any suggestions/ errors in our PDFs Feel Free to contact us via this email: admin@exampundit.in
IBPS RRB PO Prelims – Ultra Practice Bundle PDF
Wednesday
Thursday
Friday
Saturday
 The one, who belongs to the Civil department attended invigilation duty second to before the one, who
belongs to the EEE department but neither of them attended on Monday or Saturday.

CASE-1 CASE-2
Days
Professors Department Professors Department
Monday Civil Civil
Tuesday Civil
Wednesday Civil
Thursday EEE
Friday EEE
Saturday EEE EEE

 R, who doesn’t belong to the Civil department, attended the invigilation duty three days before S.

CASE-1 CASE-2
Days
Professors Department Professors Department
Monday R R
Tuesday Civil
Wednesday Civil
Thursday S EEE S
Friday EEE
Saturday

CASE-1-A CASE-2-A
Days
Professors Department Professors Department
Monday
Tuesday Civil R
Wednesday R Civil
Thursday EEE
Friday S EEE
Saturday S

Page 314 of 728

Subscribe the Xpress Video Course & Mock Test Package for Bank & Insurance Exams
If there are any suggestions/ errors in our PDFs Feel Free to contact us via this email: admin@exampundit.in
IBPS RRB PO Prelims – Ultra Practice Bundle PDF
 The number of persons who attended the invigilation duty after S is as same as the number of persons who
attended the invigilation duty before U.

 Only one person attended the invigilation duty between U and the one, who belongs to the ECE department.

CASE-1 CASE-2
Days
Professors Department Professors Department
Monday R ECE/ R ECE
Tuesday Civil
Wednesday U U Civil
Thursday S EEE S
Friday ECE/ EEE
Saturday
Case-2-A gets eliminated since we can’t satisfy the 1st condition

CASE-1-A CASE-2-A, Eliminated


Days
Professors Department Professors Department
Monday U
Tuesday Civil R
Wednesday R ECE Civil
Thursday EEE
Friday S EEE
Saturday S

 The one, who belongs to the IT department, attended the invigilation duty immediately before the one, who
belongs to the Mechanical department.

Note: In case-1, if the one who belongs to the ECE department attended the invigilation duty on Friday, then the
above condition is not possible. Thus R must belong to the ECE department in case-1.

Case-2 gets eliminated since we can’t satisfy the above condition

CASE-1 CASE-2, Eliminated


Days
Professors Department Professors Department
Monday R ECE R ECE
Tuesday Civil
Wednesday U U Civil
Thursday S EEE S
Friday IT EEE
Saturday Mechanical
Page 315 of 728

Subscribe the Xpress Video Course & Mock Test Package for Bank & Insurance Exams
If there are any suggestions/ errors in our PDFs Feel Free to contact us via this email: admin@exampundit.in
IBPS RRB PO Prelims – Ultra Practice Bundle PDF

CASE-1-A
Days
Professors Department
Monday U
Tuesday Civil
Wednesday R ECE
Thursday EEE
Friday IT
Saturday S Mechanical

 P attended the invigilation duty immediately before the one, who belongs to the CSE department.
 Q attended the invigilation duty before T.

CASE-1
Days
Professors Department
Monday R ECE
Tuesday P Civil
Wednesday U CSE
Thursday S EEE
Friday Q IT
Saturday T Mechanical

Case-1-A gets eliminated since we can’t satisfy the 1st condition

CASE-1-A, Eliminated
Days
Professors Department
Monday U
Tuesday Civil
Wednesday R ECE
Thursday EEE
Friday IT
Saturday S Mechanical

FINAL ARRANGEMENT

Page 316 of 728

Subscribe the Xpress Video Course & Mock Test Package for Bank & Insurance Exams
If there are any suggestions/ errors in our PDFs Feel Free to contact us via this email: admin@exampundit.in
IBPS RRB PO Prelims – Ultra Practice Bundle PDF
CASE-1
Days
Professors Department
Monday R ECE
Tuesday P Civil
Wednesday U CSE
Thursday S EEE
Friday Q IT
Saturday T Mechanical
8. D) U

9. C) Three

10. C) The one who belongs IT department

11. D) Wednesday and Saturday

12. D) Q-IT-Friday

SOLUTION (13-17)

 Seven packages were arrived at an Amazon facility-Mumbai on different days of the week starting from
Monday to Sunday.
 Each package contains different items such that no two packages have the same item.

Days Packages Items


Monday
Tuesday
Wednesday
Thursday
Friday
Saturday
Sunday

 Three packages arrived between Package F and the package which contains the Printer such that Package F
arrived before the package which contains Printer.

Case-1 Case-2 Case-3


Days
Packages Items Packages Items Packages Items
Monday F
Tuesday F
Page 317 of 728

Subscribe the Xpress Video Course & Mock Test Package for Bank & Insurance Exams
If there are any suggestions/ errors in our PDFs Feel Free to contact us via this email: admin@exampundit.in
IBPS RRB PO Prelims – Ultra Practice Bundle PDF
Wednesday F
Thursday
Friday Printer
Saturday Printer
Sunday Printer

 As many as packages arrived before Package F is the same as many as packages arrived after Package G
 Two packages arrived between Package G and the package which contains a Calculator.

Case-1 Case-2 Case-3


Days
Packages Items Packages Items Packages Items
Monday F
Tuesday F Calculator
Wednesday Calculator F
Thursday Calculator
Friday Printer G
Saturday G Printer
Sunday G Printer

 Two packages arrived between Package A and Package B, but neither of them contains a Calculator nor a
Printer.
 Package A arrived after Package B.

Case-1 Case-2 Case-3


Days
Packages Items Packages Items Packages Items
Monday F B B
Tuesday F A/B Calculator
Wednesday B A/B Calculator F
Thursday A/B Calculator A A
Friday A/B Printer G
Saturday A G Printer
Sunday G A/B Printer

Case-2-A
Days
Packages Items
Monday
Tuesday F
Page 318 of 728

Subscribe the Xpress Video Course & Mock Test Package for Bank & Insurance Exams
If there are any suggestions/ errors in our PDFs Feel Free to contact us via this email: admin@exampundit.in
IBPS RRB PO Prelims – Ultra Practice Bundle PDF
Wednesday A/B Calculator
Thursday B
Friday
Saturday G Printer
Sunday A

 Package E contains Mouse, which was arrived immediately after the package which contains Keyboard.

Case-1 Case-2 Case-3


Days
Packages Items Packages Items Packages Items
Monday F Keyboard B B
Tuesday E Mouse F Calculator
Wednesday B Calculator F
Thursday Calculator A Keyboard A
Friday Printer E Mouse G Keyboard
Saturday A G Printer E Mouse
Sunday G Printer

Case-2-A
Days
Packages Items
Monday
Tuesday F
Wednesday Calculator
Thursday B Keyboard
Friday E Mouse
Saturday G Printer
Sunday A

 Package C arrived after Package D and more than two packages were arrived in between them.

Case-1 and Case-2-A get eliminated since the above condition not get satisfied.

Case-1, Eliminated Case-2 Case-3


Days
Packages Items Packages Items Packages Items
Monday F Keyboard B B
Tuesday E Mouse F D Calculator
Wednesday B D Calculator F
Thursday D Calculator A Keyboard A
Page 319 of 728

Subscribe the Xpress Video Course & Mock Test Package for Bank & Insurance Exams
If there are any suggestions/ errors in our PDFs Feel Free to contact us via this email: admin@exampundit.in
IBPS RRB PO Prelims – Ultra Practice Bundle PDF
Friday C Printer E Mouse G Keyboard
Saturday A G Printer E Mouse
Sunday G C C Printer

Case-2-A, Eliminated
Days
Packages Items
Monday D
Tuesday F
Wednesday C Calculator
Thursday B Keyboard
Friday E Mouse
Saturday G Printer
Sunday A

 The package which contains Headset arrived immediately before the Package which contains Pen drive.
 The package which contains Data cable arrived after the Package which contains Pen drive.

Case-3 gets eliminated since 2nd condition was not satisfied.

Case-2 Case-3, Eliminated


Days
Packages Items Packages Items
Monday B Headset B Data cable
Tuesday F Pendrive D Calculator
Wednesday D Calculator F Headset
Thursday A Keyboard A Pendrive
Friday E Mouse G Keyboard
Saturday G Printer E Mouse
Sunday C Data cable C Printer

FINAL ARRANGEMENT

Case-2
Days
Packages Items
Monday B Headset
Tuesday F Pendrive
Wednesday D Calculator
Thursday A Keyboard
Friday E Mouse
Page 320 of 728

Subscribe the Xpress Video Course & Mock Test Package for Bank & Insurance Exams
If there are any suggestions/ errors in our PDFs Feel Free to contact us via this email: admin@exampundit.in
IBPS RRB PO Prelims – Ultra Practice Bundle PDF
Saturday G Printer
Sunday C Data cable
13. B) Sunday

14. A) One

15. D) Package F

16. E) None of the statement is True

17. A) 3 days

SOLUTION (18-22):

 Seven persons attended the interviews starting from Monday to Sunday.


 Each of them has a different weight in kg among 52kg, 56kg, 61kg, 63kg, 67kg, 71kg, and 76kg, but not
necessarily in the same order.

Days Persons Weights


Monday
Tuesday
Wednesday
Thursday
Friday
Saturday
Sunday

Weight arrangement in descending order:

Persons
Weight 76kg 71kg 67kg 63kg 61kg 56kg 52kg

 Four persons attended the interview between the heaviest person and S, who attended the interview before the
heaviest person.
 Only one person attended the interview between S and the second lightest person.

Case-1 Case-2
Days
Persons Weights Persons Weights
Monday S
Tuesday S
Page 321 of 728

Subscribe the Xpress Video Course & Mock Test Package for Bank & Insurance Exams
If there are any suggestions/ errors in our PDFs Feel Free to contact us via this email: admin@exampundit.in
IBPS RRB PO Prelims – Ultra Practice Bundle PDF
Wednesday 56kg
Thursday 56kg
Friday
Saturday 76kg
Sunday 76kg

 T attended the interview third to before the lightest person.


 The number of persons who attended the interview between S and T is as same as the number of persons who
attended the interview between T and Q.

Case-1 Case-1-A Case-2


Days
Persons Weights Persons Weights Persons Weights
Monday S S
Tuesday T S
Wednesday 56kg Q 56kg T
Thursday T Q 56kg
Friday 52kg
Saturday 76kg 76kg 52kg
Sunday Q 52kg 76kg

 P attended the interview immediately before the third heaviest person.


 Two persons attended the interview between the third heaviest person and the person, who is lighter than only
three persons.

Note: A person with weight 63kg is lighter than only three persons.

Case-1 Case-1-A Case-2


Days
Persons Weights Persons Weights Persons Weights
Monday S 63kg S P
Tuesday T S 67kg
Wednesday P 56kg Q 56kg T
Thursday T 67kg 63kg Q 56kg
Friday 52kg 63kg
Saturday 76kg P 76kg 52kg
Sunday Q 52kg 67kg 76kg

 U is heavier than R but lighter than V

Page 322 of 728

Subscribe the Xpress Video Course & Mock Test Package for Bank & Insurance Exams
If there are any suggestions/ errors in our PDFs Feel Free to contact us via this email: admin@exampundit.in
IBPS RRB PO Prelims – Ultra Practice Bundle PDF
 The number of persons who attended interview the after U is the same as the number of persons who attended
the interview before the third lightest person.

Hints: V> U>R. The person with 61kg is the third lightest person.

Case-1: V=76kg, U=71kg and R=61kg. Anyway, the 2nd condition is not possible. Hence, case-1 gets eliminated.

Case-1a: V=67kg, U=63kg, and R=52kg. Anyway, the 2nd condition is not possible. Hence, case-1-A gets
eliminated.

Case-2: V=76kg, U=63kg and R=52kg. As per 2nd condition T=61kg and finally P=71kg

Case-1, Eliminated Case-1-A, Eliminated Case-2


Days
Persons Weights Persons Weights Persons Weights
Monday S 63kg S P 71kg
Tuesday R/U 61kg/71kg T S 67kg
Wednesday P 56kg Q 56kg T 61kg
Thursday T 67kg U 63kg Q 56kg
Friday U/R 71kg/61kg R 52kg U 63kg
Saturday V 76kg P 76kg R 52kg
Sunday Q 52kg V 67kg V 76kg
FINAL ARRANGEMENT

Case-2
Days
Persons Weights
Monday P 71kg
Tuesday S 67kg
Wednesday T 61kg
Thursday Q 56kg
Friday U 63kg
Saturday R 52kg
Sunday V 76kg

Weight arrangement in descending order:

Persons V P S U T Q R
Weight 76kg 71kg 67kg 63kg 61kg 56kg 52kg

18. B) 71kg

Page 323 of 728

Subscribe the Xpress Video Course & Mock Test Package for Bank & Insurance Exams
If there are any suggestions/ errors in our PDFs Feel Free to contact us via this email: admin@exampundit.in
IBPS RRB PO Prelims – Ultra Practice Bundle PDF
19. B) Four

20. A) 13kg

21. D) T

22. B) Tuesday

SOLUTION (23-27)

 Seven workshops were held in a different city on different days of the week starting from Monday to Sunday.
 Each workshop was attended by the different number of persons among 51, 57, 59, 63, 67, 72, and 77 but not
necessarily in the same order.

Days City Persons


Monday
Tuesday
Wednesday
Thursday
Friday
Saturday
Sunday

 The number of persons attended the workshop in City R is 20 more than the number of persons attended the
workshop in City A.
 Three workshops were held between the workshop, which is held in City M and City A.
 The workshop held in City A was held after the workshop held in City M.
 Four workshops were held between the workshop, which is held in City P, and the workshop which was
attended by 57 persons.
 The workshop, which is held in City P, is not on Monday.

Hints: 1st condition i.e. City R=20 persons +City A (the only combination is 57 and 77)

We get City A workshop =57 persons and City R workshop =77 persons

Days City Persons


Monday P
Tuesday P
Wednesday M
Page 324 of 728

Subscribe the Xpress Video Course & Mock Test Package for Bank & Insurance Exams
If there are any suggestions/ errors in our PDFs Feel Free to contact us via this email: admin@exampundit.in
IBPS RRB PO Prelims – Ultra Practice Bundle PDF
Thursday
Friday
Saturday
Sunday A 57

 The workshop held in City G and the workshop held in City A is held in consecutive days.
 Only one workshop was held between the workshop, which is held in City P and City H.
 The workshop held in City H and the workshop held in City R is held in consecutive days.
 The workshop held in City S was attended by 51 persons.

Note: City R workshop =77 persons

Days City Persons


Monday S 51
Tuesday P
Wednesday M
Thursday H
Friday R 77
Saturday G
Sunday A 57

 The workshop which was attended by 63 persons was held just before or just after the workshop held in City
H.

Days City Persons


Monday S 51
Tuesday P
Wednesday M 63
Thursday H
Friday R 77
Saturday G
Sunday A 57

 The number of persons attended the workshop in City H more than the number of persons attended the
workshop in City G.
 The sum of the number of persons attended the workshop held in City H and City G is 131.

Hints: remaining numbers left are 59, 67, and 72


Page 325 of 728

Subscribe the Xpress Video Course & Mock Test Package for Bank & Insurance Exams
If there are any suggestions/ errors in our PDFs Feel Free to contact us via this email: admin@exampundit.in
IBPS RRB PO Prelims – Ultra Practice Bundle PDF
City H+ City G=131 persons (i.e. 59+72=131 persons)

We get City H workshop =72 persons and City G workshop =59 persons (City H>City G)

Finally, City P workshop=67 persons

Days City Persons


Monday S 51
Tuesday P 67
Wednesday M 63
Thursday H 72
Friday R 77
Saturday G 59
Sunday A 57
FINAL ARRANGEMENT

Days City Persons


Monday S 51
Tuesday P 67
Wednesday M 63
Thursday H 72
Friday R 77
Saturday G 59
Sunday A 57
23. D) Tuesday

24. B) Two

25. D) 72 persons

26. E) City G-Saturday

27. C) 130 persons

SOLUTION (28-31)

 Seven teachers taught different subjects in a week starting from Monday to Sunday such that one teacher
taught only one subject in a day.

Monday
Tuesday
Wednesday
Page 326 of 728

Subscribe the Xpress Video Course & Mock Test Package for Bank & Insurance Exams
If there are any suggestions/ errors in our PDFs Feel Free to contact us via this email: admin@exampundit.in
IBPS RRB PO Prelims – Ultra Practice Bundle PDF
Thursday
Friday
Saturday
Sunday
 The mathematics teacher taught on Friday.
 R and the one, who taught Tamil, taught on Tuesday and Saturday respectively.
 U taught immediately after the one, who taught Tamil.

Days Teachers Subjects


Monday
Tuesday R
Wednesday
Thursday
Friday Mathematics
Saturday Tamil
Sunday U

 Either Q or S taught on Thursday and neither of them taught after T.


 Either T or V taught on Wednesday and one among them taught Zoology.

Note: Both Q and S taught before T. This implies that V must teach on Wednesday.

Case-1 Case-2
Days Teachers Subjects Days Teachers Subjects
Monday Monday
Tuesday R Tuesday R
Wednesday V Zoology/ Wednesday V Zoology/
Thursday Q Thursday S
Friday Mathematics Friday Mathematics
Saturday Tamil Saturday Tamil
Sunday U Sunday U

 Q taught three days after the one, who taught English.


 P taught the subject before V.
 Neither V nor P taught English.

Case-1 gets eliminated since P doesn’t teach English

Page 327 of 728

Subscribe the Xpress Video Course & Mock Test Package for Bank & Insurance Exams
If there are any suggestions/ errors in our PDFs Feel Free to contact us via this email: admin@exampundit.in
IBPS RRB PO Prelims – Ultra Practice Bundle PDF
In case-2, we know Both Q and S taught before T. This implies that Q taught on Friday and T taught on
Saturday.

Case-1[Eliminated] Case-2
Days Teachers Subjects Days Teachers Subjects
Monday P English Monday P
Tuesday R Tuesday R English
Wednesday V Zoology/ Wednesday V Zoology/
Thursday Q Thursday S
Friday Mathematics Friday Q Mathematics
Saturday Tamil Saturday T Tamil
Sunday U Sunday U

 Neither U nor P taught Chemistry.


 The one, who taught Botany taught after the one, who taught Physics.

We know between V and T, one must teach Zoology. Here T teaches Tamil. Thus V must teach Zoology. Finally, S
taught Chemistry, U taught Botany and P taught Physics.

Case-2
Days Teachers Subjects
Monday P Physics
Tuesday R English
Wednesday V Zoology
Thursday S Chemistry
Friday Q Mathematics
Saturday T Tamil
Sunday U Botany

FINAL ARRANGEMENT

Case-2
Days Teachers Subjects
Monday P Physics
Tuesday R English
Wednesday V Zoology
Thursday S Chemistry
Friday Q Mathematics
Saturday T Tamil
Page 328 of 728

Subscribe the Xpress Video Course & Mock Test Package for Bank & Insurance Exams
If there are any suggestions/ errors in our PDFs Feel Free to contact us via this email: admin@exampundit.in
IBPS RRB PO Prelims – Ultra Practice Bundle PDF
Sunday U Botany
28. C) U

29. B) Three

30. C) Zoology

31. A) Tuesday

SOLUTION (32-35)

 Team India played seven cricket matches (50 over) against different teams (P to V) in a week starting from
Monday to Sunday, but not necessarily in the same order.
 Indian team scored different runs against each team such as 75, 125, 200, 250, 325, 350, and 425 but not
necessarily in the same order.

Days Teams Runs


Monday
Tuesday
Wednesday
Thursday
Friday
Saturday
Sunday
 India played a match against T on Sunday.
 India played a match against P on Tuesday or Saturday.
 The number of matches played before playing against P is as same as after playing against Q.
 Indian team scored 200 runs immediately after the match played against P.

Case-1 Case-2
Days Teams Runs Days Teams Runs
Monday Monday
Tuesday P Tuesday Q
Wednesday 200 Wednesday
Thursday Thursday
Friday Friday
Saturday Q Saturday P
Sunday T Sunday T 200

Page 329 of 728

Subscribe the Xpress Video Course & Mock Test Package for Bank & Insurance Exams
If there are any suggestions/ errors in our PDFs Feel Free to contact us via this email: admin@exampundit.in
IBPS RRB PO Prelims – Ultra Practice Bundle PDF
 Indian team scored 325 runs against the match, which is played adjacent to the matches, where India played
against V and Q.
 Two matches were played between the matches, where the Indian team scored 125 and 325 runs.
 India doesn’t score 125 runs in a match, which is played on Saturday.

Case-2 gets eliminated since 3rd condition not satisfied

Case-1 Case-2, Eliminated


Days Teams Runs Days Teams Runs
Monday Monday
Tuesday P 125 Tuesday Q
Wednesday 200 Wednesday 325
Thursday V Thursday V
Friday 325 Friday
Saturday Q Saturday P 125
Sunday T Sunday T 200
 Indian team scored the highest runs immediately before the match played against U.

Case-1
Days Teams Runs
Monday
Tuesday P 125
Wednesday 200
Thursday V 425
Friday U 325
Saturday Q
Sunday T

 Indian team scored 250 runs in a match against either U or T.

Case-1
Days Teams Runs
Monday
Tuesday P 125
Wednesday 200
Thursday V 425
Friday U 325
Saturday Q
Sunday T 250

Page 330 of 728

Subscribe the Xpress Video Course & Mock Test Package for Bank & Insurance Exams
If there are any suggestions/ errors in our PDFs Feel Free to contact us via this email: admin@exampundit.in
IBPS RRB PO Prelims – Ultra Practice Bundle PDF
 India doesn’t score 200 or 350 runs in a match against R.

Note: Match against S scored 200 runs; Match against R scored 75 runs and Match against Q scored 350 runs

Case-1
Days Teams Runs
Monday R 75
Tuesday P 125
Wednesday S 200
Thursday V 425
Friday U 325
Saturday Q 350
Sunday T 250

FINAL ARRANGEMENT

Case-1
Days Teams Runs
Monday R 75
Tuesday P 125
Wednesday S 200
Thursday V 425
Friday U 325
Saturday Q 350
Sunday T 250

32. B) 350

33. B) Three

34. B) Wednesday

35. C) 350 runs

SOLUTION (36-40)

 Seven COVID-19 patients were admitted to a hospital on different days of the week starting from Monday to
Sunday.
 Each of them likes different colors among Red, Pink, Yellow, Purple, Green, Orange, and Black but not
necessarily in the same order.
Page 331 of 728

Subscribe the Xpress Video Course & Mock Test Package for Bank & Insurance Exams
If there are any suggestions/ errors in our PDFs Feel Free to contact us via this email: admin@exampundit.in
IBPS RRB PO Prelims – Ultra Practice Bundle PDF
Days Patients Color
Monday
Tuesday
Wednesday
Thursday
Friday
Saturday
Sunday

 Only two persons were admitted to a hospital after R.


 No one was admitted to a hospital between R and V.
 Two persons were admitted to a hospital between V and the one, who likes Pink.

Case-1 Case-2
Days Patients Color Days Patients Color
Monday Monday Pink/
Tuesday Tuesday
Wednesday Pink Wednesday
Thursday Thursday V
Friday R Friday R
Saturday V Saturday
Sunday Sunday Pink/

 The one, who likes Yellow, was admitted to a hospital immediately before the one, who likes Pink.

Case-1 Case-2
Days Patients Color Days Patients Color
Monday Monday
Tuesday Yellow Tuesday
Wednesday Pink Wednesday
Thursday Thursday V
Friday R Friday R
Saturday V Saturday Yellow
Sunday Sunday Pink

 More than three persons were admitted to a hospital between the one, who likes Yellow and Purple color.
 Four persons were admitted to a hospital between T and the one, who likes Purple.

Page 332 of 728

Subscribe the Xpress Video Course & Mock Test Package for Bank & Insurance Exams
If there are any suggestions/ errors in our PDFs Feel Free to contact us via this email: admin@exampundit.in
IBPS RRB PO Prelims – Ultra Practice Bundle PDF
Case-1 Case-2
Days Patients Color Days Patients Color
Monday Monday Purple
Tuesday T Yellow Tuesday
Wednesday Pink Wednesday
Thursday Thursday V
Friday R Friday R
Saturday V Saturday T Yellow
Sunday Purple Sunday Pink

 Q was admitted to a hospital immediately before U.


 Q wasn’t admitted to a hospital on Monday.

Case-1 Case-2
Days Patients Color Days Patients Color
Monday Monday Q Purple
Tuesday T Yellow Tuesday Q
Wednesday Q Pink Wednesday U
Thursday U Thursday V
Friday R Friday R
Saturday V Saturday T Yellow
Sunday Purple Sunday Pink

 P was admitted to a hospital before S.


 Only one person was admitted to a hospital between U and the one, who likes Green.

Case-1 Case-2
Days Patients Color Days Patients Color
Monday P Monday P Purple
Tuesday T Yellow Tuesday Q
Wednesday Q Pink Wednesday U
Thursday U Thursday V
Friday R Friday R Green
Saturday V Green Saturday T Yellow
Sunday S Purple Sunday S Pink

 The one, who likes Orange, was admitted in a hospital before the one, who likes Black but after the one, who
likes Red.
Page 333 of 728

Subscribe the Xpress Video Course & Mock Test Package for Bank & Insurance Exams
If there are any suggestions/ errors in our PDFs Feel Free to contact us via this email: admin@exampundit.in
IBPS RRB PO Prelims – Ultra Practice Bundle PDF
 Not less than three persons were admitted to a hospital before the one who likes Orange.

Case-2 gets eliminated since 2nd condition not satisfied

Case-1 Case-2, Eliminated


Days Patients Color Days Patients Color
Monday P Red Monday P Purple
Tuesday T Yellow Tuesday Q Pink
Wednesday Q Pink Wednesday U Orange
Thursday U Orange Thursday V Black
Friday R Black Friday R Green
Saturday V Green Saturday T Yellow
Sunday S Purple Sunday S Pink

FINAL ARRANGEMENT

Case-1
Days Patients Color
Monday P Red
Tuesday T Yellow
Wednesday Q Pink
Thursday U Orange
Friday R Black
Saturday V Green
Sunday S Purple

36. A) Red

37. C) U and R

38. D) Saturday

39. E) None of the above

40. E) All the above

SOLUTION (41-45)

Page 334 of 728

Subscribe the Xpress Video Course & Mock Test Package for Bank & Insurance Exams
If there are any suggestions/ errors in our PDFs Feel Free to contact us via this email: admin@exampundit.in
IBPS RRB PO Prelims – Ultra Practice Bundle PDF
 Eight different flights were departed from Chennai-Mumbai on four different days in a week (Thursday,
Friday, Saturday, and Sunday) at two different slots i.e. 10.30 AM and 3 PM. On each day only two flights
were departed at different slots.

Days Slots Flights


10.30 AM
Thursday
3 PM
10.30 AM
Friday
3 PM
10.30 AM
Saturday
3 PM
10.30 AM
Sunday
3 PM
 Flight F was departed at 10.30 AM on Friday.
 Flight G was departed on Saturday.

Case-1 Case-2
Days Slots
Flights Flights
10.30 AM
Thursday
3 PM
10.30 AM F F
Friday
3 PM
10.30 AM G
Saturday
3 PM G
10.30 AM
Sunday
3 PM

 Flight D was departed immediately after Flight E’s departure on the same day.

Case-1 Case-1-A Case-2 Case-2-A


Days Slots
Flights Flights Flights Flights
10.30 AM E E
Thursday
3 PM D D
10.30 AM F F F F
Friday
3 PM
10.30 AM G G
Saturday
3 PM G G
10.30 AM E E
Sunday
3 PM D D

Page 335 of 728

Subscribe the Xpress Video Course & Mock Test Package for Bank & Insurance Exams
If there are any suggestions/ errors in our PDFs Feel Free to contact us via this email: admin@exampundit.in
IBPS RRB PO Prelims – Ultra Practice Bundle PDF
 Flight C and Flight E were departed at the same slot and not more than one flight is departed in between them.

Case-1, 1-A & 2 get eliminated since the above condition not satisfied.

Case-1, Case-1-A Case-2 Case-2-A


Days Slots Eliminated Eliminated Eliminated
Flights Flights Flights Flights
10.30 AM E C E
Thursday
3 PM D D
10.30 AM F F F F
Friday
3 PM
10.30 AM G G C C
Saturday
3 PM G G
10.30 AM C E C E
Sunday
3 PM D D

 Only two flights were departed between Flight C and Flight H.


 Flight B was departed immediately after the day of Flight A’s departure, but at different slots.

Case-2-A
Days Slots
Flights
10.30 AM A
Thursday
3 PM H
10.30 AM F
Friday
3 PM B
10.30 AM C
Saturday
3 PM G
10.30 AM E
Sunday
3 PM D
FINAL ARRANGEMENT

Case-2-A
Days Slots
Flights
10.30 AM A
Thursday
3 PM H
10.30 AM F
Friday
3 PM B
10.30 AM C
Saturday
3 PM G
Sunday 10.30 AM E
Page 336 of 728

Subscribe the Xpress Video Course & Mock Test Package for Bank & Insurance Exams
If there are any suggestions/ errors in our PDFs Feel Free to contact us via this email: admin@exampundit.in
IBPS RRB PO Prelims – Ultra Practice Bundle PDF
3 PM D
41. D) C

42. A) Thursday

43. A) None

44. C) Flight H-Thursday-3 PM

45. D) Flight C

SOLUTION (46-50)

 Eight persons have purchased things from Amazon on different days i.e. Monday to next Monday.
 Each of them was purchased for different amounts such as Rs.150, Rs.200, Rs.350, Rs.400, Rs.500, Rs.600,
Rs.800, and Rs.1600, but not necessarily in the same order.

Days Persons Amount


Monday
Tuesday
Wednesday
Thursday
Friday
Saturday
Sunday
Monday

 No one has purchased things before B.


 Two persons have purchased things between B and C.

Days Persons Amount


Monday B
Tuesday
Wednesday
Thursday C
Friday
Saturday
Sunday
Monday

Page 337 of 728

Subscribe the Xpress Video Course & Mock Test Package for Bank & Insurance Exams
If there are any suggestions/ errors in our PDFs Feel Free to contact us via this email: admin@exampundit.in
IBPS RRB PO Prelims – Ultra Practice Bundle PDF
 A was purchased thing for either Rs.150 or Rs.350 and only one person was purchased things between A and
C.
 A was purchased things before C.

Days Persons Amount


Monday B
Tuesday A Rs.150/Rs.350
Wednesday
Thursday C
Friday
Saturday
Sunday
Monday

 Not more than four persons have purchased things before H.


 No one has purchased things between H and F, who purchased two days before G.

Days Persons Amount


Monday B
Tuesday A Rs.150/Rs.350
Wednesday
Thursday C
Friday H
Saturday F
Sunday
Monday G

 D has purchased things before F.


 F has purchased things for the highest amount.

Days Persons Amount


Monday B
Tuesday A Rs.150/Rs.350
Wednesday D
Thursday C
Friday H
Saturday F Rs.1600
Sunday
Monday G

Page 338 of 728

Subscribe the Xpress Video Course & Mock Test Package for Bank & Insurance Exams
If there are any suggestions/ errors in our PDFs Feel Free to contact us via this email: admin@exampundit.in
IBPS RRB PO Prelims – Ultra Practice Bundle PDF

 E has purchased things for the second-highest amount.

Days Persons Amount


Monday B
Tuesday A Rs.150/Rs.350
Wednesday D
Thursday C
Friday H
Saturday F Rs.1600
Sunday E Rs.800
Monday G

 B has purchased things for Rs.100 more than D.


 B doesn’t purchase things for Rs.500.

Note: Given, B=Rs.100 + D (only possible combination i.e. Rs.500 and Rs.600)

Thus D=Rs.500 and B=Rs.600

Days Persons Amount


Monday B Rs.600
Tuesday A Rs.150/Rs.350

Wednesday D Rs.500
Thursday C
Friday H
Saturday F Rs.1600
Sunday E Rs.800
Monday G
 The difference between the amounts spent by C and G is Rs.50.
 C has purchased things for more amount than G.

Note: Given, C-G=Rs.50 (C>G) [Two possible combinations (Rs.150, Rs.200) and (Rs.350 and Rs.400)

1st possibility: C=Rs.200, G=Rs.150 (Here A=Rs.350)

2nd possibility: C=Rs.400, G=Rs.350 (Here A=Rs.150)

Case-1 (C=Rs.200, G=Rs.150) Case-2 (C=Rs.400, G=Rs.350)


Page 339 of 728

Subscribe the Xpress Video Course & Mock Test Package for Bank & Insurance Exams
If there are any suggestions/ errors in our PDFs Feel Free to contact us via this email: admin@exampundit.in
IBPS RRB PO Prelims – Ultra Practice Bundle PDF
Days Persons Amount Days Persons Amount
Monday B Rs.600 Monday B Rs.600
Tuesday A Rs.350 Tuesday A Rs.150
Wednesday D Rs.500 Wednesday D Rs.500
Thursday C Rs.200 Thursday C Rs.400
Friday H Friday H
Saturday F Rs.1600 Saturday F Rs.1600
Sunday E Rs.800 Sunday E Rs.800
Monday G Rs.150 Monday G Rs.350

 H doesn’t purchase things for Rs.200.

Case-1, H spent Rs.400 for purchasing things.

Case-2 gets eliminated since H doesn’t purchase things for Rs.200.

Case-1 Case-2, Eliminated


Days Persons Amount Days Persons Amount
Monday B Rs.600 Monday B Rs.600
Tuesday A Rs.350 Tuesday A Rs.150
Wednesday D Rs.500 Wednesday D Rs.500
Thursday C Rs.200 Thursday C Rs.400
Friday H Rs.400 Friday H Rs.200
Saturday F Rs.1600 Saturday F Rs.1600
Sunday E Rs.800 Sunday E Rs.800
Monday G Rs.150 Monday G Rs.350

FINAL ARRANGEMENT

Case-1
Days Persons Amount
Monday B Rs.600
Tuesday A Rs.350
Wednesday D Rs.500
Thursday C Rs.200
Friday H Rs.400
Saturday F Rs.1600
Sunday E Rs.800
Monday G Rs.150

Page 340 of 728

Subscribe the Xpress Video Course & Mock Test Package for Bank & Insurance Exams
If there are any suggestions/ errors in our PDFs Feel Free to contact us via this email: admin@exampundit.in
IBPS RRB PO Prelims – Ultra Practice Bundle PDF

46. A) H

47. A) None

48. C) Rs.400

49. D) The one who purchased things for Rs.200

50. B) Only I

16). Parallel Row Seating Arrangement


Direction (1-5): Study the following information b. Block G is not adjacent to block C.
carefully and answer the questions given below: c. Block E is not neighbour of that block which is
In Delhi there is a residential society named Dwarkapuri opposite C.
which has Twelve blocks A, B, C, D, E, F, G, H, I, J, K d. Block C is 3rd to the right of D.
and L. These blocks form two rows. Blocks on row 1 are e. None of these
facing west and blocks on row 2 are facing east. Blocks 2) Which of the following blocks are at the ends of the
C, D, I, F, G and L are in row 1 and blocks A, B, E, J, K row?
and H are in row 2. Each blocks of row 1 is exactly a. I, G
opposite the other block in row 2. Block I, which is at b. C, D
one of the ends of the row, is second to the left of block c. H, A
L. There are four blocks between block J and K. Block H d. K, B
is third to the right of B. Block J sits to the left end of the e. J, F
row. Block H is not opposite to either I or L. Block I is 3) Block I is related to block K in the same way as
third to the left of C. Block F is not opposite to H. Block block L is related to E, based on the given
B is not adjacent to E. Block E is not opposite to I. Block arrangement. Who among the following is C related
G is third to the right of the one, who is opposite to E. to, following the same pattern?
Block A is neither opposite to G nor I. a. A
1) Which is following statements is true about C? b. J
a. Block E is opposite to block C. c. B
Page 341 of 728

Subscribe the Xpress Video Course & Mock Test Package for Bank & Insurance Exams
If there are any suggestions/ errors in our PDFs Feel Free to contact us via this email: admin@exampundit.in
IBPS RRB PO Prelims – Ultra Practice Bundle PDF
d. H than the number of persons sitting between A and D. A is
e. None of these not an immediate neighbor of B. I sit in row 1. H does
4) How many blocks are there between blocks E and not sit at the extreme end of the row. D sits third to the
K? left of E.
a. Four 6) Who among the following sits at the extreme left
b. Three end of Row 1?
c. Two a. B
d. One b. J
e. None of these c. F
5) Which block is placed opposite to block B? d. E
a. A e. None of these
b. L 7) Who sits 2nd to the right of B?
c. F a. A
d. G b. B
e. I c. F
d. E
Direction (6-10): Study the following information e. None of these
carefully and answer the questions given below: 8) Who sits to the immediate right of A?
Ten persons A, B, C, D, E, F, G, H, I and J are sitting in a. B
two parallel rows such that five members are sitting in b. F
each row. The persons sitting in row 1 faces north while c. None
persons in row 2 faces south direction. Therefore, in the d. G
given seating arrangement persons sitting in row 1 are e. E
facing the persons sitting in the row 2. 9) Who among the following sits opposite to I?
C sits in middle of the row. G sits to the immediate left a. B
of the person who faces C. F sits at the extreme end of b. J
the row. B sits third to the left of the person who faces F. c. A
Number of persons sitting between H and J is two less d. H
Page 342 of 728

Subscribe the Xpress Video Course & Mock Test Package for Bank & Insurance Exams
If there are any suggestions/ errors in our PDFs Feel Free to contact us via this email: admin@exampundit.in
IBPS RRB PO Prelims – Ultra Practice Bundle PDF
e. None of these a. A
10) Four of the following five are related to each b. C
other in some way and thus formed a group. Choose c. P
the one which does not belong to that group. d. R
a. A e. None of these
b. B 13) How many persons sit between P and T in the
c. D same row?
d. F a. Three
e. J b. Four
Direction (11-15): Study the following information c. Five
carefully and answer the questions given below: d. One
Twelve persons are sitting in parallel rows. A, B, C, D, E e. Two
and F sit in row 1 while P, Q, R, S, T and U sit in row 2. 14) Who sits second to the left of R?
Persons in row 1 and row 2 face each other. Persons of a. P
row 1 face south while persons of row 2 face north. b. U
B sits second to the right of C, who faces R. One person c. T
sits between R and T, who sits at one of the extreme d. Q
ends. B does not sit at any extreme end. Q sits to the e. None of these
immediate left of P. U sits opposite to E, who is not 15) Who sits to the immediate left of U?
adjacent to B. F sits second to the right of A but not a. T
opposite to S. b. C
11) Who sits opposite to T? c. A
a. E d. R
b. D e. None of these
c. B Direction (16-20): Study the following information
d. A carefully and answer the questions given below:
e. None of these 8 persons sit in two parallel rows in such a way that in
12) Who sits to the immediate right of E? row 1, P, S, T and V sit facing south while Q, R, U and
Page 343 of 728

Subscribe the Xpress Video Course & Mock Test Package for Bank & Insurance Exams
If there are any suggestions/ errors in our PDFs Feel Free to contact us via this email: admin@exampundit.in
IBPS RRB PO Prelims – Ultra Practice Bundle PDF
W face north. Row 1 person’s face row 2 persons and d. One
vice versa. e. Two
W faces the person, who sits 2nd to the left of V. T sits 20)Four of the following five are related to each other
adjacent to V. S faces R. P doesn’t sit at the left end of in some way and thus formed a group. Choose the
his row. Q sits to the right of U. one which does not belong to that group.
16) Who faces Q? a. V
a. T b. W
b. P c. S
c. V d. U
d. Can’t be determined e. R
e. None of these Direction (21-25): Study the following information
17) Find the odd one out. carefully and answer the questions given below:
a. T Twelve persons are sitting in two parallel rows such that
b. V persons in each row are facing each other. P, Q, R, S, T
c. W and U sit in row 1 and face south while A, B, C, D, E and
d. Q F sit in row 2 and face north. Distance between each
e. P adjacent person in each row is same.
18) __ sits to the immediate right of T and W sits to Two persons are sitting between Q and U. C is sitting
the immediate right of ___. opposite to the one, who is sitting to the immediate left
a. V, U of U. Three persons are sitting between C and D. B, who
b. P, R is sitting opposite to S, is sitting adjacent to C. Two
c. V, Q persons are sitting between S and P. F is not sitting
d. S, U opposite to P and U. A is sitting adjacent to E, who is not
e. None of these sitting adjacent to the person, who is sitting opposite to
19) How many persons sit between U and R? T. R is not sitting adjacent to Q.
a. Zero 21) How many persons are sitting between Q and S?
b. Either a or d. a. 2
c. None of these. b. 1
Page 344 of 728

Subscribe the Xpress Video Course & Mock Test Package for Bank & Insurance Exams
If there are any suggestions/ errors in our PDFs Feel Free to contact us via this email: admin@exampundit.in
IBPS RRB PO Prelims – Ultra Practice Bundle PDF
c. 3 Directions (26-30): Study the following information
d. Cannot be determined and answer the given questions.
e. 4 Twelve people are sitting in two parallel rows containing
22) ______ is sitting opposite to T. six people each, in such a way that there is an equal
a. C distance between adjacent persons. In row-1, Amit,
b. F Belal, Cintu, Dinesh, Firoz, and Gaurav are seated and
c. B all of them are facing south. In row-2, Manish, Neetu,
d. E Om, Prakash, Qamar, and Rohan are seated and all of
e. None of these them are facing north. Therefore, in the given seating
23) How many persons are sitting between D and E? arrangement each member seated in a row faces another
a. 2 member of the other row.
b. 3 Prakash sits fourth to the right of Manish. Manish faces
c. 1 the person who sits to the immediate right of Gaurav.
d. Cannot be determined Two people sit between Cintu and Gaurav who faces
e. None of these Rohan. Three persons sit between Rohan and Om. Om
24) Find the odd one out. faces Dinesh who is third to the left of Amit. Manish
a. Q does not face Firoz. Neetu and Qamar are immediate
b. U neighbours of each other and Qamar is not an immediate
c. D neighbor of Rohan.
d. S 26) Belal is related to Amit in the same way as Firoz
e. B related to Dinesh. To whom of the following people is
25) How many persons are sitting in the right of A? Qamar related, if we follow the same pattern?
a. 4 a. Neetu
b. 1 b. Manish
c. 3 c. Om
d. 2 d. Prakash
e. Cannot be determined e. Rohan
27) Which of the following is true regarding Firoz?
Page 345 of 728

Subscribe the Xpress Video Course & Mock Test Package for Bank & Insurance Exams
If there are any suggestions/ errors in our PDFs Feel Free to contact us via this email: admin@exampundit.in
IBPS RRB PO Prelims – Ultra Practice Bundle PDF
a. Firoz sits second to the left of one who faces Rohan Twelve people are sitting in two parallel rows. Six
b. Firoz is not an immediate neighbor of Amit persons M, N, O, P, Q and R are facing south and the
c. Firoz faces Prakash other six U, V, W, X, Y and Z are facing north. Each
d. Firoz is third to the right of Dinesh person from one row faces exactly one person from the
e. Firoz faces Manish other row but not necessarily in the same order.
28) How many persons are seated between Manish P sits third to the right of N and one of them sits at the
and Prakash? extreme end. The one who faces N sits second to the
a. One right of Y. Only one person sits between Y and V. Z
b. Three does not face an immediate neighbor of O. V does not sit
c. More than three at the extreme end. Q sits second to the left of M. R does
d. None not face U. Immediate neighbor of V sits opposite to R.
e. Two Two people sit between X and Z.
29) Who amongst the following sits second to the 31) Four of the following five are alike in a certain
right of Neetu? way and hence form a group. Who among the
a. Who faces Dinesh following does not belong to that group?
b. Qamar a. N - V
c. Rohan b. R – Z
d. Prakash c. Q - X
e. The person who faces Firoz d. M - Y
30) Who amongst the following sit at extreme ends of e. P – U
the rows? 32) Which of the following statement/statements
a. Manish, Amit is/are true about Z?
b. Dinesh, Rohan a. Z sits opposite to P
c. Belal, Om b. More than one person sit between W and Z
d. Firoz, Neetu c. Z is an immediate neighbor of the one who faces Q
e. Om, Gaurav d. Both 1) or 3)
Direction (31-35): Read the given information e. All 1), 2) and 3)
carefully and answer the questions asked below. 33) Who among the following sit at extreme ends?
Page 346 of 728

Subscribe the Xpress Video Course & Mock Test Package for Bank & Insurance Exams
If there are any suggestions/ errors in our PDFs Feel Free to contact us via this email: admin@exampundit.in
IBPS RRB PO Prelims – Ultra Practice Bundle PDF
a. R, X D sits third to right of B. Either D or B sits at an extreme
b. P, W end of the line. The one who faces B sits second to right
c. W, N of T. Two people sit between Q and V. Neither Q nor V
d. Both 1) and 2) sits at an extreme end of the line. The immediate
e. Both 1) and 3) neighbour of Q faces the person who sits third to the left
34) How many people sit between R and N? of A. C and E are immediate neighbours of each other. R
a. One sits second to the left of P. E does not face the immediate
b. Two neighbour of S.
c. Three 36) Four of the following five are alike in a certain
d. Four way based on the given arrangement and so form a
e. None group. Which is the one that does not belong to that
35) Who among the following faces O? group?
a. W a. PT
b. V b. DF
c. Z c. CE
d. X d. QS
e. U e. DE
Direction (36-40): Study the given information 37) Which of the following statement is true?
carefully and answer the following questions. a. F faces P.
Twelve people are sitting in two parallel rows containing b. S is an immediate neighbour of T.
six people each, in such a way that there is an equal c. E faces the person who is immediate right of Q.
distance between adjacent persons. In row-1 A, B, C, D, d. Three people sit between F and C.
E and F are seated and all of them facing south. In row-2 e. V is second to left of T.
P, Q, R, S, T and V are seated and all of them are facing 38) A is related to T in the same way as F is related to
north. Therefore, in the given seating arrangement each Q based on the given arrangement. To which of the
member seated in a row faces another member of the following is D related to, following the same pattern?
other row. a. S
b. Q
Page 347 of 728

Subscribe the Xpress Video Course & Mock Test Package for Bank & Insurance Exams
If there are any suggestions/ errors in our PDFs Feel Free to contact us via this email: admin@exampundit.in
IBPS RRB PO Prelims – Ultra Practice Bundle PDF
c. T any of the extreme ends. V is not an immediate neighbor
d. Data Inadequate of T. B sits at one of the extreme ends. Only two people
e. None of these sit between B and E. E does not face V. Two persons sit
39) How many persons are seated between R and T? between R and Q. R is not an immediate neighbor of T.
a. One C does not face V. P is not an immediate neighbor of R.
b. Two 41) Who is second to the left of person who is facing
c. Three E?
d. Four a. P
e. None b. S
40) Who among the following sit at extreme ends of c. V
the rows? d. Q
a. BF e. None of these
b. TE 42) Which of the following is true regarding F?
c. SQ a. F sits second to right of V.
d. RS b. F is not an immediate neighbor of A.
e. None of these c. F sits third to left of D.
Directions (41-45): Study the following information to d. F sits at one of the extreme ends of the line.
answer the given questions: e. F faces V.
Twelve people are sitting in two parallel rows containing 43) How many persons are seated between T and S?
six people each, in such a way that there is an equal a. One
distance between adjacent persons. In row-1, P, Q, R, S, b. Two
T and V are seated and all of them are facing south. In c. Three
row-2, A, B, C, D, E and F are seated and all of them are d. Four
facing North. Therefore, in the given seating e. None
arrangement each member seated in a row faces another 44) Who amongst the following faces A?
member of the other row. a. R
A sits third to right of D. Neither A nor D sits at extreme b. T
ends. T faces D. V does not face A and V does not sit at c. P
Page 348 of 728

Subscribe the Xpress Video Course & Mock Test Package for Bank & Insurance Exams
If there are any suggestions/ errors in our PDFs Feel Free to contact us via this email: admin@exampundit.in
IBPS RRB PO Prelims – Ultra Practice Bundle PDF
d. Q d. Q
e. S e. None of these
45) Who amongst the following sit at extreme ends of 47) How many persons sit between A and D?
the rows? a. Three
a. B, E b. Two
b. S, T c. One
c. P, R d. Four
d. B, F e. None of these
e. None of these 48) Who sits to the immediate right of S?
Directions (46-50): Study the following information to a. U
answer the given questions: b. T
Twelve persons are sitting in a parallel row such that six c. R
persons sit in each row. P, Q, R, S, T and U sit in row 1 d. Q
and A, B, C, D, E and F sit in row 2. Persons in row 1 e. None of these
face south while persons in row 2 face north. Persons in 49) How many persons sit to the left of A?
row 1 and row 2 are facing towards each other. a. Three
U sits to the immediate right of T, who does not sit at b. Five
extreme end. P sits second to the right of one, who sits c. Four
opposite to E. At most two persons sit to the right of E. F d. One
sits second to the left of C. Q sits opposite to F. C is not e. Cannot be determined
adjacent to E. R sits opposite to B, who is not adjacent to 50) Who sits to the immediate right of R?
E. a. P
46) Who sits opposite to C? b. Q
a. U c. T
b. T d. S
c. S e. None of these

Page 349 of 728

Subscribe the Xpress Video Course & Mock Test Package for Bank & Insurance Exams
If there are any suggestions/ errors in our PDFs Feel Free to contact us via this email: admin@exampundit.in
IBPS RRB PO Prelims – Ultra Practice Bundle PDF
16). Parallel Row Seating Arrangement - Answer with Explanation
Answers:
1. b
2. a
3. a
4. d
5. c
Solutions 1-5
1) Block I, which is at one of the ends of the row, is
6) Bock G is third to the right of the one, who is opposite
second to the left of block L.
to E.
7) Block A is neither opposite to G nor I.
8) Block J sits to the left end of the row.

2) Block H is third to the right of B.


3) Block H is not opposite to either I or L.

9) Block I is third to the left of C.


10) Block F is not opposite to H.

4) Block B is not adjacent to E.


5) Block E is not opposite to I.
Page 350 of 728

Subscribe the Xpress Video Course & Mock Test Package for Bank & Insurance Exams
If there are any suggestions/ errors in our PDFs Feel Free to contact us via this email: admin@exampundit.in
IBPS RRB PO Prelims – Ultra Practice Bundle PDF

4. Number of persons sit between H and J is two less


than the number of persons sit between A and D.
Answers: 5. D sits third to the left of E which means A, D and E
6. c are sitting in the same row.
7. d Case I:
8. c
9. a
10. b
Solutions 6-10
1. C sits in middle of the row so, C sits either in row 1 or
row 2.
2. G sits immediate left of the person who faces C. Case II:
3. F sits at the extreme end of the row. B sits third to the
left of the person who faces F.
Case I:

6. I sits in row 1 which is not possible in case I so, case I


is invalid.
Case II:
The final arrangement is as follows:

Page 351 of 728

Subscribe the Xpress Video Course & Mock Test Package for Bank & Insurance Exams
If there are any suggestions/ errors in our PDFs Feel Free to contact us via this email: admin@exampundit.in
IBPS RRB PO Prelims – Ultra Practice Bundle PDF

Answers: 5. U sits opposite to E, who is not adjacent to B.

11. b 6. F sits second to the right of A but not opposite to S.

12. b So, case 1 is rejected.

13. a So, we have,

14. a
15. d
Solutions 11-15
1. B sits second to the right of C, who faces R.
2. One person sits between R and T, who sits at one of
the extreme ends. Answers:
3. B does not sit at any extreme end. 16. a
4. Q sits immediate left of P. 17. b
So, we have two cases, 18. c
Case-1 19. e
20. b
Solutions 16-20
1. W faces the person, who sits 2nd to the left of V.
2. T sits adjacent to V
So, there will be two cases
Case-2
Case-1

Page 352 of 728

Subscribe the Xpress Video Course & Mock Test Package for Bank & Insurance Exams
If there are any suggestions/ errors in our PDFs Feel Free to contact us via this email: admin@exampundit.in
IBPS RRB PO Prelims – Ultra Practice Bundle PDF
22. b
23. a
24. b
25. c
Solutions 21-25
1. Two persons are sitting between Q and U.
2. C is sitting opposite to the person, who is sitting to the
Case-2
immediate left of U.
3. Three persons are sitting between C and D.
4. B, who is sitting opposite to S, is sitting adjacent to C.
So there will be two cases
Case-1

4. S faces R.
5. P doesn’t sit at the left end of his row.
So case II is rejected
6. Q sits to the right of U.
The final seating arrangement is given below: Case-2

5. Two persons are sitting between S and P.


6. R is not sitting adjacent to Q.
Answers:
7. A is sitting adjacent to E, who is not sitting adjacent to
21. e
the person, who is sitting opposite to T.
Page 353 of 728

Subscribe the Xpress Video Course & Mock Test Package for Bank & Insurance Exams
If there are any suggestions/ errors in our PDFs Feel Free to contact us via this email: admin@exampundit.in
IBPS RRB PO Prelims – Ultra Practice Bundle PDF
8. F is not sitting opposite to P and U
Hence, Case-1 is eliminated.

4) Three persons sit between Rohan and Om. Thus case-


2 gets rejected
Answers: 5) Om faces Dinesh who is third to the left of Amit.
26. e 6) Manish does not face Firoz.
27. a 7) Neetu and Qamar are immediate neighbours of each
28. b other and Qamar is not an immediate neighbor of Rohan.
29. d
30. c
Solutions 26-30
1) Prakash sits fourth to the right of Manish.
2) Manish faces the person who sits to the immediate
right of Gaurav. Answers:
3) Two people sit between Cintu and Gaurav who faces 31. a
Rohan. 32. d
Case-1 33. e
34. d
35. e
Solutions 31-35
1) P sits third to the right of N and one of them sits at the
extreme end.
2) The one who faces N sits second to the right of Y.
Case-2

Page 354 of 728

Subscribe the Xpress Video Course & Mock Test Package for Bank & Insurance Exams
If there are any suggestions/ errors in our PDFs Feel Free to contact us via this email: admin@exampundit.in
IBPS RRB PO Prelims – Ultra Practice Bundle PDF
3) Only one person sits between Y and V. Vdoes not sit 36. e
at the extreme end. 37. e
4) Q sits second to the left of M. 38. a
Case-1 39. b
40. d
Solutions 36-40
1) D sits third to right of B.
2) Either D or B sits at an extreme end of the line.
3) The one who faces B sits second to right of T.

Case-2 Case 1

4) Immediate neighbor of V sits opposite to R. (This Case 2


eliminates case 1)
5) Two people sit between X and Z.
6. Z does not face an immediate neighbor of O.
7) R does not face U.

4) Two people sit between Q and V.


5) Neither Q nor V sits at an extreme end of the line.
6) The immediate neighbor of Q faces the person who
sits third to the left of A. Hence, Case- 2 is eliminated

Answers:
Page 355 of 728

Subscribe the Xpress Video Course & Mock Test Package for Bank & Insurance Exams
If there are any suggestions/ errors in our PDFs Feel Free to contact us via this email: admin@exampundit.in
IBPS RRB PO Prelims – Ultra Practice Bundle PDF
4) V does not face A and V does not sit at any of the
extreme ends.
5) V is not an immediate neighbor of T.

7) C and E are immediate neighbors of each other.


8) R sits second to the left of P.
9) E does not face the immediate neighbor of S.

6) B sits at one of the extreme ends.


7) Only two people sit between B and E.
8) E does not face V.

Answers:
41. b
42. e
43. b
9) Two persons sit between R and Q.
44. e
10) R is not an immediate neighbor of T.
45. c
11) C does not face V.
Solutions 41-45
12) P is not an immediate neighbor of R.
1) A sits third to right of D.
2) Neither A nor D sits at extreme ends.

Answers:
3) T faces D. 46. e
Page 356 of 728

Subscribe the Xpress Video Course & Mock Test Package for Bank & Insurance Exams
If there are any suggestions/ errors in our PDFs Feel Free to contact us via this email: admin@exampundit.in
IBPS RRB PO Prelims – Ultra Practice Bundle PDF
47. c
48. b
49. e
50. b
Solutions 46-50
1. P sits second to the right of one, who sits opposite to Case-3
E.
2. At most two persons sit to the right of E.
3. F sits second to the left of C.
4. Q sits opposite to F.
5. C is not adjacent to E.
So, we have three cases,
6. U sits immediate right of T, who does not sit at
Case-1
extreme end.
So, case 2 is rejected.
7. R sits opposite to B, who is not adjacent to E.
So, case 3 is rejected.

Case-2

17). Flat & Floors Based Puzzle


Directions (1-5): Answer the following questions different floors among floor 1, floor 2, floor 3 and floor
according to the given statements: 4. Each floor has two different flats viz. flat 1 and flat 2.
There are eight persons Ahmed, Parvez, Sameer, Nargis, There are 2 flats on each floor flat 1 – flat 2 from left to
Nadeem, Shirazi, Usman and Mishra, who live on four right (when seen in north direction) in such a way that
Page 357 of 728

Subscribe the Xpress Video Course & Mock Test Package for Bank & Insurance Exams
If there are any suggestions/ errors in our PDFs Feel Free to contact us via this email: admin@exampundit.in
IBPS RRB PO Prelims – Ultra Practice Bundle PDF
flat 1 of the fourth floor is exactly above flat 1 of the c. Sameer and Parvez
third floor and flat 1 of the third floor is exactly above d. Nadeem and Nargis
flat 1 of the second floor and other flats are situated in e. None of the above
the same way. 4) Who lives two floors below the floor on which
Sameer lives one floor above the floor on which Parvez Usman lives in the same flat number?
lives. Shirazi lives with Parvez who lives two floors a. Ahmed
below Nargis. Parvez lives on the second floor. Nadeem b. Shirazi
lives in flat 2 of the first floor. Usman lives on the third c. Sameer
floor but not in flat 1. Mishra does not live on the fourth d. Nadeem
floor but lives in flat 1. Parvez lives in even numbered e. None of these
flat. Ahmed and Sameer live in flat 1 but on different 5) Who live in the flat 1 of floor 2?
floors. a. Parvez
1) Who live on the fourth floor of flat 2? b. Sameer
a. Mishra c. Shirazi
b. Sameer d. Ahmed
c. Ahmed e. Usman
d. Nadeem Direction (6-10): Study the following information
e. Nargis carefully and answer the questions that follow:
2) Which of the following given pair is different from Eight people Himanshi, Nidhi, Abhi, Priya, Vinayak,
rest? Smriti, Kiran, Shweta live in a building with 4 floors but
a. Mishra and Sameer not necessarily in the same order. Each floor has two
b. Usman and Nadeem flats i.e. flat 1 and flat 2. Flat 1 is west of Flat 2. The
c. Sameer and Shirazi lowermost building floor is numbered 1, the floor above
d. Nadeem and Usman it 2 and so on till the topmost floor is numbered 4. Flat 1
e. Parvez and Nargis is west of Flat 2.
3) Who lives on the same floors? There is 1 floor between the floors on which Smriti and
a. Mishra and Usman Vinayak live and both live on the same flat but not on
b. Shirazi and Parvez even numbered flat. Shweta lives 3 floors below the
Page 358 of 728

Subscribe the Xpress Video Course & Mock Test Package for Bank & Insurance Exams
If there are any suggestions/ errors in our PDFs Feel Free to contact us via this email: admin@exampundit.in
IBPS RRB PO Prelims – Ultra Practice Bundle PDF
person who lives on the top floor. Abhi lives on an even- 9) On which floor does Abhi live?
numbered floor but he does not live on the top floor. a. 2nd
Himanshi lives on the floor immediately above Vinayak b. 4th
and both lives on same flat. Nidhi and Vinayak do not c. 3rd
live on the top floor. Himanshi live 2 floors below the d. 1st
floor on which Priya lives and both lives on same flat. e. None of these
Priya and Kiran do not live on the floor immediately next 10) Who live immediately below Priya?
to Abhi’s floor. a. Smriti
6) Who lives on Top floor? b. Kiran
a. Smriti c. Abhi
b. Kiran d. Vinayak
c. Abhi e. Both b and d
d. Priya Direction (11-15): Study the following information
e. Both b and d carefully and answer the questions that follow:
7) How many floors are there between Kiran and Ten persons Peeyush, Qasim, Rehan, Sudhir, Tahir,
Shweta? Ansh, Biden, Chris, Dinesh and Ethan are living in a five
a. Two storey building such that ground floor is numbered as 1,
b. None above it is floor 2 then top floor is numbered as 5. Each
c. One of the floor has 2 flats in it as flat-1 and flat-2. Flat-1 of
d. Either a or c floor-2 is immediately above flat-1 of floor-1 and
e. None of these immediately below flat-1 of floor-3 and so on. In the
8) Which of the following statements is correct? same way flat-2 of floor-2 is immediately above flat-2 of
a. Smriti lives on the third floor floor-1 and immediately below flat-2 of floor-3 and so
b. Priya lives on a floor above Kiran on. Flat-1 is in west of flat-2.
c. There is three persons living between Priya and Qasim lives in an odd numbered floor. Both Tahir and
Vinayak Biden livesin the same flat number. Rehan and Ansh live
d. Nidhi lives immediately above Kiran in the same floor. Chris does not live in flat number 2
e. Himanshi lives immediately below Vinayak and floor 1. There is a gap of two floors in which Qasim
Page 359 of 728

Subscribe the Xpress Video Course & Mock Test Package for Bank & Insurance Exams
If there are any suggestions/ errors in our PDFs Feel Free to contact us via this email: admin@exampundit.in
IBPS RRB PO Prelims – Ultra Practice Bundle PDF
and Peeyhush lives. Both Peeyush and Qasim do not live c. None
in the same flat number. Tahir lives immediately above d. Three
Peeyush in the same flat number. There is a gap of one e. Cannot be determined
floor in which Tahir and Biden lives. Ethan, who lives on 14) Biden lives on which of the following floor?
an even numbered floor, lives below Ansh and both lives a. Floor-1
in the same flat number. Dinesh lives above Sudhir and b. Floor-2
both live in the same flat number. Chris and Sudhir c. Floor-3
neither live in the same flat number nor in the same d. Floor-4
floor. Dinesh lives below chris, who lives in flat 1, and e. Floor-5
both lives in different flat. 15) Four of the following five are alike in a certain
11) Who among the following lives in flat-2 of 4th way and hence form a group, which among the
floor? following does not belong to that group?
a. Rehan a. Ansh
b. Ansh b. Ethan
c. Peeyush c. Sudhir
d. Ethan d. Dinesh
e. None of these e. Tahir
12) Who among the following lives with Qasim in the Direction (16-20): Study the following information
same floor? carefully and answer the questions that follow:
a. Chris There are seven persons i.e. P, Q, R, S, T, U and V who
b. Biden lives in a building having three floors such that ground
c. Ethan floor is numbered as 1 and above it is 2 and so on up to
d. Dinesh top floor numbered as 3. Each of the floor consist of 3
e. None of these flats as flat-1, flat-2 and flat-3. Flat-1 of floor-2 is
13) How many floors are there between Chris and immediately above flat-1 of floor-1 and immediately
Rehan? below flat-1 of floor-3 and in the same way flat-2 of
a. One floor-2 is immediately above flat-2 of floor-1 and
b. Two immediately below flat-2 of floor-3 and so on. Flat-2 is
Page 360 of 728

Subscribe the Xpress Video Course & Mock Test Package for Bank & Insurance Exams
If there are any suggestions/ errors in our PDFs Feel Free to contact us via this email: admin@exampundit.in
IBPS RRB PO Prelims – Ultra Practice Bundle PDF
in east of flat-1 and flat-3 is in east of flat-2. Not less a. 1st Floor
than two person lives on the same floor. Only one person b. 2nd Floor
lives in a flat. Two flats are vacant. c. 3rd Floor
Only one floor is between U and V but both live in same d. Can’t be determined
flat number. S lives immediately west to V. T lives e. Either 1st or 3rd
immediately above R and both are living in same flat. No 19) Which of the following is true regarding P?
one lives on the east of U. R does not lives in the same a. Flat 1- 2nd Floor
flat number as S. T and S do not live on the same floor. P b. Flat 3- 3rd Floor
does not live in the same flat number as R. Q live below c. Flat 1- 3rd Floor
the floor on which P lives but not in the same flat d. Flat 2- 2nd Floor
number. Q lives in an even numbered flat and on even e. None of these
numbered floor but not on the same floor on which R 20) Who among the following lives immediately below
lives. T in same flat?
16) Who among the following lives immediately above a. P
Q in same flat? b. Q
a. P c. R
b. S d. S
c. V e. U
d. U
e. None of these Direction (21-25): Study the following information
17) P live on which of the following flat? carefully and answer the questions that follow:
a. Flat 1 10 persons live in a building of 5 floors such that bottom
b. Flat 2 floor is numbered 1 and floor above it is 2 and so on up
c. Flat 3 to fifth floor. Each of the floor consist of 2 flats as flat-1,
d. Either (a) or (b) which is in west of flat-2. Flat-1 of floor-2 is
e. Either (a) or (c) immediately above flat-1 of floor-1 and immediately
18) On which of the following floor three persons below flat-1 of floor-3.
lives?
Page 361 of 728

Subscribe the Xpress Video Course & Mock Test Package for Bank & Insurance Exams
If there are any suggestions/ errors in our PDFs Feel Free to contact us via this email: admin@exampundit.in
IBPS RRB PO Prelims – Ultra Practice Bundle PDF
I live west of D. F lives above H. E lives in an odd 24) Who among the following lives to the east of C?
numbered flat of even numbered floor. G lives west of B a. H
on one of the floors below E. A life on odd numbered b. D
floor but below C. A and J lives on different numbered c. G
flats. There are three floors in between floors of B and C. d. F
Only two floors are in between the floors of C and J. C e. B
lives Northwest to J. 25) Four of the following five belongs to a group. Who
21) Who among the following lives in flat 2 of 4th does not belong to the group?
floor? a. F
a. C b. I
b. A c. J
c. D d. D
d. B e. E
e. J Direction (26-30): Study the following information
22) How many floors are there between floors of F carefully and answer the questions that follow:
and J? There is a group of 6 persons living in a 4-storey
a. two building. The persons are A, B, C, D, E and F. Each
b. None storey of the building has 2 flats. Thus, there are 8 flats
c. one in all and 2 of them are vacant. Persons living on the
d. three floors are said to be living on the any of the flat 1 or 2.
e. Cannot be determined Ex- B lives on Top floor so B can live on either Flat 1 or
Flat 2. Flat-1 is in west of flat-2.
23) On which floor does H live? C is heavier than B, who in turn is heavier than F. The
a. Floor 5 heaviest of the group lives on the top floor while the
b. Floor 4 lightest lives on the ground floor. E is lighter than D.
c. Floor 3 One of the flats on the third floor is vacant while one on
d. Floor 2 the second floor is also vacant and both of them on flat 2.
e. Floor 1 A is neither the heaviest nor the lightest. There are only
Page 362 of 728

Subscribe the Xpress Video Course & Mock Test Package for Bank & Insurance Exams
If there are any suggestions/ errors in our PDFs Feel Free to contact us via this email: admin@exampundit.in
IBPS RRB PO Prelims – Ultra Practice Bundle PDF
two people heavier than C where A is not one of them. B e. None of these
shares the floor with the heaviest on flat 2 of the group 30) Starting from the heaviest of the group, what is
while C shares it with the lightest on flat 1. the rank of B in order of weight?
26) Who among the following is the lightest in the a. 3rd
group? b. 4th
a. A c. 5th
b. C d. either 4th or 5th
c. F e. None of these
d. B Direction (31-35): Study the following information
e. None of these carefully and answer the questions that follow:
27) On which floor does A live? There are three floors in a given building such that floor
a. ground 2 is above floor 1 and floor 3 is above floor 2. In the
b. first building there are two types of flats in each floor such.
c. Second Flat A is in the west of Flat B. There are different
d. either first or second number of family members in each flat staring from 4 to
e. None of these 9.
28) Who among the following shares the floor with C? The total members of flat B are three more than flat A.
a. B There is no even number of family members in the even
b. A numbered floor. There are total members of floor number
c. D three is 11. Flat having 6 family members are live above
d. F flat having 8 family members but not necessary in the
e. None of these same type of flat.
29) Starting from the lightest of the group, what is the 31) Total numbers of family members live on Floor 2?
rank of E in order of weight? a. 13
a. 3rd b. 14
b. 4th c. 15
c. 5th d. 16
d. 2nd e. None of these
Page 363 of 728

Subscribe the Xpress Video Course & Mock Test Package for Bank & Insurance Exams
If there are any suggestions/ errors in our PDFs Feel Free to contact us via this email: admin@exampundit.in
IBPS RRB PO Prelims – Ultra Practice Bundle PDF
32) Sum of family members live on Flat A of Floor 1 There are 8 people – A, B, C, D, E, F, G and H who stay
and Flat B of Floor 2? on 5 floors (numbered 1 to 5) of a building. There are
a. 17 two flats on each of the five floors out of which two flats
b. 14 are vacant. The flats are numbered 1 and 2 on each of the
c. 15 floors and are left to right (as per north direction) on the
d. 16 floor respectively. Flat no. 1 of floor no. 2 is exactly
e. 11 above the flat no. 1 of floor no. 1 and so on. Flats which
33) Sum of Family members live in flat B? are empty do have same flat number.
a. 27 H lives on flat number 1 of floor number 1. There is one
b. 24 floor between floors of H and C. C lives above H. B lives
c. 25 on flat which is immediately above C’s and both are
d. 26 living in same flat. E lives immediately above G’s flat
e. None of these and both are living in same flat. A and E share same
34) Which of the following flat has odd number of floor. There are 2 floors between A and D and they live
family members? in same flat number. D lives on one of the flats which is
a. Flat B of Floor 2 immediately above an empty flat. B and F share same
b. Flat B of Floor 3 flat number.
c. Flat A of Floor 1 36) Who lives on flat number 2 of floor number 4?
d. Both a and C a. Empty
e. None of these b. D
35) Which floor has 16 family members? c. B
a. 3rd d. Cannot be determined
b. 2nd e. E
c. 1st 37) H shares floor with which of the following?
d. Both a and b a. D
e. None of these b. F
Direction (36-40): Study the following information c. G
carefully and answer the questions that follow: d. No one
Page 364 of 728

Subscribe the Xpress Video Course & Mock Test Package for Bank & Insurance Exams
If there are any suggestions/ errors in our PDFs Feel Free to contact us via this email: admin@exampundit.in
IBPS RRB PO Prelims – Ultra Practice Bundle PDF
e. Cannot be determined right on the floor respectively (as per north direction).
38) Who shares same floor with F? Flat no. 1 of floor no. 2 is exactly above the flat no. 1 of
a. D floor no. 1 and so on.
b. C Wasim stays on flat no. 1 of the floor just below the floor
c. H on which Sakshi stays. The flat no 2 of floor no 2 is
d. No one empty. Sakshi stays on flat no. 2 on one of the floors.
e. Cannot be determined Urvashi stays on flat no. 2. Teena stays on flat no. 3 of
39) Four of the following are similar in a certain way, same floor on which Wasim stays. Flat to the right of
and so form a group. Find the odd one out. Sakshi is empty. Flat on which Ruhi stays is below the
a. A – E flat of Sakshi such that both are not living in different
b. D-B flats. Flat no 3 of floor no 4 is empty. Jordan stays on flat
c. H-C to the right of Urvashi. Yuri and Veena stay on the same
d. C-D floor. Teena and Yuri do not share same flat number.
e. B-H Zoya does not stay on top floor.
40) How many floors are there between the floors of 41) Who lives immediately below Yuri’s flat?
B and E? a. Jordan
a. None b. Zoya
b. One c. Urvashi
c. Two d. Veena
d. Three e. None of these
e. Cannot be determined 42) How many persons live between Veena and
Direction (41-45): Study the following information Jordan?
carefully and answer the questions that follow: a. 0
There are ten people–Ruhi, Sakshi, Teena, Urvashi, b. 1
Veena, Wasim, Xavier, Yuri, Zoya and Jordan who stay c. 2
on four floors of a building. There are 3 flats on each of d. Either b or c
the 4 floors out of which 2 flats are vacant. The flats are e. None of these
numbered 1, 2 and 3 on each of the floors and are left to 43) On which floor Xavier live?
Page 365 of 728

Subscribe the Xpress Video Course & Mock Test Package for Bank & Insurance Exams
If there are any suggestions/ errors in our PDFs Feel Free to contact us via this email: admin@exampundit.in
IBPS RRB PO Prelims – Ultra Practice Bundle PDF
a. 1st North of Waqar. Firoz lives below Zuber and both lives
b. 2nd in same flat number. Amar lives North of Yuvi but South
c. 3rd of Manish. Daman and Zuber live on same odd number
d. 4th floor. Gautam lives above Varun who does not live on
e. None of these fourth floor.
44) Who among the following lives immediately above 46) Who among the following live between the one
Wasim? who lives in Flat 1 of 4th floor and the one who lives
a. Jordan in Flat 3 of same floor?
b. Zoya a. Waqar
c. Urvashi b. Yuvi
d. Veena c. Eshwar
e. Xavier d. Amar
45) Who lives Flat 3 of Floor 1? e. Yuvi
a. Jordan 47) Which of the following combination is correct
b. Zoya regarding Charlie?
c. Urvashi a. Flat 3, Floor 1
d. Veena b. Flat 1, Floor 2
e. None of these c. Flat 2, Floor4
Direction (46-50): Study the following information d. Flat 3, Floor2
carefully and answer the questions that follow: e. None of these
There are 12 persons who live on 4 different floors of a 48) In which of the direction Balram lives with
same building. Bottommost floor is 1st floor and so on. respect to Amar?
There are 3 flats on each floor – flat-1, flat-2 and flat-3 a. South
from west to east. Such that flat-1 of fourth floor is b. North
exactly above flat -1 of third floor which is exactly above c. South-West
flat -1 of second floor. d. North-East
Varun lives Northwest of Charlie. Balram lives e. Can’t be determined
Southwest to Eshwar. Balram lives west of Yuvi but
Page 366 of 728

Subscribe the Xpress Video Course & Mock Test Package for Bank & Insurance Exams
If there are any suggestions/ errors in our PDFs Feel Free to contact us via this email: admin@exampundit.in
IBPS RRB PO Prelims – Ultra Practice Bundle PDF
49) Who among the following live to the North-West 50) Four of the following five are alike in a certain
of Zuber? way and hence they form a group. Which one of the
a. Balram following does not belong to that group?
b. Manish a. Waqar, Charlie, Firoz
c. Gautam b. Yuvi, Balram, Varun
d. Charlie c. Eshwar, Gautam, Manish,
e. None of these d. Amar, Zuber, Daman
e. Yuvi, Waqar, Varun

17). Flat & Floors Based Puzzle – Solution with detailed Explanation

Answers Flat1 Flat2


1. e Floor4
2. c Floor3
3. b Floor2 Parvez
4. d Floor1
5. c
Solution 1-5 4. Shirazi lives with Parvez who lives two floors
1) Parvez lives on the second floor. below Nargis. Sameer lives one floor above the
Case I floor on which Parvez lives.
Flat1 Flat2 Case I
Floor4 Flat1 Flat2
Floor3 Floor4
Floor2 Parvez Floor3 Sameer
Floor1 Floor2 Shirazi Parvez
Floor1
Case II
Page 367 of 728

Subscribe the Xpress Video Course & Mock Test Package for Bank & Insurance Exams
If there are any suggestions/ errors in our PDFs Feel Free to contact us via this email: admin@exampundit.in
IBPS RRB PO Prelims – Ultra Practice Bundle PDF
Floor4
Case II Floor3 Sameer Usman
Flat1 Flat2 Floor2 Shirazi Parvez
Floor4 Floor1
Floor3 Sameer
Floor2 Parvez Shirazi 4) Mishra does not live on the fourth floor but in the flat1
Floor1
Flat1 Flat2
Case III Floor4
Flat1 Flat2 Floor3 Sameer Usman
Floor4 Floor2 Shirazi Parvez
Floor3 Sameer Floor1 Mishra
Floor2 Shirazi Parvez
Floor1 5) Ahmed and Sameer live in flat 1 but on different
floors. The only flat left for Ahmed is on the fourth floor.
Case IV Flat1 Flat2
Flat1 Flat2 Floor4 Ahmed
Floor4 Floor3 Sameer Usman
Floor3 Sameer Floor2 Shirazi Parvez
Floor2 Parvez Shirazi Floor1 Mishra
Floor1
6) Nadeem lives in flat2 of the first floor. The only
5. Usman live on the third floor but not in flat1. person left is Nargis
Parvez lives in even-numbered flat. Flat1 Flat2
Hence, case II, case III and Case IV are eliminated. Floor4 Ahmed Nargis
Floor3 Sameer Usman
Flat1 Flat2 Floor2 Shirazi Parvez

Page 368 of 728

Subscribe the Xpress Video Course & Mock Test Package for Bank & Insurance Exams
If there are any suggestions/ errors in our PDFs Feel Free to contact us via this email: admin@exampundit.in
IBPS RRB PO Prelims – Ultra Practice Bundle PDF
Floor1 Mishra Nadeem 3 Smriti
Answers 2 Himanshi Abhi
6. e 1 Vinayak Shweta
7. a
8. a 6. Nidhi and Vinayak do not live on the top floor.
9. a 7. Himanshi live 2 floors below the floor on which Priya
10. a lives and both lives on same flat.
Solution 6-10 Floor Flat 1 Flat 2
1. Abhi lives on an even-numbered floor and flat but he 4 Priya Kiran
does not live on the top floor. 3 Smriti Nidhi
2. Priya and Kiran do not live on the floor immediately 2 Himanshi Abhi
next to Abhi’s floor. 1 Vinayak Shweta
3. There is 1 floor between the floors on which Smriti
and Vinayak live and both live on the same flat but not Answers
on even numbered flat. 11. b
Floor Flat 1 Flat 2 12. a
4 Priya/Kiran Priya/Kiran 13. c
3 Smriti/Vinayak 14. a
2 Abhi 15. e
1 Smriti/Vinayak Solution 11-15
1. Qasim lives in an odd numbered floor.
4. Shweta lives 3 floors below the person who lives on 2. There is a gap of two floors in which Qasim and
the top floor. Peeyhush lives.
5. Himanshi lives on the floor immediately above 3. Both Peeyush and Qasim do not live in the same flat
Vinayak. number.
Floor Flat 1 Flat 2 Case-1
4 Priya/Kiran Priya/Kiran Floor Flat 1 Flat 2

Page 369 of 728

Subscribe the Xpress Video Course & Mock Test Package for Bank & Insurance Exams
If there are any suggestions/ errors in our PDFs Feel Free to contact us via this email: admin@exampundit.in
IBPS RRB PO Prelims – Ultra Practice Bundle PDF
5 Qasim 1 Qasim
4
3 4. Tahir lives immediately above Peeyush in the same
2 Peeyush flat number.
1 5. Both Tahir and Biden lives in the same flat number.
6. There is a gap of one floor in which Tahir and Biden
Case-1(a) lives.
Floor Flat 1 Flat 2 7. Ethan, who lives on an even numbered floor, lives

5 Qasim below Ansh and both lives in the same flat number.

4 8. Rehan and Ansh live in the same floor.

3 Hence, Case-1 (a), Case-2, Case -2(a) is eliminated.

2 Peeyush
1 Case-1
Floor Flat 1 Flat 2

Case-2 5 Qasim

Floor Flat 1 Flat 2 4 Rehan Ansh

5 3 Tahir

4 Peeyush 2 Peeyush Ethan

3 1 Biden

2
1 Qasim 9. Chris does not live in flat number 2.

Case -2 (a) 10. Dinesh lives above Sudhir and both live in the same
flat number.
Floor Flat 1 Flat 2
11. Chris and Sudhir neither live in the same flat number
5
nor in the same floor.
4 Peeyush
3
2

Page 370 of 728

Subscribe the Xpress Video Course & Mock Test Package for Bank & Insurance Exams
If there are any suggestions/ errors in our PDFs Feel Free to contact us via this email: admin@exampundit.in
IBPS RRB PO Prelims – Ultra Practice Bundle PDF
Floor Flat 1 Flat 2 2
5 Chris Qasim 1 S V
4 Rehan Ansh Case-3
3 Tahir Dinesh Floor Flat 1 Flat 2 Flat 3
2 Peeyush Ethan 3 S V
1 Biden Sudhir 2
1 U Nil
Answers Case-4
16. d Floor Flat 1 Flat 2 Flat 3
17. a 3 S V
18. a 2
19. c 1 U
20. c
Solution 16-20 4. T lives immediately above R.
1. Only one floor is between U and V but both live in 5. R does not live in the same flat number as S.
same flat number. 6. T and S do not live on the same floor.
2. S lives immediately west to V. So, case 3 will be eliminated.
3. No one lives on the east of U. Case-1
Case-1 Floor Flat 1 Flat 2 Flat 3
Floor Flat 1 Flat 2 Flat 3 3 U Nil
3 U Nil 2 T
2 1 S V R
1 S V Case-2(a)
Floor Flat 1 Flat 2 Flat 3
Case-2 3 U
Floor Flat 1 Flat 2 Flat 3 2 T
3 U 1 R S V

Page 371 of 728

Subscribe the Xpress Video Course & Mock Test Package for Bank & Insurance Exams
If there are any suggestions/ errors in our PDFs Feel Free to contact us via this email: admin@exampundit.in
IBPS RRB PO Prelims – Ultra Practice Bundle PDF
Solution 21-25
Case-2(b) 1. E lives in an odd numbered flat of even numbered
Floor Flat 1 Flat 2 Flat 3 floor.
3 T U 2. G lives west of B on one of the floors below E.
2 R 3. There are three floors in between floors of B and C.
1 S V There will be two possibilities either T lives on 2 nd floor
Case-4 or on 4th floor—

Floor Flat 1 Flat 2 Flat 3 Case-1

3 S V Floor Flat Flat

2 T 5 C/ C/

1 R U 4
3
7. P does not live in the same flat number as R. 2 E
8. Q lives below the floor on which P lives but not in the 1 G B
same flat number.
9. Q lives in an even number flat but not on the floor on Case-2
which R lives. Floor Flat Flat
So, case2 (a) and case2 (b) and case 4 get eliminated. 5 C/ C/
Floor Flat 1 Flat 2 Flat 3 4 E
3 P U Nil 3
2 Nil Q T 2
1 S V R 1 G B
Answers
21. c 4. Only two floors are in between the floors of C and J.
22. a 5. C lives Northwest to J.
23. c 6. This means C lives on flat-1 of 5th floor.
24. d 7. A lives on odd numbered floor but below C. A and J
25. a lives on different numbered flats.
Page 372 of 728

Subscribe the Xpress Video Course & Mock Test Package for Bank & Insurance Exams
If there are any suggestions/ errors in our PDFs Feel Free to contact us via this email: admin@exampundit.in
IBPS RRB PO Prelims – Ultra Practice Bundle PDF
Case-1 27. d
Floor Flat Flat 28. d
5 C/ C/ 29. c
4 30. d
3 A Solution 26-30
2 E J 1. C is heavier than B, who in turn is heavier than F.
1 G B 2. There are only two people heavier than C and A is not
one of them.

Case-2 3. E is lighter than D.

Floor Flat Flat 4. A is neither the heaviest nor the lightest.

5 C/ C/ D > E > C >A/B > A/B > F

4 E 5. B shares the floor with the heaviest of the group while

3 A C shares it with the lightest.


6. The heaviest of the group lives on the top floor
2 J
while the lightest lives on the ground floor.
1 G B
Floor Flat 1 Flat 2
9. I lives west of D. By this condition Case-2 will be
3 B
eliminated.
10. F lives above H. 2

We got the final arrangement 1

Floor Flat Flat Ground C

5 C F 7. One of the flats on the first floor is vacant while

4 I D one on the second floor is also vacant.


D will occupy the top floor as D is heaviest and F will
3 A H
occupy the Ground floor as F is the lightest
2 E J
Floor Flat 1 Flat 2
1 G B
3 D B
Answers
2 E/A Nil
26. c

Page 373 of 728

Subscribe the Xpress Video Course & Mock Test Package for Bank & Insurance Exams
If there are any suggestions/ errors in our PDFs Feel Free to contact us via this email: admin@exampundit.in
IBPS RRB PO Prelims – Ultra Practice Bundle PDF
1 E/A Nil Floor Flat A Flat B
Ground C F 3 5 6
Answers 2 9 7
31. d 1 4 8
32. e
33. e Answers
34. a 36. c
35. b 37. c
Solution 31-35 38. a
1. There is no even number of family members in the 39. a
even numbered floor. 40. b
2. There are total members of floor number three is 11. Solution 36-40
3. Flat having 6 family members are live above flat 1. H lives on flat number 1 of floor number 1.
having 8 family members but not necessary in the same 2. There is one floor between floors of H and C. C lives
type of flat. above H.
Case-1 3. B lives on floor which is immediately above C’s and
Floor Flat A Flat B both are living in same flat.
3 6 5 Case-1 Case-2
2 7/9 7/9 Floor Flat 1 Flat 2 Flat 1 Flat 2
1 8/4 4/8 5
Case-2 4 B B
Floor Flat A Flat B 3 C C
3 5 6 2
2 7/9 7/9 1 H H
1 8/4 4/8 4. A and E share same floor.
5. There are 2 floors between A and D and they live in
8. The total members of flat B is three more than same flat number.
flat A.
Page 374 of 728

Subscribe the Xpress Video Course & Mock Test Package for Bank & Insurance Exams
If there are any suggestions/ errors in our PDFs Feel Free to contact us via this email: admin@exampundit.in
IBPS RRB PO Prelims – Ultra Practice Bundle PDF
6. D lives on one of the flats which is immediately above 1. Wasim stays on flat no. 1 of the floor just below the
an empty flat. floor on which Sakshi stays.
7. E lives immediately above G’s flat and both are living 2. The flat no 2 of floor no 2 is empty.
in same flat. 3. Sakshi stays on flat no. 2 on one of the floors.
Case-2(a) Case- Case-1
2(b) Floor Flat 1 Flat 2 Flat 3
Floor Flat 1 Flat 2 Flat 1 Flat 2 4 Sakshi
5 D E A 3 Wasim
4 Nil B G B 2 X
3 C C 1
2 A E D Case-2
1 H G H Nil Floor Flat 1 Flat 2 Flat 3
4
9. B and F share same flat number. 3 Sakshi
Floor Flat 1 Flat 2 2 Wasim X
5 D F 1
4 Nil B 4. Teena stays on flat no. 3 of same floor on which
3 Nil C Wasim stays.
2 A E 5. Flat to the right of Sakshi is empty.
1 H G 6. Flat no 3 of floor no 4 is empty.
Answers Hence, Case-2 is eliminated.
41. b Case-1
42. a Floor Flat 1 Flat 2 Flat 3
43. d 4 Sakshi X
44. e 3 Wasim Teena
45. a 2 X
Solution 41-45 1

Page 375 of 728

Subscribe the Xpress Video Course & Mock Test Package for Bank & Insurance Exams
If there are any suggestions/ errors in our PDFs Feel Free to contact us via this email: admin@exampundit.in
IBPS RRB PO Prelims – Ultra Practice Bundle PDF
7. Jordan stays on flat to the right of Urvashi. Case -1
8. Yuri and Veena stay on the same floor. Floor Flat 1 Flat 2 Flat 3
9. Teena and Yuri do not share same flat number. 4 Manish
10. Urvashi stays on flat no. 2.
3 Amar
Floor Flat 1 Flat 2 Flat 3
2 Balram Yuvi
4 Sakshi X
3 Wasim Teena 1 Waqar

2 Yuri X Veena Case -2


1 Urvashi Jordan Floor Flat 1 Flat 2 Flat 3

4 Manish
11. Zoya does not stay on top floor.
12. Flat on which Ruhi stays is below the flat of Sakshi 3 Amar

such that both are not living in different flats. 2 Balram Yuvi
Floor Flat 1 Flat 2 Flat 3
1 Waqar
4 Xavier Sakshi X
3 Wasim Ruhi Teena Case -3

2 Yuri X Veena Floor Flat 1 Flat 2 Flat 3

1 Zoya Urvashi Jordan


4 Manish

Answers 3 Amar

46. c 2 Balram Yuvi


47. a
48. c 1 Waqar

49. c 2. Daman and Zuber lives on same odd number floor.


50. e
3. Firoz lives below Zuber and both live is same flat
Solution 46-50
number.
10. Amar lives north of Yuvi but south of Manish.
Case -1
Balram lives west of Yuvi but north of Waqar.
Page 376 of 728

Subscribe the Xpress Video Course & Mock Test Package for Bank & Insurance Exams
If there are any suggestions/ errors in our PDFs Feel Free to contact us via this email: admin@exampundit.in
IBPS RRB PO Prelims – Ultra Practice Bundle PDF
Floor Flat 1 Flat 2 Flat 3 By this case – 1 will be cancelled.
Case – 2
4 Manish
Floor Flat 1 Flat 2 Flat 3
3 Daman Amar Zuber
4 Gautam/ Gautam/ Manish
2 Balram Yuvi Firoz/
3 Daman Zuber Amar
1 Waqar Firoz/
2 Balram Varun Yuvi
Case -2
1 Waqar Firoz Charlie
Floor Flat 1 Flat 2 Flat 3

4 Manish
Case-3
3 Daman Zuber Amar Floor Flat 1 Flat 2 Flat 3

2 Balram Firoz/ Yuvi 4 Gautam/ Gautam/ Manish

1 Waqar Firoz/ 3 Zuber Daman Amar

Case -3 2 Varun Balram Yuvi


Floor Flat 1 Flat 2 Flat 3
1 Firoz Waqar Charlie
4 Manish
12. Balram lives south-west to Eshwar.
3 Zuber Daman Amar By this case-3 will be eliminated. As in Case -3 Balram
lives in Flat 2 and Eshwarcannot live in Flat 3.
2 Firoz/ Balram Yuvi
Floor Flat 1 Flat 2 Flat 3
1 Firoz/ Waqar
4 Gautam Eshwar Manish

3 Daman Zuber Amar


11. Varun lives north-west of Charlie. Gautam lives
above Varun.
Page 377 of 728

Subscribe the Xpress Video Course & Mock Test Package for Bank & Insurance Exams
If there are any suggestions/ errors in our PDFs Feel Free to contact us via this email: admin@exampundit.in
IBPS RRB PO Prelims – Ultra Practice Bundle PDF
2 Balram Varun Yuvi 1 Waqar Firoz Charlie

18). Certain Number- Circle Seating

Directions (1-5): Study the information carefully and 4. In certain way U is related to Y and P is related to
answer the questions given below. O and in same way that, who among the following
There are more than ten people sitting in a circle. Each of person is related to Y?
the seats occupied by one person. Each of them is facing (a) W
inside the circle. O is sitting third to the left of P. Q is (b) S
sitting second to the left of R. There are two people (c) X
sitting between R and S. T is an immediate neighbor of (d) V
both U and V, but not sitting second tothe right of Q. W (e) T
and Y are sitting secondto the left and third to the right 5. If one more person C is seated immediate right of
of Q respectively. X is sitting second tothe left of S, But T than what is the position of C with respect to R?
not second tothe right of Y. S is sitting third to the right (a) Second Right
of V. R is sitting third tothe left of U. (b) Third Right
1. How many people are sitting in the circle? (c) Fourth Right
(a) 9 (d) Fifth Right
(b) 16 (e) Immediate Right
(c) 11 Directions (6-10): Study the information carefully
(d) 10 and answer the questions given below
(e) None A few persons were seated around a circular table
2. How many people are sitting between S and V, facing the centre. The chairs were kept at equal distance
when counted from left of V? and the number of persons sitting in the circle was odd
(a) 4 number but not more than 20. Number of people
(b) 6 between B and E when counted from the right of B was
(c) 5 thrice as that of the number of persons seated between B
(d) 7 and N when counted from the left of B. E and N were
(e) None of these neighbouring each other. U was seated to the immediate
3. Which of the following statements is true? left of Q. Two persons between Q and H. E was 3rd to
(a) V sits third to the left of Q the right of H. C was 2nd to the right of J and 2nd to the
(b) W sits third to the right of Q left of U. E is to the immediate left of N.
(c) X sits fifth to the left of U 6. How many people can sits in the Circle?
(d) S sits to the immediate left of Y (a) 19
(e) None is true
Page 378 of 728

Subscribe the Xpress Video Course & Mock Test Package for Bank & Insurance Exams
If there are any suggestions/ errors in our PDFs Feel Free to contact us via this email: admin@exampundit.in
IBPS RRB PO Prelims – Ultra Practice Bundle PDF
(b) 17 sitting between Z and B. Q sits third to the left of B. O
(c) Cannot be determine sits third to the left of E. At least fifteen persons are
(d) 18 sitting around the circular table. T is immediate right of
(e) 15 Z.
7. How many known people are sitting between H 11. Which of the following’s pairs are immediate
and B, when counted from left of H? neighbour of each others?
(a) Three (a) Q, E
(b) One (b) Z, B
(c) Two (c) B, E
(d) Four (d) O, T
(e) Five (e) O, Z
8. What is the position of N with respect to Q? 12. Total how many persons can sits in the circle?
(a) Third to the left (a) 15
(b) Third to the right (b) 16
(c) Fifth to the left (c) 20
(d) Seventh to the right (d) 18
(e) Eight to right (e) 19
9. How many more person can sit between U and J 13. What is the position of E with respect to T?
when counted to the right of J? (a) Third to the right
(a) 5 (b) Fifth to the left
(b) 1 (c) Seventh to the right
(c) 2 (d) Third to the left
(d) 3 (e) Eight to right
(e) 4 14. Who amongst the followings is third to the right of
10. What is the position of H with respect to E? Q?
(a) Third to the right (a) E
(b) Fifth to the left (b) O
(c) Seventh to the right (c) B
(d) Third to the left (d) Z
(e) Eight to right (e) T
Directions (11-15): Study the information carefully 15. How many people can sit between Z and Q?
and answer the questions given below (a) 2
A certain number of people are seating around a circular (b) 3
table facing outside the center. E and B are immediate (c) 5
neighbors. Not more than Six persons are sitting between (d) 6
O and T when counted from left of O. Two persons are (e) Cannot be determined

Page 379 of 728

Subscribe the Xpress Video Course & Mock Test Package for Bank & Insurance Exams
If there are any suggestions/ errors in our PDFs Feel Free to contact us via this email: admin@exampundit.in
IBPS RRB PO Prelims – Ultra Practice Bundle PDF
Directions (16-20): Study the information carefully (e) None of these
and answer the questions given below 20. How many persons are sitting between O and L,
More than twelve peoples are sitting around a circular when counting right of O?
table and all are facing centre. L sits third to the left of (a) Two
M. Only one person sits between L and O. O and M are (b) One
not immediate neighbours. Three persons sit between O (c) Five
and D. Two person sits between A and D. A and O are (d) Four
not immediate neighbours. P sits immediate left of A. T (e) More than five
is immediate neighbor of P and M. S sits second to the Directions (21-25): Study the information carefully
left of T. Only Two people sit between U and S. and answer the questions given below:
Certain number of persons is sitting around the circular
16. Who is sitting second to the right of L? table. All of them are facing towards the centre. Distance
(a) S between any two adjacent persons is same. A is facing P,
(b) M who is sitting second to the right of T. S is facing L, who
(c) T is sitting fourth to the right of C. None among S, C and L
(d) P is an immediate neighbor of P. The person, who is sitting
(e) A fourth to left of or right of T, is not immediate neighbor
of C. There are six persons between M and T. M is not
17. Who sits at immediate right of A?
immediate neighbor of A. At most 13 people sit around
(a) P
the circular table.
(b) U
21. How many people are sitting around the table?
(c) O
(a) 11
(d) D
(b) 9
(e) No one
(c) 10
18. How many persons sitting in a circle?
(d) 13
(a) 17
(e) 12
(b) 15
22. How many people are sitting between M and A,
(c) 16
when counted from left of A?
(d) 14
(a) One
(e) None of these
(b) Two
(c) Three
19. Who among the following person sits sixth to the
(d) Four
right of D?
(e) None of these
(a) U
23. Which of the following statements is true?
(b) O
(a) A sits third to the left of L
(c) L
(b) S sits third to the right of P
(d) S
Page 380 of 728

Subscribe the Xpress Video Course & Mock Test Package for Bank & Insurance Exams
If there are any suggestions/ errors in our PDFs Feel Free to contact us via this email: admin@exampundit.in
IBPS RRB PO Prelims – Ultra Practice Bundle PDF
(c) T sits fifth to the left of S (e) None of these
(d) C sits to the immediate right of M 27. Who among the following person sits fourth to the
(e) None is true right of K?
24. In certain way A is related to C and T is related to (a) O
L and in same way that, who among the following (b) D
person is related to S? (c) N
(a) C (d) A
(b) M (e) E
(c) P 28. How many persons currently sitting between B
(d) L and O when counted to the left of B?
(e) None of these (a) 1
25. Who sits at fifth place when counting starts from (b) 2
T in Anti clock wise direction? (c) 3
(a) C (d) 4
(b) M (e) 5
(c) L 29. If J is an immediate neighbor of V then who
(d) S among the following person sits second to the left
(e) None of these of J?
Directions (26-30): Study the information carefully (a) A
and answer the questions given below. (b) K
An uncertain number of persons sitting in a circular table (c) C
facing centre. Three persons sit between O and K. N sits (d) B
second to the right of O. Only three persons sit between (e) O
K and U. V is immediate left of O. D is immediate left of 30. Four of the following five are alike in a certain
N. As many persons sit between N and A as many way based on a group; find the one that does not
between A and U. B sits second to the left of C, who is belong to that group?
an immediate neighbor of K. Not more than two persons (a) V, O
sit between C and Uwhen counted to the Right of U. Five (b) D, N
persons sit between U and D when counted to the left of (c) B, U
U. As many persons sit between U and E as many (d) K, C
between E and D. (e) V, D
26. How many persons sitting in a Circle? Directions (31-35): Study the information carefully
(a) 20 and answer the questions given below.
(b) 18 Some people are sitting around a circular table facing
(c) 15 centre. Distance between each of them is equal. A sits
(d) 14 immediate left of S. Two people sit between A and C.

Page 381 of 728

Subscribe the Xpress Video Course & Mock Test Package for Bank & Insurance Exams
If there are any suggestions/ errors in our PDFs Feel Free to contact us via this email: admin@exampundit.in
IBPS RRB PO Prelims – Ultra Practice Bundle PDF
Two people sit between P and C. D is immediate (a) S, D
neighbour of C. N sits third to left of D.D and S are not (b) P, D
immediate neighbours. Number of people between N and (c) D, N
S when counted to the left of N is two times the number (d) P, C
of person between N and S when counted to the right of (e) A, C
N. There are at-most 19 person sitting around circular Directions (36-40): Study the information carefully
table. and answer the questions given below.
31. How many persons sitting in a Circle? A certain number of persons are sitting in a circle facing
(a) 17 outside the centre. Three persons sit between D and E
(b) 18 and B sits third to the right of E. Number of people sit
(c) 17 between D and F when counted to the right of D is equal
(d) 14 to number of people sit between D and C when counted
(e) None of these to the left of D. F and C are immediate neighbour. B sits
32. Who among the following person sits fifth to the second to the left of F. Only five persons sit between C
left of A? and A. There are only one person sit between F and G. C
(a) P and B are not immediate neighbors
(b) C
(c) N 36. Who is sitting second to the left of F?
(d) S (a) B
(e) D (b) E
33. What is the position N with respect to S? (c) D
(a) Second to the right (d) C
(b) Third to the left (e) A
(c) Fourth to the left 37. Who sits fourth to the right of D?
(d) Fifth to the left (a) C
(e) None of these (b) E
34. If F is an immediate neighbor of N then who (c) G
among the following person sits third to left of F? (d) D
(a) P (e) B
(b) D 38. How many persons sitting in a circle?
(c) C (a) 20
(d) A (b) 24
(e) No One (c) 19
35. Four of the following five are alike in a certain (d) 14
way based on a group; find the one that does not (e) 26
belong to that group?

Page 382 of 728

Subscribe the Xpress Video Course & Mock Test Package for Bank & Insurance Exams
If there are any suggestions/ errors in our PDFs Feel Free to contact us via this email: admin@exampundit.in
IBPS RRB PO Prelims – Ultra Practice Bundle PDF
39. Who among the following person sits seventh to (e) None of these
the right of A? 43. Who is to the immediate left of J?
(a) C (a) O
(b) E (b) Q
(c) B (c) M
(d) D (d) L
(e) None of these (e) NONE
40. How many persons are sitting between G and 44. Who is third to the left of M?
Bwhen counting from left of G? (a) N
(a) Two (b) Q
(b) Five (c) L
(c) One (d) P
(d) Three (e) None of these
(e) More than five 45. How many persons are sitting between K and P
Directions (41-45): Study the following information when counted clock wise direction from K?
carefully and answer the questions below. (a) Two
A certain number of people are sitting in a circle and all (b) Five
of them are facing inside. M sits eighth to the left of J. N (c) Three
sits fourth to the right of K. O sits to the immediate right (d) Four
of P. L is immediate neighbour of K and J. Number of (e) More than five
people between J and P is same as that of between K and Directions (46-50): Refer to the data below and
J when counted to the left of K. Only six people sit answer the questions that follow.
between N and O when counted to the left of N. Five Certain number of Persons sitting in a circular table.
persons sits between P and Q. Not more than 16people They all are facing inside. Not more than three Persons
are seated in circle. M sits second to the right of N. sitting together. Total number of seats around the table is
41. How many persons sitting in a circle? a prime number more than 17.
(a) 20
(b) 19 Two persons are sitting between P and Q. Two persons
(c) 23 are sitting between P and R. Q sits to the left of P. There
(d) 16 is only one person between S and T. Two person sits
(e) None of these between Q and U. V sits second to the right of P. R is
42. Who is to the third to the right of L ? immediate neighbor of V and S. There are three persons
(a) M between T and W. X sits exactly between T and W when
(b) Q counted to the right of T. U and Y are immediate
(c) O neighbors of each other. There are exactly five persons
(d) L between Y and Z. O and Q are immediate neighbors of

Page 383 of 728

Subscribe the Xpress Video Course & Mock Test Package for Bank & Insurance Exams
If there are any suggestions/ errors in our PDFs Feel Free to contact us via this email: admin@exampundit.in
IBPS RRB PO Prelims – Ultra Practice Bundle PDF
each other. Z sits immediate left of P. There are three (a) Immediate left
persons between Y and X. No one sits to the immediate (b) Fifth to the right
right of Q. (c) Fifth to the left
46. Who among the following sits 10th to right of P? (d) Sixth to the left
(a) Y (e) Fourth to the right
(b) U 49. What is the position of O with respect to W?
(c) W (a) Fourth to the left
(d) Q (b) Fifth to the right
(e) None of these. (c) Sixth to the right
47. If D is an immediate neighbour of S then how (d) Fifth to the left
many persons are sitting between D and V, when (e) Third to the right
counted from right of V? 50. Who among the following sits third to the right of
(a) One T?
(b) Three (a) W
(c) Five (b) No one
(d) Six (c) X
(e) None of these (d) V
48. What is the position of Y with respect to U? (e) P

18). Certain Number- Circle Seating - Solution and Detailed Explanation

SOLUTION (1-5):
 S is sitting third to the right of V.
 There are two people sitting between R and S
 R is sitting third to U.

 T is an immediate neighbor of both U and V, but not


sitting second to Q.

Page 384 of 728

Subscribe the Xpress Video Course & Mock Test Package for Bank & Insurance Exams
If there are any suggestions/ errors in our PDFs Feel Free to contact us via this email: admin@exampundit.in
IBPS RRB PO Prelims – Ultra Practice Bundle PDF
 Q is sitting second to R.
 T is not sitting second to Q.
 W is sitting second to Q. (As per given instruction)
Here it is clear that we cannot add more person now

1.c
2.d
3.e
 O is sitting third to the left of P
4.d(based on number of person in clock wise
 X is sitting second to S, But not second to Y
direction)
U is 2nd right of Y
P is third right of O
V is 4th right of Y
5.d
SOLUTION (6-10):
1. E was 3rd to the right of H
2. Two person between Q and H
3. E is to the immediate left of N

Y is sitting third to Q.
Final Arrangement

Page 385 of 728

Subscribe the Xpress Video Course & Mock Test Package for Bank & Insurance Exams
If there are any suggestions/ errors in our PDFs Feel Free to contact us via this email: admin@exampundit.in
IBPS RRB PO Prelims – Ultra Practice Bundle PDF
4. U was seated to the immediate left of Q
5. C was 2nd to the right of J and 2nd to the left of U.

6. Number of person between B and E when counted Final Arrangement


from the right of B was thrice as that of the number
of persons seated between B and N when counted
from the left of B There are two people sitting
between R and S.

6.e
7.a
8.d
9.b
10.d
SOLUTION (11-15):
1. Two persons are sitting between Z and B
2. Q sits third to the left of B

Page 386 of 728

Subscribe the Xpress Video Course & Mock Test Package for Bank & Insurance Exams
If there are any suggestions/ errors in our PDFs Feel Free to contact us via this email: admin@exampundit.in
IBPS RRB PO Prelims – Ultra Practice Bundle PDF

3. E and B are immediate neighbors.


4. O sits third to the left of E.

Final Arrangement

5. T is immediate right of Z.
6. From the left of O, not more than Six persons are
sitting between O and T
7. At least fifteen persons are sitting around the circular
table.

Page 387 of 728

Subscribe the Xpress Video Course & Mock Test Package for Bank & Insurance Exams
If there are any suggestions/ errors in our PDFs Feel Free to contact us via this email: admin@exampundit.in
IBPS RRB PO Prelims – Ultra Practice Bundle PDF
7. P sits immediate left of A.

11.c
12.a
13.b
14.c
15.e (because there is no specific direction in the
question)

SOLUTION (16-20) 8. T is immediate neighbor of P and M.


1. L sits third to the left of M. 9. S sits second to the left of T.
2. Only one person sits between L and O. 10. Only Two people sit between U and S.
3. O and M are not immediate neighbours.

4. Three persons sit between O and D.


5. Two person sits between A and D.
6. A and O are not immediate neighbours.
Page 388 of 728

Subscribe the Xpress Video Course & Mock Test Package for Bank & Insurance Exams
If there are any suggestions/ errors in our PDFs Feel Free to contact us via this email: admin@exampundit.in
IBPS RRB PO Prelims – Ultra Practice Bundle PDF

1. At most 13 people sit around the circular table.


2. A is facing P, who is sitting second to the right of T.

Final Arrangements

16.a
17.e
18.b
19.c
20.b
SOLUTION (21-25):

Page 389 of 728

Subscribe the Xpress Video Course & Mock Test Package for Bank & Insurance Exams
If there are any suggestions/ errors in our PDFs Feel Free to contact us via this email: admin@exampundit.in
IBPS RRB PO Prelims – Ultra Practice Bundle PDF

3. There are six persons between M and T.


4. M is not immediate neighbor of A.
5. None among S, C and L is an immediate neighbor of 6. S is facing L, who is sitting fourth to the right of C
P. 7. The person, who is sitting fourth to T, is not
immediate neighbor of C.

Page 390 of 728

Subscribe the Xpress Video Course & Mock Test Package for Bank & Insurance Exams
If there are any suggestions/ errors in our PDFs Feel Free to contact us via this email: admin@exampundit.in
IBPS RRB PO Prelims – Ultra Practice Bundle PDF
Final Arrangement CASE 2

21.e
22.b 5. D is immediate left of N.
23.d 6. Five persons sit between U and D when counted to
24.a(number of person between them) the left of U.
A is 2nd right of C CASE 1
T is 2nd right of L
C is 2nd right of S
25.b

SOLUTION (26-30):
1. Three persons sit between O and K.
2. N sits second to the right of O.
3. Only three persons sit between K and U.
4. V is immediate left of O. CASE 2
CASE 1

Page 391 of 728

Subscribe the Xpress Video Course & Mock Test Package for Bank & Insurance Exams
If there are any suggestions/ errors in our PDFs Feel Free to contact us via this email: admin@exampundit.in
IBPS RRB PO Prelims – Ultra Practice Bundle PDF
Final Arrangement

7. B sits second to the left of C, who is an immediate


neighbor of K.
8. Not more than two persons sit between C and U
when counted to the Right of U.

26.c
27.a
28.d
29.b
9. As many persons sit between N and A as many 30.e(Person seated is immediate neighbor of each
between A and U. other)
10. As many persons sit between U and E as many
SOLUTION (31-35):
between E and D.
1. A sits immediate left of S.
2. Two people sit between A and C.
3. Two people sit between P and C.

Page 392 of 728

Subscribe the Xpress Video Course & Mock Test Package for Bank & Insurance Exams
If there are any suggestions/ errors in our PDFs Feel Free to contact us via this email: admin@exampundit.in
IBPS RRB PO Prelims – Ultra Practice Bundle PDF

4. D is immediate neighbour of C. 6. Number of person between N and S when counted to


5. N sits third to left of D. the left of N is two times the number of person
between N and S when counted to the right of N.
7. There are at-most 20 person sitting around circular
table

Page 393 of 728

Subscribe the Xpress Video Course & Mock Test Package for Bank & Insurance Exams
If there are any suggestions/ errors in our PDFs Feel Free to contact us via this email: admin@exampundit.in
IBPS RRB PO Prelims – Ultra Practice Bundle PDF

Final Arrangement

3. Number of people sit between D and F when


31.a counted to the right of D is equal to number of
32.c people sit between D and C when counted to the left
33.e of D.
34.e 4. F and C is immediate neighbour.
35.b (Number of person) 5. C and B are not immediate neighbors
SOLUTION (36-40):
1. Three persons sit between D and E and B sits third
to the right of E.
2. B sits second to the left of F

Page 394 of 728

Subscribe the Xpress Video Course & Mock Test Package for Bank & Insurance Exams
If there are any suggestions/ errors in our PDFs Feel Free to contact us via this email: admin@exampundit.in
IBPS RRB PO Prelims – Ultra Practice Bundle PDF

36.a
37.b
38.c
39.b
40.d

SOLUTION (41-45):
All the person are faces inside
1. M sits eighth to the left of J.
6. Only five people sit between C and A. 2. N sits fourth to the right of K.
7. There is only one person sit between F and G. 3. L is immediate neighbour of K and J.

Final Arrangement

Page 395 of 728

Subscribe the Xpress Video Course & Mock Test Package for Bank & Insurance Exams
If there are any suggestions/ errors in our PDFs Feel Free to contact us via this email: admin@exampundit.in
IBPS RRB PO Prelims – Ultra Practice Bundle PDF

4. O sits to the immediate right of P.


5. Only six people sit between N and O when counted
to the left of N.
6. Not more than 16 people are seated in circle.

8. Five persons sit between P and Q.


Final Arrangement

41.d
42.b
7. Number of people between J and P is same as that of 43.a
between K and J when counted to the left of K. 44.e
Page 396 of 728

Subscribe the Xpress Video Course & Mock Test Package for Bank & Insurance Exams
If there are any suggestions/ errors in our PDFs Feel Free to contact us via this email: admin@exampundit.in
IBPS RRB PO Prelims – Ultra Practice Bundle PDF
45.c

SOLUTION (46-50):

All the person are faces inside

1. V sits second to the right of P.


2. Two persons between P and Q and P and R and Q sit
to the left of P.
3. There is only one person between S and T.
4. R is immediate neighbor of V and S.
11. U and Y are immediate neighbors of each other.
12. There are exactly five persons between Y and Z.
13. There are three persons between Y and X.

5. Two persons sit between Q and U.


6. Z sits immediate left of P.
7. There are three persons between T and W.

Final Arrangements

8. No one sits to the immediate right of Q.


9. O and Q are immediate neighbors of each other
10. X sits exactly between T and W when counted to the
right of T.
Page 397 of 728

Subscribe the Xpress Video Course & Mock Test Package for Bank & Insurance Exams
If there are any suggestions/ errors in our PDFs Feel Free to contact us via this email: admin@exampundit.in
IBPS RRB PO Prelims – Ultra Practice Bundle PDF
46.c

47.e

48.a

49.b

50.b

19). Floor Based Puzzle

Directions (1-5): Study the information carefully and (d) 1st


answer the questions given below. (e) 2nd
Nine friends P, Q, R, S, T, U, V, Wand X are living in a 2. How many persons live between Q and the one
nine-storey building, but not necessarily in the same who is immediate above U?
order. The ground floor is numbered 1; the floor above it (a) One
is numbered 2; and so on, until the topmost floor is (b) Two
numbered 9. T lives three floors below the topmost floor. (c) Three
Xdoesn’t live on the topmost floor. Only four people live (d) Four
between U, who lives on the 2nd floor, and R. Only four (e) None of these
people live between X and V. Only three people live 3. If all the persons are arranged alphabetically
between S and W, who lives exactly below T. P lives on from top to bottom then how many persons
an even numbered floor. Only two people live between X remain (live) on same floor?
and T.Q does not live below X. (a) Two
1. On which of the following floor P lives? (b) One
(a) 4th (c) Three
(b) 5th (d) Four
(c) 7th (e) None
Page 398 of 728

Subscribe the Xpress Video Course & Mock Test Package for Bank & Insurance Exams
If there are any suggestions/ errors in our PDFs Feel Free to contact us via this email: admin@exampundit.in
IBPS RRB PO Prelims – Ultra Practice Bundle PDF
4. Which of the following person lives on odd 6. Who lives on the 5th floor?
number floor? (a) G
(a) V (b) F
(b) T (c) T
(c) W (d) M
(d) P (e) K
(e) U 7. Who lives on the 3rd floor?
5. If V is related to T in the same way W is related to (a) T
X, then following the same pattern, P is related (b) M
to? (c) G
(a) T (d) F
(b) S (e) Either (a) or (b)
(c) X 8. How many persons are between G and B?
(d) U (a) Five
(e) None of these (b) Three
Directions (6–10): Study the information carefully (c) Four
and answer the questions given below: (d) Seven
Ten people K, T, D, H, F, G, B, M, S and U are living in (e) None of these
a building. A building has ten floors numbered one to 9. Which of the following person live between K and
ten, in such a way that the ground floor is numbered one; G?
the floor above it is numbered two and so on. The (a) T
topmost floor numbered ten. H lives on 6th floor. T and (b) M
M floors are near to each other but both of them do not (c) F
live on 9th and 10th floors. There are three floors between (d) Can’t be determined
the F and D floors. K lives on 1 st floor. U floor is not (e) H
immediately before the B floor but S floor is 10. Which of the following person does not live on
immediately above the H floor. There are four floors odd number floor?
between D and G. D live above F. U is belowB. (a) D
Page 399 of 728

Subscribe the Xpress Video Course & Mock Test Package for Bank & Insurance Exams
If there are any suggestions/ errors in our PDFs Feel Free to contact us via this email: admin@exampundit.in
IBPS RRB PO Prelims – Ultra Practice Bundle PDF
(b) S (d) H
(c) H (e) U
(d) F 13. Which of the following combinations is true?
(e) K (a) Floor no. 3 - H
Directions (11-15): Study the information carefully (b) Floor no. 2 - W
and answer the questions given below: (c) Floor no. 4 - F
Eight persons E, F, G, H, T, U, V and W are living in (d) Floor no. 6 - V
eight floors of a building. The ground floor is numbered (e) None of the above
one and the topmost floor is numbered 8. There are as 14. Who among the following lives on topmost floor?
many persons live between E and H is one more that the (a) U
number of people between H and F. H live below E and (b) H
does not live on even number floor. The number of (c) T
persons living between G and V is more than four. E (d) V
lives on an even numbered floor but not on topmost (e) G
floor. There is only one person living between V and U. 15. How many person lives below E?
Three persons live between T and H. U lives on one of (a) 1
the floors below E. W does not lives in bottom most (b) 2
floor. (c) 3
11. How many persons are there between G and V? (d) 4
(a) None (e) 5
(b) One Directions (16-20): Study the information carefully
(c) Two and answer the questions given below
(d) Three Nine persons i.e. A, B, C, D, E, F, G, H and I are living
(e) None of the above in ten floored building in which one floor is vacant.
12. Who among the following lives on the first floor? Ground floor is numbered as 1, first floor is numbered as
(a) G 2 and so on until topmost floor is numbered as 10. There
(b) W are five floors gap between B and D. E lives at odd
(c) F numbered floor. There are three floors gap between E
Page 400 of 728

Subscribe the Xpress Video Course & Mock Test Package for Bank & Insurance Exams
If there are any suggestions/ errors in our PDFs Feel Free to contact us via this email: admin@exampundit.in
IBPS RRB PO Prelims – Ultra Practice Bundle PDF
and A. Ilive immediate below D which doesn't live in (e) None of these
even numbered floor. There are as many floors below H 20. Find the odd one out?
as above F. G lives immediate above H. C lives above (a) B
the vacant floor. There are two floors gap between G and (b) G
E. (c) E
16. How many persons live below H? (d) F
(a) None (e) D
(b) One Directions (21-25): Study the information carefully
(c) Two and answer the questions given below
(d) Five Eight persons B, I, H, U, J, O, T and Y are living in eight
(e) Four different floors of a building. The lowermost floor is
17. Who lives immediately above E? numbered 1, the one above floor is numbered 2, and so
(a) H on till the topmost floor is numbered 8 but not
(b) F necessarily in the same order. If persons place value is
(c) Floor is vacant even as per English alphabetical order, then they live on
(d) D odd numbered floor or else live on even numbered floor.
(e) None of these Two persons live between J and O who does not live
18. How many persons live between C and F? above T. U lives above B. Two persons live between Y
(a) None and T. T lives below Y but not on floor number 1. O
(b) One lives above H. Number of persons live below B are same
(c) Two as the number of Persons lives above U. H does not live
(d) Three immediately above or immediately below Y. I live below
(e) Four O.
19. Who lives immediately below B? 21. In which of the following Floors H lives?
(a) G (a) 1
(b) H (b) 2
(c) Floor is vacant (c) 3
(d) A (d) 4
Page 401 of 728

Subscribe the Xpress Video Course & Mock Test Package for Bank & Insurance Exams
If there are any suggestions/ errors in our PDFs Feel Free to contact us via this email: admin@exampundit.in
IBPS RRB PO Prelims – Ultra Practice Bundle PDF
(e) 5 Eight persons A, B, C, D, E, F, G and H are staying in an
22. How many Floors are below T? eight-storey building. The lowermost floor is numbered 1
(a) Six and the topmost floor is numbered 8. They have different
(b) Two number of Chocolate.No two persons have the same
(c) Four number of chocolates. E has 19 chocolate and stays on
(d) Five the third floor. Two persons are staying between E and
(e) None of these the one who has 30 chocolate. F has least number of
23. I is how many Floors below J? chocolates in the group and the one who has highest
(a) Three number of chocolates has 46 chocolate. A has more
(b) Five chocolate than E and two people is between A and B. G
(c) Four has 35 chocolates. The difference in the number of
(d) Six chocolates of A and D is 10. Three persons are between
(e) None of these H and D who live below H. The number of people living
24. How many Floors are between Y and O? above G is the same as the number of people living
(a) Two below D. Number of persons between B and the one
(b) Three having 16 chocolate is 2. One of the persons having 29
(c) Four Chocolate who is not H. The one who has least number
(d) One of chocolates has 11 chocolates. H doesn’t live on 6th
(e) None of these floor.
25. Which person lives immediately above U? 26. Who among the following has 29 chocolates?
(a) W (a) The one who lives on 4th floor
(b) T (b) E
(c) B (c) F
(d) K (d) D
(e) None of these (e) None of these
Directions (26-30): Study the information carefully 27. How many persons live in between A and E?
and answer the questions given below (a) Two
(b) Three
Page 402 of 728

Subscribe the Xpress Video Course & Mock Test Package for Bank & Insurance Exams
If there are any suggestions/ errors in our PDFs Feel Free to contact us via this email: admin@exampundit.in
IBPS RRB PO Prelims – Ultra Practice Bundle PDF
(c) None as 1 and above it floor is numbered as 2 and so on till the
(d) One topmost floor is numbered as 8. They teach in different
(e) More than Three classes viz. 9, 8, 7 and 6 such that only two teachers
28. C lives in which of the following floor? teach in same class. Teacher who teach in same class live
(a) 4th floor on Odd - even floors (i.e one of those teaching in same
(b) 1st floor class lives on even number floor and the other of same
(c) 6th floor class lives on odd number floor). B and H teach in same
(d) 7th floor class. The one who teaches in class 9 lives on 5th floor.
(e) 8th floor The one who lives on 3rd floor teaches in even numbered
29. Which of the following statement is true? class. The one who lives on 6thfloor and C teach in same
(a) A lives on 6th floor class. A teaches in class 7and lives on 4th floor. Only
(b) B live immediate above F three teachers live between F and A. D does not live on
(c) Dhas 11 chocolates bottommost floor. Only two teacher lives between F and
(d) A live immediate below the one who has 35 H. There are two teachers live between the one who lives
chocolates. at floor 3rd and G, who lives below the one who teaches
(e) None is true in class 6th. Both teachers who teach in class 6th live
30. Who among the following person lives on 7th above the floor of C. I lives below B.
floor? 31. Who lives on the topmost floor?
(a) The one who has 30 chocolates (a) D
(b) H (b) F
(c) The one who has least number of chocolates (c) A
(d) The one who has 26 of chocolates (d) Cannot be determined
(e) None of these (e) None of these
Directions (31-35): Study the information carefully 32. Which of the following class teach by C?
and answer the questions given below: (a) 6
There are eight teachers A, B, C, D, F, G, H and I who (b) 7
live on different floors of a eight floor building, but not (c) 8
necessarily in the same order. Ground floor is numbered (d) 9
Page 403 of 728

Subscribe the Xpress Video Course & Mock Test Package for Bank & Insurance Exams
If there are any suggestions/ errors in our PDFs Feel Free to contact us via this email: admin@exampundit.in
IBPS RRB PO Prelims – Ultra Practice Bundle PDF
(e) Cannot be determined inMumbai. F lives immediately below G and
33. On which floor I lives? immediately above H. F lives on an oddnumbered floor.
(a) 6 The person who lives in Delhi lives on the third floor.
(b) 5 The person who lives in Hyderabad lives above the
(c) 4 person who lives in Mumbai. Four persons live between
(d) 3 the one who lives in Hyderabad and the one who lives in
(e) 1 Indore. I lives above E. J does not lives in Hyderabad. K
34. How many people live between G and B? lives below the one who lives in Raipur and above the
(a) 2 one who lives in Pune.
(b) None 36. Who among the following lives on floor number
(c) 1 6?
(d) 3 (a) The one who lives in Hyderabad
(e) Cannot be determined (b) The one who lives in Mumbai
35. Who is neighbour of the person who live in floor (c) The one who lives in Raipur
number 5? (d) The one who lives in Delhi
(a) G (e) The one who lives in Chennai
(b) A 37. Which of the following person lives immediately
(c) Both G and A above the floor on which Chennai?
(d) C (a) F
(e) None of these (b) G
Directions (36-40): Study the information carefully (c) H
and answer the questions given below: (d) J
Seven persons lives on a seven storey building such that (e) None of These
the ground floor is numbered as 1 and floor just above is 38. To which of the following cities G belongs to ?
numbered as 2 and so on till the topmost floor which is (a) Chennai
numbered as 7. Each of them lives on different city. Less (b) Indore
than three persons live below E who lives inChennai. (c) Delhi
Three persons live between E and the one who lives (d) Mumbai
Page 404 of 728

Subscribe the Xpress Video Course & Mock Test Package for Bank & Insurance Exams
If there are any suggestions/ errors in our PDFs Feel Free to contact us via this email: admin@exampundit.in
IBPS RRB PO Prelims – Ultra Practice Bundle PDF
(e) Pune persons live above S. I lives above U but not
39. How many people live between the floors on immediately above.
which K and the one who live in Delhi? 41. Which of the following person lives immediately
(a) One above J?
(b) Two (a) F
(c) None (b) S
(d) More than three (c) I
(e) Three (d) V
40. Who lives on the floor immediately below J? (e) None of these
(a) I 42. How many persons live between I and U?
(b) K (a) Four
(c) G (b) One
(d) F (c) Two
(e) None of These (d) Three
Direction (41-45): Study the following information (e) None of these
carefully and answer the questions given below: 43. Which of the following person lives on lowermost
Eight persons D, F, I, J, S, U, V and X are living in eight floor?
different floors of a building. The lowermost floor is (a) X
numbered 1, the one above floor is numbered 2, and so (b) D
on till the topmost floor is numbered 8 but not (c) J
necessarily in the same order. Persons name starts with (d) U
first half of the English alphabetical series lives on odd (e) None of these
numbered floors and the remaining person lives on even 44. Who among the following person lives three floors
numbered floors. Three persons live between V and X. J below S?
lives below S but not immediate below. Three persons (a) J
live between F and D. Flives above D. X lives below V. (b) U
Number of persons live below J is same as the number of (c) D
(d) J
Page 405 of 728

Subscribe the Xpress Video Course & Mock Test Package for Bank & Insurance Exams
If there are any suggestions/ errors in our PDFs Feel Free to contact us via this email: admin@exampundit.in
IBPS RRB PO Prelims – Ultra Practice Bundle PDF
(e) X (a) 10
45. If I is related to F, S is related to X, in the same (b) 11
way J is related to which of the following? (c) 15
(a) F (d) 8
(b) X (e) None of these
(c) D 47. How many persons live between D and the one
(d) U whose birthday date is 2?
(e) None of these (a) One
Direction (46-50): Study the following information (b) Two
carefully and answer the questions given below. (c) Three
Eight persons N, S, U, I, D, H, B and Z are living on (d) Four
eight different floors of a building but not necessarily in (e) None of these
the same order. Lowermost floor is numbered 1, the 48. How many people live above the person whose
above floor is numbered 2, and so on till the topmost birthday date is 1?
floor is numbered 8. Each one of them has his birthday (a) 1
on different dates from 1 to 8. Person who lives on floor (b) 2
number does not match with their birthday dates. U lives (c) 3
on 5th floor. N’s birthday date is 1. B lives abovethe one (d) 4
whose birthday date is 2. Three persons live between U (e) 5
and the one whose birthday date is 8. Birthday date of D 49. Whose birthday date is 5?
is a square number of a number. Two persons live (a) B
between H and Z. The one whose birthday date is 3 lives (b) D
immediately below H. More than three persons live (c) Z
between B and the one whose birthday date is 2. Ilives (d) U
immediately below S. Three persons live between N and (e) None of these
the one whose birthday date is 7. H’s birthday date is 50. Which of the following person lives on fourth
below 6. Z lives above U. floor?
46. What is the sum of birthday date of Z and S? (a) N
Page 406 of 728

Subscribe the Xpress Video Course & Mock Test Package for Bank & Insurance Exams
If there are any suggestions/ errors in our PDFs Feel Free to contact us via this email: admin@exampundit.in
IBPS RRB PO Prelims – Ultra Practice Bundle PDF
(b) H (d) I
(c) S (e) None of these

19). Floor Based Puzzle - Solution and Detailed Explanation

SOLUTION (1-5):
 T lives three floors below the topmost floor.
 Only three people live between S and W, who lives
exactly below T.
 Only four people live between U, who lives on the
2nd floor, and R

 P lives on an even numbered floor.


 Q does not live below X.

 Only two people live between X and T.


 Only four people live between X and V.
 X doesn’t live on the topmost floor

Final Arrangement
Page 407 of 728

Subscribe the Xpress Video Course & Mock Test Package for Bank & Insurance Exams
If there are any suggestions/ errors in our PDFs Feel Free to contact us via this email: admin@exampundit.in
IBPS RRB PO Prelims – Ultra Practice Bundle PDF

1.a
2.e 7. T and M floors are near to each other but both of

3.b them do not live on 9th and 10th floors

4.c 8. U is below B.

5.d (gap is one on down side) 9. U floor is not immediately before the B floor

SOLUTION (6-10):
1. H lives on 6th floor
2. K lives on 1st floor.
3. S floor is immediately above the H floor.

Final Arrangement
4. There are three floors between the F and D floors.
5. There are four floors between D and G.
6. D live above F

Page 408 of 728

Subscribe the Xpress Video Course & Mock Test Package for Bank & Insurance Exams
If there are any suggestions/ errors in our PDFs Feel Free to contact us via this email: admin@exampundit.in
IBPS RRB PO Prelims – Ultra Practice Bundle PDF

4. Three persons live between T and H.


5. U lives on one of the floors below E.
6.b 6. There is only one person living between V and U
7.e
8.a
9.d
10.c
SOLUTION (11-15):
1. There are as many persons live between E and H is
one more that the number of person between H and
F.
2. H live below E and does not live on even number
floor. 7. The number of persons living between G and V is
3. E lives on an even numbered floor but not on more than four.
topmost floor.

Page 409 of 728

Subscribe the Xpress Video Course & Mock Test Package for Bank & Insurance Exams
If there are any suggestions/ errors in our PDFs Feel Free to contact us via this email: admin@exampundit.in
IBPS RRB PO Prelims – Ultra Practice Bundle PDF
8. W does not lives in bottom most floor 3. There are two floors gap between G and E.

Final Arrangement

4. G lives immediate above H.


5. There are as many floors below H as above F.

11.e
12.c
13.a
14.e
6. I live immediate below D which doesn't live in
15.e
even numbered floor
SOLUTION (16-20): 7. There are five floors gap between B and D
1. E lives at odd numbered floor.
2. There are three floors gap between E and A.
Page 410 of 728

Subscribe the Xpress Video Course & Mock Test Package for Bank & Insurance Exams
If there are any suggestions/ errors in our PDFs Feel Free to contact us via this email: admin@exampundit.in
IBPS RRB PO Prelims – Ultra Practice Bundle PDF
20.c (adjacent to vacant floor)
SOLUTION (21-25):
Given condition: If persons place value is even as per
English alphabetical order, then they live on odd
numbered floor or else live on even numbered floor.
English Alphabetical place value of all 8 persons
B=2, I=9, H=8, J=10, O=15, T=20, Y=25 and U=21
[B, H, J and T lives on Odd & I, O, Y and U lives on
even]
1. Two persons live between Y and T.
2. T lives below Y but not on floor number 1.

8. C lives above the vacant floor. 3. Person whose place value is odd lives on even floor

Final Arrangement

4. Two person live between J and O who does not live


above T.
5. O lives above I.

16. d
17.c
18.e
19.a
Page 411 of 728

Subscribe the Xpress Video Course & Mock Test Package for Bank & Insurance Exams
If there are any suggestions/ errors in our PDFs Feel Free to contact us via this email: admin@exampundit.in
IBPS RRB PO Prelims – Ultra Practice Bundle PDF
Final Arrangement

21.a
22.c
23.b
6. U lives above B 24.b
7. Number of persons live below B is same as the 25.e (J)
number of Person lives above U. SOLUTION (26-30):
8. H does not live immediately above or immediately
1. E has 19 chocolate and stays on the third floor. Two
below Y.
persons are staying between E and the one who has
30 chocolate
2. Three persons are between H and D who live below
H.
3. H doesn’t live on 6th floor.
4. G has 35 chocolates.

Page 412 of 728

Subscribe the Xpress Video Course & Mock Test Package for Bank & Insurance Exams
If there are any suggestions/ errors in our PDFs Feel Free to contact us via this email: admin@exampundit.in
IBPS RRB PO Prelims – Ultra Practice Bundle PDF

11. One of the persons having 29 Chocolate who is not


5. Two people is between A and B. H.
12. The one who has highest number of chocolates has
46 chocolate.

6. F has least number of chocolates in the group


7. The one who has least number of chocolates has 11 Final Arrangement
chocolates.
8. Number of persons between B and the one having 16
chocolate is 2.
9. A has more chocolate than E
10. The difference in the number of chocolates of A and
D is 10.

Page 413 of 728

Subscribe the Xpress Video Course & Mock Test Package for Bank & Insurance Exams
If there are any suggestions/ errors in our PDFs Feel Free to contact us via this email: admin@exampundit.in
IBPS RRB PO Prelims – Ultra Practice Bundle PDF
26.a 9. There are two teachers live between the one who livs

27.b at floor 3rd and G, who lives below the one who
teaches in class 6th.
28.c
29.d
30.d
SOLUTION (31-35):
1. The one who teaches in class 9thlives on 5th floor.
2. A teaches in class 7 and lives on 4th floor.
3. Only three teachers live between F and A.
4. Only two teacher lives between F and H.
5. There are two teachers live between the one who
teaches in class 3rd and Gwho lives below the one 10. The one who lives on 6thfloor and C teach in same
who teaches in class 6th. class.
6. Both teachers who teach in class 6th live above the 11. D does not live on bottommost floor.
floor of C.

7. B and H teach in same class. (B is in even floor i.e in


2nd floor)
8. The one who lives on 3rd floor teaches in even
Final Arrangement
numbered class

Page 414 of 728

Subscribe the Xpress Video Course & Mock Test Package for Bank & Insurance Exams
If there are any suggestions/ errors in our PDFs Feel Free to contact us via this email: admin@exampundit.in
IBPS RRB PO Prelims – Ultra Practice Bundle PDF
4. The one who teaches in class 9thlives on 5th floor.
5. A teaches in class 7 and lives on 4th floor.
6. Four persons live between the one who lives in
Hyderabad and the one who lives in indore.
7. The person who lives in Hyderabad lives above the
person who lives in Mumbai.

31.b
32.c
33.e
34.d
35.c 8. K lives below the one who lives in Raipur and above
SOLUTION (36-40): the one who lives in Pune.
1. Less than three persons live below E who lives in 9. F lives immediately below G and immediately above
Chennai. H.
2. The person who lives in Delhi lives on the third floor 10. F lives on an odd numbered floor.
3. Three persons live between E and the one who lives 11. I lives above E.
in Mumbai. 12. J does not lives in Hyderabad.

Page 415 of 728

Subscribe the Xpress Video Course & Mock Test Package for Bank & Insurance Exams
If there are any suggestions/ errors in our PDFs Feel Free to contact us via this email: admin@exampundit.in
IBPS RRB PO Prelims – Ultra Practice Bundle PDF
D, F, I and J lives on odd floor & S, U, V and X lives
on even floor
1. Three persons live between V and X.
2. X lives below V.

Final Arrangement

3. Three persons live between F and D.


4. Flives above D.

36.c
37.c
38.e
39.a
40.b
SOLUTION (41-45): 5. J lives below S but not immediate below
Given condition: Persons name starts with first half of 6. Number of persons live below J is same as the
the English alphabetical series lives on odd numbered number of persons live above S
floors and the remaining person lives on even numbered
floors.

Page 416 of 728

Subscribe the Xpress Video Course & Mock Test Package for Bank & Insurance Exams
If there are any suggestions/ errors in our PDFs Feel Free to contact us via this email: admin@exampundit.in
IBPS RRB PO Prelims – Ultra Practice Bundle PDF

41.e
42.a
7. I lives above U but not immediately above.
43.b
44.d
45.c (one person between them in down ward)
SOLUTION (46-50):
1. U lives on 5th floor.
2. Three persons live between U and the one whose
birthday date is 8
3. Z lives above U
4. Two persons live between H and Z

Final Arrangement

Page 417 of 728

Subscribe the Xpress Video Course & Mock Test Package for Bank & Insurance Exams
If there are any suggestions/ errors in our PDFs Feel Free to contact us via this email: admin@exampundit.in
IBPS RRB PO Prelims – Ultra Practice Bundle PDF
5. The one whose birthday date is 3 lives immediately 10. N’s birthday date is 1.
below H 11. H’s birthday date is below 6.

Final Arrangement
6. I lives immediately below S.
7. More than three persons live between B and the one
whose birthday date is 2.
8. B lives above the one whose birthday date is 2.
9. Birthday date of D is a square number of a number.

46.d
47.c
48.d
49.c
50.a

Page 418 of 728

Subscribe the Xpress Video Course & Mock Test Package for Bank & Insurance Exams
If there are any suggestions/ errors in our PDFs Feel Free to contact us via this email: admin@exampundit.in
IBPS RRB PO Prelims – Ultra Practice Bundle PDF
20). BOX Based Puzzle

Directions (1-5): Study the following information (b) R


carefully to answer the given question: (c) C
Eight boxes B, C, D, P, Q, R, S and T are placed one (d) S
above another. Only three boxes are placed between box (e) None of these
P and R. Box C is placed immediately below box Q. Two 4. Which of the following box is placed at bottom?
boxes are placed between C and D. Only one box is (a) T
placed between box P and B who is above P. More than (b) R
one box is placed between box B and S. Box D is placed (c) C
above box S. Box T is neither placed at top nor below (d) S
box S. Box R is not placed at the top. (e) None of these
1. Which of the following box is placed at top? 5. Which of the following is true regarding P?
(a) S (a) P is placed above B
(b) B (b) P is at 2nd position from bottom
(c) D (c) D is just below P
(d) S (d) P is placed below B
(e) None of these (e) None of these
2. How many boxes are in between box Q and box Directions (6-10): Study the following information
S? carefully to answer the given question:
(a) Three Ten Boxes are placed one above the other in any
(b) Two particular order. Box number 1 is at the bottom and Box
(c) Four no 10 is at the top. Arrange them as per the information
(d) More than four given here. There are four boxes kept between U and X.
(e) None of these Box Y is placed at the top i.e. at box no. 10. There are
3. Which of the following box is placed just above two boxes between Box P and Box T. Box X is placed at
box T? odd number position. Box R is at 7th number position
(a) B from bottom. Box S and Box T is kept in a place
Page 419 of 728

Subscribe the Xpress Video Course & Mock Test Package for Bank & Insurance Exams
If there are any suggestions/ errors in our PDFs Feel Free to contact us via this email: admin@exampundit.in
IBPS RRB PO Prelims – Ultra Practice Bundle PDF
immediately above and immediately below Box R. Box (e) Box T
Q is placed exactly between the box V and box X. One 10. Which boxes are placed in the middle of the
of the boxes placed is W. arrangement?
6. Which box is placed at the fifth from bottom? (a) Box U and Box R
(a) Box P (b) Box T and Box Q
(b) Box U (c) Box E and Box F
(c) Box Q (d) Box V and Box W
(d) Box T (e) Box Q and Box S
(e) None of these Directions (11-15): Study the information carefully
7. Which of the following box are between S and Q? and answer the questions given below.
(a) Box R, T Nine boxes are placed one above another in a stack. Five
(b) Box X, Y boxes are placed between L and M which is below L.
(c) Box T, Q Three boxes are placed between box N and O, which is
(d) Box P, W placed above box M. There are four boxes are placed
(e) Box P, V between box Q and box P, which is placed just above
8. Which of the following Box is placed at position box L. More than two boxes are placed between box O
no. 8? and box R, which is placed just below box N. Less than
(a) Box Y Four boxes are placed between box S and box T, which
(b) Box X is placed below box S.
(c) Box S 11. Which of the following box is placed two box
(d) Box R above box R?
(e) None of these (a) N
9. Which of the following Box is placed immediately (b) S
below Box W? (c) Q
(a) Box P (d) T
(b) Box U (e) None of these
(c) Box Q 12. Which of the following box is placed fourth from
(d) Box V bottom of the stack?
Page 420 of 728

Subscribe the Xpress Video Course & Mock Test Package for Bank & Insurance Exams
If there are any suggestions/ errors in our PDFs Feel Free to contact us via this email: admin@exampundit.in
IBPS RRB PO Prelims – Ultra Practice Bundle PDF
(a) M Direction (16-20): Read the given information
(b) T carefully and answer the questions;
(c) R Six boxes G, H, I, J, K and L are placed one above
(d) C another, also they are occupied with different items i.e.
(e) None of these R, T, U, V, W and X (but not necessarily in the same
13. How Many boxes are placed between box L and order). G is placed either top or bottom, only 3 boxes are
Box T? placed between G and J, which is occupied with V. Only
(a) Four 2boxes are placed between J and L. Box K is not placed
(b) Three just above and below J. Box K is not placed just above
(c) More than four the box which is occupied with R. The box which is
(d) One placed just below H is occupied with T. The box which
(e) None is placed at bottom occupied with W. Box which is
14. Which of the following statement is True about occupied with X is placed somewhere between J and L.
box N? 16. Box I is occupied with which of the following
(a) Box N is placed just above box S item?
(b) Not more than two boxes are placed between box (a) R
M and N (b) T
(c) One box is placed between L and N (c) U
(d) Less than three boxes are placed above box N (d) V
(e) Both (b) and (d) (e) None of these
15. Which of the following box is placed just below 17. Which of the following box placed at top box?
box Q? (a) K
(a) T (b) H
(b) M (c) J
(c) R (d) I
(d) N (e) L
(e) S 18. How many boxes are placed between box H and
box G?
Page 421 of 728

Subscribe the Xpress Video Course & Mock Test Package for Bank & Insurance Exams
If there are any suggestions/ errors in our PDFs Feel Free to contact us via this email: admin@exampundit.in
IBPS RRB PO Prelims – Ultra Practice Bundle PDF
(a) One Hyderabad. The box Q is not storing at Raipur. The box
(b) Three U is stored at Bhopal and the box S is stores at Pune.
(c) Two Only four boxes kept between V and W which is below
(d) More than three V. The box W is not storing at Indore. As many boxes
(e) None of these between W and Q as between W and S. P kept
19. Which of the following item is placed just above immediate above the box placed at Delhi. Only three
box L? boxes kept between the box stores at Mumbai and
(a) T Indore. Box S is not kept below box W.
(b) The box which is occupied with J 21. T box stores at which city?
(c) X (a) Delhi
(d) Both (b) and (c) (b) Mumbai
(e) None of these (c) Hyderabad
20. Box H is occupied with which of the following (d) Raipur
item? (e) None of these
(a) T 22. Which of the following box is placed at top?
(b) X (a) U
(c) U (b) T
(d) W (c) P
(e) None of these (d) V
Direction (21-25): Read the given information (e) None of these
carefully and answer the questions 23. How many boxes are placed between P and the box
Eight boxes P, Q, R, S, T, U, V and W are arranging in which stores at Chennai?
top to bottom. Each box are stores in different cities- (a) One
Raipur, Indore, Bhopal. Pune, Mumbai, Delhi, (b) Three
Hyderabad, and Chennai, but not necessary in the same (c) Two
order. Only two boxes kept between V and U. Both the (d) More than three
box which is placed at Mumbai and Indore kept above R. (e) None of these
Only one box kept between T and the box which stores at 24. Which of the following box stores at Raipur?
Page 422 of 728

Subscribe the Xpress Video Course & Mock Test Package for Bank & Insurance Exams
If there are any suggestions/ errors in our PDFs Feel Free to contact us via this email: admin@exampundit.in
IBPS RRB PO Prelims – Ultra Practice Bundle PDF
(a) S are placed between the boxes in which Mango fruit and
(b) P Apple fruit is placed. Box P is not placed on Mango
(c) R fruit.
(d) Q 26. What is the fruit of box U?
(e) None of these (a) Pineapple
25. How many boxes above W? (b) Guava
(a) 1 (c) Apple
(b) 2 (d) Mango
(c) 3 (e) None of these
(d) 4 27. Which of the following is Papaya box?
(e) 5 (a) Q
Direction (26 30): Study the following information (b) T
carefully to answer the given questions. (c) R
Seven boxes P, Q, R, S, T, U and V of different fruits (d) U
viz. Apple, Orange, Guava, Mango, Litchi, Pineapple (e) None of these
and Papaya are placed one above the other, but not 28. Which of the following box is two boxes below V?
necessarily in the same order. The box at bottom is (a) P
numbered as 1, the above box is numbered as 2 and so on (b) R
till topmost box is numbered as 7. Box U is placed (c) U
immediately above box R. less than three boxes are (d) T
placed below the Apple box. Box T is placed below the (e) Q
box in which Applefruits. The box which having mango 29. Which of the following box placed at bottom?
fruit is placed immediately above box V. Box P is placed (a) U
immediately above the box in which Pineapple fruit. (b) R
Guava fruit box is placed above the box U. Only two (c) T
boxes are placed between Guava box and Papaya box. (d) Q
The Orange box is not placed on the bottom. Box T is (e) None of these
placed immediately above Box Q. There are three boxes 30. How many boxes are placed below box S?
Page 423 of 728

Subscribe the Xpress Video Course & Mock Test Package for Bank & Insurance Exams
If there are any suggestions/ errors in our PDFs Feel Free to contact us via this email: admin@exampundit.in
IBPS RRB PO Prelims – Ultra Practice Bundle PDF
(a) Four (d) Q, M
(b) Two (e) None of these
(c) None 32. Which of following statement istrue?
(d) Three (a) P is kept immediately above L.
(e) More than Five (b) Only two boxes between L and Q.
Directions (31-35): Study the following information (c) K is placed immediately above R
carefully and answer the questions below. (d) All of these.
Nine boxes – J, K, L, M, N, O, P, Q and R are arranged (e) None of these.
one above another indifferent shelves numbered 1 to 9 33. How many boxes are between Pand J?
from bottom to top. The lowermost shelf is numbered as (a) 1
1 and the shelf above 1 is numbered as 2 and so on. All (b) 3
the information is not necessary to be in the same order. (c) 2
The shelf number and the sum of digit of place value of (d) 4
the alphabet is not same in any shelves. (E.g. Place value (e) 0
of Box K in alphabet series is 11 and sum of digits of K 34. Which of the following box placed top and bottom
is 1+1= 2; so box k is not placed in the shelf numbered respectively?
2). Box O is placed at one of the positions above Box N (a) Q, M
in an even numbered position. Box M is two places (b) L, J
below box N. Number of boxes below N is as same as (c) K, N
the number of boxes above L. Box L is placed at one of (d) P, M
the positions above N. Four boxes are placed between (e) P, O
Box P and R. Box K is placed below J but above Q. 35. Which of the following keptimmediately above
More than two boxes are placed between Box P and Box box K?
Q. (a) R
31. Which among the following box is adjacent to J? (b) N
(a) L, K (c) K
(b) K, R (d) Q
(c) L, O (e) None of these
Page 424 of 728

Subscribe the Xpress Video Course & Mock Test Package for Bank & Insurance Exams
If there are any suggestions/ errors in our PDFs Feel Free to contact us via this email: admin@exampundit.in
IBPS RRB PO Prelims – Ultra Practice Bundle PDF
Directions (36-40): Study the following information 38. Which among the following boxes contains the
carefully and answer the questions given below: maximum number of balls?
Seven boxes P, Q, R, S, T, U and V are kept one above (a) R
the other containing different number of ball ranging (b) S
from 30- 80. Box U is below box S. Three boxes are kept (c) T
between P and the box containing 43balls, which is kept (d) V
above Box P. Box S contains twice number of balls than (e) W
box Q. Box R contains 55numbers of balls and is not 39. Which of the following combination is true?
kept at the bottom. The number of balls in box V is a (a) R- 55
cube of a number. Only two boxes are kept between box (b) T- 76
containing 43balls and 37balls. Box S has a greater (c) Q- 37
number of balls than box P. One of the boxes contains (d) P – 64
76balls. Five boxes are kept between box containing (e) None of these
66balls and Box R. Box V is immediately below box T. 40. Which among the following boxes is kept
Box S is not kept immediately below or above box Q. immediately below box T?
Three boxes are kept between box S and box U. (a) Q
36. How many balls does box V has? (b) U
(a) 55 (c) P
(b) 76 (d) V
(c) 43 (e) None of these
(d) 64 Directions (41-45): Study the information carefully
(e) 37 and answer the questions given below.
37. How many balls are kept in box P? Six boxes G, K, M, N, S and T are placed one above
(a) 55 another in a stack. Each box has different height i.e.
(b) 64 18cm, 7cm, 45cm, 51cm, 14cm and 40cm. All
(c) 38 information is not necessarily in same order. Box M has
(d) 37 height that is odd number. Three box is placed between
(e) 66 M and N and N’s height is 40cm. The box whose height
Page 425 of 728

Subscribe the Xpress Video Course & Mock Test Package for Bank & Insurance Exams
If there are any suggestions/ errors in our PDFs Feel Free to contact us via this email: admin@exampundit.in
IBPS RRB PO Prelims – Ultra Practice Bundle PDF
is prime number is placed just below Box N. Two boxes (a) Two
are place between box G and box S which is not placed (b) One
below N. One box is placed between the box which (c) Three
height is 18cm and the box which height is 51cm. Two (d) Four
boxes is placed between the box which height is 18cm (e) None of these
and the box which height is 45cm. Box K is placed 45. What is the difference between the height of box
below the box T, which isn’t placed below box G. Box G G and the box which is just above box N?
height is not greater than 40cm. Sum of the heights of the (a) 28
boxes placed adjacent to Box K is more than 60cm. (b) 7
41. Which of the following box height is 51cm? (c) 27
(a) S (d) 4
(b) N (e) None of these
(c) T Directions (46-50): Study the following information
(d) M carefully and answer the given questions:
(e) K Seven boxes P, Q, R, S, A, B and C are placed one above
42. How many boxes are placed above box G? other, but not necessarily in the same order. Each of
(a) One them store different food items viz. Eggs, Bread,
(b) Three Maggie, Meat, Chicken, Toast and Biscuit but not
(c) None necessarily in the same order.
(d) Four Box B does not store Maggie. Box Q is not placed below
(e) five box S. Box which store Eggs is placed immediately
43. Which of the following box height is 7cm? above box P. There are two boxes placed between box S
(a) T and the box which store Eggs. Box which store Maggie
(b) G is placed just above box S. Three boxes are placed
(c) N between box R and the box which stores Maggie. The
(d) S box which stores Biscuit is placed immediately above
(e) K box R. The box which stores Meat is placed immediately
44. How many boxes have Height more than box G? above box C. Only two boxes is placed between box Q
Page 426 of 728

Subscribe the Xpress Video Course & Mock Test Package for Bank & Insurance Exams
If there are any suggestions/ errors in our PDFs Feel Free to contact us via this email: admin@exampundit.in
IBPS RRB PO Prelims – Ultra Practice Bundle PDF
and box A. Box Q neither store Maggie nor Biscuit. Box (a) C, Q
A does not store Meat. Box which store Bread is placed (b) P, S
above chicken. Two Box between the box which stores (c) B, R
Bread and Chicken. Box R and Q do not have eggs. (d) R, C
46. Which of the following food items stores by C? (e) B, S
(a) Biscuit 49. Which of the following combination is true?
(b) Bread (a) Q – Meat
(c) Meat (b) A – Bread
(d) Eggs (c) B - Toast
(e) Chicken (d) S- Chicken
47. Which of the following Box contains Maggie? (e) A- Biscuit
(a) Q 50. How many boxes are placed below the box which
(b) R contains Bread?
(c) P (a) 2
(d) S (b) 3
(e) A (c) 4
48. Which of the following pair of boxes are placed at (d) 5
top and bottom respectively? (e) 1

20). BOX Based Puzzle - Solution and Detailed Explanation


SOLUTION (1-5):
 Only three boxes are placed between box P and R
 Only one box is placed between box P and B who is
above P
 Box R is not placed at the top. (It means that
minimum 1 box above R)

Page 427 of 728

Subscribe the Xpress Video Course & Mock Test Package for Bank & Insurance Exams
If there are any suggestions/ errors in our PDFs Feel Free to contact us via this email: admin@exampundit.in
IBPS RRB PO Prelims – Ultra Practice Bundle PDF

 More than one box is placed between box B and


S.
 Box D is placed above box S.
 Box T is neither placed at Top nor below box S.
 Box C is placed immediately below box Q.
 Two boxes are placed between C and D.
 Box D is placed above box S. (So we cannot
place D at bottom)

1.b
2.a
3.c

Page 428 of 728

Subscribe the Xpress Video Course & Mock Test Package for Bank & Insurance Exams
If there are any suggestions/ errors in our PDFs Feel Free to contact us via this email: admin@exampundit.in
IBPS RRB PO Prelims – Ultra Practice Bundle PDF
4.d
5.d
SOLUTION (6-10):
1. Box Y is placed at the top i.e at box no. 10
5. Box Q is placed exactly between the box V and box
2. Box R is at 7th number position from bottom. Box S
X.
and Box T is kept in a place immediately above and
immediately below Box R.

3. There are four boxes kept between U and X.


4. There are two boxes between Box P and Box T. FINAL ARRANGEMENT

Page 429 of 728

Subscribe the Xpress Video Course & Mock Test Package for Bank & Insurance Exams
If there are any suggestions/ errors in our PDFs Feel Free to contact us via this email: admin@exampundit.in
IBPS RRB PO Prelims – Ultra Practice Bundle PDF

3. Three boxes are placed between box N and O, which


6.c is placed above box M.
7.a 4. More than two boxes are placed between box O and
8.c box R, which is placed just below box N.
9.d
10.b
SOLUTION (11-15):
1. Five boxes are placed between L and M which is
below L.
2. There are four boxes are placed between box Q and
box P, which is placed just above box L.

Page 430 of 728

Subscribe the Xpress Video Course & Mock Test Package for Bank & Insurance Exams
If there are any suggestions/ errors in our PDFs Feel Free to contact us via this email: admin@exampundit.in
IBPS RRB PO Prelims – Ultra Practice Bundle PDF
5. Less than three boxes are placed between box S and 15.a
box T, which is placed below box S. SOLUTION (16-20):
1. G is placed either top or bottom.
2. Only 3 boxes are placed between G and J, which is
occupied with V.
3. The box which is placed at bottom occupied with W

FINAL ARRANGEMENT
4. Only 2 boxes are placed between J and L.
5. Box K is not placed just above and below J.

6. The box which is placed just below H is occupied


11.b with T.
12.c 7. Box which is occupied with X is placed somewhere
13.a between J and L.
14.c

Page 431 of 728

Subscribe the Xpress Video Course & Mock Test Package for Bank & Insurance Exams
If there are any suggestions/ errors in our PDFs Feel Free to contact us via this email: admin@exampundit.in
IBPS RRB PO Prelims – Ultra Practice Bundle PDF
8. The box which is placed just below H is occupied 18.c
with T 19.a
20.b
SOLUTION (21-25):
1. Only four boxes kept between V and W which is
below V.
2. As many boxes between W and Q as between W and
S.
3. Only two boxes kept between V and U.
9. Box K is not placed just above the box which is
occupied with R.

4. The box U is stores at Bhopal and the box S is stores


FINAL ARRANGEMENTS at Pune
5. Only one box kept between T and the box which
stores at Hyderabad.
6. Both the box which is placed at Mumbai and Indore
kept above R.
7. P kept immediate above the box placed at Delhi.

16.a
17.d

Page 432 of 728

Subscribe the Xpress Video Course & Mock Test Package for Bank & Insurance Exams
If there are any suggestions/ errors in our PDFs Feel Free to contact us via this email: admin@exampundit.in
IBPS RRB PO Prelims – Ultra Practice Bundle PDF
Final Arrangement

8. Only three boxes kept between the box stores at


Mumbai and Indore. 21.a
9. The box W is not storing at Indore. 22.d
10. The box Q is not storing at Raipur 23.d
24.c
25.e

SOLUTION (26-30):
1. Less than three boxes are placed below the Apple
box.
2. Box T is placed below the box in which Apple fruits.
3. Box T is placed immediately above Box Q.
4. There are three boxes are placed between the boxes
in which Mango fruit and Apple fruit is placed.

Page 433 of 728

Subscribe the Xpress Video Course & Mock Test Package for Bank & Insurance Exams
If there are any suggestions/ errors in our PDFs Feel Free to contact us via this email: admin@exampundit.in
IBPS RRB PO Prelims – Ultra Practice Bundle PDF
11. The Orange box is not placed on the bottom.

5. The box which having mango fruit is placed


immediately above box V. .

6. Guava fruit box is placed above the box U. FINAL ARRANGEMENT

7. Only two boxes are placed between Guava box and


Papaya box
8. Box U is placed immediately above box R.

26.a
27.b
28.c
29.d
30.e
9. Box P is not placed on Mango fruit. SOLUTION (31-35):
10. Box P is placed immediately above the box in which 1. Box O is placed at one of the position above Box N
Pineapple fruit. in an even numbered position.
Page 434 of 728

Subscribe the Xpress Video Course & Mock Test Package for Bank & Insurance Exams
If there are any suggestions/ errors in our PDFs Feel Free to contact us via this email: admin@exampundit.in
IBPS RRB PO Prelims – Ultra Practice Bundle PDF
2. Box M is two places below box N.

5. Four boxes are placed between Box P and R.

3. Number of boxes below N is as same as the number


of boxes above L.
4. Box L is placed at one of the positions above N.

6. Box K is placed below J but above Q.


7. More than two boxes are placed between Box P and
Box Q.

Page 435 of 728

Subscribe the Xpress Video Course & Mock Test Package for Bank & Insurance Exams
If there are any suggestions/ errors in our PDFs Feel Free to contact us via this email: admin@exampundit.in
IBPS RRB PO Prelims – Ultra Practice Bundle PDF
2. Box R contains 55 numbers of balls and is not kept at
the bottom.
3. Three boxes are kept between P and the box
containing 43 balls, which is kept above Box P.

FINAL ARRANGEMETS

4. Only two boxes are kept between box containing 43


balls and 37 balls
5. Three boxes are kept between box S and box U
6. Box S is above box U

31.a
32.c
33.c
34.d
7. Box V is immediately below box T.
35.e
8. Box S is not kept immediately below or above box Q
SOLUTION (36-40):
9. The number of balls in box V is a cube of a number
1. Five boxes are kept between box containing 66 balls
and Box R.
Page 436 of 728

Subscribe the Xpress Video Course & Mock Test Package for Bank & Insurance Exams
If there are any suggestions/ errors in our PDFs Feel Free to contact us via this email: admin@exampundit.in
IBPS RRB PO Prelims – Ultra Practice Bundle PDF
36.d
37.e
38.b
39.a
40.d
SOLUTION (31-35):
1. Three box is placed between M and N and N’s height
is 40cm
2. The box whose height is prime number is placed just
10. Box S has a greater number of balls than box P.
below Box N
11. One of the boxes contains 76 balls.
3. Two boxes are place between box G and box S which
12. Box S contains twice number of balls than box Q.
is not placed below N

FINAL ARRANGEMENT 4. Box K is placed below the box T, which isn’t placed
below box G.
5. One box is placed between the box which height is
18cm and the box which height is 51cm.
6. Two boxes is placed between the box which height is
18cm and the box which height is 45cm.
7. Box G height is not greater than 40cm.
Case-1 Case-2

Page 437 of 728

Subscribe the Xpress Video Course & Mock Test Package for Bank & Insurance Exams
If there are any suggestions/ errors in our PDFs Feel Free to contact us via this email: admin@exampundit.in
IBPS RRB PO Prelims – Ultra Practice Bundle PDF
Box Box 2. The box which stores Biscuit is placed immediately
Boxes Height Boxes Height
Number Number above box R.
6 M 45cm 6 S 45cm 3. There are two boxes placed between box S and the
5 T 51cm 5 N 40cm box which store Eggs.
4 S 14cm 4 T 7cm 4. Box which store Maggie is placed just above box S.
3 K 18cm 3 G 18cm
2 N 40cm 2 K 14cm
1 G 7cm 1 M 51cm
8. Sum of the heights of the boxes placed
adjacent to Box K is more than 60cm.
With this condition (8) case-1 gets eliminated.
FINAL ARRANGEMENT

5. Box which store Eggs is placed immediately above


box P.
6. Box P does not store Bread
7. Box which store Bread is not placed at the bottom.
8. Two Box between the one who stores Bread and
Chicken.

41.d
42.b
43.a
44.c
45.c
SOLUTION (46-50):
1. Three boxes are placed between box R and the box
9. Only two boxes is placed between box Q and box A
which stores Maggie.
Page 438 of 728

Subscribe the Xpress Video Course & Mock Test Package for Bank & Insurance Exams
If there are any suggestions/ errors in our PDFs Feel Free to contact us via this email: admin@exampundit.in
IBPS RRB PO Prelims – Ultra Practice Bundle PDF
10. Box Q neither store Maggie nor Biscuit. FINAL ARRANGEMENTS
11. The box which stores Meat is placed immediately
above box C.

46.d
47.e
12. Box B does not store Maggie.
48.e
49.a
50.d

Page 439 of 728

Subscribe the Xpress Video Course & Mock Test Package for Bank & Insurance Exams
If there are any suggestions/ errors in our PDFs Feel Free to contact us via this email: admin@exampundit.in
IBPS RRB PO Prelims – Ultra Practice Bundle PDF
21). Month Based Puzzle
Directions (1-5): Study the following information (b) Indore
carefully to answer the given question: (c) Delhi
Seven People L, S, K, M, H, C and I have a Interview on (d) Mumbai
seven different months (of the same year), namely (e) Raipur
January, February, March, April, May, July and 2. In which of the followingcity does C have his
Augustbut not necessarily in the same order inand each interview?
of them have interview in different city namely Indore, (a) Raipur
Delhi, Mumbai, Chennai, Goa, Raipur and Pune but not (b) Mumbai
necessarily in the same order. The one who has interview (c) Goa
in Raipur has in a month which has less than 30 days. K (d) Indore
has interview in a month which has less than 31 days. (e) Delhi
Only two people have interview between K and M. M 3. How many people has/have Interview between the
has interview immediately before the one who has month in which H and K have?
interview in Chennai. S has interview immediately after (a) None
the one who has interview in Raipur. Only three people (b) 2
have interview between S and the one who has interview (c) 1
in Delhi. H has interview immediately before L. Ihas (d) 3
interview in Pune. The person whose interview in Goa in (e) 4
the month which has less than 31 days. The one who has 4. Who among the following have Interview in
interview immediately after February monthnot in the January and May respectively?
city Indore. (a) K, S
1. As per the given arrangement, Iis related to (b) S, C
Raipur and S is related to Goa in a certain way. (c) C, M
Which of the following is M related to in the same (d) M, H
way? (e) I, M
(a) Chennai
Page 440 of 728

Subscribe the Xpress Video Course & Mock Test Package for Bank & Insurance Exams
If there are any suggestions/ errors in our PDFs Feel Free to contact us via this email: admin@exampundit.in
IBPS RRB PO Prelims – Ultra Practice Bundle PDF
5. In which of the following months does M have his (d) Either (b) or (c)
interview? (e) None of these
(a) January 7. On which of the following month Mithali has
(b) May exam?
(c) July (a) May
(d) August (b) August
(e) Cannot be determined (c) July
Directions (6-10): Study the following information (d) Can’t be determined
carefully to answer the given question: (e) None of these
There are seven persons namely Virat, Rahul, Sachin, 8. Which among the following statements is true?
Kapil, Saina, Sania and Mithali. They studying in (a) Two persons have exam between Kapil
different college have exams in different months of the andSachin.
year viz. January, March, May, July, August, October, (b) Saina has exam before Sachin.
December not necessarily in the same order. No two (c) Sania has exam in May.
persons have a exam on the same month of the year. (d) Virat doesn’t have exam in March
Three persons have exam between the months in which (e) None of these
Virat and Kapil have. Neither Mithali nor Sania has a 9. If Sania has exam in August. Then, who among the
exam on January and December. Only one person has a following has exam immediately before Mithali?
exam between the months in which Sania and Mithali (a) Sachin
has. There are more than two persons who have exams (b) Virat
between the months in which Rahul and Sachin have. (c) Kapil
Sachin has exam neither in March nor in May. Virat has (d) Can’t be determined
a exam before Kapil but not in May. Rahul has a exam in (e) None of these
December 10. Four of the five are alike in a certain way and
6. Who among the following has a exam in July? hence form a group. Find the one that doesn’t belong
(a) Sania to that group.
(b) Mithali (a) Virat, Sachin
(c) Saina (b) Kapil, Rahul
Page 441 of 728

Subscribe the Xpress Video Course & Mock Test Package for Bank & Insurance Exams
If there are any suggestions/ errors in our PDFs Feel Free to contact us via this email: admin@exampundit.in
IBPS RRB PO Prelims – Ultra Practice Bundle PDF
(c) Saina, Sania 12. Four of the following five are alike in a certain
(d) Mitahli, Sania way and hence form a group. Which one of the
(e) Mithali, Saina followingdoes not belong to that group?
Directions (11-15): Study the following information (a) P
carefully to answer the given question: (b) U
Twelve friends M, N, O, P, Q, R, S, T, U, V, W and X (c) S
were going for picnic in different months of the same (d) X
year. From January to December.M goes to picnic in the (e) V
month which has least number of days. S goes to picnic 13. Who among the following goes picnic in the
in the month of July. V was going to picnic immediately month of June?
after the month in which W goes and immediately before (a) M
the month in which O goes. V goes to picnic neither in (b) P
the month of September nor in April. There is a gap of (c) Q or T
three months between the one who goes picnic between (d) U
X and N. There were 31 days in the month in which X (e) X
goes to picnic. P goes to picnic before the month in 14. How many friends go to picnic after S?
which U goes. There is a gap of two month between the (a) None
month in which N and R goes. Q and T were goes to (b) Three
picnic in that months which had 30 days each.There were (c) Four
30 days in the month in which N goes to picnic. (d) Five
11. In which of the following months Vgoes to picnic? (e) Six
(a) December 15. How many goes to picnic between W and M?
(b) June (a) 2
(c) March (b) 3
(d) November (c) 4
(e) September (d) 6
(e) 7

Page 442 of 728

Subscribe the Xpress Video Course & Mock Test Package for Bank & Insurance Exams
If there are any suggestions/ errors in our PDFs Feel Free to contact us via this email: admin@exampundit.in
IBPS RRB PO Prelims – Ultra Practice Bundle PDF
Directions (16-20): Study the following information (e)None of the above is true
carefully to answer the given question: 18. If everyone’s postponed their vacation by one
There are seven employees of a company-E, F, G, H, I, J month, who will be going on March?
and K goes to different water park for vacation i.e., Fun (a)I
and food, Dream world, Wonderlands. Each of them (b) F
goes on different months of the year i.e. February, (c) E
March, April, May, June, July and August. Minimum (d) G
two people goes to each water park and Fun and Food is (e)None of these
the only city to which three people goes. E goes to Fun 19. On which month and to which water park does K
and food on March. I go to Dream world but neither on go?
July nor on April. J goes on August but not to (a) March, Fun & Food
Wonderlands. The one who goes to Wonderland goes on (b) May, Wonderland
May. H goes on June. K goes to Fun and Food but not on (c) July, Fun & food
April. F neither goes to Wonderland nor Fun and food. I (d) June, Dream world
do not go after K. (e) None of these
16. Which of the following employees goes Water 20. Who among the following employee goes for
Park in July? vacation on April?
a. E (a) I
b. F (b) E
c. G (c) F
d. I (d) H
e. K (e) K
17. Which one of the following combinations is true Directions (21-25): Study the following information
according to the given information? carefully to answer the given question:
(a)E –April– Dream world Six Bride grooms P, Q, R, S, T and U married within a
(b) G –March– Fun & Food Year in the months March, May, June, August,
(c)H –June – Wonderland September and December. The Brides are A, B, C, D, E
(d) J – July – Wonderland and F. All of them got married in different Place of India
Page 443 of 728

Subscribe the Xpress Video Course & Mock Test Package for Bank & Insurance Exams
If there are any suggestions/ errors in our PDFs Feel Free to contact us via this email: admin@exampundit.in
IBPS RRB PO Prelims – Ultra Practice Bundle PDF
as Goa, Mumbai, Delhi, Chennai, Hyderabad and (c) Delhi
Lucknow. All the information is not necessarily in same (d) Lucknow
order. The following are some facts about their (e) Goa
marriages. R’s wedding took place in the month of 23. Which of the following combinations of Bride -
August but neither in Goa nor in Hyderabad, however he Groom - month is correct?
was not married A or F. P’s wedding took place in (a) A - Q - June
Mumbai and T’s wedding in Lucknow, but P was not (b) B- U- September
married to B or D. The wedding in Lucknow took in the (c) Q- C- May
month which has 30days. The Bride C’s wedding took in (d) P- F- March
the month of which has 31days, but in Goa. E and A got (e) None of these
married in the month which has 31 and 30 days 24. Q married in which place and in which month?
respectively and in Delhi and Lucknow, but not (a) Hyderabad, December
necessarily in same order. S's marriage was held in the (b) Lucknow, August
month of March. U was married to B but not in 31 days (c) Goa, May
month. Q’s marriage did not take place in neither (d) Chennai, December
Chennai nor Goa. D’s marriage took place in Hyderabad. (e) None of these
The couple who’s married are in the month of March and 25. Who is married to R?
December not took place in Mumbai. Bride T is not (a) A
married after R. (b) B
21. In which month does B’s married? (c) C
(a) March (d) E
(b) May (e) Can’t be determine
(c) June Directions (26-30): Study the following information
(d) December carefully to answer the given question:
(e) September Seven persons A, B, C, D, E, F and G are born in
22. In which City D’smarried? different months viz., February, April, June, August,
(a) Chennai October, November and December in the same year.
(b) Hyderabad Each of them likes a different fruit viz., Apple, banana,
Page 444 of 728

Subscribe the Xpress Video Course & Mock Test Package for Bank & Insurance Exams
If there are any suggestions/ errors in our PDFs Feel Free to contact us via this email: admin@exampundit.in
IBPS RRB PO Prelims – Ultra Practice Bundle PDF
Grape, Orange, Guava, Pineapple, Litchi but not 28. In a certain way E is related to Banana and C is
necessary in the same order. B was born in the month of related to Orange. Then D is related to which of
which has 31 day but not in October and does not like the following?
either litchi or Banana. C likes apple and was not born in (a) Orange
the month which have 31 days. E was born in the month (b) Litchi
which has least number of days and likes Grape. The one (c) Guava
who likes Guava was born in the month of November. D (d) Pineapple
was born in the month which has 31days but he is not (e) None of these
youngest. G likes Pineapple. A was not born in the 29. Which of the following combinations is not
month of November. The one who likes Banana was correct?
born in the month which has 30days but not in April. The (a) April-C-Apple
one who like pineapple fruit born after the one who like (b) August-B- Orange
Litchi. The person who likes Orange does not born after (c) October-D-Litchi
pineapple. (d) December –G-Pinneaple
26. F was born in which of the following months? (e) All are correct
(a) November 30. Who was born in the month of October?
(b) December (a) A
(c) June (b) B
(d) October (c) D
(e) None of these (d) F
27. A likes which of the following Fruit? (e) G
(a) Grape Directions (31-35): Study the following information
(b) Apple carefully to answer the given question:
(c) Banana Eight persons E, F, G, H, I, J, K and L are born in
(d) Orange different months of the same year viz., January,
(e) None of these February, March, April, June, August, September and
November but not necessary in the same order.

Page 445 of 728

Subscribe the Xpress Video Course & Mock Test Package for Bank & Insurance Exams
If there are any suggestions/ errors in our PDFs Feel Free to contact us via this email: admin@exampundit.in
IBPS RRB PO Prelims – Ultra Practice Bundle PDF
K is born in one of the months which have 30 days (b) K
before September. Three persons are born between L and (c) G
K. Only two persons are born between L and G. As many (d) E
as persons are born after G is two less than the number of (e) Can’t be determined
persons born before E. As many persons born between G 35. Four of the following five are alike in a certain
and I is one more than the number of persons born way and hence form a group. Which of the
between H and G. J is born in one of the months before following onethat does not belong to the group?
H. F is older than H. (a) K
31. Who among the following is the eldest person? (b) G
(a) E (c) H
(b) I (d) L
(c) F (e) E
(d) J Directions (36-40): Study the following information
(e) Can’t be determined carefully to answer the given question:
32. E was born in which of the following months? Seven persons Q, R, S, T, U, V and W are going to USA
(a)June for vacation in seven different months of the same year
(b) August i.e. March, April, May, June, July, September and
(c) September October but not necessary in same order.
(d) November T goes in the month which has odd number of days.
(e) None of these Three persons go between T and W. R goes before T.
33. How many persons are born between H and I? Less than two people go between T and S. V goes just
(a) One before S. Not less than to two persons go between R and
(b) Two W. Q does not go in the month which has even number
(c) Three of day. S does not go in July.R does not go for vacation
(d) More than three before Q. W and V are not immediate neighbour.
(e) None 36. In which of the following month R goes?
34. Who among the following is immediately after F? (a) June
(a) I (b) April
Page 446 of 728

Subscribe the Xpress Video Course & Mock Test Package for Bank & Insurance Exams
If there are any suggestions/ errors in our PDFs Feel Free to contact us via this email: admin@exampundit.in
IBPS RRB PO Prelims – Ultra Practice Bundle PDF
(c) August (d) T
(d) March (e) None of these
(e) None of these Directions (41-45): Study the following information
37. Who of the following person goes on April? carefully to answer the given question:
(a) U Eight persons i.e. A, B, C, D, E, F, G and H are
(b) R celebrating their anniversary in different months i.e.
(c) T January, February, April, May, June, July, August and
(d)V September. Each of them likes different flower i.e. Rose,
(e) None of these Lily, Dairy, Tulip, Lotus, Orchid, Sunflower and Iris. All
38. How many persons go between V and U? information is not necessarily in the same order.
(a) Three Not less than three people born between C and the one
(b) Two who likes lotus, both persons born in month having 30
(c) Four days. B born in June and likes Dairy. The person who
(d) Five likes Tulip born just after the one who likes Orchid. H
(e) None of these does like Tulip. Only four persons born between E and
39. Four of the following five are alike in certain way the one who likes Sunflower. Number of persons born
based from a group, find the one that does not after the one who likes Iris and before the one who likes
belong to that group? Dairyis same. E born in month having 30 days and after
(a) W the person who likes Sun flower. F born after D but
(b) Q before H but not just before and after. A born in the
(c) T month having 31 days and does like Rose. No one born
(d) S between A and the one who likes Lily. Both D and F
(e) V were born before G. G was not born in month of august.
40. Which of the following person goes just before Q? 41. How many persons were born in between G and
(a) W D?
(b) U (a) Five
(c) R (b) Two
(c) Four
Page 447 of 728

Subscribe the Xpress Video Course & Mock Test Package for Bank & Insurance Exams
If there are any suggestions/ errors in our PDFs Feel Free to contact us via this email: admin@exampundit.in
IBPS RRB PO Prelims – Ultra Practice Bundle PDF
(d) Three Directions (46-50): Study the following information
(e) None carefully to answer the given question:
42. D born in which of the following month? Ten friends P, Q, R, S, T, U, V, W, X, Y and Z who had
(a) February celebrated their birthday in the same year were No two
(b) April persons have celebrate their birthday in the same month
(c) May of a year and neither of the friends had birthday in the
(d) January months From Januaryto October.
(e) None of these P was the Last person to celebrate his birthday. R will
43. Who among the following person were born in celebrate his birthday not in the 30 days and maximum
May? number of days. At most 2 persons have their birthday
(a) B after V. V was celebrated his birthday just after W and
(b) G just before X. The birthday of Q is before that of X. The
(c) H birthday of T is after the birthday of U and before the
(d) F birthday of S. The birthday months of T and Y have 30
(e) E days.
44. Who among the following person like Iris Flower? 46. Who among the following Celebrate his birthday
(a) A in May?
(b) D (a) U
(c) C (b) Q
(d) F (c) T
(e) None of these (d) S
45. Which of the following combination is true? (e) R
(a) C-lily 47. Four of the five are alike in a certain way and
(b) B-Iris thusform a group. Find the option which does not
(c) H-Tulip belongto the group.
(d) G-Lotus (a) U
(e) None of these (b) Q
(c) S
Page 448 of 728

Subscribe the Xpress Video Course & Mock Test Package for Bank & Insurance Exams
If there are any suggestions/ errors in our PDFs Feel Free to contact us via this email: admin@exampundit.in
IBPS RRB PO Prelims – Ultra Practice Bundle PDF
(d) W (b) Q
(e) R (c) R
48. How many persons will celebrate theirBirthday (d) S
betweenS and P? (e) Can’t be determined
(a) 1 50. Identify the true statement among the following.
(b) 2 (a) The Birthday of Y is in May.
(c) 3 (b) V had birthday before T.
(d) 4 (c) W has his birthday in October.
(e) 5 (d) X will celebrate his birthday after T.
49. Who was the First person to celebrate his (e) The month of V's birthday has 30 days.
birthday among theten friends?
(a) U

21). Month Based Puzzle - Solution and Detailed Explanation


SOLUTION (1-5):
 The one who has interview in Raipur has in a
month which has less than 30 days.
 K has interview in a month which has less than
31 days.
 Only two people have interview between K and
M.  S has interview immediately after the one who
 M has interview immediately before the one who has interview in Raipur.
has interview in Chennai.  Only three people have interview between S and
the one who has interview in Delhi.
 The person whose interview in Goa in the month
which has less than 31 days.

Page 449 of 728

Subscribe the Xpress Video Course & Mock Test Package for Bank & Insurance Exams
If there are any suggestions/ errors in our PDFs Feel Free to contact us via this email: admin@exampundit.in
IBPS RRB PO Prelims – Ultra Practice Bundle PDF
1.a
2.c
3.d
4.e
5.b
Solution 6-10
1. Rahul has a exam in December
2. Three persons have exam between the months in
 H has interview immediately before L. which Virat and Kapil have.
 I has interview in Pune. 3. Virat has a exam before Kapil but not in May.
 The one who has interview immediately after Months Case 1 Case 2
February month is not in the city Indore. January Virat
March Virat
May
July
August Kapil
October Kapil
December Rahul Rahul

Final Arrangement 4. Neither Mithali nor Sania has a exam on January and
December.
5. Only one person has a exam between the months in
which Sania and Mithali has
Month Case 1 Case 1.1 Case 2
s
Januar Virat Virat
y

Page 450 of 728

Subscribe the Xpress Video Course & Mock Test Package for Bank & Insurance Exams
If there are any suggestions/ errors in our PDFs Feel Free to contact us via this email: admin@exampundit.in
IBPS RRB PO Prelims – Ultra Practice Bundle PDF
March Mitahli / Sania Virat (Case 1, Case 1.1 and Case 1.2 get wrong) and in Case
May Mithali / 2, remaining seat will be occupied by
Sania Saina. Thus, the final arrangement will be as follows:
July Mithali / Sania / Mitahli Months Case 2
Sania January Sachin
August Kapil Kapil Sania / March Virat
Mithali May Mithali / Sania
Octobe Sania / Kapil July Saina
r Mithali August Sania / Mithali
Decem Rahul Rahul Rahul October Kapil
ber December Rahul

6. There are more than two persons who have exams 6. C


between the months in which Rahul and Sachin have 7. D
8. E
9. B
10. D
SOLUTION (11-15):
1. M goes to picnic in the month which has least
number of days.
2. S goes to picnic in the month of July.
3. V was going to picnic immediately after the month
in which W goes and immediately before the month
in which O goes.
4. V goes to picnic neither in the month of September
nor in April.

7. Sachin has exam neither in March nor in May.

Page 451 of 728

Subscribe the Xpress Video Course & Mock Test Package for Bank & Insurance Exams
If there are any suggestions/ errors in our PDFs Feel Free to contact us via this email: admin@exampundit.in
IBPS RRB PO Prelims – Ultra Practice Bundle PDF

5. There is a gap of three months between the one who


goes picnic between X and N.
10. P goes to picnic before the month in which U.
6. There were 31 days in the month in which X goes to
picnic
7. There were 30 days in the month in which N goes to
picnic
8. Q and T were goes to picnic in that months which
had 30 days each.

FINAL ATRRANGEMENT

9. There is a gap of two month between the month in


which N and R goes.

Page 452 of 728

Subscribe the Xpress Video Course & Mock Test Package for Bank & Insurance Exams
If there are any suggestions/ errors in our PDFs Feel Free to contact us via this email: admin@exampundit.in
IBPS RRB PO Prelims – Ultra Practice Bundle PDF
5. I go to Dream world but neither on July nor on
April.
6. K goes to Fun and Food but not on April.
7. I do not go after K.

11.d
12.e (In this Number of days is 30) 8. F neither goes to Wonderland nor Fun and food.
13.c 9. J not goes to Wonderlands. (Means J go to either fun
14.d & food or Dream world)
15.e
SOLUTION (16-20):
1. E goes to Fun and food on March
2. J goes on August
3. The one who goes to Wonderland goes on May.
4. H goes on June

Final Arrangement

Page 453 of 728

Subscribe the Xpress Video Course & Mock Test Package for Bank & Insurance Exams
If there are any suggestions/ errors in our PDFs Feel Free to contact us via this email: admin@exampundit.in
IBPS RRB PO Prelims – Ultra Practice Bundle PDF

4. E and A got married in the month which has 31 and


30 days respectively and in Delhi and Lucknow, but
not necessarily in same order.
16.e 5. T’s wedding in Lucknow,
17.c 6. The wedding in Lucknow and took in the month
18.a which has 30days.
19.c 7. Bride R and T not married in consecutive months.
20.c
SOLUTION (21-25):
1. R’s wedding took place in the month of August but
neither in Goa nor in Hyderabad, however he was
not married A or F.
2. S's marriage was held in the month of March.
3. U was married to B but not in 31 days month.

8. P’s wedding took place in Mumbai

Page 454 of 728

Subscribe the Xpress Video Course & Mock Test Package for Bank & Insurance Exams
If there are any suggestions/ errors in our PDFs Feel Free to contact us via this email: admin@exampundit.in
IBPS RRB PO Prelims – Ultra Practice Bundle PDF
9. The couple whose married are in the month of
March and December not took place in Mumbai.

21.e
22.b
10. The Bride C’s wedding took in the month of which 23.b
has 31days, but in Goa. 24.a
11. Q’s marriage did not take place in neither Chennai 25.d
nor Goa. SOLUTION (26-30):
12. D’s marriage took place in Hyderabad. 1. B was born in the month of which has 31 day but
not in October
2. E was born in the month which has least number of
days and likes Grape.
3. The one who likes Guava was born in the month of
November.

Final Arrangement

Page 455 of 728

Subscribe the Xpress Video Course & Mock Test Package for Bank & Insurance Exams
If there are any suggestions/ errors in our PDFs Feel Free to contact us via this email: admin@exampundit.in
IBPS RRB PO Prelims – Ultra Practice Bundle PDF
4. The one who likes Banana was born in the month
which has 30days but not in April
5. C likes apple and was not born in the month which
have 31 days.

Final Arrangement

6. D was born in the month which has 31days but he is


not youngest.
7. G likes Pineapple.
8. The one who like pineapple fruit born after the one
who like Litchi. 26.a
27.c
28.d
29.e
30.c
SOLUTION (31-35):
1. K is born in one of the months which have 30 days
9. The person who likes Orange does not born after
before September.
pineapple.
2. Three persons are born between L and K.
10. A was not born in the month of November.
3. Only two persons are born between L and G.

Page 456 of 728

Subscribe the Xpress Video Course & Mock Test Package for Bank & Insurance Exams
If there are any suggestions/ errors in our PDFs Feel Free to contact us via this email: admin@exampundit.in
IBPS RRB PO Prelims – Ultra Practice Bundle PDF

4. As many as persons are born after G is two less than


the number of persons born before E. Final Arrangements

31.e
5. As many persons born between G and I is one more
32.b
than the number of persons born between H and G.
33.d
6. J is born in one of the months before H.
34.e
7. F is older than H.
35.e

SOLUTION (36-40):
1. T goes in the month which has odd number of days.

Page 457 of 728

Subscribe the Xpress Video Course & Mock Test Package for Bank & Insurance Exams
If there are any suggestions/ errors in our PDFs Feel Free to contact us via this email: admin@exampundit.in
IBPS RRB PO Prelims – Ultra Practice Bundle PDF
2. Three persons go between T and W.

8. Q does not go in the month which has even number


of day.
3. R goes before T. 9. R does not go for vacation before Q.
4. Not less than to two persons go between R and W.

5. Less than two people go between T and S. Final Arrangements


6. V goes just before S.
7. S does not go in July.

Page 458 of 728

Subscribe the Xpress Video Course & Mock Test Package for Bank & Insurance Exams
If there are any suggestions/ errors in our PDFs Feel Free to contact us via this email: admin@exampundit.in
IBPS RRB PO Prelims – Ultra Practice Bundle PDF
6. The person who likes Tulip born just after the one
36.a who likes Orchid.
37.a
38.a
39.e
40.b
SOLUTION (41-45):
1. Only four persons born between E and the one who
likes Sunflower.
2. E born in month having 30 days and after the person
who likes Sun flower. 7. F born after D but before H but not just before and
3. Number of persons born after the one who likes Iris after.
and before the one who likes Dairy is same. 8. A born in the month having 31 days and does like
4. B born in June and likes Dairy. Rose.
9. Both D and F were born before G.
10. G was not born in month of august
11. No one born between A and the one who likes Lily
12. A born in the month having 31 days and does like
Rose

5. Not less than three people born between C and the


one who likes lotus, both persons born in month
having 30 days.

Page 459 of 728

Subscribe the Xpress Video Course & Mock Test Package for Bank & Insurance Exams
If there are any suggestions/ errors in our PDFs Feel Free to contact us via this email: admin@exampundit.in
IBPS RRB PO Prelims – Ultra Practice Bundle PDF
Final Arrangements

5. The birthday months of T and Y have 30 days.


41.d
6. The birthday of T is after the birthday of U and
42.a
before the birthday of S
43.d
44.d
45.c
SOLUTION (46-50):
1. P was the Last person to celebrate his birthday.
2. R will celebrate his birthday not in the 30 days and
maximum number of days.
3. At most 2 persons have their birthday after V.
4. V was celebrated his birthday just after W and just
before X.

7. The birthday of Q is before that of X.

Page 460 of 728

Subscribe the Xpress Video Course & Mock Test Package for Bank & Insurance Exams
If there are any suggestions/ errors in our PDFs Feel Free to contact us via this email: admin@exampundit.in
IBPS RRB PO Prelims – Ultra Practice Bundle PDF

46.d
47.e
FINAL ARRANGEMENTS
48.d
49.e
50.d

22). Month and Date Based Puzzle

Directions (1-5): Study the following information product immediately after N on different month. G and N
carefully to answer the given question: sell their product on some month. F sells their product in
Eight employees M, N, O, G, F, H, C and D are going to the month of May. H sells his product before O. The
sell their product on four different months of the year number of employees who sells his product between C
viz. May, June, July and November, such that not more and B is one less than the number of employees who sell
than two employees sell their products in each of the their product between N and F. C and B is not last person
months. They sell their product on either 5th or 16th day to sell their product.
of the month. No two employees sell on same day. There 1. Which of the following date and month H sells his
are four persons sell their product between O and H. D product?
sell his a) 5th June

Page 461 of 728

Subscribe the Xpress Video Course & Mock Test Package for Bank & Insurance Exams
If there are any suggestions/ errors in our PDFs Feel Free to contact us via this email: admin@exampundit.in
IBPS RRB PO Prelims – Ultra Practice Bundle PDF
b) 5th May b) B
c) 5th July c) Either a or b
d) 5thNovember d) G
e) 16th June e) N
2. Who among the following sells his product on Directions (6-10): Study the following information
16th November? carefully to answer the given question:
a) G Eight friends – P, Q, R, S, T, U, V and W were born on
b) N in January, April, November and December on either 5th
c) D or 12th (No two people born on same on dates). The ones
d) O who were born in a month having 30 days like different
e) F fast food – Maggie, Pasta, Chowmein and Sandwich not
3. Who among the following sells his product necessarily in the same order. The ones who were born in
immediately after C? a month having 31 days like different city – Indore,
a) F Bhopal, Raipur and Mumbai not necessarily in the same
b) H order. T was born on 12th of the month which has
c) G 30days. Two people were born between T and S. S does
d) Can’t be determined not like any fast food. Q likes Maggie. Number of people
e) None of these was born after P as before S are same. One person was
4. How many employees sells their products between born between the ones who like Pasta and Raipur city.
H and D? Two people were born between T and one who likes
a) Two Mumbai city. No person was born between Q and one
b) One who likes Indore city. P does not like Indore city. The
c) None ones who like Bhopal city and Indore were born on same
d) Three month. Two people were born between P and W. U do
e) More than three not like any City. One person was born between R and
5. Who among the following person sells his product one who likes Maggie. Number of persons was born
first? between the ones who like Sandwich and Raipur city is
a) C two.
Page 462 of 728

Subscribe the Xpress Video Course & Mock Test Package for Bank & Insurance Exams
If there are any suggestions/ errors in our PDFs Feel Free to contact us via this email: admin@exampundit.in
IBPS RRB PO Prelims – Ultra Practice Bundle PDF
6. Who likes Maggie Food? a) Mumbai
a) P b) Indore
b) Q c) Bhopal
c) R d) Raipur
d) S e) Pasta
e) T Directions (11-15): Study the following information
7. How many people were born between P and one carefully to answer the given question:
who likes Sandwich? Ten persons have birthday in five different months’ viz.
a) Four March, May, September, November and December, but
b) Two not necessarily in the same order. Their Birthdays are on
c) Three 2 different dates viz. 17 and 24. Each of the ten people
d) None has Birthday on different dates. There is only three
e) One person’s birthdays after G’s birthday. Four person’s
8. Four of the following forms a group based on birthdays between G and I. D and A has birthday in
certain pattern. Who does not belong to this December. Only two people have birthday between the D
group? and S. The number of birthdays between A and C’s is
a) V same as the number of birthdays between B and T. T’s
b) U birthday in a month having 30 days. There is no birthday
c) W between the birthday of U and L, whose birthday is
d) S exactly between D and B. U’s birthday, is not in same
e) R month as L’s birthday.
9. Who was born on 12th April? 11. In which month S has birthday?
a) U a) May
b) W b) March
c) T c) September
d) S d) November
e) R e) December
10. P likes which of the following food/city?
Page 463 of 728

Subscribe the Xpress Video Course & Mock Test Package for Bank & Insurance Exams
If there are any suggestions/ errors in our PDFs Feel Free to contact us via this email: admin@exampundit.in
IBPS RRB PO Prelims – Ultra Practice Bundle PDF
12. Who among the following has birthday on 17th IBPS Exam of Ten candidates have scheduled in five
May? different months viz. January, April, July, October and
a) B December, but not necessarily in the same order. The
b) I exam will conduct 18th and 28th date of each month and
c) U each candidate have exam on different dates. There is
d) C four candidate’s exams between the exam of Z and S.
e) None of these Only four candidates take exam before the exam of T.
13. Who among the following has birthday on 24th There are same number of exams between S and U’s
December? exam as between V and T’s exam. Exam of both Z and X
a) A falls in the month having 30 days but X’s exam is after
b) L Z’s exam. There is no exam between the exam of K and
c) G L. The exam date of U and L is not same. The exam
d) D month of W and S is same. One of the persons is M.
e) None of these 16. How many candidates take exam between the
14. Which of the following Date I has birthday? exam of L and S?
a) 24th December a) 1
b) 17th December b) 4
c) 17th May c) 5
d) 24th September d) 3
e) 24th March e) 2
15. How many people are there between C and T? 17. Who among the following has exam on 28th
a) 1 December?
b) 2 a) V
c) 3 b) T
d) 4 c) W
e) 5 d) M
Directions (16-20): Study the following information e) U
carefully to answer the given question: 18. V writes his exam on which date?
Page 464 of 728

Subscribe the Xpress Video Course & Mock Test Package for Bank & Insurance Exams
If there are any suggestions/ errors in our PDFs Feel Free to contact us via this email: admin@exampundit.in
IBPS RRB PO Prelims – Ultra Practice Bundle PDF
a) 28 July between Q and U. T attends seminar immediately after
b) 18 July V. V does not attend the seminar in December. W
c) 18 April attends before R. W and Q are adjacent. Two people
d) 28 January between R and S. W is not the first person to attend the
e) Cannot be determined seminar.
19. Which among the following pair is odd one out? 21. How many persons attend seminar between W
a) TU and T?
b) TV a) 5
c) WS b) 4
d) UM c) 3
e) ZX d) No one
20. How many people between K and V? e) None of these
a) 1 22. Who attend the seminar immediately before R?
b) 2 a) S
c) 3 b) W
d) 4 c) Q
e) 5 d) P
Directions (21-25): Study the following information e) None of these
carefully to answer the given question: 23. T attend seminar on which of the following date?
Eight persons – P, Q, R, S, T, U, V and W have attended a) 6th July
seminar on three different dates on 6, 13, 21 of the b) 13th September
months of July, September and December of year 2020. c) 6th December
No two persons attend the seminar on the same date of d) 13th July
the same month. One of the dates of in the given months e) None of these.
is a holiday. S attends seminar on even numbered date of 24. Who attend the seminar after U?
a month which has odd number of days. Holiday is come a) R
in a date of 21st of the month. Only three persons attend b) P
seminar between P and S. Less than five persons c) T
Page 465 of 728

Subscribe the Xpress Video Course & Mock Test Package for Bank & Insurance Exams
If there are any suggestions/ errors in our PDFs Feel Free to contact us via this email: admin@exampundit.in
IBPS RRB PO Prelims – Ultra Practice Bundle PDF
d) No one 27. How many persons go in between F and H?
e) None of these a) Four
25. Who attend the seminar in September? b) Three
a) S c) Two
b) W d) One
c) Q e) None of these
d) P 28. Who among the following organized party on 23th
e) T June?
Directions (26-30): Study the following information a) E
carefully to answer the given question: b) F
E, F, G, H, T, U, V and W have a party in either June or c) G
August of the same year which will be organized only on d) T
the four days viz. 8th, 11th, 18th and 23th of each month. e) None of these
No two people have a party on the same day. E has a 29. Four of the following are alike in a certain way so
party in June on an even numbered date. Only four form a group then which of the following does not
people have a party between E and G. There are three belong to that group?
people have party between G and F. Only two people a) E
have a party between T and F. Both W and V have to b) F
arrange a party on the same date. W has to party on one c) H
of the days after V. No one has to arrange a party d) T
between W and U. H has to arrange a party on 18th June. e) U
26. Who among the following organized party 30. Who among the following organize party 8th
immediately after U? August?
a) F a) E
b) H b) W
c) V c) H
d) W d) T
e) None of these e) None of these
Page 466 of 728

Subscribe the Xpress Video Course & Mock Test Package for Bank & Insurance Exams
If there are any suggestions/ errors in our PDFs Feel Free to contact us via this email: admin@exampundit.in
IBPS RRB PO Prelims – Ultra Practice Bundle PDF
a) J
Directions (31-35): Study the following information b) F
carefully to answer the given question: c) I
Ten people were born in different days of the same year. d) A
The months in which they were born were March, April, e) None of these
May, October and November and the dates were 17 and 34. How many persons go before F?
28 of these months. A was born on 17th October. B was a) Two
the second oldest person. Six people were born between b) One
C and D where C was born after D. There is a gap of one c) Four
person between E and F who born after E. G was born d) Five
immediately after H on the same date as C. I was born e) None of these
after J but after F 35. Which of the following combination is true?
31. Who among the following person goes on 17th a) 17th April-J
March? b) 28th April-D
a) A c) 17th October -A
b) B d) 17th May-I
c) C e) None is true
d) D Directions (36-40): Study the following information
e) E carefully to answer the given question:
32. How many persons go between E and C? Eight students E, F, G, H, I, J, K and L appear in an
a) One examination in two different shifts either at 8AM or
b) Three 12PM. They give exam on different months of the same
c) Four year starting from July to October. Only three persons
d) Five give exam in between L and I, whose exam is in the
e) None of these month which have odd number of days. E gives exam in
33. Who among the following person goes on even the month having 30 days. H gives exam in morning but
number date of the month which has even number after L. Four persons give exam between J and G. L
of days? gives exam after I. K gives an exam in afternoon shift of
Page 467 of 728

Subscribe the Xpress Video Course & Mock Test Package for Bank & Insurance Exams
If there are any suggestions/ errors in our PDFs Feel Free to contact us via this email: admin@exampundit.in
IBPS RRB PO Prelims – Ultra Practice Bundle PDF
the month which has odd number of days but before L. J e) More than 3
gives the exam in the month having 31days but not in 40. Four of the following belongs to a group in a
October. Only one person gives exam between E and F. certain way, find which of the one does not belong to
36). Which of the following month L gives exam? that group?
a) July a) J
b) August b) L
c) September c) H
d) October d) I
e) None of these e) K
37. Who among the following gives exam on August Directions (41-45): Study the following information
at 12pm? carefully to answer the given question:
a) L Fourteen students joined coaching on either 17th or 28th
b) I of seven different months of the same year i.e. March,
c) J May, June, August, September, October and November.
d) K A joined coaching on 28 th June. The number of students
e) F joined between B and C is same as between C and D. D
38. Who among the following gives exam immediately joined after B. Six students joined between A and H.
after J? Three students joined between H and I. F joined
a) F immediately after I. J joined immediately after K in the
b) L different month. Less than two people joined between L
c) E and D, who joined after L. L joined on 28th October.
d) H Four persons joined between L and M. E and N joined
e) G after G. N joined after F.
39. How many person gives exam between K and H? 41. How many persons Joined coaching between A
a) 2 and L?
b) 1 a) 3
c) None b) 2
d) 3 c) 1
Page 468 of 728

Subscribe the Xpress Video Course & Mock Test Package for Bank & Insurance Exams
If there are any suggestions/ errors in our PDFs Feel Free to contact us via this email: admin@exampundit.in
IBPS RRB PO Prelims – Ultra Practice Bundle PDF
d) 0 Directions (46-50): Study the following information
e) None of these carefully to answer the given question:
42. On which day does M joined coaching? Eight persons A, B, C, D, E, F, G, H were born in
a) 17th June different months i.e. March, June, September, and
b) 17th May November on two different dates 17th or 26th. Only One
c) 17th August person was born on one date. They all like different
d) 17th November colors i.e. Red, Yellow, Green, Black, Purple, Pink,
e) 17th March White and Blue but not necessarily in the same order. B
43. Who the last person is to joined coaching? born in the month of September. Only two persons born
a) F between B and the one who likes Blue, who does not
b) N born on an odd number date. Only one person born
c) L between H and the one who like blue. H and G, who
d) D likes White color born in the different month. Three
e) H persons born between E and the one who likes White
44. How many persons joined after I joined coaching? color. D likes Pink color and born immediately below E.
a) Three A does not like Red color and born in the month having
b) One 31 days on Odd date. Three persons born between A and
c) Two the one who likes Yellow color. H does not like Yellow
d) Five color. F does not like blue color born immediately after
e) None of these C, but not in the different month. The one who likes
45. Who among the following joined coaching on 17th green color born after the one who likes Red, who does
March? not born in the November month. C does not like Green
a) G color.
b) B 46. Who among the following has born on 17th of
c) E November?
d) K a) B
e) J b) G
c) C
Page 469 of 728

Subscribe the Xpress Video Course & Mock Test Package for Bank & Insurance Exams
If there are any suggestions/ errors in our PDFs Feel Free to contact us via this email: admin@exampundit.in
IBPS RRB PO Prelims – Ultra Practice Bundle PDF
d) F c) Blue
e) B d) Yellow
47. Who among the following likes Red color? e) Either a or b
a) B 50. Which of the following pair born in the same
b) H month?
c) C a) A and D
d) G b) B and C
e) F c) D and H
48. How many persons born between E and B? d) G and E
a) One e) None of these
b) Three
c) Four
d) Two
e) Five
49. C likes which of the following Color?
a) Purple
b) Black

22). Month and Date Based Puzzle - Solution and Detailed Explanation

SOLUTION (1-5):
 There are four people sell their product between O
and H.
 H sells his product before O.
 F sells their product in the month of May.

Page 470 of 728

Subscribe the Xpress Video Course & Mock Test Package for Bank & Insurance Exams
If there are any suggestions/ errors in our PDFs Feel Free to contact us via this email: admin@exampundit.in
IBPS RRB PO Prelims – Ultra Practice Bundle PDF
 D sells his product immediately after N on different
month.
 G and N sell their product on some month.

1.a
 C and B is not last person to sell their product. 2.d
 The number of employees who sells his product 3.d
between C and B is one less than the number of 4.d
employees who sell their product between N and F. 5.c
SOLUTION (6-10):
1. T was born on 12th of the month which has 30days.
2. Two people were born between T and S.
3. S does not like any fast food. (it means S not born on
the month which has 30 days)
4. Number of people was born after P as before S are
same.
Final Arrangement

5. Q likes Maggie.
Page 471 of 728

Subscribe the Xpress Video Course & Mock Test Package for Bank & Insurance Exams
If there are any suggestions/ errors in our PDFs Feel Free to contact us via this email: admin@exampundit.in
IBPS RRB PO Prelims – Ultra Practice Bundle PDF
6. Two people were born between P and W.
7. U do not like any City (it means U born on even
number of days of month)
8. One person was born between R and one who likes
Maggie

12. Two people were born between T and one who likes
Mumbai city.
13. One person was born between the ones who like
Pasta and Raipur city.
14. Number of persons was born between the ones who
like Sandwich and Raipur city is two.

9. No person was born between Q and one who likes


Indore city.
10. P does not like Indore city.
11. The ones who like Bhopal city and Indore were born
Final arrangements
on same month.

Page 472 of 728

Subscribe the Xpress Video Course & Mock Test Package for Bank & Insurance Exams
If there are any suggestions/ errors in our PDFs Feel Free to contact us via this email: admin@exampundit.in
IBPS RRB PO Prelims – Ultra Practice Bundle PDF

6.b 4. Only two people have birthday between the D and S.


7.b 5. T’s birthday in a month having 30 days
8.e (even date) 6. There is no birthday between the birthday of U
9.c and L,
10.d 7. U’s birthday is not in same month as L’s
SOLUTION (11-15): birthday.
1. There are only three person’s birthdays after G’s
birthday.
2. Four person’s birthdays between G and I.
3. D and A has birthday in December.

8. L birthday is exactly between D and B.

Page 473 of 728

Subscribe the Xpress Video Course & Mock Test Package for Bank & Insurance Exams
If there are any suggestions/ errors in our PDFs Feel Free to contact us via this email: admin@exampundit.in
IBPS RRB PO Prelims – Ultra Practice Bundle PDF
9. The number of birthdays between A and C’s is same
as the number of birthdays between B and T.

5. There are same number of exams between S and U’s


exam as between V and T’s exam
6. There is no exam between the exam of K and L.
7. The exam date of U and L is not same.

11.c
12.d
13.a
14.e
15.d
SOLUTION (16-20):
1. Only four candidates take exam before the exam of
T.
2. Exam of both Z and X falls in the month having 30
days but X’s exam is after Z’s exam.
3. There is four candidate’s exams between the exam
Final Arrangement
of Z and S.
4. The exam month of W and S is same

Page 474 of 728

Subscribe the Xpress Video Course & Mock Test Package for Bank & Insurance Exams
If there are any suggestions/ errors in our PDFs Feel Free to contact us via this email: admin@exampundit.in
IBPS RRB PO Prelims – Ultra Practice Bundle PDF

3. Two people between R and S.


16.c
4. W and Q are adjacent
17.d
5. W is not the first person to attend the seminar.
18.a
19.a
20.d
SOLUTION (21-25):
1. S attends seminar on even numbered date of a month
which has odd number of days
2. Only three persons attend seminar between P and S.

6. Less than five persons between Q and U.


7. T attends seminar immediately after V.
8. V does not attend the seminar in December.

Page 475 of 728

Subscribe the Xpress Video Course & Mock Test Package for Bank & Insurance Exams
If there are any suggestions/ errors in our PDFs Feel Free to contact us via this email: admin@exampundit.in
IBPS RRB PO Prelims – Ultra Practice Bundle PDF
2. E has a party in June on an even numbered date.
3. Only four people have a party between E and G.

9. Holiday is come in a date of 21st of the month.


4. There are three people have party between G and F.
5. Only two people have a party between T and F.

21.b
6. Both W and V have to arrange a party on the same
22.c
date.
23.c
7. W has to party on one of the days after V.
24.d
8. No one have to arrange a party between W and U.
25.d
SOLUTION (26-30):
1. H has to arrange a party on 18th June

Page 476 of 728

Subscribe the Xpress Video Course & Mock Test Package for Bank & Insurance Exams
If there are any suggestions/ errors in our PDFs Feel Free to contact us via this email: admin@exampundit.in
IBPS RRB PO Prelims – Ultra Practice Bundle PDF

Final Arrangement

4. G was born immediately after H on the same date as


C.
5. There is a gap of one person between E and F
who born after E.
6. I was born after J but after F

26.d
27.e
28.e
29.b
30.d
SOLUTION (31-35):
1. A was born on 17th October.
2. B was the second oldest person.
3. Six people were born between C and D where C was
born after D.

Page 477 of 728

Subscribe the Xpress Video Course & Mock Test Package for Bank & Insurance Exams
If there are any suggestions/ errors in our PDFs Feel Free to contact us via this email: admin@exampundit.in
IBPS RRB PO Prelims – Ultra Practice Bundle PDF

4. J gives the exam in the month having 31days but not


in October.
5. Four persons give exam between J and G.
31.d
6. E gives exam in the month having 30 days
32.c
7. Only one person gives exam between E and F
33.a
34.c
35.c
SOLUTION (36-40):
1. Only three persons give exam in between L and I,
whose exam is in the month which have odd number
of days
2. L gives exam after I.
3. H gives exam in morning but after L.

Page 478 of 728

Subscribe the Xpress Video Course & Mock Test Package for Bank & Insurance Exams
If there are any suggestions/ errors in our PDFs Feel Free to contact us via this email: admin@exampundit.in
IBPS RRB PO Prelims – Ultra Practice Bundle PDF
8. K gives an exam in afternoon shift of the month
which has odd number of days but before L.

5. Three students joined between H and I.


6. Four persons joined between L and M.
7. F joined immediately after I.

36.c
37.e
38.a
39.e
40.e
SOLUTION (41-45):
1. A joined coaching on 28th June
2. Six students joined between A and H.
3. Less than two people joined between L and D, who 8. The number of students joined between B and C is
joined after L. same as between C and D.
4. L joined on 28th October 9. D joined after B.
10. J joined immediately after K in the different month.

Page 479 of 728

Subscribe the Xpress Video Course & Mock Test Package for Bank & Insurance Exams
If there are any suggestions/ errors in our PDFs Feel Free to contact us via this email: admin@exampundit.in
IBPS RRB PO Prelims – Ultra Practice Bundle PDF
44.d
45.a
SOLUTION (46-50):
1. B born in the month of September.
2. Only two person born between B and the one who
likes Blue, who does not born on an odd number
date.
3. Only one person born between H and the one who
like blue.

11. N joined after F.


12. E and N joined after G

4. A does not like Red color and born in the month


having 31 days on Odd date.
5. F does not like blue color born immediately after C,
but not in the different month.
6. Three persons born between E and the one who likes
White color.
7. D likes Pink color and born immediately below E
8. H and G, who likes White color born in the different
month.

41.e
42.c
43.d
Page 480 of 728

Subscribe the Xpress Video Course & Mock Test Package for Bank & Insurance Exams
If there are any suggestions/ errors in our PDFs Feel Free to contact us via this email: admin@exampundit.in
IBPS RRB PO Prelims – Ultra Practice Bundle PDF

9. The one who likes green color born after the one
who likes Red, who does not born in the November
month.
10. C does not like Green color. 46.c
As A does not like red color 47.b
48.d
49.e
50.c

Final Arrangement

23). Profession Based Puzzle


Directions [1-5]: Answer the questions based on the DBA, Project Head, Vice President. All the above
information given below. information is not necessarily in the same order.
Seven persons P, Q, R, S, T, U and V works in different R does not work in Xiomi but work as a Project Head.
mobile companies among Apple, Intex, Micromax, The one, who works in Samsung, works as DBA and
Oppo, Oneplus, Samsung and Xiomi in different who works in Apple work as a Analyst. S works in One
positions as Tester, Developer, Analyst, Team Lead, plus work as Developer. P works in Intex but not as
Page 481 of 728

Subscribe the Xpress Video Course & Mock Test Package for Bank & Insurance Exams
If there are any suggestions/ errors in our PDFs Feel Free to contact us via this email: admin@exampundit.in
IBPS RRB PO Prelims – Ultra Practice Bundle PDF
Tester. The one, who works in Xiomi works as Team d. U
lead and who works in Micromax work as Tester. V e. V
doesn’t work in Oppo and Xiomi. Q works as Tester but 5) What is the profession of T?
not in Xiomi. U does not work as Team lead but works in a. Analyst
Samsung. b. Team Lead
1) V works as? c. DBA
a. Analyst d. Project Head
b. Team Lead e. Vice President.
c. DBA Directions [6-10]: Study the following information
d. Project Head and answer the given questions.
e. Vice President.
2) Who works in Apple? Arjun, Brij, Charu, Diljit, Gautam and Fardeen are six
a. R friends having their birthdays on same date of different
b. S month viz. January, February, March, April, July and
c. T November of same year i.e. year 1998. They are engaged
d. U in different professions/services viz. Engineer, Advocate,
e. V Guard, Consultant, IPS and Director. All the above
3) Who among the following working as the Vice information is not necessarily in the same order.
President? Brij is born in the month having 30 days, but not the
a. P youngest. Diljit is older than Brij. Gautam is a Director
b. S and born in February. Fardeen is neither a Engineer nor a
c. T IPS. The youngest person is a IPS. The difference of age
d. U between an Director and Arjun is exactly the same as
e. V between Diljit and a Guard. The one who is older to
4) Who is the Tester of Micromax? Fardeen but not the oldest is a Consultant. Arjun is six
a. P months older to the one who is Advocate.
b. Q 6) Whose birthday is exactly between Gautam and
c. T Brij?
Page 482 of 728

Subscribe the Xpress Video Course & Mock Test Package for Bank & Insurance Exams
If there are any suggestions/ errors in our PDFs Feel Free to contact us via this email: admin@exampundit.in
IBPS RRB PO Prelims – Ultra Practice Bundle PDF
a. Fardeen b. Advocate
b. Charu c. Guard
c. Diljit d. Cannot be determined
d. Arjun e. None of these
e. None of these Directions [11-15]: Study the following information
7) Persons in which of the following pairs are born in and answer the given questions.
the month having 31 days? Seven persons A , B, C, E, F, G and H are working in a
a. Arjun – Diljit company with different grade position such as CEO,
b. Charu – Fardeen MD, GM, DGM, Senior Manager, Assistant Manager
c. Brij – Diljit and Clerk. Their positions are given in descending order,
d. Charu – Gautam such as CEO as the highest position and Clerk as the
e. None of these lowest position. Consecutive alphabetical name of the
8) Four of the following five are alike in a certain way persons doesn’t hold a consecutive grade position.
and thus form a group. Which one of the following Three persons are hold a position between H and E.
does not belong to that group? Number of persons hold a position higher than G is twice
a. Diljit – 31 – Guard as that of number of people hold a position higher than
b. Brij – 31 – Consultant A. As many persons ranking higher than E is same as
c. Charu – 30 – Advocate that of lower than G. C‟s grade position is immediately
d. Gautam – 31 – Consultant higher than F‟s grade position. The grade position of B is
e. Brij – 30 – Guard higher than G.
9) Which one of the following is an Engineer? 11) Which is the correct combination?
a. Diljit a. A – CEO
b. Brij b. B – Clerk
c. Charu c. C– MD
d. Arjun d. G- GM
e. None of these e. None of above
10) Find the profession of Diljit? 12) Rank of H from Bottom?
a. Consultant a. first
Page 483 of 728

Subscribe the Xpress Video Course & Mock Test Package for Bank & Insurance Exams
If there are any suggestions/ errors in our PDFs Feel Free to contact us via this email: admin@exampundit.in
IBPS RRB PO Prelims – Ultra Practice Bundle PDF
b. third descending order, such as DGP as the highest position
c. second and Constable as the lowest position.
d. fifth Brahmdutt is a SI. Two persons are hold a position
e. None of the above between Ayan Ranjan and Brahmdutt. More than four
13) Which of the following cannot be true? person are hold a position between Chitragupt and
a. F rank third from top Heeralal. As many person senior to Chitragupt is same as
b. B rank sixth from top that of junior to Dharambir who is junior to Heeralal.
c. C rank second from top Eshwar Singh is junior to Farooq Ali and senior to
d. G rank fourth from bottom Giriraj. Dharambir is not a SSP.
e. None of the above 16) What is the profession of Ayan Ranjan?
14) Who is Senior Manager? a. SSP
a. H b. SP
b. A c. DSP
c. G d. SI
d. B e. Cannot be determined.
e. None of above 17) What is the rank of Dharambir from top?
15) Who ranked as MD? a. Fifth
a. H b. Sixth
b. A c. Seventh
c. G d. Eight
d. B e. Cannot be determined.
e. None of above 18) How many positions are there between Giriraj
Direction [16-20]: Study the following information and Heeralal?
carefully and answer the questions given below. a. One
Eight persons are working in a Police Department with b. Two
different grade position such as DGP, SSP, SP, DSP, c. Three
SHO, SI, ASI and Constable. Their positions are given in d. Four
e. Cannot be determined.
Page 484 of 728

Subscribe the Xpress Video Course & Mock Test Package for Bank & Insurance Exams
If there are any suggestions/ errors in our PDFs Feel Free to contact us via this email: admin@exampundit.in
IBPS RRB PO Prelims – Ultra Practice Bundle PDF
19) Who is ranked as DGP? liking Swimming does not live on the ground floor or
a. Ayan Ranjan top-most floor.
b. Brahamdutt 21) Who are the three Swimmer?
c. Dharambir a. A, B, C
d. Chitragupt b. D, E, F
e. Farooq Ali c. C, D, E
20) Who is Constable in the Police Department? d. E, F, G
a. Ayan Ranjan e. C, A, D
b. Brahamdutt 22) Who occupy the second floor?
c. Dharambir a. B, C
d. Eshwar Singh b. C, D
e. Farooq Ali c. B, D
Directions [21-25]: Refer to the data below and d. D, E
answer the questions that follow. e. A, C
A, B, C, D, E, F, and G are seven friends who live in the 23) What is the profession of G?
hostel. Some of them occupy different floors of the a. Swimmer
hostel building but not more than two people are on the b. Doctor
same floor. c. Engineer
There are total of four floors (Ground, first, second and d. Either 1) or 2)
third) in the hostel building. They are Engineer, e. None of these
Swimmer or Doctor. 24) Which two persons are Doctor?
Among the friends three are swimmer, two are Engineer a. G, E
and two are Doctor. Neither G nor B is a Swimmer. D b. B, C
and E are best friends and thus stay in the same room on c. C, G
the first floor and in same profession. B shares the room d. B, G
with the person who is a Swimmer. F is neither a e. F, G
Swimmer nor a Doctor. A is a Engineer and does not 25) What is the profession of C?
share his room with anyone on the ground floor. Persons a. Swimmer
Page 485 of 728

Subscribe the Xpress Video Course & Mock Test Package for Bank & Insurance Exams
If there are any suggestions/ errors in our PDFs Feel Free to contact us via this email: admin@exampundit.in
IBPS RRB PO Prelims – Ultra Practice Bundle PDF
b. Doctor b. Professor
c. Engineer c. Engineer
d. Either 1) or 2) d. None of these
e. None of these e. Cannot be determined
Directions [26-30]: Refer to the data below and 28) Who is the professor?
answer the questions that follow. a. B
Seven professionals A, B, C, D, E, F and G are b. C
practicing their professions in different cities Chennai, c. D
Bangalore, Hyderabad, Mumbai, Ahmedabad, Jaipur and d. E
Bhubaneswar not necessarily in the same order. Each has e. None of these
a different profession Doctor, Engineer, Pharmacist, 29) Which of the following combinations of
Lawyer, Counselor, Professor and Artist not necessarily profession and place is correct?
in the same order. a. Pharmacist—Jaipur
A is a Pharmacist and practices in Bhubaneswar. D b. Engineer—Chennai
practices in Bangalore but is not a Doctor or an Artist. c. Doctor—Bangalore
The one who practices in Hyderabad is a Professor. G is d. Artist—Mumbai
a Counselor and does not practice in Mumbai or e. None of these
Chennai. E is a Lawyer and practices in Ahmedabad. F 30) Which of the following persons work in Jaipur?
practices in Chennai but is not an artist. C practices in a. B
Mumbai. b. G
26) Who is the Doctor? c. C
a. D d. B or G
b. B e. None of these
c. C Directions [31-35]: Study the following information
d. B or C carefully and answer the questions that follow:
e. None of these Six people: Anirudh, Pawan, Aanchal, Amis, Shankar
27) What is D’s profession? and Amit live on six different floors of a building,
a. Doctor starting from floor one to floor six. Their professions are
Page 486 of 728

Subscribe the Xpress Video Course & Mock Test Package for Bank & Insurance Exams
If there are any suggestions/ errors in our PDFs Feel Free to contact us via this email: admin@exampundit.in
IBPS RRB PO Prelims – Ultra Practice Bundle PDF
different which are Painter, Youtuber, Editor, Writer, d. Aanchal
Director, and Consultant but not necessarily in the same e. Shankar
order. 34) Who lives in topmost floor?
Anirudh lives three floors above the Director. There are a. Shankar
three floors between Amis and Pawan, who doesn’t live b. Amis
in fifth floor. Aanchal is not a Director. Amit neither c. Pawan
lives on extreme floors nor he lives on any odd d. Aanchal
numbered floor. Youtuber lives two floors above the e. None of these
Painter and both of them lives on odd numbered floor. 35) Who among the following is the Editor?
Shankar does not live on any odd numbered floor. Writer a. Amit
lives four floors below the Editor on even numbered b. Amis
floor. c. Pawan
31) Pawan is a _________? d. Aanchal
a. Editor e. Shankar
b. Painter Direction [36-40] : Study the below details and
c. Writer answer the following questions.
d. Director There are 8 members in a family – P, Q, R, S, T, U, V
e. Consultant and W. Each has a different profession – Doctor,
32) Who lives in the 5th floor? Teacher, Student, Homemaker, Journalist, RJ, Engineer
a. Editor and Manager, however not necessarily in the same order.
b. Aanchal Each likes different sports – Kabaddi, Hockey, Golf,
c. Amis Athletics, Cricket, Badminton, Football and Tennis, not
d. Painter necessarily in the same order. No two persons like the
e. None of these same sport and no two persons work in the same
33) Who is a Writer? profession.
a. Amit P is a Doctor and likes Football. T is a Manager and Q
b. Amis likes Tennis. R likes Kabaddi and W is a Journalist. S
c. Pawan does not like either Badminton or Hockey. The person
Page 487 of 728

Subscribe the Xpress Video Course & Mock Test Package for Bank & Insurance Exams
If there are any suggestions/ errors in our PDFs Feel Free to contact us via this email: admin@exampundit.in
IBPS RRB PO Prelims – Ultra Practice Bundle PDF
who likes Tennis is not an Engineer. The person who b. Engineer
likes Kabaddi is RJ. The person who is a Manager does c. Homemaker
not like Badminton or Hockey or Athletics. V is a d. Doctor
Teacher and likes Hockey. The Journalist does not like e. Teacher
either Golf, Athletics or Cricket. U is an Engineer and 40) Who among the following likes Golf?
likes Cricket. The person who likes Athletics is a a. Q
Homemaker. b. P
36) What is the profession of the person who likes c. T
Golf? d. S
a. Engineer e. R
b. Doctor Direction [41-45]: Study the below details and answer
c. teacher the following questions.
d. Manager There are 7 persons P, Q, R, S, T, U and V from different
e. RJ states – Nagaland, Sikkim, Meghalaya, Kerala, Goa,
37) Who among the following is a Homemaker? Gujarat and Bihar, but not necessarily in the same order.
a. S Each of them has a different profession – Manager,
b. V Artist, Hacker, Influencer, Principal, Politician and
c. U Cricketer, but not necessarily in the same order.
d. W R is from Goa and is a Artist. P is not from Bihar. The
e. R person from Bihar is an Influencer. The Hacker is from
38) Who among the following likes Badminton? Kerala. U is a Politician and from Gujarat. The one from
a. Teacher Nagaland is a Principal. V is from Sikkim. Q is an
b. Engineer Cricketer. S is not an Influencer. P is not a Hacker.
c. Student 41) Which state does P belong to?
d. Doctor a. Delhi
e. Journalist b. Bihar
39) What is the profession of Q? c. Nagaland
a. Student d. Can’t say
Page 488 of 728

Subscribe the Xpress Video Course & Mock Test Package for Bank & Insurance Exams
If there are any suggestions/ errors in our PDFs Feel Free to contact us via this email: admin@exampundit.in
IBPS RRB PO Prelims – Ultra Practice Bundle PDF
e. None of these Seven people namely Nitin, Manoj, Ojaswi, Parul, Qadir,
42) What is the profession of V? Rahul and Satbir like seven different genres of music
a. Influencer namely Blues, Folk, Country, Soul, Dance, Rap and
b. Manager Hardcore but not necessarily in the same order. Each
c. Principal person also works in some office but at a different
d. Can’t say position on the basis of seniority namely Trainee (TE),
e. Cricketer Assistant Manager (AM), Manager (MG), Senior
43) What is the profession of R? manager (SM), Chief Manager (CM), Executive Director
a. Influencer (ED) and director (DR) but not necessary inthe same
b. Manager order. (Please Note: The Position have been given
c. Principal increasing order of seniority with TE being the joiner
d. Artist most position whilst DR being the senior most position)
e. Cricketer Only one person is senior than Ojaswi. The one who
44) What is profession of S? senior than Ojaswi likes Folk.Only two people are junior
a. Influencer than Rahul.Manoj is junior than Rahul and like
b. Manager Hardcore. Manoj is the not junior most. Qadir is the
c. Principal junior than the CM but senior than the one who like
d. None of these Soul. The one who like Rap is not the ED. Satbir like
e. Cricketer Blues. The one who likes Country is senior than Qadir
45) Which state does T belong to? but not the ED. Parul does not like Country.
a. Delhi 46) Which of following music genres does Rahul like?
b. Bihar a. Hardcore
c. Kerala b. Country
d. Can’t say c. Rap
e. None of these d. Folk
Direction [46-50]: Study the following information e. Soul
carefully and answer the question given below 47) Which combination represent the position at
which Nitin works and the music genres that he likes?
Page 489 of 728

Subscribe the Xpress Video Course & Mock Test Package for Bank & Insurance Exams
If there are any suggestions/ errors in our PDFs Feel Free to contact us via this email: admin@exampundit.in
IBPS RRB PO Prelims – Ultra Practice Bundle PDF
a. CM – Rap a. Manoj, Satbir
b. CM – Country b. Rahul, Ojaswi
c. CM – Blues c. Nitin, Parul
d. MG – Dance d. Manoj, Ojaswi
e. MG – Soul e. Nitin, Ojaswi
48) Who amongst the following works as TE? 50) Who amongst the following works as a MG?
a. Parul a. Rahul
b. Nitin b. Qadir
c. Rahul c. Satbir
d. Satbir d. Parul
e. Ojaswi e. Other than those given answer.
49) Which of the following pairs represents the people
who have less experience than Parul and more
experience than Qadir?

23). Profession Based Puzzle - Solutions


1. a Person Company Profession
2. e P
3. a Q Xiomi Tester
4. b R Xiomi Project Head
5. b S
Solutions 1-5 T
1. R does not work in Xiomi but work as a Project Head. U Samsung Team lead
2. Q works as Tester but not in Xiomi. V
3. U does not work as Team lead but works in Samsung.

4. S works in One plus work as Developer.

Page 490 of 728

Subscribe the Xpress Video Course & Mock Test Package for Bank & Insurance Exams
If there are any suggestions/ errors in our PDFs Feel Free to contact us via this email: admin@exampundit.in
IBPS RRB PO Prelims – Ultra Practice Bundle PDF
5. P works in Intex but not as Tester. 9. d
6. The one, who works in Xiomi works as Team lead and 10. a
who works in Micromax work as Tester. Solutions 6-10
7. V doesn’t work in Oppo and Xiomi. 1. Gautam is an Director and born in February.
Hence, T works in Xiomi
Person Company Profession 2. The youngest person is a IPS.
P Intex
Q Micromax Tester
R Project Head Months Days Persons Profession
S One Plus Developer January 31
T Xiomi Team lead February 28 Gautam Director
U Samsung March 31
V Oppo, April 30
8. The one, who works in Samsung, works as DBA and July 31
who works in Apple work as a Analyst. November 30 CA
Person Company Profession
P Intex Vice President
Q Micromax Tester 3. The youngest person is a IPS.
R Oppo Project Head
S One Plus Developer Months Days Persons Profession
T Xiomi Team lead January 31 Arjun
U Samsung DBA February 28 Gautam Director
V Apple Analyst March 31
Answers April 30
6. c July 31 Advocate
7. a November 30 IPS
8. e

Page 491 of 728

Subscribe the Xpress Video Course & Mock Test Package for Bank & Insurance Exams
If there are any suggestions/ errors in our PDFs Feel Free to contact us via this email: admin@exampundit.in
IBPS RRB PO Prelims – Ultra Practice Bundle PDF
July 31 Fardeen Advocate
4. The difference of age between a Director and Arjun is November 30 Charu IPS
exactly the same as between Diljit and a Guard.
8. Only left Charu must be the IPS.
5. Brij is born in the month having 30 days, but not the
youngest. Diljit is older than Brij. Months Days Persons Profession
January 31 Arjun Engineer
Months Days Persons Profession February 28 Gautam Director
January 31 Arjun March 31 Diljit Consultant
February 28 Gautam Director April 30 Brij Guard
March 31 Diljit July 31 Fardeen Advocate
April 30 Brij Guard November 30 Charu IPS
July 31 Advocate
November 30 IPS Answers
11. c
6. Fardeen is neither a Engineer nor a IPS. 12. b
13. d
So, Fardeen is neither a Engineer nor a IPS, so he must 14. a
be a Advocate. 15. e
Solutions 11-15
7. Only left Charu must be the IPS. 1. As many persons ranking higher than E is same
as that of lower than G.
Months Days Persons Profession 2. Three persons are hold a position between H and
January 31 Arjun E.
February 28 Gautam Director 3. Number of persons hold a position higher than G
March 31 Diljit is twice as that of number of persons hold a
April 30 Brij Guard position higher than A.
Case-1
Page 492 of 728

Subscribe the Xpress Video Course & Mock Test Package for Bank & Insurance Exams
If there are any suggestions/ errors in our PDFs Feel Free to contact us via this email: admin@exampundit.in
IBPS RRB PO Prelims – Ultra Practice Bundle PDF
Position Person Senior Manager H
CEO E Assistant Manager B
MD Clerk G
GM
DGM A Case-2
Senior Manager H Position Person
Assistant Manager CEO H
Clerk G MD A
GM G
Case-2 DGM B
Position Person Senior Manager E
CEO H Assistant Manager C
MD A Clerk F
GM G
DGM 5. The grade position of B is higher than G.
Senior Manager E Hence, Case-2 is eliminated.
Assistant Manager
Clerk Position Person
CEO E
4. C‟s grade position is immediately higher than F‟s MD C
grade position. GM F
Case-1 DGM A
Position Person Senior Manager H
CEO E Assistant Manager B
MD C Clerk G
GM F
DGM A Answers

Page 493 of 728

Subscribe the Xpress Video Course & Mock Test Package for Bank & Insurance Exams
If there are any suggestions/ errors in our PDFs Feel Free to contact us via this email: admin@exampundit.in
IBPS RRB PO Prelims – Ultra Practice Bundle PDF
16. b DSP
17. d SHO
18. a SI Brahmdutt
19. d ASI Heeralal
20. c Constable Dharambir
Solutions 16-20
1. Brahmdutt is a SI. 5. Eshwar Singh is junior to Farooq Ali and senior to
2. Two persons are hold a position between Ayan Ranjan Giriraj.
and Brahmdutt. 6. Dharambir is not a SSP.
Position Person Position Person
DGP DGP Chitragupt
SSP SSP Farooq Ali
SP Ayan Ranjan SP Ayan Ranjan
DSP DSP Eshwar Singh
SHO SHO Giriraj
SI Brahmdutt SI Brahmdutt
ASI ASI Heeralal
Constable Constable Dharambir
Answers
3. More than four persons are hold a position between 21. c
Chitragupt and Heeralal. 22. a
4. As many persons senior to Chitragupt is same as that 23. b
of junior to Dharambir who is junior to Heeralal. 24. d
Position Person 25. a
DGP Chitragupt Solution: 21-25
SSP 1) A is a Engineer and does not share his room with
SP Ayan Ranjan anyone on the ground floor.

Page 494 of 728

Subscribe the Xpress Video Course & Mock Test Package for Bank & Insurance Exams
If there are any suggestions/ errors in our PDFs Feel Free to contact us via this email: admin@exampundit.in
IBPS RRB PO Prelims – Ultra Practice Bundle PDF
Name Profession Floor E First
A Engineer Ground F Engineer
B G
C
D 4) Neither G nor B is a Swimmer.
E Name Profession Floor
F A Engineer Ground
G B Doctor
C
2) F is neither a Swimmer nor a Doctor which clearly D First
indicates he is a Engineer. E First
Name Hobby Floor F Engineer
A Engineer Ground G Doctor
B
C 5) B is sharing a room with a Swimmer.
D Name Profession Floor
E A Engineer Ground
F Engineer B Doctor
G C Swimmer
D First
3) D and E are sharing same room on first floor as well E First
as the same profession F Engineer
Name Profession Floor G Doctor
A Engineer Ground
B 6) Persons liking Swimming does not live on the ground
C floor or top-most floor. (Thus, C does not live on the
D First ground floor that implies B also doesn't live there too.

Page 495 of 728

Subscribe the Xpress Video Course & Mock Test Package for Bank & Insurance Exams
If there are any suggestions/ errors in our PDFs Feel Free to contact us via this email: admin@exampundit.in
IBPS RRB PO Prelims – Ultra Practice Bundle PDF
This also means that they occupy the second floor as the 28. a
first floor is being occupied by D and E.) 29. d
Name Profession Floor 30. b
A Engineer Ground Solution: 26-30
B Doctor Second 1. A is a Pharmacist and practices in Bhubaneswar. D
C Swimmer Second practices in Bangalore but is not a Doctor or an Artist.
D First Person Profession City Condition
E First A Pharmacist Bhubaneswar
F Engineer B
G Doctor C
D Bangalore Doctor ×
(F and G live on the third floor as the ground floor, first Artist ×
floor and second floor have been occupied and not more E
than two people can be on the same floor.) F
Name Profession Floor G
A Engineer Ground 2. G is a Counselor and does not practice in Mumbai or
B Doctor Second Chennai. E is a Lawyer and practices in Ahmedabad.
C Swimmer Second Person Profession City Condition
D Swimmer First A Pharmacist Bhubaneswar
E Swimmer First B
F Engineer Third C
G Doctor Third D Bangalore Doctor ×
Artist ×
E Lawyer Ahmedabad
Answers F
26. e G Counselor Mumbai ×
27. c Chennai ×

Page 496 of 728

Subscribe the Xpress Video Course & Mock Test Package for Bank & Insurance Exams
If there are any suggestions/ errors in our PDFs Feel Free to contact us via this email: admin@exampundit.in
IBPS RRB PO Prelims – Ultra Practice Bundle PDF
3. F practices in Chennai but is not an artist. C practices Answers
in Mumbai. 31. d
The one who practices in Hyderabad is a Professor. ⇒ B 32. c
is practicing in Hyderabad ∵ G is a Counselor ⇒ G 33. a
practices in Jaipur 34. a
Person Profession City Condition 35. e
A Pharmacist Bhubaneswar Solution 31-35
B Professor Hyderabad 1) Writer lives four floors below the Editor on even
C Mumbai numbered floor.

D Bangalore Doctor × (Writer must live in 2 nd floor because he lives on even

Artist × numbered floor.)

E Lawyer Ahmedabad 2) Youtuber lives two floors above the Painter and both

F Chennai Artist × of them lives on odd numbered floor.

G Counselor Jaipur Mumbai × 3) Anirudh lives three floors above the Director.

Chennai × (Therefore, Youtuber lives on 5th floor and Painter lives

4. Professions left to be allotted = Doctor, Engineer and on 3rd floor. The only floor left for Director is 1 st floor.

Artist. Therefore, Director lives on 1st floor)

D isn’t doctor & artist ⇒ D is an Engineer. F is a Doctor Floor Person Profession

because he can’t be an artist ⇒ C is an artist. 6th Editor

Person Profession City 5th Youtuber

A Pharmacist Bhubaneswar 4th Anirudh

B Professor Hyderabad 3rd Painter

C Artist Mumbai 2nd Writer

D Engineer Bangalore 1st Director

E Lawyer Ahmedabad
F Doctor Chennai 5) Amit neither lives on extreme floors nor he lives on
any odd numbered floor.
G Counselor Jaipur
(Only floor left for Amit is 2nd floor.)
Page 497 of 728

Subscribe the Xpress Video Course & Mock Test Package for Bank & Insurance Exams
If there are any suggestions/ errors in our PDFs Feel Free to contact us via this email: admin@exampundit.in
IBPS RRB PO Prelims – Ultra Practice Bundle PDF
(The only occupation left is Consultant. Therefore, Solution 36-40
Anirudh is the Consultant.) 1) P is a Doctor and likes Football.
Floor Person Profession 2) T is a Manager and Q likes Tennis.
6th Editor 3) R likes Kabaddi and W is a Journalist.
5th Youtuber Person Sport Profession
4th Anirudh Consultant P Football Doctor
3rd Painter Q Tennis
2nd Amit Writer R Kabaddi
1st Director S
T Manager
6) Shankar does not live on any odd numbered floor U
7) There are three floors between Amis and Pawan, who V
doesn’t live in fifth floor. Aanchal is not a Director. W Journalist

Floor Person Profession 4) S does not like either Badminton or Hockey.


6th Shankar Editor 5) The person who likes Tennis is not an Engineer.
5th Amis Youtuber 6) The person who likes Kabaddi is RJ.
4th Anirudh Consultant Person Sport Profession
3rd Aanchal Painter P Football Doctor
2nd Amit Writer Q Tennis (Not
1st Pawan Director Engineer)
R Kabaddi RJ
Answers S (Not
36. d Badminton)
37. a (Not
38. e Hockey)
39. a T Manager
40. c
Page 498 of 728

Subscribe the Xpress Video Course & Mock Test Package for Bank & Insurance Exams
If there are any suggestions/ errors in our PDFs Feel Free to contact us via this email: admin@exampundit.in
IBPS RRB PO Prelims – Ultra Practice Bundle PDF
U Engineer)
V R Kabaddi RJ
W Journalist S (Not
Badminton)
7) The person who is a Manager does not like Badminton T (Not Manager
or Hockey or Athletics. Badminton,
Person Sport Profession Athletics)
P Football Doctor U
Q Tennis (Not V Hockey Teacher
Engineer) W Journalist
R Kabaddi RJ 9) The Journalist does not like either Golf, Athletics or
S (Not Cricket.
Badminton) (Then only Badminton is left, thus the Journalist i.e. W
(Not likes Badminton)
Hockey) Person Sport Profession
T (Not Manager P Football Doctor
Badminton, Q Tennis (Not
Hockey, Engineer)
Athletics) R Kabaddi RJ
U S
V T (Not Manager
W Journalist Athletics)
U
8) V is a Teacher and likes Hockey. V Hockey Teacher
Person Sport Profession W Badminton Journalist
P Football Doctor
Q Tennis (Not 10) U is an Engineer and likes Cricket.

Page 499 of 728

Subscribe the Xpress Video Course & Mock Test Package for Bank & Insurance Exams
If there are any suggestions/ errors in our PDFs Feel Free to contact us via this email: admin@exampundit.in
IBPS RRB PO Prelims – Ultra Practice Bundle PDF
(Also, since T does not like Athletics, therefore S likes 44. d
Athletics and T likes Golf) 45. b
Person Sport Profession Solution 41-45
P Football Doctor 1. R is from Goa and is a Artist.
Q Tennis 2. U is a Politician and from Gujarat.
R Kabaddi RJ 3. Q is an Cricketer
S Athletics Person State Profession
T Golf Manager P
U Cricket Engineer Q Cricketer
V Hockey Teacher R Goa Artist
W Badminton Journalist S
T
11) The person who likes Athletics is a Homemaker. U Gujarat Politician
(Also, only Studentsis left. Therefore, Q is a student) V
Person Sport Profession 4. V is from Sikkim.
P Football Doctor 5. P is not from Bihar.
Q Tennis Student 6. The person from Bihar is an Influencer.
R Kabaddi RJ Person State Profession
S Athletics Homemaker P
T Golf Manager Q Cricketer
U Cricket Engineer R Goa Artist
V Hockey Teacher S Bihar/ Influencer/
W Badminton Journalist T Bihar/ Influencer/
Answers U Gujarat Politician
41. c V Sikkim
42. b 7. The Hacker is from Kerala.
43. d 8. The one from Nagaland is a Principal.

Page 500 of 728

Subscribe the Xpress Video Course & Mock Test Package for Bank & Insurance Exams
If there are any suggestions/ errors in our PDFs Feel Free to contact us via this email: admin@exampundit.in
IBPS RRB PO Prelims – Ultra Practice Bundle PDF
9. S is not an Influencer. Manoj Hardcore AM
10. P is not a Hacker. Ojaswi ED
Person State Profession Parul
P Nagaland Principal Qadir
Q Meghalaya Cricketer Rahul MG
R Goa Artist Satbir Blues
S Kerala Hacker
T Bihar Influencer 6) Qadir is the junior than the CM but senior than the one
U Gujarat Politician who like Soul.
V Sikkim Manager (Hence, Qadir is Senior manager (SM) as he is senior
Answers than one who like Soul here Soul like by Rahul as he is
46. e Manager)
47. b 7) Parul does not like Country.
48. d Name Music Genres Position
49. e
50. a Nitin
Solutions 46-50 Manoj Hardcore AM
1) Only one person is senior than Ojaswi.
Ojaswi ED
(Therefore, Ojaswi is Executive Director (ED))
Parul ×Country
2) Only two people are junior than Rahul.
Qadir SM
(Thus, Rahul is a Manager (MG))
3) Manoj is junior than Rahul and like Hardcore. Rahul Soul MG

4) Manoj is the not junior most. Satbir Blues


(Thus, Manoj is Assistant Manager (AM).)
5) Satbir like Blues 8) The one who senior than Ojaswi likes Folk.
Name Music Genres Position (Thus DR → Folk, as Ojaswi is ED and senior then ED
is DR)
Nitin
Page 501 of 728

Subscribe the Xpress Video Course & Mock Test Package for Bank & Insurance Exams
If there are any suggestions/ errors in our PDFs Feel Free to contact us via this email: admin@exampundit.in
IBPS RRB PO Prelims – Ultra Practice Bundle PDF
9) The one who likes Country is senior than Qadir but Manoj Hardcore AM
not the ED. Ojaswi ED
(Hence CM like Country as ED → does not like Country Parul Folk DR
and, DR → Folk) Qadir SM
(Thus, Nitin is CM and Like Country) Rahul Soul MG
Name Music Genres Position Satbir Blues TE

Nitin Country CM 10) The one who like Rap is not the ED.
Manoj Hardcore AM (Therefore, Qadir like Rap and Ojaswi like Dance)
Ojaswi ED Name Music Genres Position

Parul
Nitin Country CM
Qadir SM
Manoj Hardcore AM
Rahul Soul MG
Ojaswi Dance ED
Satbir Blues
Parul Folk DR
Qadir Rap SM
→ Now leftover combination is DR → Folk and Parul is
Rahul Soul MG
leftover person
Satbir Blues TE
→ Satbir will be TE as only that position left for Satbir.
Name Music Genres Position

Nitin Country CM

24). Syllogism Questions

Directions (1-50): Read both the conclusions and then 1. Statements:


decide which of the given conclusions logically follows All P are W
from the given statements disregarding commonly Some W are G
known facts: No G is H

Page 502 of 728

Subscribe the Xpress Video Course & Mock Test Package for Bank & Insurance Exams
If there are any suggestions/ errors in our PDFs Feel Free to contact us via this email: admin@exampundit.in
IBPS RRB PO Prelims – Ultra Practice Bundle PDF
Conclusions: I. All ribbon being socks is a possibility.
I) All P are H is a possibility II. All ribbon can never be lace.
II) Some W are Not H (a) Only conclusion I follows
III) All P are G is a possibility (b) Only conclusion II follows
(a) Only I follow (c) Both I and II follows
(b) Only II and III follows (d) Neither I nor II follows
(c) Only III follows (e) Either I or II follows
(d) All follow 4. Statements:
(e) None of these Few hills are mountain
2. Statements: Each rivers are mountains
Some meat are Chicken Only a few Dam are rivers
All chicken are fresh Conclusions:
No chicken are fish I. All Dam are mountain is a possibility
Some fish is whale II. Some hills are not rivers
Conclusions: III. All Dam are Hills is possibility.
I. Some fresh are not fish (a) Only conclusion I follows
II. All meat being Whale is a possibility (b) Only conclusion II and III follows
III. No meat is Fish (c) Only I and III follows
(a) Only conclusion I follows (d) None follows
(b) Only conclusion I and II follows (e) All Follows
(c) Only I and III follows 5. Statements:
(d) None follows All lyrics are songs.
(e) All Follows Some songs are not rhythm.
3. Statements: Some rhythm is poem.
Only a few Chain are ribbon Conclusions:
Only a few ribbon are lace I. Some poem is not Songs.
No lace are socks II. At least some lyrics being rhythm is a possibility.
Conclusions: (a) Both I and II follow
Page 503 of 728

Subscribe the Xpress Video Course & Mock Test Package for Bank & Insurance Exams
If there are any suggestions/ errors in our PDFs Feel Free to contact us via this email: admin@exampundit.in
IBPS RRB PO Prelims – Ultra Practice Bundle PDF
(b) Neither I nor II follows (d) All of I, II and III follow
(c) Either I or II follows (e) None of the above
(d) Only I follows 8. Statements:
(e) Only II follows All light are Flame
6. Statements: All flame is air
Minimum watch is quartz No flame is burns
No quartz is clock Conclusions:
A few clocks are second. I. Some burn is not air
Conclusions: II. Some air is not light.
I. Some seconds are not quartz. III. Some air is burn.
II. No clock is watches. (a) All the conclusions follow
III. All watch being second is a possibility. (b) Either I or III follows
(a) Only I and II follow (c) Only II and III follow
(b) Only I and III follow (d) None follows
(c) Only II and III follow (e) Only I and II follow
(d) Only I follows 9. Statements:
(e) Only II follows Some Square are Circle
7. Statements: No circle are Triangle
Only a few TV are Radio All Triangle are hexagon
No radio is jockeys No hexagon are square
All Jockey are Transmitter Conclusions:
Conclusions: I. Some square are not hexagon.
I. No radio is a jockey is a possibility. II. All circle being hexagon is a possibility.
II. Some TV is not Transmitter (a) Both I and II follow
III. All Transmitter being TV is a possibility. (b) Neither I nor II follows
(a) Only II and III follow (c) Either I or II follows
(b) Only I and II follow (d) Only II follows
(c) Either I, II or III follows (e) Only I follows
Page 504 of 728

Subscribe the Xpress Video Course & Mock Test Package for Bank & Insurance Exams
If there are any suggestions/ errors in our PDFs Feel Free to contact us via this email: admin@exampundit.in
IBPS RRB PO Prelims – Ultra Practice Bundle PDF
10. Statements: All Apple are Guava
Some pink are not green At least Some Guava are banana
All green are yellow All banana are Kiwi
Only a few blue are green No apple are Kiwis
Conclusions: Conclusions:
I. No green are pink is a possibility. I. All kiwis being apple is a possibility.
II. All blue are yellow is a possibility. II. Some guava are not banana
III. Some pink are not blue. III. Some banana is definitely not guava.
(a) Only I follows (a) Only II follows
(b) Only III follows (b) Only I and III follows
(c) Only II follows (c) Only I, II and III follows
(d) Only I and II follows (d) Either I or III follow
(e) Only II and III follow (e) Only II and III follow
11. Statements: 13. Statements:
Each Chinese are Japanese All arrow are Line
No Chinese are Indian Only a few Line are star
No Indian are Mexicans Only a few Star are circle
Conclusions: No circle is Line
I. Some Japanese are not Indian. Conclusions:
II. Some Mexicans are not Japanese. I. No Arrow is circle
III. All Chinese being Mexicans is a possibility. II. All stars being Line is possibility.
IV. All Indians are Japanese is a possibility. III. Some arrow are not stars
(a) Only II and III follows (a) Only II follows
(b) Only I, III and IV follows (b) Only I or II follows
(c) Either II or III follow (c) Only I follows
(d) Only I, II and III follow (d) None of I, II, III follows
(e) Only III follows (e) None follows
12. Statements: 14. Statements:
Page 505 of 728

Subscribe the Xpress Video Course & Mock Test Package for Bank & Insurance Exams
If there are any suggestions/ errors in our PDFs Feel Free to contact us via this email: admin@exampundit.in
IBPS RRB PO Prelims – Ultra Practice Bundle PDF
Some bottles are wine Conclusions:
Some wine are dot I. No interest are hates
No dot are bottle II. Some Topic are hates
Only a few wine are cashew III. No topic are hates
Conclusions: (a) Only II follows
I. Some cashew are not wine (b) Only I and III follows
II. All wine are cashew. (c) Only I, II and III follows
(a) Both I and II follow (d) Either II or III follow
(b) Neither I nor II follows (e) Only II and III follow
(c) Either I or II follows 17. Statements:
(d) Only II follows All administration are amazing
(e) Only I follows Only a few Amazing are attention
15. Statements: Only a few certificates are attention.
Only a few Portent are Impute All culture are certificate
All solicit are impute Conclusions:
Only a few Impute are Covert I. All certificate being administration is a possibility
Conclusions: II. Some administration being attention is a possibility.
I. All solicit can never be Portent. III. No administration is Culture.
II. All portent being Impute is a possibility. (a) Only I follows
(a) Both I and II follow (b) Only I or II follows
(b) Neither I nor II follows (c) Only II follows
(c) Either I or II follows (d) None of I, II, III follows
(d) Only II follows (e) Only I and II follows
(e) Only I follows 18. Statements:
16. Statements: All nature are beauty
At least Some Interest are topic Only a few beauty are wonder
All topic are Book No wonder is worlds
Only a few Book are Hates Conclusions:
Page 506 of 728

Subscribe the Xpress Video Course & Mock Test Package for Bank & Insurance Exams
If there are any suggestions/ errors in our PDFs Feel Free to contact us via this email: admin@exampundit.in
IBPS RRB PO Prelims – Ultra Practice Bundle PDF
I. Some nature are worlds II. No tree is heavy
II. Some wonder are nature III. Some coconut are not heavy
III. No wonder are nature (a) Only I and II follows
(a) Only I follows (b) Only III follows
(b) Only I, II and III follows (c) Only I follows
(c) Only I and III follows (d) Either I or II follow
(d) Either I or II follow (e) Only II and III follow
(e) Either II or III follow 21. Statements:
19. Statements: Only a few wood are glass
Some number are palindrome No glass is Cup
Some palindrome are Digit No cup is plate
No digit are Prime Conclusions:
No prime are number I. Some wood are not plate
Conclusions: II. Some cup are not wood
I. No Digit are Number III. All Wood are cup
II. All prime are number is a possibility (a) None follows
III. No palindrome are Prime (b) Only II follows
(a) Only I follows (c) Only III follows
(b) None follows (d) Only I or III follow
(c) Only II follows (e) None of these
(d) Either I or II follow 22. Statements:
(e) Only I or III follow All bulls are Nets
20. Statements: No bulls are pipe
Only a few coconut are tree All pipes are Monkey
All tree are Ball Some Monkey are cow
No ball are heavy Conclusions:
Conclusions: I. All Nets being monkey is a possibility
I. All Coconut being tree is a possibility II. No bulls are cow
Page 507 of 728

Subscribe the Xpress Video Course & Mock Test Package for Bank & Insurance Exams
If there are any suggestions/ errors in our PDFs Feel Free to contact us via this email: admin@exampundit.in
IBPS RRB PO Prelims – Ultra Practice Bundle PDF
III. Some cow are not nets. (a) Only I, II and III follow
(a) Only I and II follow (b) I and II follow
(b) Only II follows (c) Only II and III follow
(c) Only III follows (d) Only I and either II or III follow
(d) Only I and III follow (e) None of these
(e) None of the above 25. Statements:
23. Statements: Only goat are lion
Only floor are flat All Shark are goat
Some floor are building No Shark are nest
Some building are collage Conclusions:
Conclusions: I. Some lion are not nest
I. No flat are collage II. All goat being shark is a possibility
II. All building being flat is a possibility III. Some nest are not shark
III. Some floor being collage is a possibility (a) Only I and III follows
(a) Only I and II follows (b) Only II follows
(b) Only II and III follows (c) Only III follows
(c) Only I and III follows (d) Only II and III follow
(d) All follow (e) None of these
(e) None of these 26. Statements:
24. Statements: Only stall are hall
Some law are grew No ball are Stall
All grew are House Some call are ball
Some house are pink Conclusions:
No grew are car I. No hall are call
Conclusions: II. Some ball are not call
I. Some law are not Car III. All call being Stall is a possibility
II. No pink are car (a) None follows
III. Some pink are car (b) Only I follows
Page 508 of 728

Subscribe the Xpress Video Course & Mock Test Package for Bank & Insurance Exams
If there are any suggestions/ errors in our PDFs Feel Free to contact us via this email: admin@exampundit.in
IBPS RRB PO Prelims – Ultra Practice Bundle PDF
(c) Only II follows (b) Only II follows
(d) Only III follows (c) None follows
(e) Only I and II follow (d) Only III follows
27. Statements: (e) Only I and II follow
All summer are winter 29. Statements:
No summer are Rainy Some Wednesday are Monday
Only rainy are string All Monday are Tuesday
Some Rainy are autumn Some Tuesday are not Thursday
Conclusions: Only Friday are Thursday
I. All winter can never be Rainy Conclusions:
II. Some string are not winter I. All Wednesday being Tuesday is a possibility
III. Some summer are not autumn II. All Friday being Monday is a possibility
(a) None follows III. Some Wednesday are not Thursday
(b) Only I follows IV. Some Tuesday are Friday
(c) Only II follows (a) I, II and III follow
(d) Only III follows (b) I, II and IV follow
(e) Only I and II follow (c) II and III follow
28. Statements: (d) I, II and either III or IV follow
Some switch are bulb (e) None of these
All bulb are light 30. Statements:
Some light are cooler Some year are month
Some switch are fan All date are month
No fan are cooler Some week are month
Conclusions: No date are days
I. Some bulb are fan Conclusions:
II. Some light are not fan I. All month can never be days
III. Some fan are not bulb II. Some year being week is a possibility.
(a) Only I follows (a) Only conclusion I follow.
Page 509 of 728

Subscribe the Xpress Video Course & Mock Test Package for Bank & Insurance Exams
If there are any suggestions/ errors in our PDFs Feel Free to contact us via this email: admin@exampundit.in
IBPS RRB PO Prelims – Ultra Practice Bundle PDF
(b) Only conclusion II follows. III. Some wood not being aluminium is possibility
(c) Both conclusion I and II follow. (a) Only III follows
(d) Neither conclusion I nor conclusion II follows. (b) Only I follows
(e) None of the above. (c) None follows
31. Statements: (d) Only II follows
Some hair are Nail (e) Only I and II follow
Some limb are hands 33. Statements:
Some hand are leg Only a few hill are peak
No leg are hair Only a few peak are events
No nail are limb Only a few plates are event
Conclusions: No plates are motion
I. Some limb are not leg is a possibility Conclusions:
II. All hair being hand is a possibility I. Some hills are not plates
III. Some hand are not hair II. All events being plates is a possibility
(a) Only I and II follows III. Some Peak are not plates
(b) Only III follows (a) Only I follows
(c) All follows (b) Only II follows
(d) Only III follows (c) None follows
(e) Only I and III follow (d) Only III follows
32. Statements: (e) Only I and II follow
Only wood are iron 34. Statements:
Some wood are Bronze No bike are Vehicle
No gold is Bronze All bus are bicycle
All silver are gold At least some bus are vehicle
Some bronze are Aluminium All truck are Bicycle
Conclusions: Conclusions:
I. Some aluminium are not silver I. Some bike are not bus
II. Some gold are definitely not iron II. No truck are vehicle
Page 510 of 728

Subscribe the Xpress Video Course & Mock Test Package for Bank & Insurance Exams
If there are any suggestions/ errors in our PDFs Feel Free to contact us via this email: admin@exampundit.in
IBPS RRB PO Prelims – Ultra Practice Bundle PDF
III. All bicycle are bike is possibility (b) All follows
(a) Only I and III follows (c) Only I and II follows
(b) Only I and II follows (d) Only I follows
(c) Only III follows (e) Either II or III follow
(d) None follows 37. Statements:
(e) Only I and II follow Some birds are house
35. Statements: Some bird are lion
Some lily are rose No lion are house
No rose are begonia Some house are tiger
All rose are lotus Some tiger are lion
Conclusions: Conclusions:
I. Some lily are not Begonia I. Some tiger are not lion
II. Some lotus are not begonia II. Some bird are not tiger
III. Some begonia are lotus III. No bird is tiger
(a) Only I and II follows (a) Only II and either I or III follows
(b) Only II follows (b) Only I follows
(c) Only I follows (c) Only III follows
(d) All follows (d) All follows
(e) Either II or III follow (e) Only I and Either II or III follow
36. Statements: 38. Statements:
Only handle are chair Some stick are lamp
Some handle are pots Some lamp are flow
Some pots are mat All lamp are shirt
Conclusions: No shirt are dress
I. No chair are mat Conclusions:
II. All pots are handle is a possibility I. Some flow are not stick
III. Some handle are not mat II. No dress are stick
(a) Only II follows III. Some stick being dress is a possibility
Page 511 of 728

Subscribe the Xpress Video Course & Mock Test Package for Bank & Insurance Exams
If there are any suggestions/ errors in our PDFs Feel Free to contact us via this email: admin@exampundit.in
IBPS RRB PO Prelims – Ultra Practice Bundle PDF
(a) Only I follows (b) Only II and III follows
(b) All follows (c) None follows
(c) Only III follows (d) Only III follows
(d) Only I and III follows (e) Only I and II follow
(e) Either I or III follow 41. Statements:
39. Statements: Some exam are not test
Only a few dew is Drop All test are answer
Only drop is stone Some answer are not question
Some drop is cement Conclusions:
Conclusions: I. No question are exam is a possibility
I. No dew is cement II. Some answer are not exam
II. All drop being cement is a possibility III. All test being exam is a possibility
III. Every cements being dew is a possibility (a) Only I follows
(a) Only I follows (b) Only II follows
(b) Only II follows (c) None follows
(c) None follows (d) Only III follows
(d) Only III follows (e) Only I and II follow
(e) Only I and II follow 42. Statements:
40. Statements: No single are Rock
All bunkers are guns All English are single
All dagger are pistol Only a few Hindi are english
Only a few bullet are pistol No English are Compress
No bunkers are dagger Conclusions:
Conclusions: I. All hindi being single is a possibility
I. All bullets are dagger II. Some hindi are not Compress
II. Some guns being bullets is a possibility III. Some rock are not compress
III. All pistol being gun is a possibility (a) Only II follows
(a) Only II follows (b) Only I follows
Page 512 of 728

Subscribe the Xpress Video Course & Mock Test Package for Bank & Insurance Exams
If there are any suggestions/ errors in our PDFs Feel Free to contact us via this email: admin@exampundit.in
IBPS RRB PO Prelims – Ultra Practice Bundle PDF
(c) All follows (d) Only I and II follow
(d) Only I and II follows (e) None of the above
(e) Only I and III follow 45. Statements:
43. Statements: Only a few men are women
All wins are trophy All child are women
All trophy are Price Some women are adult
Only a few loss are wins All son are adult
Conclusions: No son are men
I. All loss being price is a possibility Conclusions:
II. Some price are not trophy I. Some men are not child
III. All price being loss is a possibility II. All adult being men is a possibility
(a) Only I and III follows III. Some women are not adult
(b) Only III follows (a) Only I follows
(c) None follows (b) Only III follows
(d) Only II follows (c) None follows
(e) Only II and III follow (d) Only II follows
44. Statements: (e) Only II and III follow
Some baskets are casket 46. Statements:
Only a few casket are fan Some exam are paper
All fan are trucks Some exam are college
No trucks are baskets No college are paper
Conclusions: Only a few students are paper
I. Some casket are not trucks Conclusions:
II. Some fan are not casket I. All exam are college is a possibility
III. No fan are basket II. Some students are not exam
(a) Only II follows III. No college are students
(b) Only II follows (a) Only I and III follows
(c) Only III follows (b) Only II follows
Page 513 of 728

Subscribe the Xpress Video Course & Mock Test Package for Bank & Insurance Exams
If there are any suggestions/ errors in our PDFs Feel Free to contact us via this email: admin@exampundit.in
IBPS RRB PO Prelims – Ultra Practice Bundle PDF
(c) All follows (d) Only III follows
(d) None follows (e) Only I and II follow
(e) Only I and II follow 49. Statements:
47. Statements: Only a few elephant are tiger
At least some king queen Some tiger are rat
All card are queen All rat are cat
Only a few card are joker No lion are rat
Conclusions: Conclusions:
I. No king are card I. Some elephant are not rat
II. Some joker are not queen is a possibility II. Some tiger are not lion
III. All queen being joker is a possibility III. All lion being elephant is a possibility
(a) Only I follows (a) Only III follows
(b) Only II follows (b) Only I follows
(c) None follows (c) None follows
(d) Only III follows (d) Only II follows
(e) Only I and II follow (e) Only II and III follow
48. Statements: 50. Statements:
All august are January All sky are bird
Some august are February All bird are Fly
No august are March All fly are rock
Some march are February No Sky are Moon
Conclusions: Conclusions:
I. Some February are not August I. All rock are moon is possibility
II. All august being February is a possibility II. Some fly are not moon
III. Some January are march III. Some moon are not Bird
(a) Only I follows (a) Only II follows
(b) Only II follows (b) Only I follows
(c) All follows (c) None follows
Page 514 of 728

Subscribe the Xpress Video Course & Mock Test Package for Bank & Insurance Exams
If there are any suggestions/ errors in our PDFs Feel Free to contact us via this email: admin@exampundit.in
IBPS RRB PO Prelims – Ultra Practice Bundle PDF
(d) Only III follows
(e) Only II and III follow

24). Syllogism Questions - Solution and Detailed Explanation

SOLUTION (1-5):

1). b

4). c

2). b

5). e
3). b

Page 515 of 728

Subscribe the Xpress Video Course & Mock Test Package for Bank & Insurance Exams
If there are any suggestions/ errors in our PDFs Feel Free to contact us via this email: admin@exampundit.in
IBPS RRB PO Prelims – Ultra Practice Bundle PDF
6). b

7). e (only III follows) 10). d

11). b

8). d

12). a
9). e
Page 516 of 728

Subscribe the Xpress Video Course & Mock Test Package for Bank & Insurance Exams
If there are any suggestions/ errors in our PDFs Feel Free to contact us via this email: admin@exampundit.in
IBPS RRB PO Prelims – Ultra Practice Bundle PDF

13). c

16). d

14). b

17). e

15). b

Page 517 of 728

Subscribe the Xpress Video Course & Mock Test Package for Bank & Insurance Exams
If there are any suggestions/ errors in our PDFs Feel Free to contact us via this email: admin@exampundit.in
IBPS RRB PO Prelims – Ultra Practice Bundle PDF

18). e
21). a

19). b 22). e (only I follows)

20). e 23). c

Page 518 of 728

Subscribe the Xpress Video Course & Mock Test Package for Bank & Insurance Exams
If there are any suggestions/ errors in our PDFs Feel Free to contact us via this email: admin@exampundit.in
IBPS RRB PO Prelims – Ultra Practice Bundle PDF

24). d 27). b

28). b

25). a

26). b 29). e (Only 1 follows)

Page 519 of 728

Subscribe the Xpress Video Course & Mock Test Package for Bank & Insurance Exams
If there are any suggestions/ errors in our PDFs Feel Free to contact us via this email: admin@exampundit.in
IBPS RRB PO Prelims – Ultra Practice Bundle PDF

33). b
30). c

34). d
31). c

32). e

35). a

Page 520 of 728

Subscribe the Xpress Video Course & Mock Test Package for Bank & Insurance Exams
If there are any suggestions/ errors in our PDFs Feel Free to contact us via this email: admin@exampundit.in
IBPS RRB PO Prelims – Ultra Practice Bundle PDF

39). d
36). c

37). b
40). b

38). c
41). d
Page 521 of 728

Subscribe the Xpress Video Course & Mock Test Package for Bank & Insurance Exams
If there are any suggestions/ errors in our PDFs Feel Free to contact us via this email: admin@exampundit.in
IBPS RRB PO Prelims – Ultra Practice Bundle PDF

45). a
42). d

43). a 46). d

47). b
44). c

Page 522 of 728

Subscribe the Xpress Video Course & Mock Test Package for Bank & Insurance Exams
If there are any suggestions/ errors in our PDFs Feel Free to contact us via this email: admin@exampundit.in
IBPS RRB PO Prelims – Ultra Practice Bundle PDF

48). e

50). a

49). e

Page 523 of 728

Subscribe the Xpress Video Course & Mock Test Package for Bank & Insurance Exams
If there are any suggestions/ errors in our PDFs Feel Free to contact us via this email: admin@exampundit.in
IBPS RRB PO Prelims – Ultra Practice Bundle PDF
25). Conditional Coding Decoding Questions
Directions (1 – 5): In each question below is given a 1. %BNE#
group of numbers / symbols followed by five a) 49054
combinations of letter codes numbered (1), (2), (3), b) 49505
(4) and (5). You have to find out which of the c) 49504
combinations correctly represents the group of d) 49507
numbers / symbols based on the following coding e) 79507
system and the conditions and mark the number of 2. ADO&@
that combination as your answer: a) 82638
Element & B A % D E # N O @ b) 82631
Symbol 3 9 1 4 2 0 7 5 6 8 c) 12631
Conditions: d) 12638
1. If the first element is a vowel and the last element is a e) 86231
symbol, then the codes of both the elements are 3. B%&NO
interchanged. a) 65346
2. If the first element is a consonant and the last element b) 95346
is a vowel, then the codes of all the elements are to be c) 65439
reversed in order. d) 65349
3. If both the first and the last elements are symbols, e) 63549
then both the elements are to be coded as code of the 4. B%&#N
first symbol. a) 53475
4. If both the first and the last elements are consonants, b) 54735
then both the elements are to be coded as code as code c) 94739
of the last letter. d) 94379
Each element is denoted by a number as shown above. e) 54375
Further, consider the given conditions and answer the 5. DA%&O
questions accordingly for a given element: a) 21436
Page 524 of 728

Subscribe the Xpress Video Course & Mock Test Package for Bank & Insurance Exams
If there are any suggestions/ errors in our PDFs Feel Free to contact us via this email: admin@exampundit.in
IBPS RRB PO Prelims – Ultra Practice Bundle PDF
b) 64312 ii) If the first letter is Vowel and the last digit is
c) 63418 perfectly divisible by 2, then both are to be coded as 'Z'.
d) 23416 iii) If both third and fifth element are exact multiples of
e) 63412 three, the smaller of the two numbers is to be coded as '-
Directions (6 – 10): In each of the questions given '.
below, a group of digits/letter is given followed by iv) If second last element is a consonant and fourth digit
four combinations of symbols numbered (a), (b), (c) is a perfect square, then both are to be coded as 'X'.
and (d). Find out which of the four combinations Answer following questions based on above
correctly represents the group of digits/letters based information:
on the symbol codes and the conditions given below. 6. 1B3U64
If none of the four combinations represents the a) +%∞÷#$
group of digits correctly, give (e) i.e. 'None of these' b) +%-÷#$
as the answer. c) +%#÷#$
Note: d) $%-÷#+
1. If more than one condition applies, all are to be e) None of these
applied for coding. 7. AL81M6
2. Counting is done from left to right. a) Z/&XXZ
3. Consider 1 as a perfect square. b) XZ/&XZ
Digi 7 A 1 6 B I 8 M F 9 L U 3 2 E 4 c) ZXX&*/
t d) Z&/XXZ
Sy ! @ $ # % ^ & * © → / ÷ ∞ ∑ ₹ + e) None of these
mb 8. 3BU2I4
ol a) +!@∑^∞
Condition for coding the group elements: b) +%-∑^∞
i) If the first element is an odd number and the last c) +%÷∑^∞
element a perfect square, then the codes for these two d) $+%÷∑^
are to be interchanged. e) None of these
9. 7IU9M1
Page 525 of 728

Subscribe the Xpress Video Course & Mock Test Package for Bank & Insurance Exams
If there are any suggestions/ errors in our PDFs Feel Free to contact us via this email: admin@exampundit.in
IBPS RRB PO Prelims – Ultra Practice Bundle PDF
a) $^÷XX% c) ∞! $-XX
b) ! $^÷XX
d) $^÷XX! b) @Z! #$Z
e) None of these c) Z*-₹#Z
10. UM3E68 d) @#^%!*
a) Z₹*-%Z e) None of these

Directions (11 – 15): In each question below is given a group of numbers / symbols followed by five
combinations of letter codes numbered (1), (2), (3), (4) and (5). You have to find out which of the combinations
correctly represents the group of numbers / symbols based on the following coding system and the conditions
and mark the number of that combination as your answer:
Number * > ! ^ $ # + 7 2 { 5 3 8 1 4 ) 6 0 9
/
Symbols
Letter Z Q D L H A P f U O Y B J y R G w I X
Code

Conditions: If more than one conditions follow then the order of


(1) If a number is immediately preceded by a symbol precedence will be in ascending order of the condition
and immediately succeeded by a number, then all the number.
symbols are to be coded as ‘y’. 11. How *0{7+65^1 will be coded?
(2) If the second element is an even number and is a) ZIOfPwYLy
immediately succeeded by a symbol then that even b) yIyfywYyy
number is to be coded as the code for the symbol. c) yFyyyyyyy
(3) If there are no odd numbers then the codes of the d) YfFWwHAYF
second and last element have to be interchanged. e) None of these
12. How {2!>4^2 will be coded?
a) None of these
Page 526 of 728

Subscribe the Xpress Video Course & Mock Test Package for Bank & Insurance Exams
If there are any suggestions/ errors in our PDFs Feel Free to contact us via this email: admin@exampundit.in
IBPS RRB PO Prelims – Ultra Practice Bundle PDF
b) DQOUUIDQ Code # ¥ β µ ^ € @ $ © & π %
c) OUDQRLD Each element in the 1st row is denoted by a symbol in its
d) ODODRUIQ respective row below it. Further, consider the give
e) DDDQRODI conditions and answer accordingly.
13. How 8)646+2 will be coded? (i) If the first element is odd and it is immediately
a) JGwRwpU followed by a consonant then code of the both the
b) jUrwFPT element should be interchanged.
c) JGWRWPU (ii) If the last element is even and the letter preceding it
d) YRPjwRg is a vowel then the vowel takes the code of the even
e) JUwRwyy number following it.
14. How 8>^$+26* will you code? (iii) If the third element is preceded by a vowel and
a) JyyyyUwy immediately followed by a consonant then code for that
b) fUOyYYwW vowel and consonant should be interchanged.
c) HAPPYyuy 16. RTM5E8
d) JUUyyAHA a) β ^ © $ µ @
e) JYBDLyyy b) β ^ © $ µ µ
15. What will be the result of ^4{>#${5 after coding? c) β ^ © & µ µ
a) LrOQAHOY d) β ^ © $ @ µ
b) LooQAHOY e) None
c) LOOQAHOY 17. 8AWT9R
d) LROQAHOY a) ^ π # ¥ β µ
e) LOOQAOHY b) µ ¥ π # ^ β
c) µ # π ^ ¥ β
Directions (16 – 20): Given below are the codes for d) µ ^ π # ¥ β
the digits/symbols. Study the conditions given below e) None
and answer the questions that follow. 18. 9MR4WA
a) © ¥ β & π #
Symbol A 9 R 8 T 3 E 5 M 4 W U b) © © β & π #
Page 527 of 728

Subscribe the Xpress Video Course & Mock Test Package for Bank & Insurance Exams
If there are any suggestions/ errors in our PDFs Feel Free to contact us via this email: admin@exampundit.in
IBPS RRB PO Prelims – Ultra Practice Bundle PDF
c) ¥ © β & π # (2) If the left most element is a symbol and the right
d) © ¥ β & π © most element is an even digit then the code for both will
e) None be Y.
19. 9ATUW8 (3) If the left most element is an odd digit and the right
a) ¥ % ^ # π µ most element is a symbol then the codes of these two
b) # ¥ ^ % µ µ elements got interchanged.
c) # ¥ ^ % π µ (4) If the left most element is an odd digit and the right
d) ¥ # ^ % π µ most element is an even digit then the code for both will
e) None be W.
20. 89TMA4 (5) If the left most element in the group is an even digit
a) µ ¥ ^ © # & and the right most element is an odd digit then reverse
b) µ ¥ ^ © & # the order of the code for the group.
c) µ ¥ ^ © && 21. 836@497
d) µ ¥ ^ © # # a) PDECHFA
e) None b) BFHCEDA
Directions (21 – 25): Given below are the codes for c) ADECHFB
the digits/symbols. Study the conditions given below d) XDECHFX
and answer the questions that follow. e) YDECHFY
22. 3#%7964
a) HEFBJGD
b) WGJBFEW
Conditions: c) XGJBFEX
(1) If both the left most and the right most elements are d) HGJBFED
symbols, then the codes for the first two elements get e) None of these
interchanged between them and the codes for the last 23. $4783&6
two elements get interchanged between them. a) EKDABHQ
b) ZHBADKZ
c) EHBADKQ
Page 528 of 728

Subscribe the Xpress Video Course & Mock Test Package for Bank & Insurance Exams
If there are any suggestions/ errors in our PDFs Feel Free to contact us via this email: admin@exampundit.in
IBPS RRB PO Prelims – Ultra Practice Bundle PDF
d) YHBADKE 25. %8@521*
e) YHBADKY a) AJCTLNR
24. 51&269@ b) NACTLRJ
a) YRKLEFY c) ZACTLRZ
b) ZRKLEFZ d) YACTLRY
c) XRKLEFX e) None of these
d) CRKLEFT
e) None of these
Directions (26 – 30): Given below are the codes for the digits/symbols. Study the conditions given below and
answer the questions that follow.

Conditions:
1. If both the first and last elements are prime number, e) EBLGMV
then interchange their codes. 27. 3*8!#5
2. If the second element is an even digit and the fourth a) IOXYGR
element is a symbol, then code both of them with the b) ROYXGI
code of the fourth element. c) IOYXGI
3. If the first element is a perfect square and the last d) IOYXGR
element is a symbol, then code both of them as Z. e) ROVXGR
4. If the third element is an even prime and the fifth 28. 9%72^@
element is an odd prime, then code both of them as S. a) ZEMLKZ
26. %&2#7@ b) ZWMLKZ
a) EBSVSG c) ZEMLKV
b) EBSGSV d) AEMLKZ
c) EDSGSV e) WAMLZK
d) GBSVSB 29. @4#&92

Page 529 of 728

Subscribe the Xpress Video Course & Mock Test Package for Bank & Insurance Exams
If there are any suggestions/ errors in our PDFs Feel Free to contact us via this email: admin@exampundit.in
IBPS RRB PO Prelims – Ultra Practice Bundle PDF
a) VBGABL c) GAYERK
b) BVGBAL d) KAYGRA
c) VBGBAL e) None of these
d) VQGBAL
e) None of these
30. 8#9^3%
a) YGAKRM
b) YGARKE
Directions (31 – 35): Study the following information carefully to answer the given questions
Letter M P F Q V L U G S E X K T Z C
Code @ 0 7 * 9 % 1 # 5 ! ? 3 © 8 $
Condition: d) 38*$5!
i)If the first letter is a vowel and the last letter is a e) None of these
consonant, the codes are to be interchanged. 33. UMLGTP
ii)If the first letter is consonant and the last letter is a a) 0@#©%0
vowel, both are to be coded as the code for the last letter b) $@%#©$
iii)If both the first and the last letters are vowels, both c) 1@%#©0
are to be coded as ‘&’. d) 0@%#©1
31. QGMCLE e) None of these
a) !#@$%! 34. EGXKQU
b) &#%$?& a) &#?3*&
c) 7#?$%8 b) !#?3*1
d) *#@$%! c) 13?#*!
e) None of these d) !#3*?&
32. KZQCSE e) None of these
a) 38*$53 35. PEZTVL
b) !8*$5! a) %!8©90
c) 58*$5! b) @0!5#?
Page 530 of 728

Subscribe the Xpress Video Course & Mock Test Package for Bank & Insurance Exams
If there are any suggestions/ errors in our PDFs Feel Free to contact us via this email: admin@exampundit.in
IBPS RRB PO Prelims – Ultra Practice Bundle PDF
c) 0!8©9% d) $5$46
d) 1!*?9% e) None of these
e) None of these 37. What would be the code of the word ‘QJCWM’?
Directions (36 – 40): In each of the following a) 71$3%
questions given below, a group of digits/letters is b) 0736%
given followed by four combinations of symbols c) 78&%4
numbered A, B, C and D. you have to find out which d) %073&
of the following four combinations correctly e) None of these
represents the group of digits/letters based on the 38. What would be the code of the word ‘RIWED’?
symbol codes and the conditions given below. a) $73%5
Le Q D I P S E H R C U M W N A J B O b) !82#@
tte c) $52?@
r d) $#53@
Co 7 @ 5 4 1 # 6 $ % 9 & 3 2 ? 0 8 ! e) None of these
de 39. What would be the code of the word ‘SUPAE’?
a) 5@4?1
Condition 1:If there are more than two vowels in the b) #?491
word then the code will be written in reversed order. c) 496?#
Condition 2:If there is no vowel in the word then first d) #5!91
three letters of the code will be written in reversed order. e) None of these
Condition 3:If any two letters of the word are same then 40. What would be the code of the word ‘HNRBH’?
last two letters of the code will be interchanged. a) $2?28
Note: If more than one condition are to be applied, all b) 0$06%
will applied. c) $2668
36. What would be the code of the word ‘PMIMN’? d) $2686
a) $4$68 e) None of these
b) 1%3%6 Directions (41 – 45): Study the following data
c) 4&52& carefully and answer the questions accordingly.
Page 531 of 728

Subscribe the Xpress Video Course & Mock Test Package for Bank & Insurance Exams
If there are any suggestions/ errors in our PDFs Feel Free to contact us via this email: admin@exampundit.in
IBPS RRB PO Prelims – Ultra Practice Bundle PDF
b) &>≤+=∆!
c) &+≤==∆!

Conditions: d) &>≤==∆!

(i) If the first, as well as the last number, is even, then e) None of these

both are to be coded by the code for the first number. 44. 6B952GU

(ii) If the first number is even and the last letter is a) !&>≥=*≤

consonant, then both are to be coded by the code for the b) !&>+∆*≤

last letter. c) !&>≥∆M≤

(iii) If the first number is odd and the last letter is a d) !&>≥∆*≤

vowel, then the codes for the first and the last digits are e) None of these

to be interchanged. 45. 5GU6BDU

(iv) If the first, as well as the last letter, is a consonant, a) ≤*≤!+%≥

then both are to be coded by the code for the third b) ≤*≤!&M≥

element. c) ≤*≤=&%≥

41. 9J6VJD7 d) ≤*≤!&%≥

a) >/!+/%+ e) None of these

b) >/!&/%= Directions (46 – 50): Study the following information

c) >/!=/<= carefully and answer the questions given below.In

d) >/!=/%= each question below is given a group of letters

e) None of these followed by four combinations of digits/symbols

42. D73WJ6V numbered (1), (2), (3) and (4). You have to find out

a) +=+</&+ which of the combinations correctly represents the

b) +=+<>!+ group of letters based on the following coding system

c) +=+</!+ and mark the number of that combination as the

d) +=+%/!+ answer. If none of the four combinations correctly

e) None of these represents the group of letters, mark 5), i.e. ‘None of

43. B9UV726 these’, as the answer.


a) &><==∆! Lett R U S B O X A D K Q C Z G L F H
Page 532 of 728

Subscribe the Xpress Video Course & Mock Test Package for Bank & Insurance Exams
If there are any suggestions/ errors in our PDFs Feel Free to contact us via this email: admin@exampundit.in
IBPS RRB PO Prelims – Ultra Practice Bundle PDF
er a) 5%4@78
Dig 6 3 % & 4 @ 5 $ 1 & 7 9 # 2 © 8 b) 8%4@75
it / c) 5%4@75
Sy d) 8%4@78
mb e) 5%47@8
ol 48. ALBDZO
a) 520004
Conditions: b) 50&004
1) If the first letter is a vowel and the last letter is a c) 500$04
consonant their codes are to be interchanged. d) 500004
2) If both the first and the last letters is consonant both e) 400005
are to be coded as ?. 49. OLBDRX
3) If both the first and the last letters is vowel and in a) @&2$64
between if there are two or more consonants, then all the b) @2&6$4
consonants are to be coded as ‘0’. c) @&26$4
4) If the first letter is consonant and the last letter is a d) @&2$64
vowel, both are to be coded as the code for the vowel. e) @2&$64
46. XLDABO 50. OKSXGA
a) @2$5&@ a) 400005
b) ?2$5&? b) 40%005
c) 42$5&4 c) 400@05
d) 42$&54 d) 410005
e) ?2$&5? e) 4000#5
47. ASOXCH

Page 533 of 728

Subscribe the Xpress Video Course & Mock Test Package for Bank & Insurance Exams
If there are any suggestions/ errors in our PDFs Feel Free to contact us via this email: admin@exampundit.in
IBPS RRB PO Prelims – Ultra Practice Bundle PDF
25). Conditional Coding Decoding Questions - Answers with Explanation:
1. Answer: C) 7. Answer: A)
By rule 3, Using condition (ii) and (iv), the code becomes
% and # is to be coded as 4. Z/&XXZ
B-9, N-5, E-0 8. Answer: C)
So, the code is 49504. Using condition (i), the code becomes +%÷∑^∞
2. Answer: B) 9. Answer: D)
By rule 1, Using condition (i) and (iv), the code becomes $^÷XX!
A and @ code will be interchanged. 10. Answer: C)
D-2, O-6, &-3 Using condition (ii) and (iii), the code becomes Z*-₹#Z
So, the code is 82631. 11. Answer: B)
3. Answer: D) Here, only 1st rule applies. As per 1st rule, a number is
By rule 2, immediately preceded by a number and immediately
The codes of all the elements are to be reversed. succeeded by a symbol. So, the symbols are to be coded
B-9, %-4, &-3, N-5, O-6. as ‘y’.
Hence our answer will be 65349 Therefore, *0{7+65^1 will be coded as yIyfywYyy.
4. Answer: E) 12. Answer: C)
By rule 4, Here, 2nd and 3rd rule applies. As per 2nd rule, second
Both B and N are coded as 5. element is an even number and is immediately
%-4, &-3, #-7 succeeded by a symbol. So, even number is to be coded
So, the code is 54375. as the code for the symbol i.e. ODDQRLU. Now as per
5. Answer: E) 3rd rule, there are no odd numbers. So, the codes of the
By rule 2, second and last element have to be interchanged i.e.
The codes of all elements are reversed. OUDQRLD.
So, the code is 63412. Therefore, {2!>42^ will be coded as OUDQRLD.
6. Answer: B) 13. Answer: E)
Using condition (i) and (iii), the code becomes +%-÷#$

Page 534 of 728

Subscribe the Xpress Video Course & Mock Test Package for Bank & Insurance Exams
If there are any suggestions/ errors in our PDFs Feel Free to contact us via this email: admin@exampundit.in
IBPS RRB PO Prelims – Ultra Practice Bundle PDF
Here, Rule 1 applies for ")64" so ')' and '+' will be coded By Rule iii. The third element W is preceded by A and
as 'y'. followed by T. So, code for A and T must be
3rd rule also applies. As per 3rd rule, there are no odd interchanged.
numbers. So, the codes of the second and last element 8 A W T 9 R
have to be interchanged. µ ^ π # ¥ β
Therefore, 8)646+2 will be coded as JUwRwyy.
14. Answer: A) 18. Answer: A)
Here, 1st rule and 3rd rule applies. As per 1st rule, number By Rule i. The first element is 9 and it is immediately
is immediately preceded by a number and immediately followed by M. So, code for 9 and M must be
succeeded by a symbol. So, symbols are to be coded as interchanged.
‘y’ i.e. JyyyyUwy. As per 3rd rule, there are no odd 9 M R 4 W A
numbers. So, the codes of the second and last element © ¥ β & π #
have to be interchanged i.e. JyyyyUwy. 19. Answer: D)
Therefore, 8>^$+26* is coded as JyyyyUwy. No Rule follows. Hence, codes will remain as it is.
15. Answer: C) 9 A T U W 8
nd nd
Here, only 2 rule applies. So as per 2 rule, second ¥ # ^ % π µ
element is an even number and is immediately 20. Answer: C)
succeeded by a symbol. So, even number is to be coded By Rule ii. Last element is 4 and is preceded by 'A'.
as the code for the symbol. Thus, A should be coded as code for 4. i.e. &
Therefore, ^4{>#${5 is coded as LOOQAHOY. 8 9 T M A 4
16. Answer: B) µ ¥ ^ © & &
By Rule ii. Last element is 8 and is preceded by E. Thus,
21. Answer: B)
E should be coded as code for 8. i.e. µ It satisfies condition (5).
R T M 5 E 8 If the left most element in the group is an even digit and
β ^ © $ µ µ the right most element is an odd digit then reverse the
17. Answer: D) order of the code for the group.
Hence, code for 836@497 is BFHCEDA.
22. Answer: B)
Page 535 of 728

Subscribe the Xpress Video Course & Mock Test Package for Bank & Insurance Exams
If there are any suggestions/ errors in our PDFs Feel Free to contact us via this email: admin@exampundit.in
IBPS RRB PO Prelims – Ultra Practice Bundle PDF
It satisfies condition (4). Hence, the code is IOYXGR.
If the left most element is an odd digit and the right most 28. Answer: A)
element is an even digit then the code for both will be It satisfies condition number 3.
W. Hence, the code is ZEMLKZ.
Hence, code for 3#%7964 is WGJBFEW. 29. Answer: C)
23. Answer: E) It satisfies the condition number 2.
It satisfies condition (2). Hence, the code is VBGBAL.
If the left most element is a symbol and the right most 30. Answer: E)
element is an even digit then the code for both will be Y. It does not satisfy any of the given conditions.
Hence, code for $4783&6 is YHBADKY. Hence, the code is YGAKRE.
24. Answer: D) 31. Answer: A)
It satisfies condition (3). QGMCLE = *#@$%! = !#@$%!
If the left most element is an odd digit and the right most 32. Answer: B)
element is a symbol then the codes of these two KZQCS => 38*$5! = !8*$5!
elements got interchanged. 33. Answer: D)
Hence, code for 51&269@ is CRKLEFT. UMLGTP = 1@%#©0 = 0@%#©1
25. Answer: A) 34. Answer: A)
It satisfies condition (1). EGXKQU = !#?3*1 = &#?3*&
If both the left most and the right most elements are 35. Answer: C)
symbols, then the codes for the first two elements get PEZTVL = 0!8©9%
interchanged between them and the codes for the last 36. Answer: C)
two elements get interchanged between them. Here, we can see that the two letters ‘M’ of the word
Hence, code for %8@521* is AJCTLNR. ‘PMIMN’ are same. Therefore, Condition 3 can be
26. Answer: B) applied:
It satisfies the condition number 4. Then, the code of the word ‘PMIMN’ is ‘4&5&2’ after
Hence, the code is EBSGSV. applying the conditions ‘4&52&’.
27. Answer: D) 37. Answer: D)
It satisfies the condition number 1.
Page 536 of 728

Subscribe the Xpress Video Course & Mock Test Package for Bank & Insurance Exams
If there are any suggestions/ errors in our PDFs Feel Free to contact us via this email: admin@exampundit.in
IBPS RRB PO Prelims – Ultra Practice Bundle PDF
Here, we can see that there is no vowel in the word 43. Answer: D)
‘QJCWM’. Therefore, Condition 2 can be applied: None of the above conditions will be applied. So the
Then, the code of the word ‘QJCWM’ is ‘70%3&’ after given letters and digits will be coded according to their
reversing first three letters ‘%073&’. respective codes given.
38. Answer: E) B9UV726 - &>≤==∆!
Here, we can see that there is no condition applicable in 44. Answer: D)
this case. So, we can write the code of ‘RIWED’ directly None of the above conditions will be applied. So the
from the given table: given letters and digits will be coded according to their
Then, the code of the word ‘RIWED’ is ‘$53#@’. respective codes given.
39. Answer: B) 6B952GU - !&>≥∆*≤
Here, we can see that there are more than two vowels in 45. Answer: D)
the word ‘SUPAE’. Therefore, Condition 1 can be Here, condition (iii) will be applied.
applied: 5GU6BDU - ≤*≤!&%≥
Then, the code of the word ‘SUPAE’ is ‘194?#’ after 46. Answer: C)
reversing the code ‘#?491’. Here, condition (iv) will be applied.
40. Answer: C) XLDABO = 42$5&4
Here, we can see that there is no vowel in the word 47. Answer: B)
‘HNRBH’ and the two letters ‘H’ are same. Therefore, Here, condition (i) will be applied.
both Conditions 2 and 3 can be applied: ASOXCH = 8%4@75
Then, the code of the word ‘HNRBH’ is ‘62$86’ after 48. Answer: D)
applying the conditions ‘$2668’. Here, condition (iii) will be applied
41. Answer: D) ALBDZO = 500004
None of the above conditions will be applied. So the 49. Answer: E)
given letters and digits will be coded according to their Here, condition (i) will be applied.
respective codes given. 9J6VJD7 - >/!=/%= OLBDRX =@2&$64
42. Answer: C) 50. Answer: A)
Here, condition (iv) will be applied. Here, condition (iii) will be applied.
D73WJ6V - +=+</!+ OKSXGA = 400005
Page 537 of 728

Subscribe the Xpress Video Course & Mock Test Package for Bank & Insurance Exams
If there are any suggestions/ errors in our PDFs Feel Free to contact us via this email: admin@exampundit.in
IBPS RRB PO Prelims – Ultra Practice Bundle PDF
IBPS RRB PO Prelims – Quantitative Aptitude

S.no Topic Page number


1. Approximation
Questions 539
Solutions 544

2. Simplification
Questions 550
Solutions 556

3. Missing Number
Questions 560
Solutions 565

4. Wrong Number
Questions 572
Solutions 578

5. Quadratic Equation
Questions 584
Solutions 593

6. DI – Table
Questions 609
Solutions 622

7. DI- Bar
Questions 631
Solutions 646

8. DI- Line
Questions 653
Solutions 668

9. DI- Pie
Page 538 of 728

Subscribe the Xpress Video Course & Mock Test Package for Bank & Insurance Exams
If there are any suggestions/ errors in our PDFs Feel Free to contact us via this email: admin@exampundit.in
IBPS RRB PO Prelims – Ultra Practice Bundle PDF
Questions 682
Solutions 702

10. Caselet DI
Questions 708
Solutions 718

1). Approximation Questions


Directions (1-50): What approximate value will come c) 17
in place of the question mark (?) in the following d) 23
question (Note: You are not expected to calculate the e) 19
exact value. 4. ? = ((30.982.02 – 11.981.98) + 24.32)0.49
1. 39.8% of (62.05 × 4.99) + 67.12 = ? a) 15
a) 100 b) 19
b) 270 c) 24
c) 230 d) 35
d) 190 e) 29
e) 150 5. ? = 20.91% of (36.2 × (144.01 + 5.95))
2. ? = 30.41% of 54.5% of 4198 – 92.55 a) 1200
a) 500 b) 1080
b) 600 c) 1440
c) 700 d) 900
d) 800 e) 720
e) 900 6. ? = (41.98 + 21.02) ÷ (18.09 + 1.97)
3. (343.78 - ?2)= 5351.8 ÷ 24.01 a) 9
a) 11 b) 7
b) 7 c) 5

Page 539 of 728

Subscribe the Xpress Video Course & Mock Test Package for Bank & Insurance Exams
If there are any suggestions/ errors in our PDFs Feel Free to contact us via this email: admin@exampundit.in
IBPS RRB PO Prelims – Ultra Practice Bundle PDF
d) 3 b) 4610
e) 1 c) 4665
7. ? = (√780 × 4.89) ÷ 1.98 d) 4625
a) 40 e) 4650
b) 70 12. (180.20 - 59.74) ÷ 40.04 = 79.81 - 61.72 - ?
c) 120 a) 7
d) 150 b) 15
e) 200 c) 18
8. ? = 576.124 – 241.12 + 4.013.011 d) 9
a) 400 e) 10
b) 300 13. (27.84 + 31.90) ÷ 14.84 = 97.79 ÷ 13.98 - ?
c) 200 a) 3
d) 500 b) 10
e) 600 c) 5
9. ? = 17.12 × 14.89 + 121.12 d) 11
a) 375 e) 2
b) 300 14. (91.87 + 118.10) ÷ 14.20 = 90.08 - 14.88% of ?
c) 225 a) 50
d) 150 b) 300
e) 450 c) 390
10. (1619.89 + 1280.45 – 800.35) ÷ 29.67 = ? ÷ 19.88 d) 450
a) 1800 e) 500
b) 1400 15. (229.77 ÷ 1.84) ÷ 22.88 = 37.88 - 34.01 + ?
c) 1600 a) 1
d) 1500 b) 3
e) 2000 c) 9
11. 2301 ÷ 20.01 × 34.99 + 600.01 = ? d) 7
a) 4645 e) 10
Page 540 of 728

Subscribe the Xpress Video Course & Mock Test Package for Bank & Insurance Exams
If there are any suggestions/ errors in our PDFs Feel Free to contact us via this email: admin@exampundit.in
IBPS RRB PO Prelims – Ultra Practice Bundle PDF
16. (21.40 × 4.80) ÷ 2.94 = 117.80 ÷ 2.01 - ? d) 250
a) 34 e) 3139
b) 26 21. 59.99% of 249.98 + 79.99% of 199.99 = ?
c) 4 a) 290
d) 24 b) 310
e) 12 c) 330
17. 67.1 × 451.89 ÷ 403 =? d) 350
a) 80 e) 300
b) 70 22. 117.97 + 189.95 - 229.99 = 65.99 + ?
c) 720 a) 16
d) 750 b) 22
e) 75 c) 28
18. 46324 ÷ 484 = ? d) 32
a) 109 e) 12
b) 81 23. (885.96 + 961.99) ÷ 41.98 + 65.96 = ?
c) 96 a) 140
d) 47 b) 160
e) 15 c) 80
19. 15.002 × 13.897 × 10.034 = ? d) 110
a) 4100 e) 90
b) 4300 24. 391.99 ÷ 13.94 × 4.95 – 130.91 = √?
c) 2500 a) 3
d) 2100 b) 9
e) 3100 c) 27
20. 3038 - 1002 ÷ 20.04 + 51 d) 243
a) 3000 e) 81
b) 4039 25. 24.95 + 26.95 × 3.9 - 18.95 = 17.95 + ?
c) 3039 a) 68
Page 541 of 728

Subscribe the Xpress Video Course & Mock Test Package for Bank & Insurance Exams
If there are any suggestions/ errors in our PDFs Feel Free to contact us via this email: admin@exampundit.in
IBPS RRB PO Prelims – Ultra Practice Bundle PDF
b) 88 30. 3419.95 ÷ 90.04 x 51.07 = ? + 17.05
c) 106 a) 1981
d) 96 b) 2056
e) 118 c) 2011
26. 20.07 x 31.93 - 9.94 = ? x 29.96 d) 1816
a) 16 e) 1921
b) 21
31.
c) 33
a) 250
d) 23
b) 150
e) 8
c) 50
27. 4.97% of 639.96 + 10.04% of 389.95 = ?
d) 175
a) 71
e) 125
b) 61
c) 171 32.
d) 141 a) 5
e) -9 b) 4
28. 24.06 x 7.96 + 47.99 x 21.02 = ? c) 3
a) 1173 d) 6
b) 1190 e) 2
c) 1220
33.
d) 1194
a) 5
e) 1200
b) 8
29. (?)2 + 5.053 = 27.992 - 37.01
c) 4
a) 33
d) 2
b) 25
e) 9
c) 41
34.
d) 45
a) 205
e) 13
Page 542 of 728

Subscribe the Xpress Video Course & Mock Test Package for Bank & Insurance Exams
If there are any suggestions/ errors in our PDFs Feel Free to contact us via this email: admin@exampundit.in
IBPS RRB PO Prelims – Ultra Practice Bundle PDF
b) 195 d) 2680
c) 209 e) 2780
d) 215 39.
e) 200 a) 49
35. b) 37
a) 3400 c) 41
b) 3350 d) 45
c) 3200 e) 33
d) 3300 40.
e) 3250
a) 7
36.
b) 15
a) 1550
c) 2.5
b) 1600
d) 19
c) 1490
e) 10.5
d) 1400
41. 24.98% of 239.89 + 29.83% of 299.98 = ?
e) 1300
a) 120
37.
b) 80
a) 11360
c) 170
b) 11304
d) 180
c) 11500
e) 150
d) 11250
42. ?2 × 8.95 = 199.89 × 17.93
e) None of these
a) 16
38. b) 20
a) 2830 c) 10
b) 2710 d) 12
c) 2850 e) 15
43. 7.98 × 3.94 + 2.98 × 35.98 ÷ 5.96 = ?
Page 543 of 728

Subscribe the Xpress Video Course & Mock Test Package for Bank & Insurance Exams
If there are any suggestions/ errors in our PDFs Feel Free to contact us via this email: admin@exampundit.in
IBPS RRB PO Prelims – Ultra Practice Bundle PDF
a) 40 a) 105
b) 36 b) 110
c) 50 c) 95
d) 46 d) 90
e) 56 e) 85
44. 29.98% of 249.98 ÷ 24.98 ÷ 2.98 = ? 48. (329.98 ÷ 10.98 × 143.98 ÷ 16.16) = ?
a) 3 a) 320
b) 5 b) 240
c) 9 c) 420
d) 11 d) 270
e) 1 e) 180
45. 19.89% of 149.98 + ∛1727.98 – 5.952 = ? 49. 189.99 – 35.96 ÷ 11.93 + 11.96 × 2.93 = ?
a) 12 a) 151
b) 6 b) 161
c) 16 c) 223
d) 22 d) 221
e) 2 e) 233
46. (109.95 × 2.97) ÷ (3.14 × 4.95) = ? 50. √575.96 + √1023.98 – ? × 7.98 = 39.98
a) 12 a) 2
b) 26 b) 6
c) 36 c) 4
d) 22 d) 3
e) 48 e) 8
47. (∛728.98 + 2.93)2 = (6.98)2 + ?

1). Approximation Questions - Solution with Explanation


1. Answer: D) 39.8% of (62.05 × 4.99) + 67.12 = ?
Page 544 of 728

Subscribe the Xpress Video Course & Mock Test Package for Bank & Insurance Exams
If there are any suggestions/ errors in our PDFs Feel Free to contact us via this email: admin@exampundit.in
IBPS RRB PO Prelims – Ultra Practice Bundle PDF
40% of (62 × 5) + 67 ~ ? ? ~ 63 ÷ 20
124 + 67 ~ ? ?~3
? ~ 191 ~ 190 7. Answer: B)
2. Answer: B) ? = (√780 × 4.89) ÷ 1.98
? = 30.41% of 54.5% of 4198 – 92.55 ? ~ (√784 × 5) ÷ 2
? ~ 30% of 55% of 4200 – 93 ? ~ (28 × 5) ÷ 2
? ~ 693 – 93 ? ~ 70
? ~ 600 8. Answer: A)
3. Answer: A) ? = 576.124 – 241.12 + 4.013.011
(343.78 - ?2)= 5351.8 ÷ 24.01 ? ~ 576 – 241 + 43
(344 - ?2) ~ 5352 ÷ 24 ? ~ 576 – 241 + 64
?2 ~ 344 – 223 ? ~ 399 ~ 400
?2 ~ 121 9. Answer: A)
? ~ 11 ? = 17.12 × 14.89 + 121.12
4. Answer: E) ? ~ 17 × 15 + 120
? = ((30.982.02 – 11.981.98) + 24.32)0.49 ? ~ 255 + 120
? ~ ((312 – 122) + 24)0.5 ? ~ 375
? ~ (961 - 144 + 24)0.5 10. Answer: B)
? ~ 8410.5 ~ 29 (1619.89 + 1280.45 – 800.35) ÷ 29.67 = ? ÷ 19.88
5. Answer: B) (1620 + 1280 – 800) ÷ 30 ~ ? ÷ 20
? = 20.91% of (36.2 × (144.01 + 5.95)) ? ~ 70 × 20
? ~ 20% of (36 × (144 + 6)) ? ~ 1400
? ~ 20% of (36 × 150) 11. Answer: D)
? ~ 20% of 5400 2301 ÷ 20.01 × 34.99 + 600.01 = ?
? ~ 1080 ⇒ ? ≈ 2301 ÷ 20 × 35 + 600
6. Answer: D) ⇒ ? ≈ 115 × 35 + 600
? = (41.98 + 21.02) ÷ (18.09 + 1.97) ⇒ ? ≈ 4625
? ~ (42 + 21) ÷ (18 + 2) 12. Answer: B)
Page 545 of 728

Subscribe the Xpress Video Course & Mock Test Package for Bank & Insurance Exams
If there are any suggestions/ errors in our PDFs Feel Free to contact us via this email: admin@exampundit.in
IBPS RRB PO Prelims – Ultra Practice Bundle PDF
(180.20 - 59.74) ÷ 40.04 = 79.81 - 61.72 - ? ⇒ (92 + 118) ÷ 14 = 90 - 15% of ?
We can write the given values as: ⇒ (92 + 118) ÷ 14 = 90 - [(15/100) × ?]
180.20 ≈ 180 and 59.74 ≈ 60 ⇒ 210 ÷ 14 = 90 - [15?/100]
40.04 ≈ 40 and 79.81 ≈ 80 and 61.72 ≈ 62 ⇒ 15 = 90 - (15?/100)
Then, ⇒ (15?/100) = 90 - 15
⇒ (180 - 60) ÷ 40 = 80 - 62 - ? ⇒ (15?/100) = 75
⇒ 120 ÷ 40 = 80 - 62 - ? ⇒ 15? = 75 × 100
⇒ 3 = 80 - 62 - ? ⇒ ? = 7500/15
⇒ 3 = 18 - ? ⇒ ? ≈ 500
⇒ ? = 18 - 3 15. Answer: A)
⇒ ? ≈ 15 (229.77 ÷ 1.84) ÷ 22.88 = 37.88 - 34.01 + ?
13. Answer: A) We can write the given values as:
(27.84 + 31.90) ÷ 14.84 = 97.79 ÷ 13.98 - ? 229.77 ≈ 230 and 1.84 ≈ 2
We can write the given values as: 22.88 ≈ 23 and 37.88 ≈ 38 and 34.01 ≈ 34
27.84 ≈ 28 and 31.90 ≈ 32 ⇒ (230 ÷ 2) ÷ 23 = 38 - 34 + ?
14.84 ≈ 15 and 97.79 ≈ 98 and 13.98 ≈ 14 ⇒ 115 ÷ 23 = 38 - 34 + ?
Then, ⇒ 5 = 38 - 34 + ?
⇒ (28 + 32) ÷ 15 = 98 ÷ 14 - ? ⇒5=4+?
⇒ 60 ÷ 15 = 98 ÷ 14 - ? ⇒?=5-4
⇒4=7-? ⇒?≈1
⇒?=7-4 16. Answer: D)
⇒?≈3 (21.40 × 4.80) ÷ 2.94 = 117.80 ÷ 2.01 - ?
14. Answer: E) We can write the given values as:
(91.87 + 118.10) ÷ 14.20 = 90.08 - 14.88 % of ? 21.40 ≈ 21 and 4.80 ≈ 5
We can write the given values as: 2.94 ≈ 3 and 117.80 ≈ 118 and 2.01 ≈ 2
91.87 ≈ 92 and 118.10 ≈ 118 ⇒ (21 × 5) ÷ 3 = 118 ÷ 2 - ?
14.20 ≈ 14 and 90.08 ≈ 90 and 14.88 ≈ 15 ⇒ 105 ÷ 3 = 118 ÷ 2 - ?
Then, ⇒ 35 = 59 - ?
Page 546 of 728

Subscribe the Xpress Video Course & Mock Test Package for Bank & Insurance Exams
If there are any suggestions/ errors in our PDFs Feel Free to contact us via this email: admin@exampundit.in
IBPS RRB PO Prelims – Ultra Practice Bundle PDF
⇒ ? = 59 - 35 3038 - 1000 ÷ 20 + 51
⇒ ? ≈ 24 = 3038 - 50 + 51
17. Answer: E) = 3039
∴ we can use approximate values of the given numbers. 21. Answer: B)
67.1 ≈ 67 59.99% of 249.98 + 79.99% of 199.99 = ?
451.89 ≈ 450 ⇒ 60% of 250 + 80% of 200 = ?
403 ≈ 402 ⇒ 150 + 160 = ?
So, the given expression becomes: ∴ ? = 310
67 × 450 ÷ 402 22. Answer: E)
= 67 × 450/402 117.97 + 189.95 - 229.99 = 65.99 + ?
= 450/6 = 75 ⇒ 118 + 190 - 230 = 66 + ?
18. Answer: C) ⇒ 308 - 230 = 66 + ?
46324 ≈ 46000 ⇒ 78 = 66 + ?
484 ≈ 500 ⇒ 78 - 66 = ?
46324 ÷ 484 = ? ≈ 46000 ÷ 500 ∴ ? = 12
⇒ ? ≈ 92 ≈ 96 23. Answer: D)
19. Answer: D) (885.96 + 961.99) ÷ 41.98 + 65.96 = ?
We can write the given values as: ⇒ (886 + 962) ÷ 42 + 66 = ?
15.002 ≈ 15 ⇒ 1848 ÷ 42 + 66 = ?
13.897 ≈ 14 ⇒ 44 + 66 = ?
10.034 ≈ 10 ∴ ? = 110
Now, given expression: 24. Answer: E)
15.002 × 13.897 × 10.034 391.99 ÷ 13.94 × 4.95 – 130.91 = √?
≈ 15 × 14 × 10 ⇒ 392 ÷ 14 × 5 – 131 = √?
≈ 2100 ⇒ 28 × 5 – 131 = √?
20. Answer: C) 140 – 131 = √?
∴ approximating 1002 ≈ 1000 and 20.04 ≈ 20, we get the ⇒ 9 = √?
given expression as: ∴ ? = 81
Page 547 of 728

Subscribe the Xpress Video Course & Mock Test Package for Bank & Insurance Exams
If there are any suggestions/ errors in our PDFs Feel Free to contact us via this email: admin@exampundit.in
IBPS RRB PO Prelims – Ultra Practice Bundle PDF
25. Answer: D)
24.95 + 26.95 × 3.9 - 18.95 = 17.95 + ?
⇒ 25 + 27 × 4 - 19 = 18 + ?
⇒ 25 + 108 - 19 = 18 + ?
⇒ 133 - 19 = 18 + ? 32. Answer: D)

⇒ 114 = 18 + ?
∴ ? = 96
26. Answer: B)
? = (20 x 32 - 10)/30
= (640 - 10)/30 33. Answer: B)
= 630/30 = 21
27. Answer: A)
? ≈ (5 x 640/100) + (10 x 390/100)
= (3200/100) + (3900/100)
= 32 + 39 = 71
28. Answer: E) 34. Answer: C)

? ≈ 24 x 8 + 48 x 21
= 192 + 1008
= 1200 35. Answer: B)

29. Answer: B)
(?)2 ≈ 282 - 37 - 53
= 784 - 37 - 125 = 622
So, ? ≈ 25
30. Answer: E)
36. Answer: C)
? = 3420 ÷ 90 x 51 - 17
= 38 x 51 - 17
= 1938 - 17 = 1921
31. Answer: B)
Page 548 of 728

Subscribe the Xpress Video Course & Mock Test Package for Bank & Insurance Exams
If there are any suggestions/ errors in our PDFs Feel Free to contact us via this email: admin@exampundit.in
IBPS RRB PO Prelims – Ultra Practice Bundle PDF
⇒ 60 + 90 = ?
⇒ 150 = ?
42. Answer: B)
?2 × 8.95 = 199.89 × 17.93
⇒ ?2 × 9 = 200 × 18
37. Answer: B) ⇒ ?2 × 9 = 200 × 18
3453.9 + 6119.8 + 1729.9 ~? ⇒ ?2 × 9 = 3600
?~3454+6120+1730 ⇒ ?2 = 3600/9
?~ 11304 ⇒ ?2 = 400
38. Answer: E) ⇒ ? = 20
43. Answer: C)
7.98 × 3.94 + 2.98 × 35.98 ÷ 5.96 = ?
⇒ 8 × 4 + 3 × 36 ÷ 6 = ?
⇒8×4+3×6=?
⇒ 32 + 18 = ?

39. Answer: D) ⇒ ? = 50
44. Answer: E)
29.98% of 249.98 ÷ 24.98 ÷ 2.98 = ?
⇒ 30% of 250 ÷ 25 ÷ 3 = ?
⇒ 75 ÷ 25 ÷ 3 = ?
40. Answer: B)
⇒3÷3=?
⇒?=1
45. Answer: B)
19.89% of 149.98 + ∛1727.98 – 5.952 = ?
41. Answer: E) ⇒ 20% of 150 + ∛1728 – 62 = ?
24.98% of 239.89 + 29.83% of 299.98 = ?
⇒ 30 + 12 – 36 = ?
⇒ 25% of 240 + 30% of 300 = ?
⇒ 42 – 36 = ?
⇒ 240 × (25/100) + 300 × (30/100) = ?
⇒6=?
Page 549 of 728

Subscribe the Xpress Video Course & Mock Test Package for Bank & Insurance Exams
If there are any suggestions/ errors in our PDFs Feel Free to contact us via this email: admin@exampundit.in
IBPS RRB PO Prelims – Ultra Practice Bundle PDF
46. Answer: D) ⇒ ? = 270
(109.95 × 2.97) ÷ (3.14 × 4.95) = ? 49. Answer: C)
⇒ (110 × 3) ÷ (3 × 5) = ? 189.99 – 35.96 ÷ 11.93 + 11.96 × 2.93 = ?
⇒ 330 ÷ 15 = ? ⇒ 190 – 36 ÷ 12 + 12 × 3 = ?
⇒ 22 = ? ⇒ 190 – 3 + 12 × 3 = ?
47. Answer: C) ⇒ 190 – 3 + 36 = ?
(∛728.98 + 2.93)2 = (6.98)2 + ? ⇒ 226 – 3 = ?
⇒ (∛729 + 3)2 = (7)2 + ? ⇒ ? = 223
⇒ (9 + 3)2 = (7)2 + ? 50. Answer: A)
⇒ (12)2 = 49 + ? √575.96 + √1023.98 – ? × 7.98 = 39.98
⇒ 144 = 49 + ? ⇒ √576 + √1024 – ? × 8 = 40
⇒ ? = 95 ⇒ 24 + 32 – ? × 8 = 40
48. Answer: D) ⇒ 56 – ? × 8 = 40
(329.98 ÷ 10.98 × 143.98 ÷ 16.16) = ? ⇒ 56 – 40 = ? × 8
⇒ (330 ÷ 11 × 144 ÷ 16) = ? ⇒ 16 = ? × 8
⇒ (30 × 9) = ? ⇒?=2

2). Simplification Questions


Directions: What value will come in place of question a) 792
mark (?) in the following questions? b) 784
1. 2628 ÷ 73 = ? + 30 c) 798
a) 10 d) 782
b) 0 e) 794
c) 6 3. 31250.23 x 31250.17 = ?
d) 8 a) 125
e) -2 b) 1
2. 38/270 x 45/266 x ? = 19 c) 5

Page 550 of 728

Subscribe the Xpress Video Course & Mock Test Package for Bank & Insurance Exams
If there are any suggestions/ errors in our PDFs Feel Free to contact us via this email: admin@exampundit.in
IBPS RRB PO Prelims – Ultra Practice Bundle PDF
d) 25 b) 967
e) 625 c) 973
4. √729 + ? = 4 1/5 of 25 d) 943
a) 81 e) 961
b) 60 9. 20/2 + 12 x 5 = ? + 225/5 - 16
c) 78 a) 51
d) 82 b) 35
e) 90 c) 41
5. 29 x 29 - 1 = ? x 15 d) 47
a) 51 e) 31
b) 66 10. 16.66% of 192 + 77.77% of 252 = ?
c) 47 a) 228
d) 73 b) 234
e) 56 c) 225
6. 492 + 472 + 402 = ? d) 240
a) 6150 e) 216
b) 6120 11. 16/15 of 21/28 of 321.5 = ?
c) 6130 a) 257.2
d) 6210 b) 251.4
e) 6190 c) 264.5
7. 115 x 7 = ? - 180 x 7 d) 224.3
a) 2145 e) None of these
b) 2105 12. √4624 + √5776 = (?)2
c) 2125 a) 12
d) 2115 b) 22
e) 2065 c) 32
8. 312 - 2401¼ + 343⅓ = ? d) 18
a) 982 e) 28
Page 551 of 728

Subscribe the Xpress Video Course & Mock Test Package for Bank & Insurance Exams
If there are any suggestions/ errors in our PDFs Feel Free to contact us via this email: admin@exampundit.in
IBPS RRB PO Prelims – Ultra Practice Bundle PDF
13. √400 - √256 + √529 = ? d) 9
a) 12 e) 81
b) 22 18. (37)2 - (32)2 - (29)2 = ?
c) 27 a) -474
d) 2 b) -500
e) 37 c) -473
14. (?)2 + 93 = 292 - 12 d) -475
a) 26 e) -496
b) 6 19. ∛125 + ∛8000 + ∛1331 = ?
c) 14 a) 36
d) 22 b) 28
e) 10 c) 42
15. 33 x 42 + 8 x 21 = ? d) 39
a) 1567 e) 29
b) 1576 20. √121 + ? = 2 1/4 of 52
c) 1555 a) 94
d) 1544 b) 106
e) 1554 c) 88
16. 1610 ÷ 35 = ? + 11 d) 92
a) 32 e) 120
b) 35 21. 132 ÷ 11 - 15 × 8 + 128 = ?
c) 36 a) 10
d) 39 b) 20
e) 31 c) 25
17. 2430.39 x 2430.41 = ? d) 15
a) 27 e) 30
b) 1 22. 45% of 300 - ? = 27 × 40 ÷ 9
c) 3 a) 12
Page 552 of 728

Subscribe the Xpress Video Course & Mock Test Package for Bank & Insurance Exams
If there are any suggestions/ errors in our PDFs Feel Free to contact us via this email: admin@exampundit.in
IBPS RRB PO Prelims – Ultra Practice Bundle PDF
b) 22 27. 59/576 x 72/413 x ? = 13
c) 15 a) 730
d) 25 b) 728
e) 35 c) 732
23. 65% of 200 + 35% of 400 = ? d) 722
a) 270 e) 724
b) 310 28. √2500 - √484 - √1936 = ?
c) 350 a) -10
d) 240 b) -13
e) 210 c) -16
24. 430 - 15 × 8 + 43 - 13 = ?2 d) -9
a) 21 e) -15
b) 29 29. 160.04 x 160.46 = ?
c) 31 a) 256
d) 19 b) 1
e) 11 c) 16
25. 336 ÷ 24 × ∛343 + (7)2 = ? d) 64
a) 151 e) 4
b) 157 30. (11)2 - (17)2 - (22)2 = ?
c) 161 a) -650
d) 167 b) -657
e) 147 c) -644
26. 1155 ÷ 35 = ? + 48 d) -652
a) -15 e) -641
b) -22 31. ∛9261 - ∛125 + ∛8000 = ?
c) -14 a) 36
d) -21 b) 29
e) -23 c) 30
Page 553 of 728

Subscribe the Xpress Video Course & Mock Test Package for Bank & Insurance Exams
If there are any suggestions/ errors in our PDFs Feel Free to contact us via this email: admin@exampundit.in
IBPS RRB PO Prelims – Ultra Practice Bundle PDF
d) 31 b) -36
e) 40 c) -35
32. (?)2 + 93 = 292 - 12 d) -39
a) 22 e) -37
b) 10 37. 320.19 x 320.01 = ?
c) 26 a) 1
d) 14 b) 2
e) 30 c) 4
33. ?/4 of √225 = 45/? of ∛1728 d) 8
a) 33 e) 16
b) 36 38. ∛4096 + ∛1000 - ∛27 = ?
c) 27 a) 23
d) 12 b) 19
e) 15 c) 21
34. 165 x 15 = ? - 210 x 10 d) 26
a) 4655 e) 25
b) 4605 39. √1681 + ? = 4 4/5 of 100
c) 4525 a) 439
d) 4615 b) 426
e) 4575 c) 425
35. 22.22% of 270 + 77.77% of 342 = ? d) 450
a) 350 e) 453
b) 338 40. 23 x 26 - 38 = ? x 40
c) 347 a) 1
d) 311 b) 11
e) 326 c) 13
36. √5329 - √4761 - √1369 = ? d) 14
a) -33 e) -5
Page 554 of 728

Subscribe the Xpress Video Course & Mock Test Package for Bank & Insurance Exams
If there are any suggestions/ errors in our PDFs Feel Free to contact us via this email: admin@exampundit.in
IBPS RRB PO Prelims – Ultra Practice Bundle PDF
41. 142 – 152 + 182 - 35 =? d) 33.723
a) 250 e) 35.443
b) 260 46. ? × (242 ÷ 22) + ? × (175 ÷ 35) = 320
c) 330 a) 15
d) 290 b) 10
e) 310 c) 12
42. 365 + 72 % of 850 = 900 +? d) 25
a) 84 e) 20
b) 97 47. 15 × 20 + 12 × 15 + 18 × 15 + 15 × 10 = ?
c) 104 a) 450
d) 77 b) 600
e) 112 c) 750
43. 6594 ÷ 21 × (0.3)2 =? d) 400
a) 28.26 e) 900
b) 18.46 48. (24% of 600) ÷ (25% of 576) = ?
c) 15.66 a) 1
d) 22.86 b) 4
e) 25.36 c) 1.5
44. ? ÷ √162 = √128 ÷? d) 2
a) 169 e) 3
b) 144 49. 30% of 60% of 5000 = ? × 45
c) 72 a) 30
d) 16 b) 32
e) 12 c) 18
45. 16.189 + 31.153 - 23.899 + 7.65 =? d) 24
a) 30.113 e) 20
b) 36.063 50. 152 + ?2 + 53 + (?2 – 102) = 1050
c) 31.093 a) 25
Page 555 of 728

Subscribe the Xpress Video Course & Mock Test Package for Bank & Insurance Exams
If there are any suggestions/ errors in our PDFs Feel Free to contact us via this email: admin@exampundit.in
IBPS RRB PO Prelims – Ultra Practice Bundle PDF
b) 12 d) 20
c) 15 e) 17

2). Simplification Questions - Solutions with Explanation

1. Answer: C) ? = 115 x 7 + 180 x 7


? = 2628 ÷ 73 - 30 = 805 + 1260 = 2065
= 36 - 30 8. Answer: E)
=6 312 = 961, 74 = 2401, 73 = 343
2. Answer: C) ? = 961 - 7 + 7
? = (19 x 266 x 270)/(38 x 45) = 961
= 19 x (266/38) x (270/45) = 19 x 7 x 6 = 798 9. Answer: C)
3. Answer: D) ? = (20/2) + 12 x 5 - (225/5) + 16
? = 3125(0.23 + 0.17) = 31250.40 = 31252/5 = 10 + 60 - 45 + 16
Since, 31251/5 = 5, ? = 52 = 25 =41
4. Answer: C) 10. Answer: A)
? = 25 x (21/5) - √729 We note that 16.66% = 1/6, 77.77% = 7/9
= 5 x 21 - 27 So, ? = 192 x (1/6) + 252 x (7/9)
= 105 - 27 = 78 = (6 x 32) x (1/6) + (9 x 28) x (7/9)
5. Answer: E) = (1 x 32) + (7 x 28) = 228
? = (29 x 29 - 1)/15 11. Answer: A)
= (841 - 1)/15 16/15 of 21/28 of 321.5 = ?
= 840/15 = 56 16/15 × 3/4 × 321.5 = ?
6. Answer: D) 4/5 × 321.5 = ?
? = 492 + 472 + 402 ? = 257.2
= 2401 + 2209 + 1600 = 6210 12. Answer: A)
7. Answer: E) √4624 + √5776 = (?)2
Page 556 of 728

Subscribe the Xpress Video Course & Mock Test Package for Bank & Insurance Exams
If there are any suggestions/ errors in our PDFs Feel Free to contact us via this email: admin@exampundit.in
IBPS RRB PO Prelims – Ultra Practice Bundle PDF
68 + 76 = ?2 ? = ∛125 + ∛8000 + ∛1331
? = √144 = 5 + 20 + 11 = 36
? = 12 20. Answer: B)
13. Answer: C) ? = 52 x (9/4) - √121
? = √400 - √256 + √529 = 13 x 9 - 11
= 20 - 16 + 23 = 117 - 11 = 106
= 27 21. Answer: B)
14. Answer: E) 132 ÷ 11 - 15 × 8 + 128 = ?
(?)2 = 292 - 12 - 93 ⇒ 12 - 15 × 8 + 128 = ?
= 841 - 12 - 729 = 100 ⇒ 12 - 120 + 128 = ?
So, ? = 10 ⇒ 12 - 120 + 128 = ?
15.Answer: E) ⇒ 140 - 120 = ?
? = 33 x 42 + 8 x 21 ∴ ? = 20
= 1386 + 168 22. Answer: C)
= 1554 45% of 300 - ? = 27 × 40 ÷ 9
16. Answer: B) ⇒ 135 - ? = 120
? = 1610 ÷ 35 - 11 ⇒ 135 - 120 = ?
= 46 - 11 ∴ ? = 15
= 35 23. Answer: A)
17. Answer: E) 65% of 200 + 35% of 400 = ?
(0.39 + 0.41) 0.80 4/5
? = 243 = 243 = 243 ⇒ 130 + 140 = ?
1/5 4
Since, 243 = 3, ? = 3 = 81 ∴ ? = 270
18. Answer: E) 24. Answer: D)
2 2 2
? = 37 - 32 - 29 430 - 15 × 8 + 43 - 13 = ?2
= 1369 - 1024 - 841 ⇒ 430 - 120 + 64 - 13 = ?2
= -496 ⇒ 494 -120 - 13 = ?2
19. Answer: A) ⇒ 494 - 133 = ?2
53 = 125, 203 = 8000, 113 = 1331 ⇒ 361 = ?2
Page 557 of 728

Subscribe the Xpress Video Course & Mock Test Package for Bank & Insurance Exams
If there are any suggestions/ errors in our PDFs Feel Free to contact us via this email: admin@exampundit.in
IBPS RRB PO Prelims – Ultra Practice Bundle PDF
∴ ? = 19 32. Answer: B)
25. Answer: E) (?)2 = 292 - 12 - 93
336 ÷ 24 × ∛343 + (7)2 = ? = 841 - 12 - 729 = 100
⇒ 14 × 7 + 49 = ? So, ? = 10
⇒ 98 + 49 = ? 33. Answer: D)
∴ ? = 147 ?2 = (4 x 45 x ∛1728)/(√225)
26. Answer: A) = 4 x 45 x 12/15
? = 1155 ÷ 35 - 48 = 144 = 122
= 33 - 48 So, ? = 12
= -15 34. Answer: E)
27. Answer: B) ? = 165 x 15 + 210 x 10
? = (13 x 413 x 576)/(59 x 72) = 2475 + 2100 = 4575
= 13 x (413/59) x (576/72) = 13 x 7 x 8 = 728 35. Answer: E)
28. Answer: C) We note that 22.22% = 2/9, 77.77% = 7/9
502 = 2500, 222 = 484, 442 = 1936 So, ? = 270 x (2/9) + 342 x (7/9)
? = √2500 - √484 - √1936 = (9 x 30) x (2/9) + (9 x 38) x (7/9)
= 50 - 22 - 44 = -16 = (2 x 30) + (7 x 38) = 326
29. Answer: E) 36. Answer: A)
? = 16(0.04 + 0.46) = 160.50 = 161/2 732 = 5329, 692 = 4761, 372 = 1369
Since, 161/2 = 4, ? = 41 = 4 ? = √5329 - √4761 - √1369
30. Answer: D) = 73 - 69 - 37 = -33
? = 112 - 172 - 222 37. Answer: B)
= 121 - 289 - 484 ? = 32(0.19 + 0.01) = 320.20 = 321/5
= -652 Since, 321/5 = 2, ? = 21 = 2
31. Answer: A) 38. Answer: A)
213 = 9261, 53 = 125, 203 = 8000 163 = 4096, 103 = 1000, 33 = 27
? = ∛9261 - ∛125 + ∛8000 ? = ∛4096 + ∛1000 - ∛27
= 21 - 5 + 20 = 36 = 16 + 10 - 3 = 23
Page 558 of 728

Subscribe the Xpress Video Course & Mock Test Package for Bank & Insurance Exams
If there are any suggestions/ errors in our PDFs Feel Free to contact us via this email: admin@exampundit.in
IBPS RRB PO Prelims – Ultra Practice Bundle PDF
39. Answer: A) 31.093 =?
? = 100 x (24/5) - √1681 46. Answer: E)
= 20 x 24 - 41 ? × (242 ÷ 22) + ? × (175 ÷ 35) = 320
= 480 - 41 = 439 ? × (11 + 5) = 320
40. Answer: D) ? = 320 ÷ 16
? = (23 x 26 - 38)/40 ? = 20
= (598 - 38)/40 47. Answer: E)
= 560/40 = 14 15 × 20 + 12 × 15 + 18 × 15 + 15 × 10 = ?
41. Answer: B) 15 × (20 + 12 + 18 + 10) = ?
142 – 152 + 182 - 35 =? 15 × 60 = ?
196 - 225 + 324 - 35 =? ? = 900
260 =? 48. Answer: A)
42. Answer: D) (24% of 600) ÷ (25% of 576) = ?
365 + 72 % of 850 = 900 +? (0.24 × 600) ÷ (0.25 × 576) = ?
365 + 612 = 900 +? ? = 144 ÷ 144
977 = 900 +? ?=1
77 =? 49. Answer: E)
43. Answer: A) 30% of 60% of 5000 = ? × 45
6594 ÷ 21 × (0.3)2 =? 0.30 × 0.60 × 5000 = ? × 45
314 × 0.3 ×0.3 =? ? = 900 ÷ 45
28.26 =? ? = 20
44. Answer: E) 50. Answer: D)
? ÷√162 = √128 ÷? 152 + ?2 + 53 + (?2 – 102) = 1050
?2 =√(128 × 162) 2 × ?2 = 1050 – 225 – 125 + 100
?2 = √20736 ?2 = 800/2
?2 = 144 ?2 = 400
45. Answer: C) ? = ± 20
16.189 + 31.153 - 23.899 + 7.65 =?
Page 559 of 728

Subscribe the Xpress Video Course & Mock Test Package for Bank & Insurance Exams
If there are any suggestions/ errors in our PDFs Feel Free to contact us via this email: admin@exampundit.in
IBPS RRB PO Prelims – Ultra Practice Bundle PDF

3). Missing Number series


Directions (1-5): What will come in the place of 5) 91, 174, 321, 593,? , 1912
question mark? a) 1083
1) 123, 139, 175, 239, 339,? b) 1071
a) 482 c) 1081
b) 473 d) 1091
c) 483 e) 1061
d) 493 6) 178, 182, 174, 186, 170,?
e) 503 a) 192
2) 324, 341, 392, 477, 596,? b) 160
a) 729 c) 174
b) 744 d) 190
c) 739 e) 194
d) 749 7) 4000 2000 500 250 ?
e) None of the above a) 62.25
3) 214, 207, 193, 172, 144,? b) 61.75
a) 99 c) 31.25
b) 109 d) 125
c) 89 e) 62.5
d) 119 8) 217 ? 247 277 317 367
e) 129 A) 227
4) 64, 16, 20, 45, 146.25,? B) 237
a) 621.5625 C) 222
b) 621.3125 D) 232
c) 621.0125 E) 247
d) 623.125 9) 16 8 8 ? 64
e) None of the above a) 12
Page 560 of 728

Subscribe the Xpress Video Course & Mock Test Package for Bank & Insurance Exams
If there are any suggestions/ errors in our PDFs Feel Free to contact us via this email: admin@exampundit.in
IBPS RRB PO Prelims – Ultra Practice Bundle PDF
b) 16 e) None of these
c) 24 14. 1672, ?, 1765, 1423, 1936, 1208
d) 32 a) 1558
e) 40 b) 1448
10) 344 ? 126 63 28 7 c) 1458
a) 215 d) 1548
b) 217 e) 1468
c) 218 15. 42, 122, ?, 2416, 14492, 101440
d) 219 a) 524
e) 220 b) 448
11) 34 43 61 88 124 ? c) 484
a) 196 d) 464
b) 144 e) 564
c) 169 16) 13, 26, 78, 312, ?, 9360
d) 225 a) 1020
e) 179 b) 1980
c) 1560
12. 472, 489, 508, ?, 560, 591 d) 1990
a) 532 e) 1120
b) 531 17) 11, 26, 47, 74, 107, ?
c) 533 a) 146
d) 534 b) 128
e) 535 c) 132
13. 1156, 1444, 1849, 2401, ? , 4096 d) 158
a) 3136 e) 169
b) 3025 18) 17, 81, 131, 169, 197, ?
c) 3249 a) 206
d) 2916 b) 229
Page 561 of 728

Subscribe the Xpress Video Course & Mock Test Package for Bank & Insurance Exams
If there are any suggestions/ errors in our PDFs Feel Free to contact us via this email: admin@exampundit.in
IBPS RRB PO Prelims – Ultra Practice Bundle PDF
c) 217 a) 650
d) 201 b) 346
e) 218 c) 745
19) 18, 22, 34, 70, 178, ? d) 542
a) 418 e) 645
b) 502 24. 5, 12, 9, 9, 13, ?, 17
c) 398 a) 13
d) 385 b) 6
e) 391 c) 17
20) 16, 40, 140, 630, ?, 22522.5 d) 9
a) 3485 e) 5
b) 3380 25. 142, 134, 123, 107, 84, ?
c) 3930 a) 55
d) 3465 b) 32
e) 3845 c) 56
21. 12, 7, 9 , 21 , 88 , 709 , ? d) 52
a) 11350 e) 64
b) 12360 26) 1681, 1369, 961, 841, 529, ?
c) 11250 a) 289
d) 11360 b) 361
e) 15320 c) 576
22. 15, 18 , 16 , 19 , 17 , 20 , ? d) 400
a) 23 e) 169
b) 22 27) 36, 69, 203, 807, 4029, ?
c) 18 a) 26147
d) 20 b) 21467
e) 17 c) 24167
23. 5, 6, 10, 33, 128, ?, 3864 d) 21147
Page 562 of 728

Subscribe the Xpress Video Course & Mock Test Package for Bank & Insurance Exams
If there are any suggestions/ errors in our PDFs Feel Free to contact us via this email: admin@exampundit.in
IBPS RRB PO Prelims – Ultra Practice Bundle PDF
e) 24617 c) 96
28)?, 122, 125, 141, 266, 1562 d) 102
a) 121 e) 92
b) 120 33) 23, 40, 94, 189, 329, ?
c) 119 a) 518
d) 122 b) 498
e) 117 c) 483
29) 266, 284, 423, ?, 1328, 2286 d) 576
a) 713 e) 550
b) 623 34) 19, 30, 54, 93, 149, ?
c) 667 a) 274
d) 731 b) 224
e) 633 c) 298
30) 12, 48, 130, 268, 408, ? d) 219
a) 412 e) 239
b) 416 35) 11, 46, 187, ?, 3013, 12058
c) 420 a) 729
d) 424 b) 731
e) 428 c) 752
31) 8, 72, 193, ?, 678, 1078 d) 745
a) 398 e) 746
b) 372 36) -20, -84, -415, ?, - 17465, -139728
c) 343 a) -2496
d) 364 b) 2496
e) 389 c) -2596
32) 11, 14, 22, 37, 61, ? d) 2596
a) 90 e) -2498
b) 98 37) 4, 60, 325, 1316, ?, 7918, 7919
Page 563 of 728

Subscribe the Xpress Video Course & Mock Test Package for Bank & Insurance Exams
If there are any suggestions/ errors in our PDFs Feel Free to contact us via this email: admin@exampundit.in
IBPS RRB PO Prelims – Ultra Practice Bundle PDF
a) 3958 e) None of these
b) 3957 42) 29 41 55 71 89 (?)
c) 3940 a) 129
d) 3945 b) 119
e) 3959 c) 109
38) 2, 9, ?, 105, 436, 2195 d) 112
a) 28 e) 117
b) 29 43) 60 120 30 180 22.5 (?)
c) 30 a) 105
d) 31 b) 205
e) 35 c) 215
39) 8544, 1420, 280, ?, 18, 5 d) 225
a) 22 e) None of these
b) 33 44) 10 11 13 19 (?) 163
c) 55 a) 43
d) 77 b) 53
e) 66 c) 63
40) 7, 38.5, 192.5, 866.25, 3465, ? d) 73
a) 12117.5 e) None of these
b) 12111.5 45) 11 12 (?) 81 328 1645
c) 12129.5 a) 36
d) 12127.5 b) 46
e) 12128.5 c) 20
41) 7 19 34 52 73 (?) d) 32
a) 87 e) None of these
b) 97 46) 3, 10, 32, 111, 460, ?
c) 107 a) 2215
d) 100 b) 2320
Page 564 of 728

Subscribe the Xpress Video Course & Mock Test Package for Bank & Insurance Exams
If there are any suggestions/ errors in our PDFs Feel Free to contact us via this email: admin@exampundit.in
IBPS RRB PO Prelims – Ultra Practice Bundle PDF
c) 2310 e) 102
d) 2314 49) 58, 128, 260, 442, 714, ?
e) 2315 a) 1129
47) 40, 48, 60, 78, ? , 147.42 b) 1135
a) 105.5 c) 1056
b) 106.5 d) 1133
c) 105.3 e) 1115
d) 106.3 50) 999, 1647, 2457, 3429, 4563, ?
e) 112.3 a) 5868
48) 16, 19, 24, 33, 50, ? b) 5879
a) 83 c) 5869
b) 99 d) 5881
c) 66 e) 5859
d) 77

3). Missing Number series - Solutions and Explanations:

1) Answer: C 324 341 392 477


596 749
123 139 175 239
339 483 17 51 85 119
153
16 36 64 100 144
1*17 3*17 5*17 7*17
9*17
42 62 82 102
122 3) Answer: B

2) Answer: D 214 207 193 172


144 109

Page 565 of 728

Subscribe the Xpress Video Course & Mock Test Package for Bank & Insurance Exams
If there are any suggestions/ errors in our PDFs Feel Free to contact us via this email: admin@exampundit.in
IBPS RRB PO Prelims – Ultra Practice Bundle PDF
-7 -14 -21 -28 250/4 = 62.5
-35
8) Answer: A
4) Answer: A
217 + 10 = 227
64*0.25 = 16
227 + 20 = 247
16*1.25= 20
247 + 30 = 277
20*2.25=45

45*3.25= 146.25 277 + 40 = 317

146.25*4.25= 621.5625 317 + 50 = 367

5) Answer: C 9) Answer: B

91 174 321 593 16 x 0.5 = 8


1081 1912
8x1=8
83 147 272 488
831 8 x 2 = 16

64 125 216 16 x 4 = 64
343
10) Answer: A
6) Answer: D
344 = 7³ +1
178 182 174 186
170 190 215 = 6³ - 1

+4 -8 +12 -16 126 = 5³ + 1


+20
63 = 4³ - 1
7) Answer: E
28 = 3³ + 1
4000/2 = 2000
7 = 2³ - 1
2000/4 = 500
11) Answer: C
500/2 = 250
34 + 9 = 43
Page 566 of 728

Subscribe the Xpress Video Course & Mock Test Package for Bank & Insurance Exams
If there are any suggestions/ errors in our PDFs Feel Free to contact us via this email: admin@exampundit.in
IBPS RRB PO Prelims – Ultra Practice Bundle PDF
43 + 18 = 61 15. Answer: C

61 + 27 = 88 42 122 484 2416


14492 101440
88 + 36 = 124
*3-4 *4-4 *5-4 *6-4
124 + 45 = 169 *7-4

16) Answer: C
12. Answer: B
13 * 2 = 26
Given series is,
26 * 3 = 78
472 489 508 531
78 * 4 = 312
560 591
312 * 5 = 1560
17 19 23 29
31 1560 * 6 = 9360
Difference is prime number series, 17) Answer: A
13. Answer: A 11 26 47 74 107 146
1156 1444 1849 2401 15 21 27 33 39
3136 4096
18) Answer: C
342 382 432 492 562
642 17 + 82+ 0 = 81

4 5 6 7 8 81 + 72+ 1 = 131

14. Answer: D 131 + 62+ 2 = 169

1672 1548 1765 1423 169 + 52+ 3 = 197


1936 1208
197 + 42+ 4 = 217
-124 +217 -342 +513
17 81 131 169 197 217
-728
64 50 38 28 20
53-1 63+1 73-1 83+1
93-1 -14 -12 -10 -8

Page 567 of 728

Subscribe the Xpress Video Course & Mock Test Package for Bank & Insurance Exams
If there are any suggestions/ errors in our PDFs Feel Free to contact us via this email: admin@exampundit.in
IBPS RRB PO Prelims – Ultra Practice Bundle PDF
19) Answer: B 6*2 – 2 = 10
10*3 + 3 = 33
18 + 4 = 22 33*4 – 4 = 128
22+ 12 = 34 128*5 + 5 = 645
645*6 – 6 = 3864
34 + 36 = 70
24.Answer: B
70 + 108 = 178
First:
178 + 324 = 502 5+4=9
9 + 4 = 13
20) Answer: D 13 + 4 =17
Second:
16 * 2.5 = 40
12 - 3 = 9
40 * 3.5 = 140 9–3=6

140*4.5 = 630 25. Answer: D


630*5.5 = 3465 142 134 123 107
84 52
3465*6.5 = 22522.5 -8 -11 -16 -23
- 32
21. Answer: A
3 5 7
12 * 0.5 +1 = 7
9
7*1+2=9
26) Answer: B
9 * 2 + 3 = 21
21 * 4 + 4 = 88 Given series is
88 * 8 + 5 = 709
709 *16 + 6 = 11350 1681 1369 961 841 529 361
22. Answer: C
412 372 312 292 232 192
15 +3 =18
18 – 2 = 16 Prime number squares
16 + 3 = 19
19 – 2 = 17 27) Answer: C
17 + 3 = 20 36*2-3 = 69
20 – 2 =18
69*3-4= 203
23. Answer: E
203*4-5=807
5*1 + 1 = 6
Page 568 of 728

Subscribe the Xpress Video Course & Mock Test Package for Bank & Insurance Exams
If there are any suggestions/ errors in our PDFs Feel Free to contact us via this email: admin@exampundit.in
IBPS RRB PO Prelims – Ultra Practice Bundle PDF
807*5-6=4029 408 * 1 + 4 = 412

4029*6-7=24167 31) Answer: E

28Answer: A 8 + 82 = 72

Given series is 72 + 112 = 193

121 122 125 141 266 193 + 142 = 389


1562
389 + 172 = 678
1 3 16 125
1296 678 + 202 = 1078

20 31 42 53 32) Answer: C
64 11 + (3 * 1) = 14
Therefore, 122- (20) =121 14 + (4 * 2) = 22
29) Answer: D 22 + (5 * 3) = 37
Given series is 37 + (6 * 4) = 61
266 284 423 731 61 + (7 * 5) = 96
1328 2286
11 14 22 37 61 ? (96)
18 139 308 597
958 +3 +8 +15 +24 +35

112 132 172 192 +5 +7 +9 +11

Therefore 423+308= 731 33) Answer: A

30)Answer: A 23 + (1 * 17) = 40

12 * 3 + 12 = 48 40 + (3 * 18) = 94

48 * 2.5 + 10 = 130 94 + (5 * 19) = 189

130 * 2 + 8 = 268 189 + (7 * 20) = 329

268 * 1.5 + 6 = 408 329 + (9 * 21) = 518

Page 569 of 728

Subscribe the Xpress Video Course & Mock Test Package for Bank & Insurance Exams
If there are any suggestions/ errors in our PDFs Feel Free to contact us via this email: admin@exampundit.in
IBPS RRB PO Prelims – Ultra Practice Bundle PDF
34) Answer: B 4 𝑥 6 + 62 = 60

19 + (11 * 1) = 30 60 𝑥 5 + 52 = 325

30 + (12 * 2) = 54 325 𝑥 4 + 42 = 1316

54 + (13 * 3) = 93 1316 𝑥 3 + 32 = 3957


93 + (14 * 4) = 149 3957 𝑥 2 + 22 = 7918
149 + (15 * 5) = 224 7918 𝑥 1 + 12 = 7919
19 30 54 93 149 ? (224) 38) Answer: C
+11 +24 +39 +56 +75 2 x 1 +7 x 1 = 9
+13 +15 +17 +19 9 x 2 + 6 x 2 =30
35) Answer: C 30 x 3 + 5 x 3 = 105
11 * 4 + 2 = 46 105 x 4 + 4 x 4= 436
46 * 4 + 3 = 187 436 x 5 + 3 x 5 = 2195
187 * 4 + 4 = 752 39) Answer: E
752 * 4 + 5 = 3013 1420 x 6 + 24 = 8544
3013 * 4 + 6 = 12058 280 x 5 +20 = 1420
36) Answer: A 66 x 4 + 16 = 280
-20 x 4 - 4 = -84 18 x 3 + 12 = 66
-84 x 5 + 5 = -415 5 x 2 + 8 = 18
-415 x 6 - 6 = -2496 40) Answer: D
-2496 x 7 +7 = -17465 7 x 5.5 = 38.5
-17465 x 8 - 8 = -139728 38.5 x 5 = 192.5
37) Answer: B 192.5 x 4.5 = 866.25

Page 570 of 728

Subscribe the Xpress Video Course & Mock Test Package for Bank & Insurance Exams
If there are any suggestions/ errors in our PDFs Feel Free to contact us via this email: admin@exampundit.in
IBPS RRB PO Prelims – Ultra Practice Bundle PDF
866.25 x 4 = 3465 22.5 x 10 = 225

3465 x 3.5 = 12127.5 44) Answer: A

41) Answer: B 10 + 1! = 11

7 + 12 = 19 11 + 2! = 13

19 + 15 = 34 13 + 3! = 19

34 + 18 = 52 19 + 4! = 43

52 + 21 = 73 43 + 5! = 163

73 + 24 = 97 45) Answer: E

42) Answer: C 11 x 1 + 1 = 12

5² + 4 = 29 12 x 2 + 2 = 26

6² + 5 = 41 26 x 3 + 3 = 81

7² + 6 = 55 81 x 4 + 4 = 328

8² + 7 = 71 328 x 5 + 5 = 1645

9² + 8 = 89 46) Answer: E

10² + 9 = 109 3 x 1 + (1 x 7) = 10

29 41 55 71 89 (?)109 10 x 2 + (2 x 6) = 32

+12 +14 +16 +18 +20 32 x 3 + (3 x 5) = 111

43) Answer: D 111 x 4 + (4 x 4) = 460

60 x 2 = 120 460 x 5 + (5 x 3) = 2315

120 ÷ 4 = 30 47) Answer: C

30 x 6 = 180 40 x 1.2 = 48

180 ÷ 8 = 22.5 48 x 1.25 = 60

Page 571 of 728

Subscribe the Xpress Video Course & Mock Test Package for Bank & Insurance Exams
If there are any suggestions/ errors in our PDFs Feel Free to contact us via this email: admin@exampundit.in
IBPS RRB PO Prelims – Ultra Practice Bundle PDF
60 x 1.3 = 78 58 + 7 x 10 = 128

78 x 1.35 = 105.3 128 + 11 x 12 = 260

105.3 x 1.4 = 147.42 260 + 13 x 14 = 442

48) Answer: A 442 + 17 x 16= 714

16 + (21 + 1) = 19 714 + 19 x 18 = 1056

19 + (22 + 1) = 24 50) Answer: E

24 + (23 + 1) = 33 123 – 93 = 999

33 + (24 + 1) = 50 153 - 123 = 1647

50 + (25 + 1) = 83 183 - 153 = 2457

16 19 24 33 50 ?(83) 213 - 183 = 3429

+3 +5 +9 +17 +33 243 - 213 = 4563

+2 +4 +8 +16 273 - 243 = 5859

49) Answer: C

4). Wrong Number series


Direction (1-50): Find out the wrong number in the b) 148
following number series. c) 102
1. 8, 12, 24, 60, 160, 630 d) 94
a) 160 e) 121
b) 24 3. 8, 20, 44, 90, 188, 380
c) 60 a) 44
d) 630 b) 380
e) 12 c) 188
2. 121, 130, 112, 139, 102, 148, 94 d) 20
a) 130 e) 90
Page 572 of 728

Subscribe the Xpress Video Course & Mock Test Package for Bank & Insurance Exams
If there are any suggestions/ errors in our PDFs Feel Free to contact us via this email: admin@exampundit.in
IBPS RRB PO Prelims – Ultra Practice Bundle PDF
4. 17, 33, 95, 375, 1869, 11207 d) 36
a) 375 e) 9
b) 17 9)12 13 16 44 60 185
c) 1869 a) 185
d) 95 b) 60
e) 11207 c) 16
5. 252, 284, 329, 387, 457, 542 d) 13
a) 284 e) 12
b) 329 10) 4 3 4 8 32 155
c) 457 a) 3
d) 387 b) 4 (1st)
e) All are correct c) 4 (3rd)
6) 10 14 23 39 64 99 149 d) 8
a) 14 e) 155
b) 39 11) 17 18 26 53 117 252
c) 64 a) 252
d) 99 b) 117
e) 149 c) 53
7)18 36 90 264 945 3780 d) 18
a) 36 e) 17
b) 264 12) 8 27 64 98 136 178
c) 3780 a) 8
d) 90 b) 27
e) 945 c) 98
8) 9 15 36 71 120 183 d) 136
a) 183 e) 178
b) 120 13) 10000 2000 1000 200 50
c) 71 20
Page 573 of 728

Subscribe the Xpress Video Course & Mock Test Package for Bank & Insurance Exams
If there are any suggestions/ errors in our PDFs Feel Free to contact us via this email: admin@exampundit.in
IBPS RRB PO Prelims – Ultra Practice Bundle PDF
a) 200 e) 60
b) 20 18) 1, 8, 64, 343, 1296, 3125
c) 50 a) 8
d) 1000 b) 343
e) 10000 c) 64
14) 20 21 44 132 544 2725 d) 1296
a) 20 e) 3125
b) 44 19. 2800, 700, 4200, 1100, 6300, 1575
c) 2725 a) 2800
d) 132 b) 1100
e) 544 c) 6300
15) 12,133,261,401,552,721 d) 1575
a) 133 e) 4200
b) 261 20. 212, 244, 282, 332, 404, 492
c) 552 a) 212
d) 721 b) 492
e) 401 c) 244
16) 580, 527, 468, 407, 342, 269 d) 404
a) 407 e) 332
b) 342 21. 378, 442, 466, 683, 732, 1244
c) 580 a) 466
d) 527 b) 378
e) 269 c) 442
17) 60, 29, 30, 60, 240, 1920 d) 683
a) 29 e) 1244
b) 30 22. 14, 32, 105, 435, 2205, 13266
c) 1920 a) 13266
d) 240 b) 32
Page 574 of 728

Subscribe the Xpress Video Course & Mock Test Package for Bank & Insurance Exams
If there are any suggestions/ errors in our PDFs Feel Free to contact us via this email: admin@exampundit.in
IBPS RRB PO Prelims – Ultra Practice Bundle PDF
c) 2205 a) 13
d) 14 b) 52
e) 435 c) 71
23. 60, 48, 84, 231, 866.25, 4114.6875 d) 22
a) 866.25 e) 32
b) 84 28) 11, 60, 97, 121, 137, 146
c) 60 a) 97
d) 4114.6875 b) 11
e) 231 c) 60
24) 3, 6, 15, 42, 123, 364 d) 146
a) 123 e) 121
b) 42 29) 98, 114, 78, 103, 59, 90
c) 15 a) 78
d) 364 b) 114
e) 6 c) 98
25) 12, 18, 36, 65, 108, 162 d) 59
a) 18 e) 90
b) 162 30) 24, 60, 180, 670, 2520, 11340
c) 108 a) 670
d) 36 b) 2520
e) 65 c) 11340
26) 50, 67, 101, 152, 220, 304 d) 60
a) 220 e) 180
b) 304 31) 82, 95, 112, 135, 166,211
c) 101 a) 112
d) 152 b) 166
e) 67 c) 211
27) 11, 13, 22, 32, 52, 71 d) 82
Page 575 of 728

Subscribe the Xpress Video Course & Mock Test Package for Bank & Insurance Exams
If there are any suggestions/ errors in our PDFs Feel Free to contact us via this email: admin@exampundit.in
IBPS RRB PO Prelims – Ultra Practice Bundle PDF
e) 95 c) 521
32) 21, 33, 69, 141, 261, 440 d) 720
a) 261 e) 201
b) 141 37) 54 86 126 174 232 294
c) 33 a) 232
d) 440 b) 294
e) 69 c) 126
33) 24, 52, 100, 175, 280, 424 d) 174
a) 100 e) 54
b) 52 38) 6 4 5 12 52 424
c) 424 a) 52
d) 280 b) 12
e) 175 c) 5
34) 200, 300, 600, 1200, 1800, 2700 d) 4
a) 1200 e) 6
b) 600 39) 90 180 70 120 40 80
c) 300 a) 180
d) 1800 b) 70
e) 2700 c) 120
35) 30, 47, 73, 110, 160, 220 d) 40
a) 110 e) 80
b) 47 40) 2184 1320 720 335 120 24
c) 220 a) 1320
d) 73 b) 2184
e) 160 c) 335
36) 65, 110, 201, 338, 521, 720 d) 24
a) 338 e) 720
b) 110 41) 39, 41, 49, 98, 314, 939
Page 576 of 728

Subscribe the Xpress Video Course & Mock Test Package for Bank & Insurance Exams
If there are any suggestions/ errors in our PDFs Feel Free to contact us via this email: admin@exampundit.in
IBPS RRB PO Prelims – Ultra Practice Bundle PDF
a) 41 d) 160
b) 314 e) 53760
c) 98 46) 27 34 51 62 80 98
d) 39 a) 80
e) 49 b) 98
42) 14, 40, 92, 196, 376, 664 c) 51
a) 664 d) 27
b) 196 e) 34
c) 376 47) 8 8 15 41 105 228
d) 92 a) 15
e) 40 b) 41
43) 162, 163, 170, 182, 202, 232 c) 228
a) 162 d) 105
b) 170 e) None of these
c) 182 48) 280 285 292 302 318 329
d) 163 a) 329
e) 202 b) 280
44) 1700 1550 1400 1350 1325 c) 302
1312.5 d) 285
a) 1700 e) 318
b) 1550 49) 2, 27, 224, 1575, 9452, 47285
c) 1400 a) 27
d) 1312.5 b) 47285
e) 1325 c) 9452
45) 53760 6710 960 160 32 8 d) 224
a) 32 e) 1575
b) 960 50) 36, 54, 132, 472.5, 2126.25, 11694.375
c) 6710 a) 472.5
Page 577 of 728

Subscribe the Xpress Video Course & Mock Test Package for Bank & Insurance Exams
If there are any suggestions/ errors in our PDFs Feel Free to contact us via this email: admin@exampundit.in
IBPS RRB PO Prelims – Ultra Practice Bundle PDF
b) 132 d) 2126.25
c) 11694.375 e) 54

4). Wrong Number series - Solutions and Explanations:

1. Answer: A 18×2-3 = 33

Given series is 33×3-4 = 95

8 12 24 60 180 630 95×4-5 = 375

×1.5 ×2 ×2.5 ×3 x3.5 375×5-6 =1869

2. Answer: C 1869×6-7 =11207

Given series is 5. Answer: C

121 130 112 139 Given series is


103 148 94
252 284 329 387 458 542
+9 -18 +27 -36 +45
-54 32 45 58 71 84

3. Answer: E 13 13 13 13

8×2+4 =20 6) Answer: D

20×2+4 =44 10 + 2² = 14

44×2+4 = 92 14 + 3² = 23

92×2+4 =188 23 + 4² = 39

188x2+4=380 39 + 5² = 64

4. Answer: B 64 + 6² = 100

100 + 72 = 149
Page 578 of 728

Subscribe the Xpress Video Course & Mock Test Package for Bank & Insurance Exams
If there are any suggestions/ errors in our PDFs Feel Free to contact us via this email: admin@exampundit.in
IBPS RRB PO Prelims – Ultra Practice Bundle PDF
7) Answer: B 4x3–3=9

18 x 2 = 36 9 x 4 – 4 = 32

36 x 2.5 = 90 32 x 5 – 5 = 155

90 x 3 = 270 11) Answer: A

270 x 3.5 = 945 17 + 1 = 18

945 x 4 = 3780 18 + 8 = 26

8) Answer: E 26 + 27 = 53

183 – 7 x 9 = 120 53 + 64 = 117

120 – 7 x 7 = 71 117 + 125 = 242

71 – 7 x 5 = 36 12) Answer: B

36 – 7 x 3 = 15 8 + 26 = 34

15 – 7 x 1 = 8 34 + 30 = 64

9) Answer: C 64 + 34 = 98

12 + 1³ = 13 98 + 38 = 136

13 + 2² = 17 136 + 42 = 178

17 + 3³ = 44 13) Answer: C

44 + 4² = 60 10000/5 = 2000

60 + 5³ = 185 2000/2 = 1000

10) Answer: D 1000/5 = 200

4x1–1=3 200/2 = 100

3x2–2=4 100/5 = 20
Page 579 of 728

Subscribe the Xpress Video Course & Mock Test Package for Bank & Insurance Exams
If there are any suggestions/ errors in our PDFs Feel Free to contact us via this email: admin@exampundit.in
IBPS RRB PO Prelims – Ultra Practice Bundle PDF
14) Answer: D ×0.5 ×1 ×2 ×4
×8
(20 + 1) x 1 = 21

(21 + 1) x 2 = 44
18) Answer: A
(44 + 1) x 3 = 135
Given series is
(135 + 1) x 4 = 544
1 9 64 343 1296 3125
(544 + 1) x 5 = 2725 100 91 82 73 64 55
15) Answer: E
19. Answer: B
Given series is
2800 700 4200 1050
12 133 261 400 6300 1575
552 721
÷4 *6 ÷4 *6
+121 +128 +139 ÷4
+152 +169
20. Answer: D
+7 +11 +13
+17 212 244 282 332 400
492
16) Answer: B
32 38 50 68
Given series is
92
580 527 468 407
6 12 18 24
340 269

-53 -59 -61 -67 21. Answer: A


-71
378 442 467 683
Successive prime number 732 1244

17) Answer: A 64 25 216 49


512
60 30 30 60
240 1920 43 52 63 72
83
Page 580 of 728

Subscribe the Xpress Video Course & Mock Test Package for Bank & Insurance Exams
If there are any suggestions/ errors in our PDFs Feel Free to contact us via this email: admin@exampundit.in
IBPS RRB PO Prelims – Ultra Practice Bundle PDF
22. Answer: E 108 +(6*9)= 162

14*2+22= 32 26) Answer: B

32*3+32= 105 50 +17= 67

67 +34= 101
105*4+42 = 436
101 +51= 152
436*5+52 = 2205
152 +68= 220
2205*6+62 = 13266
220 +85= 305
23. Answer: C
27) Answer: B
64 48 84 231
866.25 4114.6875 11+(1*2)=13

13+(3*3)=22
*0.75 *1.75 *2.75 *3.75
*4.75 22+(5*2)=32

32+(7*3)=53
24) Answer: D
53+(9*2)=71
3 + 31 = 6
28) Answer: A
6+ 32 = 15
11 + 72 = 60
3
15 + 3 = 42
60 + 62 = 96
4
42+ 3 = 123
96 + 52 = 121
5
123 + 3 = 366
121 + 42 = 137
25) Answer: E
137 + 32 = 146
12 +(6*1)= 18
29) Answer: D
18 +(6*3)= 36
98+42=114
36 +(6*5)= 66
114 – 62 = 78
66+(6*7)= 108

Page 581 of 728

Subscribe the Xpress Video Course & Mock Test Package for Bank & Insurance Exams
If there are any suggestions/ errors in our PDFs Feel Free to contact us via this email: admin@exampundit.in
IBPS RRB PO Prelims – Ultra Practice Bundle PDF
78 + 52 = 103 24 52 100 174 280
424
103 – 72 = 54
28 48 74 106 144
54 + 62 = 90
20 26 32 38
30) Answer: A
34) Answer: A
24*2.5= 60
200 * 1.5 = 300
60*3= 180
300 * 2 = 600
180*3.5= 630
600 * 1.5 = 900
630 *4= 2520
900 * 2 = 1800
2520 * 4.5 = 11340
1800 * 1.5 = 2700
31) Answer: C
35) Answer: C
82 + 13 = 95
30 + 42 + 1 = 47
95 + 17 = 112
47 + 52 + 1 = 73
112 + 23 = 135
73 + 62 + 1 = 110
135 + 31 = 166
110 + 72 + 1 = 160
166 + 41 = 207
160 + 82 + 1 = 225
32) Answer: D
30 47 73 110 160 220
21 + 12 * 1 = 33
+17 +26 +37 +50 +60
33 + 18 * 2 = 69
+9 +11 +13 +15(225)
69 + 24 * 3 = 141
36) Answer: D
141 + 30 * 4 = 261
65 + (45 * 1) + 0= 110
261 + 36 * 5 = 441
110 + (45 * 2) + 1 = 201
33) Answer: E
201 + (45 * 3) + 2 = 338
Page 582 of 728

Subscribe the Xpress Video Course & Mock Test Package for Bank & Insurance Exams
If there are any suggestions/ errors in our PDFs Feel Free to contact us via this email: admin@exampundit.in
IBPS RRB PO Prelims – Ultra Practice Bundle PDF
338 + (45 * 4) + 3 = 521 40 x 2 = 80

521 + (45 * 5) + 4 = 750 40) Answer: C

37) Answer: A 13³ - 13 = 2184

7² + 5 = 54 11³ - 11 = 1320

9² + 5 = 86 9³ - 9 = 720

11² + 5 = 126 7³ - 7 = 336

13² + 5 = 174 5³ - 5 = 120

15² + 5 = 230 3³ - 3 = 24

17² + 5 = 294 41) Answer: D

54 86 126 174 230 294 Given series is

+32 +40 +48 +56 +64 41 41 49 98


314 939
38) Answer: E
0 8 49 216 625
7 x 0.5 + 0.5 = 4
90 81 72 63
4x1+1=5 54
5 x 2 + 2 = 12
42) Answer: C
12 x 4 + 4 = 52 14 + (13*2) = 40
52 x 8 + 8 = 424 40+ (13*4) = 92
39) Answer: B 92+ (13*8) = 196
90 x 2 =180 196 + (13*14) = 378
180/3 = 60 378 + (13*22) = 664
60 x 2 = 120
43) Answer:D
120/3 = 40 162 + (2*1) =164

Page 583 of 728

Subscribe the Xpress Video Course & Mock Test Package for Bank & Insurance Exams
If there are any suggestions/ errors in our PDFs Feel Free to contact us via this email: admin@exampundit.in
IBPS RRB PO Prelims – Ultra Practice Bundle PDF
164 + (3*2) = 170 15 + 3³ - 1 = 41

170 + (4*3) = 182 41 + 4³ - 1 = 104

182 + (5*4) = 202 104+ 5³ - 1 = 228

202 + (6*5) = 232 48) Answer: E

44) Answer: B 280 285 292 302 314


329
1700 – 200 = 1500
5 7 10 12
1500 – 100 = 1400 15
1400 – 50 = 1350 2 3 2
1350 – 25 = 1325 3

1325 – 12.5 = 1312.5 49) Answer: C

45) Answer: C 2 * 9 + 9 = 27

Given series is, 27 * 8 + 8 = 224

53760 6720 960 160 32 224 * 7 + 7 = 1575


8 1575 * 6 + 6 = 9456
÷8 ÷7 ÷6 ÷5 9456 * 5 + 5 = 47285
÷4
50) Answer: B
46) Answer: A
36 * 1.5 = 54
27 34 51 62 83 98
54 * 2.5 = 135

52+2 62-2 72+2 82-2 92+2 102-2 135 * 3.5 = 472.5

47) Answer: D 472.5 * 4.5 = 2126.25

8 + 1³ - 1 = 8 2126.25 * 5.5 = 11694.375

8 + 2³ - 1 = 15

Page 584 of 728

Subscribe the Xpress Video Course & Mock Test Package for Bank & Insurance Exams
If there are any suggestions/ errors in our PDFs Feel Free to contact us via this email: admin@exampundit.in
IBPS RRB PO Prelims – Ultra Practice Bundle PDF

5). Quadratic Equations


Directions: In the following questions two equations d) x ≥ y
numbered I and II are given. You have to solve both
e) x = y or relationship cannot be established
the equations and choose the correct option.
4. I. x2 -25x +154 = 0
1. I. x2 +35x +276 = 0

II. y2 +23y +132 = 0 II. y2 -32y +247 = 0

a) x< y a) x< y

b) x> y
b) x> y
c) x ≤ y
c) x ≤ y

d) x ≥ y d) x ≥ y

e) x = y or relationship cannot be established e) x = y or relationship cannot be established

2. I. x2 +6x -216 = 0 5. I. x2 +2x -3 = 0

II. y2 +31y +238 = 0


II. y2 +41y +414 = 0
a) x< y
a) x< y

b) x> y b) x> y

c) x ≤ y c) x ≤ y

d) x ≥ y d) x ≥ y

e) x = y or relationship cannot be established


e) x = y or relationship cannot be established
6. I. x2 + 26x + 168 = 0
3. I. x2 -19x +84 = 0

II. y2 -10y -39 = 0 II. y2 + 29y + 210 = 0

a) x< y a) x> y

b) x< y
b) x> y
c) x = y or the relationship cannot be established
c) x ≤ y
Page 585 of 728

Subscribe the Xpress Video Course & Mock Test Package for Bank & Insurance Exams
If there are any suggestions/ errors in our PDFs Feel Free to contact us via this email: admin@exampundit.in
IBPS RRB PO Prelims – Ultra Practice Bundle PDF
d) x ≥ y e) x ≤ y

e) x ≤ y 10. I. x2 + x – 240 = 0

7. I. 20y – 13x = 40 II. y2 – 17y + 60 = 0

II. 5x + 4y = 160 a) x> y

a) x> y b) x< y

b) x< y c) x = y or the relationship cannot be established

c) x = y or the relationship cannot be established d) x ≥ y

d) x ≥ y e) x ≤ y

e) x ≤ y 11. I. x2 + 2x – 48 = 0

8. I. x2 – 28x + 192 = 0 II. y2 – 17y + 66 = 0

II. y2 – 34y + 288 = 0 a) x> y

a) x> y b) x< y

b) x< y c) x = y or the relationship cannot be established

c) x = y or the relationship cannot be established d) x ≥ y

d) x ≥ y e) x ≤ y

e) x ≤ y 12. I. x2 – 27x + 180 = 0

9. I. x2 – x – 240 = 0 II. y2 – 7y – 60 = 0

II. y2– 5y – 204 = 0 a) x> y

a) x> y b) x< y

b) x< y c) x = y or the relationship cannot be established

c) x = y or the relationship cannot be established d) x ≥ y

d) x ≥ y e) x ≤ y

Page 586 of 728

Subscribe the Xpress Video Course & Mock Test Package for Bank & Insurance Exams
If there are any suggestions/ errors in our PDFs Feel Free to contact us via this email: admin@exampundit.in
IBPS RRB PO Prelims – Ultra Practice Bundle PDF
13. I. x2 + 36x + 320 = 0 II. 2y2 + 15y + 25 = 0

II. y2 – 6y – 315 = 0 a) x< y

a) x> y b) x> y

b) x< y c) x = y OR the relationship cannot be determined

c) x = y or the relationship cannot be established d) x ≥ y

d) x ≥ y e) x ≤ y

e) x ≤ y 17. I. x2 -42x +437 = 0

14. I. x2 + 25x – 116 = 0 II. y2 -13y +42 = 0

II. y2 – 18y + 56 = 0 a) x< y

a) x< y b) x> y

b) x> y c) x ≤ y

c) x = y OR the relationship cannot be determined d) x ≥ y

d) x ≥ y e) x = y or relationship cannot be established

e) x ≤ y 18. I. x2 +4x -221 = 0

15. I. 2x2 – 25x + 72 = 0 II. y2 +22y +85 = 0

II. 2y2 + 3y – 54 = 0 a) x< y

a) x< y b) x> y

b) x> y c) x ≤ y

c) x = y OR the relationship cannot be determined d) x ≥ y

d) x ≥ y e) x = y or relationship cannot be established

e) x ≤ y 19. I. 6x2 -5x -21 = 0

16. I. 20x2 + 48x – 5 = 0 II. y2 -28y +132 = 0

Page 587 of 728

Subscribe the Xpress Video Course & Mock Test Package for Bank & Insurance Exams
If there are any suggestions/ errors in our PDFs Feel Free to contact us via this email: admin@exampundit.in
IBPS RRB PO Prelims – Ultra Practice Bundle PDF
a) x< y b) x< y

b) x> y c) x = y or the relationship cannot be established

c) x ≤ y d) x ≥ y

d) x ≥ y e) x ≤ y

e) x = y or relationship cannot be established 23. I. x2 – 30x + 189 = 0

20. I. x2 – 26x + 165 = 0 II. y2– 44y + 484 = 0

II. y2 – 24y + 143 = 0 a) x> y

a) x> y b) x< y

b) x< y c) x = y or the relationship cannot be established

c) x = y or the relationship cannot be established d) x ≥ y

d) x ≥ y e) x ≤ y

e) x ≤ y 24. I. x2 + 27x + 162 = 0

21. I. x2 + x – 156 = 0 II. y2 + 23y + 132 = 0

II. y2– 32y + 240 = 0 a) x> y

a) x> y b) x< y

b) x< y c) x = y or the relationship cannot be established

c) x = y or the relationship cannot be established d) x ≥ y

d) x ≥ y e) x ≤ y

e) x ≤ y 25. I. x2 – 28x + 187 = 0

22. I. x2 + 26x + 153 = 0 II. y2– 38y + 360 = 0

II. y2+ 36y + 324 = 0 a) x> y

a) x> y b) x< y

Page 588 of 728

Subscribe the Xpress Video Course & Mock Test Package for Bank & Insurance Exams
If there are any suggestions/ errors in our PDFs Feel Free to contact us via this email: admin@exampundit.in
IBPS RRB PO Prelims – Ultra Practice Bundle PDF
c) x = y or the relationship cannot be established d) x ≥ y

d) x ≥ y e) x ≤ y

e) x ≤ y 29. I. 5x2 – 16x + 3 = 0

26. I. x2 – 11x – 26 = 0 II. 4y2 + 11y – 3 = 0

II. 4y2 – 24y – 64 = 0 a) x< y

a) x< y b) x> y

b) x> y c) x = y OR the relationship cannot be determined

c) x = y or the relationship cannot be determined d) x ≥ y

d) x ≥ y e) x ≤ y

e) x ≤ y 30. I. x2 -21x +110 = 0

27. I. 2x2 – 7x + 5 = 0 II. y2 -26y +165 = 0

II. 6y2 – 5y + 1 = 0 a) x< y

a) x< y b) x> y

b) x> y c) x ≤ y

c) x = y OR the relationship cannot be determined d) x ≥ y

d) x ≥ y e) x = y or relationship cannot be established

e) x ≤ y 31. I. x2 – 36x + 323 = 0

28. I. 2x2 – 7x + 5 = 0 II. y2– 11y – 102 = 0

II. 2y2 – 3y + 1 = 0 a) x> y

a) x< y b) x< y

b) x> y c) x = y or the relationship cannot be established

c) x = y OR the relationship cannot be determined d) x ≥ y

Page 589 of 728

Subscribe the Xpress Video Course & Mock Test Package for Bank & Insurance Exams
If there are any suggestions/ errors in our PDFs Feel Free to contact us via this email: admin@exampundit.in
IBPS RRB PO Prelims – Ultra Practice Bundle PDF
e) x ≤ y 35. I. 3x2 – 31x – 104 = 0

32. I. x2 – 26x + 160 = 0 II. 2y2 – 29y – 48 = 0

II. y2– 23y + 120 = 0 a) x< y

a) x> y b) x> y

b) x< y c) x = y OR the relationship cannot be determined

c) x = y or the relationship cannot be established d) x ≥ y

d) x ≥ y e) x ≤ y

e) x ≤ y 36. I. 8x2 - 15x + 7 = 0

33. I. x2 + 27x + 162 = 0 II. y2 - 7y + 6 = 0

II. y2+ 16y + 63 = 0 a) x> y

a) x> y b) x< y

b) x< y c) x ≥ y

c) x = y or the relationship cannot be established d) x ≤ y

d) x ≥ y e) x = y or relation cannot be established

e) x ≤ y 37. I. 3x2 – 8x + 4 = 0

34. I. x2 – 33x + 270 = 0 II. 6y2 – 5y + 1 = 0

II. y2– 22y + 117 = 0 a) x> y

a) x> y b) x< y

b) x< y c) x ≥ y

c) x = y or the relationship cannot be established d) x ≤ y

d) x ≥ y e) x = y or no relationship could be established

e) x ≤ y 38. I. 5x2 + 6x + 1 = 0

Page 590 of 728

Subscribe the Xpress Video Course & Mock Test Package for Bank & Insurance Exams
If there are any suggestions/ errors in our PDFs Feel Free to contact us via this email: admin@exampundit.in
IBPS RRB PO Prelims – Ultra Practice Bundle PDF
II. y2+ 3y– 4 = 0 a) x< y

a) x ≤ y b) x> y

b) x> y c) x ≤ y

c) x< y d) x ≥ y

d) x = y or relationship between x and y cannot be e) x = y or relationship cannot be established


established.
42. I. x2 -7x -120 = 0
e) x ≥ y
II. y2 -33y +270 = 0
39. I. x2 + 15x + 56 = 0
a) x< y
2
II. y - 23y + 132 = 0
b) x> y
a) x< y
c) x ≤ y
b) x ≤ y
d) x ≥ y
c) x> y
e) x = y or relationship cannot be established
d) x ≥ y
43. I. x2 -41x +418 = 0
e) x = y or the relationship cannot be established.
II. y2 -5y -266 = 0
40. I. 7x2 – 39x + 20 = 0
a) x< y
II. 5y + 3y – 2 = 0
2
b) x> y
a) x> y
c) x ≤ y
b) x ≥ y
d) x ≥ y
c) x< y
e) x = y or relationship cannot be established
d) x ≤ y
44. I. x2 -5x -300 = 0
e) x = y or the relationship cannot be established
II. y2 -37y +340 = 0
41. I. x2 -6x -27 = 0
a) x< y
2
II. y -43y +460 = 0
Page 591 of 728

Subscribe the Xpress Video Course & Mock Test Package for Bank & Insurance Exams
If there are any suggestions/ errors in our PDFs Feel Free to contact us via this email: admin@exampundit.in
IBPS RRB PO Prelims – Ultra Practice Bundle PDF
b) x> y c) x ≤ y

c) x ≤ y d) x ≥ y

d) x ≥ y e) x = y or relationship cannot be established

e) x = y or relationship cannot be established 48. I. x2 -38x +352 = 0

45. I. 2a2 - 13a + 20 = 0 II. y2 +7y -368 = 0

II. 2b2 - 5b + 3 = 0 a) x< y

a) a< b b) x> y

b) a> b c) x ≤ y

c) a ≤ b d) x ≥ y

d) a ≥ b e) x = y or relationship cannot be established

e) a = b or the relationship cannot be determined 49. I. x2 +28x +160 = 0

46. I. 34a2 + 44a + 10 = 0 II. y2 +3y -304 = 0

II. 18b2 + 26b + 8 = 0 a) x< y

a) a< b b) x> y

b) a> b c) x ≤ y

c) a ≤ b d) x ≥ y

d) a ≥ b e) x = y or relationship cannot be established

e) a = b or the relationship cannot be determined 50. I. 2x2 – 15x + 27 = 0

47. I. x2 +4x -252 = 0 II. 3y2 – 11y + 6 = 0

II. y2 -34y +280 = 0 a) x > y

a) x< y b) x < y

b) x> y c) x ≥ y

Page 592 of 728

Subscribe the Xpress Video Course & Mock Test Package for Bank & Insurance Exams
If there are any suggestions/ errors in our PDFs Feel Free to contact us via this email: admin@exampundit.in
IBPS RRB PO Prelims – Ultra Practice Bundle PDF
d) x ≤ y e) x = y or relation cannot be established

5). Quadratic Equations - Solutions with Explanation


1. Answer: C) So, x ≥ y

From equation I: 3. Answer: E)

x2 +35x +276 = (x + 12)(x + 23)= 0 From equation I:

=> x = -12, -23 x2 -19x +84 = (x -12)(x -7)= 0

From equation II: => x = 12, 7

y2 +23y +132 = (y + 11)(y + 12) = 0 From equation II:

=> y = -11, -12 y2 -10y -39 = (y + 3)(y -13) = 0

X = -12 X = -23 => y = -3, 13


Y = -11 x < y x<y
X = 12 X = 7
Y = -12 x = y x<y
Y = -3 x > y x>y
So, x ≤ y
Y = 13 x < y x<y
2. Answer: D) So, relationship cannot be established between x and y

From equation I: 4. Answer: E)

x2 +6x -216 = (x + 18)(x -12)= 0 From equation I:

=> x = -18, 12 x2 -25x +154 = (x -11)(x -14)= 0

From equation II: => x = 11, 14

y2 +41y +414 = (y + 23)(y + 18) = 0 From equation II:

=> y = -23, -18 y2 -32y +247 = (y -13)(y -19) = 0

X = -18 X = 12 => y = 13, 19


Y = -23 x > y x>y
X = 11 X = 14
Y = -18 x = y x>y
Page 593 of 728

Subscribe the Xpress Video Course & Mock Test Package for Bank & Insurance Exams
If there are any suggestions/ errors in our PDFs Feel Free to contact us via this email: admin@exampundit.in
IBPS RRB PO Prelims – Ultra Practice Bundle PDF
Y = 13 x < y x>y y(y + 14) + 15(y + 14) = 0
Y = 19 x < y x<y
(y + 14)(y + 15) = 0
So, relationship cannot be established between x and y
y = -14, -15
5. Answer: B)
X Relation Y
From equation I:
-12 > -14
x2 +2x -3 = (x + 3)(x -1)= 0 -12 > -15
-14 = -14
=> x = -3, 1
-14 > -15
From equation II: So, x ≥ y.

y2 +31y +238 = (y + 17)(y + 14) = 0 7. Answer: A)

=> y = -17, -14 From I:

X = -3 X = 1 20y – 13x = 40
Y = -17 x > y x > y
20y = (40 + 13x)
Y = -14 x > y x > y
So, x > y y = (40 + 13x)/20

6. Answer: D) From II:

From I: 5x + 4y = 160

x2 + 26x + 168 = 0 Putting value of y from (I),

x2 + 12x + 14x + 168 = 0 5x + (40 + 13x)/5 = 160

x(x + 12) + 14(x + 12) = 0 25x + 40 + 13x = 800

(x + 12)(x + 14) = 0 38x = 760

x = -12, -14 x = 20

From II: y = (40 + 260)/20 = 15

y2 + 29y + 210 = 0 Therefore, y < x.

y2 + 14y + 15y + 210 = 0 8. Answer: E)

Page 594 of 728

Subscribe the Xpress Video Course & Mock Test Package for Bank & Insurance Exams
If there are any suggestions/ errors in our PDFs Feel Free to contact us via this email: admin@exampundit.in
IBPS RRB PO Prelims – Ultra Practice Bundle PDF
From I: x = 16, -15

x2 – 28x + 192 = 0 From II:

x2 – 16x – 12x + 192 = 0 y2 – 5y – 204 = 0

x(x – 16) – 12(x – 16) = 0 y2 – 17y + 12y – 204 = 0

(x – 12)(x – 16) = 0 y(y – 17) +12(y – 17) = 0

x = 12, 16 (y + 12)(y – 17) = 0

From II: y = -12, 17

y2 – 34y + 288 = 0 X Relation Y


16 > -12
y2 – 16y – 18y + 288 = 0
16 < 17
y(y – 16) – 18(y – 16) = 0 -15 < -12
-15 < 17
(y – 16)(y – 18) = 0 So, no relation can be established between x and y.
y = 16, 18 10. Answer: C)
X Relation Y From I:
12 < 16
12 < 18 x2 + x – 240 = 0
16 = 16
x2 – 15x + 16x – 240 = 0
16 < 18
So x ≤ y x(x – 15) + 16(x – 15) = 0

9. Answer: C) (x – 15)(x + 16) = 0

From I: x = 15, -16

x2 – x – 240 = 0 From II:

x2 – 16x + 15x – 240 = 0 y2 – 17y + 60 = 0

x(x – 16) + 15(x – 16) = 0 y2 – 12y – 5y + 60 = 0

(x – 16)(x + 15) = 0 y(y – 12) – 5(y – 12)= 0

Page 595 of 728

Subscribe the Xpress Video Course & Mock Test Package for Bank & Insurance Exams
If there are any suggestions/ errors in our PDFs Feel Free to contact us via this email: admin@exampundit.in
IBPS RRB PO Prelims – Ultra Practice Bundle PDF
(y – 12)(y – 5) = 0 -8 < 11
Therefore, x ≤ y
y = 12, 5
12. Answer: D)
X Relation Y
15 > 12 From I:
15 > 5
x2 – 27x + 180 = 0
-16 < 12
-16 < 5 x2 – 12x – 15x + 180 = 0
Therefore, no relation can be established between x and
y. x(x – 12) – 15(x – 12) = 0

11. Answer: E) (x – 12)(x – 15) = 0

From I: x = 12, 15

x2 + 2x – 48 = 0 From II:

x2 – 6x + 8x – 48 = 0 y2 – 7y – 60 = 0

x(x – 6) + 8(x – 6) = 0 y2 – 12y + 5y – 60 = 0

(x – 6)(x + 8) = 0 y(y – 12) + 5(y – 12) = 0

x = 6, -8 (y – 12)(y + 5) = 0

From II: y = 12, -5

y2 – 17y + 66 = 0 X Relation y
12 = 12
y2 – 11y – 6y + 66 = 0 12 > -5
15 > 12
y(y – 11) – 6(y – 11)= 0
15 > -5
(y – 11)(y – 6) = 0 Therefore, x ≥ y

y = 11, 6 13. Answer: B)

X Relation y From I:
6 = 6
x2 + 36x + 320 = 0
6 < 11
-8 < 6 x2 + 16x + 20x + 320 = 0
Page 596 of 728

Subscribe the Xpress Video Course & Mock Test Package for Bank & Insurance Exams
If there are any suggestions/ errors in our PDFs Feel Free to contact us via this email: admin@exampundit.in
IBPS RRB PO Prelims – Ultra Practice Bundle PDF
x(x + 16) + 20(x + 16) = 0 ⇒ (y – 14)(y – 4) = 0

(x +16)(x + 20) = 0 Then, y = + 14 or y = + 4

x = -16, -20 So, when x = - 29, x < y for y = + 14 and x < y for y = +
4
From II:
And when x = + 4, x < y for y = + 14 and x = y for y = +
y2 – 6y – 315 = 0
4
y2 – 21y + 15y – 315 = 0
∴So, we can observe that x ≤ y.
y(y – 21) + 15(y – 21)= 0
15. Answer: D)
(y – 21)(y + 15) = 0
I. 2x2 – 25x + 72 = 0
y = 21, -15
⇒ 2x2 – 16x – 9x + 72 = 0
X Relation y
⇒ 2x(x – 8) – 9(x – 8) = 0
-16 < -15
-16 < 21 ⇒ (x – 8)(2x – 9) = 0
-20 < -15
Then, x = + 8 or x = + 9/2
-20 < 21
Therefore, x < y II. 2y2 + 3y – 54 = 0
14. Answer: E) ⇒ 2y2 + 12y – 9y – 54 = 0
I. x2 + 25x – 116 = 0 ⇒ 2y(y + 6) – 9(y + 6) = 0
⇒x2 + 29x – 4x – 116 = 0 ⇒ (y + 6)(2y – 9) = 0
⇒x(x + 29) – 4(x + 29) = 0 Then, y = - 6 or y = + 9/2
⇒ (x + 29)(x – 4) = 0 So, when x = + 8, x > y for y = - 6 and x > y for y = +
9/2
Then, x = - 29 or x = + 4
And when x = + 9/2, x > y for y = - 6 and x = y for y = +
II. y2 – 18y + 56 = 0
9/2
⇒ y2 – 14y – 4y + 56 = 0
∴So, we can clearly observe that x ≥ y.
⇒y(y – 14) – 4(y – 14) = 0
16. Answer: D)
Page 597 of 728

Subscribe the Xpress Video Course & Mock Test Package for Bank & Insurance Exams
If there are any suggestions/ errors in our PDFs Feel Free to contact us via this email: admin@exampundit.in
IBPS RRB PO Prelims – Ultra Practice Bundle PDF
I. 20x2 + 48x – 5 = 0 X = 23 X = 19
Y=7 x>y x>y
⇒ 20x2 + 50x – 2x – 5 = 0
Y=6 x>y x>y
⇒10x(2x + 5) – 1(2x + 5) = 0 So, x > y

⇒ (2x + 5)(10x – 1) = 0 18. Answer: E)

Then, x = - 5/2 or x = 1/10 From equation I:

II. 2y2 + 15y + 25 = 0 x2 +4x -221 = (x + 17)(x -13)= 0

⇒ 2y2 + 10y + 5y + 25 = 0 => x = -17, 13

⇒ 2y(y + 5) + 5(y + 5) = 0 From equation II:

⇒ (y + 5)(2y + 5) = 0 y2 +22y +85 = (y + 5)(y + 17) = 0

Then, y = - 5 or y = - 5/2 => y = -5, -17

So, when x = - 5/2, x > y for y = - 5 and x = y for y = - X = -17 X = 13


5/2 Y = -5 x < y x>y
Y = -17 x = y x>y
And when x = 1/10, x > y for y = - 5 and x > y for y = - So, relationship cannot be established between x and y
5/2
19. Answer: A)
∴So, we can observe that x ≥ y.
From equation I:
17. Answer: B)
6x2 -5x -21 = (2x + 3)(3x -7)= 0
From equation I:
=> x = -3/2, 7/3
x2 -42x +437
From equation II:
= (x -23)(x -19)= 0
y2 -28y +132 = (y -6)(y -22) = 0
=> x = 23, 19
=> y = 6, 22
From equation II:
X = -3/2 X = 7/3
y2 -13y +42 = (y -7)(y -6) = 0 Y=6 x<y x<y
=> y = 7, 6 Y = 22 x < y x<y
So, x < y
Page 598 of 728

Subscribe the Xpress Video Course & Mock Test Package for Bank & Insurance Exams
If there are any suggestions/ errors in our PDFs Feel Free to contact us via this email: admin@exampundit.in
IBPS RRB PO Prelims – Ultra Practice Bundle PDF
20. Answer: C) (x + 13)(x – 12) = 0

From I: x = -13, 12

x2 – 26x + 165 = 0 From II:

x2 – 11x – 15x + 165 = 0 y2 – 32y + 240 = 0

x(x – 11) – 15(x – 11) = 0 y2 – 12y – 20y + 240 = 0

(x – 11)(x – 15) = 0 y(y – 12) – 20(y – 12) = 0

x = 11, 15 (y – 20)(y – 12) = 0

From II: y = 20, 12

y2 – 24y + 143 = 0 X Relation Y


-13 < 20
y2 – 13y – 11y + 143 = 0
-13 < 12
y(y – 13) – 11(y – 13) = 0 12 < 20
12 = 12
(y – 13)(y – 11) = 0 So, y ≥ x
y = 13, 11 22. Answer: A)
X Relation Y From I:
11 < 13
11 = 11 x2+ 26x + 153 = 0
15 > 13
x2+ 9x + 17x + 153 = 0
15 > 11
So, no relation can be established between x and y. x(x + 9) + 17(x + 9) = 0

21. Answer: E) (x + 9)(x + 17) = 0

From I: x = -9, -17

x2 + x – 156 = 0 From II:

x2 – 12x + 13x – 156 = 0 y2 + 36y + 324 = 0

x(x – 12) + 13(x – 12) = 0 y2+ 18y + 18y + 324 = 0

Page 599 of 728

Subscribe the Xpress Video Course & Mock Test Package for Bank & Insurance Exams
If there are any suggestions/ errors in our PDFs Feel Free to contact us via this email: admin@exampundit.in
IBPS RRB PO Prelims – Ultra Practice Bundle PDF
y(y + 18) + 18(y + 18) = 0 9 < 22
9 < 22
(y + 18)(y + 18) = 0
So x < y
y = -18, -18
24. Answer: C)
X Relation Y
From I:
-9 > -18
-9 > -18 x2 + 27x + 162 = 0
-17 > -18
x2 + 9x + 18x + 162 = 0
-17 > -18
So x > y. x(x + 9) + 18(x + 9) = 0
23. Answer: B) (x + 18)(x + 9) = 0
From I: x = -18, -9
x2– 30x + 189 = 0 From II:
x2– 21x – 9x + 189 = 0 y2 + 23y + 132 = 0
x(x – 21) – 9(x – 21) = 0 y2 + 12y + 11y + 132 = 0
(x – 21)(x – 9) = 0 y(y + 12) + 11(y + 12) = 0
x = 21, 9 (y + 12)(y + 11) = 0
From II: y = -12, -11
y2 – 44y + 484 = 0 X Relation Y
-18 < -12
y2 – 22y – 22y + 484 = 0
-18 < -11
y(y – 22) – 22(y – 22) = 0 -9 > -12
-9 > -11
(y – 22)(y – 22) = 0
So, no relation can be established between x and y.
y = 22, 22
25. Answer: B)
X Relation Y
From I:
21 < 22
21 < 22 x2– 28x + 187 = 0

Page 600 of 728

Subscribe the Xpress Video Course & Mock Test Package for Bank & Insurance Exams
If there are any suggestions/ errors in our PDFs Feel Free to contact us via this email: admin@exampundit.in
IBPS RRB PO Prelims – Ultra Practice Bundle PDF
x2– 17x – 11x + 187 = 0 ⇒ y2 – 8y + 2y – 16 = 0

x(x – 17) – 11(x – 17) = 0 ⇒y(y – 8) + 2(y – 8) = 0

(x – 17)(x – 11) = 0 ⇒ (y – 8)(y + 2) = 0

x = 17, 11 Then, y = + 8 or y = - 2

From II: So, when x = + 13, x > y for y = + 8 and x > y for y = - 2

y2– 38y + 360 = 0 And when x = - 2, x < y for y = + 8and x = y for y = - 2

y2– 18y – 20y + 360 = 0 ∴So, we can observe that no clear relationship cannot be
determined between x and y.
y(y – 18) – 20(y – 18) = 0
27. Answer: B)
(y –18)(y – 20) = 0
I. 2x2 – 7x + 5 = 0
y = 18, 20
⇒ 2x2 – 5x – 2x + 5 = 0
X Relation Y
17 < 18 ⇒x(2x – 5) – 1(2x – 5) = 0
17 < 20
11 < 18 ⇒ (2x – 5)(x – 1) = 0
11 < 20 Then, x = + 5/2 or x = + 1
So, x < y
II. 6y2 – 5y + 1 = 0
26. Answer: C)
⇒ 6y2 – 3y – 2y + 1 = 0
I. x2 – 11x – 26 = 0
⇒3y(2y – 1) – 1(2y – 1) = 0
⇒x2 – 13x + 2x – 26 = 0
⇒ (2y – 1)(3y – 1) = 0
⇒x(x – 13) + 2(x – 13) = 0
Then, y = + ½ or y = + 1/3
⇒ (x – 13)(x + 2) = 0
So, when x = + 5/2, x > y for y = + ½ and x > y for y = +
Then, x = + 13 or x = - 2 1/3
II. 4y2 – 24y – 64 = 0 And when x = + 1, x > y for y = + ½ and x > y for y = +
1/3
⇒ y2 – 6y – 16 = 0 [Dividing both sides by 4 ]

Page 601 of 728

Subscribe the Xpress Video Course & Mock Test Package for Bank & Insurance Exams
If there are any suggestions/ errors in our PDFs Feel Free to contact us via this email: admin@exampundit.in
IBPS RRB PO Prelims – Ultra Practice Bundle PDF
∴We can clearly see that x> y. II. 4y2 + 11y – 3 = 0

28. Answer: D) ⇒ 4y2 + 12y – y – 3 = 0

I. 2x2 – 7x + 5 = 0 ⇒ 4y(y + 3) – 1(y + 3) = 0

⇒ 2x2 – 5x – 2x + 5 = 0 ⇒ (y + 3)(4y – 1) = 0

⇒x(2x – 5) – 1(2x – 5) = 0 Then, y = - 3 or y = + 1/4

⇒ (2x – 5)(x – 1) = 0 So, when x = + 3, x > y for y = - 3 and x > y for y = + ¼

Then, x = + 5/2 or x = + 1 And when x = + 1/5, x > y for y = - 3 and x < y for y = +
¼
II. 2y2 – 3y + 1 = 0
∴So, we can observe that no clear relationship cannot be
⇒ 2y2 – 2y – y + 1 = 0 determined between x and y.
⇒ 2y(y – 1) – 1(y – 1) = 0 30. Answer: C)
→ (y – 1)(2y – 1) = 0 From equation I:
Then, y = + 1 or y = + ½ x2 -21x +110 = (x -10)(x -11)= 0
So, when x = + 5/2, x > y for y = + 1 and x > y for y = + => x = 10, 11
½
From equation II:
And when x = + 1, x = y for y = + 1 and x > y for y = +
½ y2 -26y +165 = (y -15)(y -11) = 0

∴So, we can clearly observe that x ≥ y. => y = 15, 11

29. Answer: C) X = 10 X = 11
Y = 15 x < y x<y
I. 5x2 – 16x + 3 = 0
Y = 11 x < y x=y
⇒ 5x2 – 15x – x + 3= 0 So, x ≤ y

⇒ 5x(x – 3) – 1(x – 3) = 0 31. Answer: D)

⇒ (x – 3)(5x – 1) = 0 From I:

Then, x = + 3 or x = + 1/5 x2– 36x + 323 = 0

Page 602 of 728

Subscribe the Xpress Video Course & Mock Test Package for Bank & Insurance Exams
If there are any suggestions/ errors in our PDFs Feel Free to contact us via this email: admin@exampundit.in
IBPS RRB PO Prelims – Ultra Practice Bundle PDF
x2 – 19x – 17x + 323 = 0 y2 – 23y + 120 = 0

x(x – 19) – 17(x – 19) = 0 y2 – 15y – 8y + 120 = 0

(x – 17)(x – 19) = 0 y(y – 15) – 8(y – 15) = 0

x = 17, 19 (y – 15)(y – 8) = 0

From II: y = 15, 8

y2 – 11y – 102 = 0 X Relation Y


10 < 15
y2 – 17y + 6y – 102 = 0
10 > 8
y(y – 17) + 6(y – 17) = 0 16 > 15
16 > 8
(y – 17)(y + 6) = 0 So, no relation can be established between x and y
y = 17, -6 33. Answer: E)
X Relation Y From I:
17 = 17
17 > -6 x2+ 27x + 162 = 0
19 > 17
x2 + 9x + 18x + 162 = 0
19 > -6
So, x ≥ y x(x + 9) + 18(x + 9) = 0

32. Answer: C) (x + 9)(x + 18) = 0

From I: x = -9, -18

x2– 26x + 160 = 0 From II:

x2 – 16x – 10x + 160 = 0 y2 + 16y + 63 = 0

x(x – 16) – 10(x – 16) = 0 y2 + 7y + 9y + 63 = 0

(x – 16)(x – 10) = 0 y(y + 7) + 9(y + 7) = 0

x = 10, 16 (y + 7)(y + 9) = 0

From II: y = -7, -9

Page 603 of 728

Subscribe the Xpress Video Course & Mock Test Package for Bank & Insurance Exams
If there are any suggestions/ errors in our PDFs Feel Free to contact us via this email: admin@exampundit.in
IBPS RRB PO Prelims – Ultra Practice Bundle PDF
X Relation Y I. 3x2 – 31x – 104 = 0
-9 = -9
⇒ 3x2 – 39x + 8x – 104 = 0
-9 < -7
-18 < -9 ⇒ 3x(x – 13) + 8(x – 13) = 0
-18 < -7
So, x ≤ y ⇒ (x – 13)(3x + 8) = 0

34. Answer: A) Then, x = + 13 or x = - 8/3

From I: II. 2y2 – 29y – 48 = 0

x2– 33x + 270 = 0 ⇒ 2y2 – 32y + 3y – 48 = 0

x2 – 15x – 18x + 270 = 0 ⇒ 2y(y – 16) + 3(y – 16) = 0

x(x – 15) – 18(x – 15) = 0 ⇒ (y – 16)(2y + 3) = 0

(x – 15)(x – 18) = 0 Then, y = + 16 or y = - 3/2

x = 15, 18 So, when x = + 13, x < y for y = + 16 and x > y for y = -


3/2
From II:
And when x = - 8/3, x < y for y = + 16 and x < y for y = -
y2 – 22y + 117 = 0 3/2
y2 – 9y – 13y + 117 = 0 ∴So, we can observe that no clear relationship cannot be
determined between x and y.
y(y – 9) – 13(y – 9) = 0
36. Answer: D)
(y – 9)(y – 13) = 0
I. 8x2 - 15x + 7 = 0
y = 9, 13
⇒ 8x2– 8x – 7x + 7 = 0
X Relation Y
15 > 9 ⇒ 8x(x - 1) - 7(x - 1) = 0
15 > 13
18 > 9 ⇒ (8x - 7) (x - 1) = 0
18 > 13 Then, x = +7/8 = +0.875 or x = + 1
So, x > y
II. y2 - 7y + 6 = 0
35. Answer: C)

Page 604 of 728

Subscribe the Xpress Video Course & Mock Test Package for Bank & Insurance Exams
If there are any suggestions/ errors in our PDFs Feel Free to contact us via this email: admin@exampundit.in
IBPS RRB PO Prelims – Ultra Practice Bundle PDF
⇒ y2 - 6y - 1y + 6 = 0 When x = 2, y = 1/3, then x > y

⇒y(y - 6) - 1(y - 6) = 0 ∴x and y are related as x > y

⇒ (y - 1)(y – 6) = 0 38. Answer: D)

Then, y = 1 or y = 6 Solving the equation for x:

So, when x = + 0.875, x < y for y = + 1 and x < y for y = 5x2 + 6x + 1 = 0


+6
Formula for roots = {−b ± √(b2 − 4ac)}/2a
And when x = + 1, x = y for y = + 1 and x < y for y = +
⇒ Roots = {− (6) ± √ ({[6]2 – 4 × 5 × 1)}/ (2 × 5)
6
⇒ Roots = {-6 ± √16}/(2 × 5)
∴So, we can observe that x ≤ y.
⇒ Roots = {-6 ± 4}/10
37. Answer: A)

I. 3x2 – 8x + 4 = 0 ⇒ Roots = -10/10 or -2/10

⇒ 3x2 – 6x – 2x + 4 = 0 ⇒ Roots = -1 or -1/5

⇒ 3x(x – 2) – 2(x – 2) = 0 Solving the equation for y:

⇒ (3x – 2) (x – 2) = 0 ⇒ y2 + 3y– 4 = 0

⇒ x = 2/3, 2 ⇒ y2 + 4y - y– 4 = 0

II. 6y2 – 5y + 1 = 0 ⇒y(y + 4) – 1(y + 4) = 0

⇒ 6y2 – 3y – 2y + 1 = 0 ⇒ y = -4 or y = 1

⇒ 3y (2y – 1) – 1(2y – 1) = 0 When x = −1, y = -4 then x > y

When x = −1, y = 1 then x < y


⇒ (2y – 1) (3y – 1) = 0
When x = −1/5, y = -4 then x > y
⇒ y = 1/2, 1/3
When x = -1/5, y = 1 then x < y
When x = 2/3, y = 1/2, then x > y
∴ x = y or relationship cannot be established
When x = 2/3, y = 1/3, then x > y
39. Answer: A)
When x = 2, y = 1/2, then x > y

Page 605 of 728

Subscribe the Xpress Video Course & Mock Test Package for Bank & Insurance Exams
If there are any suggestions/ errors in our PDFs Feel Free to contact us via this email: admin@exampundit.in
IBPS RRB PO Prelims – Ultra Practice Bundle PDF
I. x2 + 15x + 56 = 0 ⇒ 5y2 + 5y – 2y – 2 = 0

⇒x2 + 7x + 8x + 56 = 0 ⇒ 5y(y + 1) – 2(y + 1) = 0

⇒x(x + 7) + 8(x + 7) = 0 ⇒ y = -1 or y = 2/5

⇒ (x + 7)(x + 8) = 0 ∴x>y

Then, x = -7 or x = -8 41. Answer: A)

II. y2-23y + 132 = 0 From equation I:

⇒ y2 – 11y – 12y + 132 = 0 x2 -6x -27 = (x + 3)(x -9)= 0

⇒y(y – 11) – 12(y – 11) = 0 => x = -3, 9

⇒ (y – 11)(y – 12) = 0 From equation II:

Then, y = +11 or y = +12 y2 -43y +460 = (y -23)(y -20) = 0

So, when x = -7, x < y for y = +11 and x < y for y = +12 => y = 23, 20

And when x = -8, x < y for y = +11 and x < y for y = +12 X = -3 X = 9
Y = 23 x < y x < y
∴We can clearly observe that x < y.
Y = 20 x < y x < y
40. Answer: A) So, x < y

Equation: 1 42. Answer: C)

⇒ 7x2 – 39x + 20 = 0 From equation I:

⇒ 7x2 – 35x – 4x + 20 = 0 x2 -7x -120 = (x + 8)(x -15)= 0

⇒ 7x(x – 5) – 4(x – 5) = 0 => x = -8, 15

⇒ (x – 5)(7x – 4) = 0 From equation II:

⇒ x = 5 or x = 4/7 y2 -33y +270 = (y -15)(y -18) = 0

Equation: 2 => y = 15, 18

⇒ 5y2 + 3y – 2 = 0 X = -8 X = 15
Y = 15 x < y x = y
Page 606 of 728

Subscribe the Xpress Video Course & Mock Test Package for Bank & Insurance Exams
If there are any suggestions/ errors in our PDFs Feel Free to contact us via this email: admin@exampundit.in
IBPS RRB PO Prelims – Ultra Practice Bundle PDF
Y = 18 x < y x<y I. 2a2 - 13a + 20 = 0
So, x ≤ y
⇒ 2a2 - 8a - 5a + 20 = 0
43. Answer: D)
⇒2a(a - 4) - 5(a - 4) = 0
From equation I:
⇒ (2a - 5)(a - 4) = 0
2
x -41x +418 = (x -19)(x -22)= 0
Then, a = 2.5 or a = 4
=> x = 19, 22
II. 2b2 - 5b + 3 = 0
From equation II:
⇒ 2b2 - 2b - 3b + 3 = 0
y2 -5y -266 = (y -19)(y + 14) = 0
⇒2b(b - 1) - 3(b - 1) = 0
=> y = 19, -14
⇒ (2b - 3)(b - 1) = 0
X = 19 X = 22
Then, b = 1 or b = 1.5
Y = 19 x = y x>y
Y = -14 x > y x>y So, when a = 2.5, a > b for both b = 1 and 1.5
So, x ≥ y
And when a = 4, a > b for both b = 1 and 1.5
44. Answer: E)
∴So, we can observe that a > b.
From equation I:
46. Answer: E)
x2 -5x -300 = (x + 15)(x -20)= 0
I. 34a2 + 44a + 10 = 0
=> x = -15, 20
⇒ 2(17a2 + 22a + 5) = 0
From equation II:
⇒ 17a2 + 17a + 5a + 5 = 0
2
y -37y +340 = (y -17)(y -20) = 0
⇒17a(a + 1) + 5(a + 1) = 0
=> y = 17, 20
⇒ (17a + 5)(a + 1) = 0
X = -15 X = 20
Y = 17 x < y x>y Then, a = -5/17 or a = -1
Y = 20 x < y x=y
II. 18b2 + 26b + 8 = 0
So, relationship cannot be established between x and y
⇒ 18b2 + 18b + 8b + 8 = 0
45. Answer: B)

Page 607 of 728

Subscribe the Xpress Video Course & Mock Test Package for Bank & Insurance Exams
If there are any suggestions/ errors in our PDFs Feel Free to contact us via this email: admin@exampundit.in
IBPS RRB PO Prelims – Ultra Practice Bundle PDF
⇒18b(b + 1) + 8(b + 1) = 0 => y = -23, 16

⇒ (18b + 8)(b + 1) = 0 X = 16 X = 22
Y = -23 x > y x>y
Then, b = -8/18 or b = -1
Y = 16 x = y x>y
So, when a = -5/17, a > b for b = -8/18 and a > b for b = - So, x ≥ y
1
49. Answer: E)
And when a = -1, a < b for b = -8/18 and a = b for b = -1
From equation I:
∴So, the relationship cannot be determined.
x2 +28x +160 = (x + 8)(x + 20)= 0
47. Answer: C)
=> x = -8, -20
From equation I:
From equation II:
2
x +4x -252 = (x -14)(x + 18)= 0
y2 +3y -304 = (y -16)(y + 19) = 0
=> x = 14, -18
=> y = 16, -19
From equation II:
X = -8 X = -20
y2 -34y +280 = (y -20)(y -14) = 0 Y = 16 x < y x < y
Y = -19 x > y x < y
=> y = 20, 14 So, relationship cannot be established between x and y
X = 14 X = -18 50. Answer: C)
Y = 20 x < y x<y
Y = 14 x = y x<y Equation I:
So, x ≤ y
2x2 – 15x + 27 = 0
48. Answer: D)
⇒ 2x2 – 6x - 9x + 27 = 0
From equation I:
⇒ 2x(x - 3) -9(x - 3) = 0
2
x -38x +352 = (x -16)(x -22)= 0
(2x - 9)(x - 3) = 0
=> x = 16, 22
x = 3, 9/2
From equation II:
Equation II:
y2 +7y -368 = (y + 23)(y -16) = 0
3y2 – 11y + 6 = 0
Page 608 of 728

Subscribe the Xpress Video Course & Mock Test Package for Bank & Insurance Exams
If there are any suggestions/ errors in our PDFs Feel Free to contact us via this email: admin@exampundit.in
IBPS RRB PO Prelims – Ultra Practice Bundle PDF
3y2 – 9y – 2y + 6 = 0 y = 2/3, 3

⇒ 3y(y - 3) – 2(y - 3) = 0 Thus, x = 3, 9/2 and y = 2/3, 3

⇒ (3y - 2)(y - 3) = 0 Comparing these values of x and y, we get x ≥ y

6). DI Based on Table


Directions (1 – 5): Study the following information and answer the following questions:

Given Table show the total number of Water bottle (3 different type- x, y and z) sold by 6 different sellers in
2016 and ratio between type z to x and y to z.

Seller Total water bottle sold Type z:x Type y: z

Big Bazaar 1005 7:9 19:21

Best Price 1350 1:2 2:1

Metro 1656 4:5 6:5

Big Basket 1710 9:5 5:9

7 –Eleven 1440 1:1 2:3

Open Market 1494 4:5 5:7


Note - Total water bottle sold = Type x + Type y + Type Z water bottle sold

1. Total number of Y type of water bottle sold by Big e) None of the above
basket is how much percentage more or less than the
total number of Z type of water bottle sold by Open 2. What is the ratio between total water bottle of z
market? type sold by Big Bazaar and Metro together to total
water bottle of Y type sold by open market?
a) 9.32% less
a) 41:32
b) 10.71% less
b) 23:51
c) 9.52% less
c) 53:24
d) 8.52% less
Page 609 of 728

Subscribe the Xpress Video Course & Mock Test Package for Bank & Insurance Exams
If there are any suggestions/ errors in our PDFs Feel Free to contact us via this email: admin@exampundit.in
IBPS RRB PO Prelims – Ultra Practice Bundle PDF
d) 23:21 b) 325

e) None of these c) 320

3. Find the average number of water bottle sold by 7- d) 180


Eleven and Best Price of X type?
e) None of the above
a) 540
5. Price of one water bottle of X type is 25 rupees then
b) 590 find out total revenue of Metro and Big Basket after
selling all the water bottle of X type? (Price of water
c) 525
bottle is same in all the shop)
d) 530 a) 26250
e) None of these b) 28800
4. What is the difference between total number of c) 27500
water bottle of Y type sold by Best Price and 7-
Eleven? d) 25000

a) 330 e) None of these


Directions (6 – 10): Answer the questions based on the information given below.

Table given below shows the total number of Covid-19 cases in the month of March, April and May from 6
different states.

Total number of Covid- Total number of Covid- Total number of Covid-


State 19 cases in the month of 19 cases in the month of 19 cases in the month of
March April May
Maharashtra 25000 55000 95000

Delhi 24800 33500 50500

Punjab 12800 22500 32100

UP 10020 11550 12800

MP 5800 8700 22000

Rajasthan 12500 16700 25200

Page 610 of 728

Subscribe the Xpress Video Course & Mock Test Package for Bank & Insurance Exams
If there are any suggestions/ errors in our PDFs Feel Free to contact us via this email: admin@exampundit.in
IBPS RRB PO Prelims – Ultra Practice Bundle PDF

6. Average number of Covid-19 cases form the month b) 1220: 1253


of March from all the state is how much more or less
c) 1250: 5153
than the average number of Covid-19 cases from the
month of May form all the state? d) 810:5192
a) 20617 e) None of these
b) 22177 9. % increase in total number of Covid-19 cases from
c) 24447 March to April is 2nd highest in which of the following
state?
d) 15525
a) Punjab
e) None of these
b) UP
7. Total number of Covid-19 case in the month of
c) MP
April is how much more or less than the total number
of Covid-19 case in UP, MP and Maharashtra from d) Rajasthan
the month of May?
e) Maharashtra
a) 13750
10. Total number of Covid-19 cases form Delhi in the
b) 14150 month of March and May is how much % of total
c) 13150 number of Covid-19 cases from Rajasthan in the
month of March and April?
d) 17150
a) 257.87%
e) None of these
b) 201.83%
8. Find the ratio between total number of Covid-19
c) 202.83%
cases from UP and MP in the month of April to total
number of Covid-19 case in UP, MP and d) 287.55%
Maharashtra from the month of May?
e) None of these
a) 1450 :810
Directions (11 – 15): Answer the questions based on the information given below.

The table given below shows the data regarding the number of total visitors of a theater who are going to
watch to different Acts like Classical music show and Drama show in five different theater.
Page 611 of 728

Subscribe the Xpress Video Course & Mock Test Package for Bank & Insurance Exams
If there are any suggestions/ errors in our PDFs Feel Free to contact us via this email: admin@exampundit.in
IBPS RRB PO Prelims – Ultra Practice Bundle PDF
Visitors who like
Ratio between
Theater Total Visitors Classical music show:
Male: Female (Visitors)
Drama Show
X 750 11:14 7:3

Y 420 22:20 4:3

Z 720 2:1 7:3

A 385 17:18 3:4

B 540 17:10 1:1

11. What is the difference between the total number e) None of these
of visitors of Theater A whom like Drama show and
total number of visitors of theater B who like 13. Find the difference between average numbers of
Classical music show? Visitors who like Drama show from all the theater
together to average number of male visitors from all
a) 135 the theater together?

b) 142 a) 68.5

c) 144 b) 85.6

d). 195 c) 195.2

e) 124 d) 89.2

12. What is the respective ratio of the number of male e) 32.5


Visitors of theater Y and Z to the number of Female
visitors of X and A? 14. Find the Total number of female visitors of
theater from all the theater together?
a) 175:112
a) 1252
b) 744 : 445
b) 1155
c) 73:129
c) 1192
d) 164:125
d) 1141

Page 612 of 728

Subscribe the Xpress Video Course & Mock Test Package for Bank & Insurance Exams
If there are any suggestions/ errors in our PDFs Feel Free to contact us via this email: admin@exampundit.in
IBPS RRB PO Prelims – Ultra Practice Bundle PDF
e) None of these b) 25.28%

15. Female visitors of theater A is how much c) 26.25%


percentage more or less than the total visitors of Y
d) 10.00%
who like drama show?

a) 33.25% e) None of these

Directions (16-20): Answer the questions based on the information given below.

Table given below show the data about total number of Laptop sold by 6 different sellers in the month of May
2020. While another column shows the % increase or decrease in the laptop sold for 2021 as compare to 2020.

% increase or decrease in Laptop


Seller Total Laptop Sold in 2020
sold in 2021 as compare to 2020
Aman 12500 +12.4%
Raman 11440 +25%
Ravan 12800 -16%
Raghav 13400 -25%
Raju 22100 -11%
Rama 12100 +12%

16. Find the difference between total number of 17. Find the total number of laptops sold in 2021 by
Laptop sold by Ravan in 2020 and 2021 together to all the sellers together?
total number of laptops sold by Raju and Rama in
a) 92500
2021?
b) 82373
a) 9669
c) 86273
b) 9356

c) 9290 d) 85273

d) 9255 e) None of these

e) None of the above


Page 613 of 728

Subscribe the Xpress Video Course & Mock Test Package for Bank & Insurance Exams
If there are any suggestions/ errors in our PDFs Feel Free to contact us via this email: admin@exampundit.in
IBPS RRB PO Prelims – Ultra Practice Bundle PDF
18. What is the ratio between total laptop sold by b) 27781250
Ravan, Raju and Aman together in 2021 to total
c) 375881250
number of laptops sold by Raghav, Raju and Rama in
2020? d) 252558125
a) 44125:31200 e) None of these
b) 45:31 20. Difference between number of laptops sold in
c) 44471:47600 2020 and 2021 is 2nd lowest for which of the following
seller? (Ignore +/- sign if any for calculating
d) 45455:47600 difference)

e) Can’t be determined a) Raman

19. Price of one laptop sold by Aman in 2020 is 12500 b) Ravan


which is 20% less than the price of one laptop sold in
c) Raghav
2021. Then find out total revenue generated by Aman
by selling all the laptop on 2020 and 2021? d) Raju
a) 22587500 e) None of these
Directions (21 – 25): Answer the questions based on the information given below.

In the Exam of IBPS RRB PO and Clerk of 2020, total number of students who appeared is 458000. Table
given below show the % of total studentsappeared from different state and ratio of Male studentsappeared to
female studentsappeared form 6 different state.

Total studentsappeared = Total male studentsappeared + Total female studentsappeared

% of studentsappeared from Ratio between


State
different state out of total student Male: Female
Rajasthan 12% 5:7
UP 18% 11:7
HP 15% 7:8
UK 25% 14:11
Assam 13% 9:4
Tamil Nadu 17% 9:8

Page 614 of 728

Subscribe the Xpress Video Course & Mock Test Package for Bank & Insurance Exams
If there are any suggestions/ errors in our PDFs Feel Free to contact us via this email: admin@exampundit.in
IBPS RRB PO Prelims – Ultra Practice Bundle PDF
21. Maximum number of Male appeared form which b) 5581
of the following state in the IBPS RRB PO and Clerk
c) 8551
Exam of 2020?

a) UK d) 7551

b) HP e) 5454

c) Assam 24. Find the ratio between total number of males who
applied from UK and Assam to total number of
d) UP females who applied form Rajasthan and HP?

e) None of these a) 1448:2125

22. Find the average number of students who b) 1528:3125


appeared form states HP, UP, Assam and UK
c) 2626:3312
together in the exam of IBPS RRB PO and Clerk
Exam of 2020? d) 26335:17175
a) 81550 e) None of these
b) 81295 25.Total number of studentswho appeared in the
c) 82520 IBPS RRB PO and Clerk exam form HP and MP
together is what % of total number of female who
d) 82550 Applied from UK and UP together?

e) None of these a) 71.25%

23. Out of Total female form UP state appeared only b) 77.5%


10% qualified the exam, while out of total male
c) 72.8%
studentsappeared, only 8% qualified the exam. Then
find out overall qualified no of male and female d) Can’t be determined
together from the state of UP?
e) None of these
a) 8051
Direction (26-30) Direction : Answer the question based on the information given below

Table given below shows the total number of players in six different state and other collumn of the tablw
shows percentage of Football players in each of the givn state.

Page 615 of 728

Subscribe the Xpress Video Course & Mock Test Package for Bank & Insurance Exams
If there are any suggestions/ errors in our PDFs Feel Free to contact us via this email: admin@exampundit.in
IBPS RRB PO Prelims – Ultra Practice Bundle PDF
Note – Total number of Player in a State = Total number of football players + Total number of Volley Ball
Players

State Total players % of Football Players


P 1050 42%
Q 1400 70%
R 1950 33.33%
S 1800 62.5%
T 1560 40%
U 1380 55%

26. Find the ratio of number of Volley Ball players in 28. Number of boys playing Football in T is 40%
P and Q together to the number of Football players more than the number of girls playing same game in
form state T? the school then, find the difference between Male and
Female who like to play football from T?
a) 343 : 208
a) 120
b) 75:22
b) 84
c) 71:52
c) 104
d) 3:2
d) 102
e) None of these
e) 105
27. Average number of Volley Ball players in R and S
together is how much more or less than the average 29. 4/7 of the Volley Ball players left Q and joined R.
number of football players in P and U together? Find number of Football players of R is
approximately what percentage of Volley ball players
a) 387.5
in R?
b) 387.5
a) 15.525%
c) 883.5
b) 23.505%
d) 375.5
c) 32.205%
e) 225.8
d) 42.207%

Page 616 of 728

Subscribe the Xpress Video Course & Mock Test Package for Bank & Insurance Exams
If there are any suggestions/ errors in our PDFs Feel Free to contact us via this email: admin@exampundit.in
IBPS RRB PO Prelims – Ultra Practice Bundle PDF
e) 25.550% b) 48%

30. Find number of Volley Ball players in state T is c) 45%


how much more or less than the number of Football
d) 40%
players in S and U together?

a) 45.5% e) None of these

Directions (31 – 35): Answer the question based on the following information

Table given below shows the export of Almond from country Japan and Import of Almond to Russia in five
different years. Read the given data carefully and answer the Question.

Export of Almond from Japan Import of Almond to Russia


Year
( in Tons) (in Tons)
2015 3300 2800

2016 3600 5500

2017 5100 3800

2018 4500 4600

2019 3400 4200

31. If ratio between export to import of Almond in e) None of these


Japan in year 2019 is 17:16 and export of Almond in
Russia in 2019 is 16-2/3% more than its import then, 32. Average quantity of Almond exported by Japan in
its import than find Export of Almond in Russia is year 2015, 2016 and 2017 is how much percent more
how much more than the import of Almond in Japan or less than average quantity of Almond imported by
in the year 2019? Russia in year 2016, 2017 and 2019?

a) 1700 a) 11.11%

b) 33.33%
b) 1280
c) 21.21%
c) 1320

d)1 350 d) 21.22%

Page 617 of 728

Subscribe the Xpress Video Course & Mock Test Package for Bank & Insurance Exams
If there are any suggestions/ errors in our PDFs Feel Free to contact us via this email: admin@exampundit.in
IBPS RRB PO Prelims – Ultra Practice Bundle PDF
e) 31.33% c) 8:3

33. Find the total Export of Almond by Russia in d) 9:2


2018. If 20% of total Export and Import together of
e) 11:3
each year is export of Almond by Russia?

a) 1150 35.If ratio of Quantity of Almond exported to Almond


imported by Japan in 2015 is 11:10 and in 2020
b) 1220 Almond imported by Russia increased by 20 % as
compared to previous year, then Almond imported by
c) 1120 Japan in 2015 is what percentage of Almond
d) 1125 imported by Russia in 2020?

e) None of these a) 75.75%

34. What is the ratio of Almond imported by Russia b) 59.52%


in year 2015, 2016, 2017 and 2018 together to Almond c) 49.55%
exported by USA in year 2018 and 2019?
d) 51.55%
a) 7:3
e) None of the above
b) Can’t be determined
Directions (36 – 40): Answer the question based on the information given below

Table given below shows the number of Hen in 6 different poultry Farm, also number of eggs by each hen on 3
different days.

Note price of 1 eggs is = rupees 6 (same in all the poultry farm)

Number of eggs by Number of eggs by Number of eggs by


Total number of
Farm each hen on each hen on each hen on
Hen
Monday Tuesday Wednesday
A 1250 4 2 6
B 1155 3 3 3
C 1235 5 4 8
D 1144 6 5 4
E 1530 2 1 5
F 1420 8 6 3

Page 618 of 728

Subscribe the Xpress Video Course & Mock Test Package for Bank & Insurance Exams
If there are any suggestions/ errors in our PDFs Feel Free to contact us via this email: admin@exampundit.in
IBPS RRB PO Prelims – Ultra Practice Bundle PDF
36. Find the total revenue generated by A on Monday, b) 27.52% less
and Wednesday together is how much more or less
c) 22.80% less
than the total revenue generated by D on same day?

a) 6360 d) 26.77% less

b) 6232 e) None of theses

c) 6565 39. Find the total number of eggs produced by A, B


and C on Wednesday?
d) 5480
a) 20845
e) None of these
b) 22845
37. What is the ratio of total number of eggs given by
c) 19845
all hens on Monday to Wednesday?

a) 35942:33245 d) 14445

b) 35924:33271 e) None of these

c) 35924:37331 40. Revenue of which of the following farm is 2nd


highest on Wednesday?
d) 25242:33254
a) C
e) None of these
b) B
38. Total number of Eggs produced by C and E on
c) A
Tuesday is how much % more or less than the total
number of Eggs produced by D and F on d) D
Wednesday?
e) E
a) 25.55% less

Direction (41-45) – Answer the question based on information given below.

Table given bellow show the total number of Vote (Valid + Invalid) in 5 different state, % or male Vote out of
total Votes and % of invalid vote casted by Male out of total vote by male. Read the given data carefully and
answer the question given below.

Page 619 of 728

Subscribe the Xpress Video Course & Mock Test Package for Bank & Insurance Exams
If there are any suggestions/ errors in our PDFs Feel Free to contact us via this email: admin@exampundit.in
IBPS RRB PO Prelims – Ultra Practice Bundle PDF
% of invalid vote casted
Total Vote (Valid + % of Total vote casted by
State by Male out of total vote
Invalid) Male
by Male
UP 15000 55% 20%

MP 12000 45% 20%

UK 18000 49% 5%

HP 10000 58% 15%

RJ 8000 45% 20%


Note – 10% of the vote casted by each states female is invalid

41. Find the difference between valid vote by Male in the total number of vote casted by female form same
UP and MP? state?

a) 2280 a) 0.1181% less

b) 2008 b) 0.1081% more

c) 2010 c) .0.2% more

d) 3050 d) 0.9% less

e) None of these e) Can’t be determined

42. Find the average number of invalid vote by 44. Find the total number of vote casted by Female
Female from all the state together? from all the state together?

a) 665 a) 30500

b) 660 b) 31130

c) 623 c) 31600

d) 664 d) 31440

e) 675 e) 31660

43. Total number of vote casted by Male form state of 45. Find the difference between total invalid vote
UK, HP and MP is how much % more or less than casted by male and female form UK and HP state
together?
Page 620 of 728

Subscribe the Xpress Video Course & Mock Test Package for Bank & Insurance Exams
If there are any suggestions/ errors in our PDFs Feel Free to contact us via this email: admin@exampundit.in
IBPS RRB PO Prelims – Ultra Practice Bundle PDF
a) 37 d) 35

b) 71 e) None of these

c) 72

Directions (46 – 50): Study the following graph and answer the following questions.

Table given below show the total number of PDF Publish by Exam Pundit of different subject and percentage
of PDF got 5 stars rating out of total number of PDFs published. Read the date carefully and answer the
question based on it.

Percentage of PDFs got 5 Start


Subject Number of PDFs published
rating
DI for Mains 3150 77.77%

DI for Prelims 2700 55.55%

English 5500 45.45%

Reasoning 4500 88.88%

Current Affairs 4200 71.428%

GK 2500 37.2%

46. Total number of DI for Mains and Reasoning 47. GK PDFs which does not get 5-star rating is how
PDF together got 5 start rating is what percentage much percentage more or less than the English PDFs
more or less than total number of DI for prelims and which do not get 5 star rating?
English PDF together got 5 start rating by Exam
a) 25.25%
Pundit?
b) 85.20%
a) 61.25%

b) 52.25% c) 56.20%

c) 51.75% d) 47.66%

d) 75.25% e) Can’t be determined

e) None of these 48. Find the average number of Reasoning and


Current affairs PDFs got 5-star rating?
Page 621 of 728

Subscribe the Xpress Video Course & Mock Test Package for Bank & Insurance Exams
If there are any suggestions/ errors in our PDFs Feel Free to contact us via this email: admin@exampundit.in
IBPS RRB PO Prelims – Ultra Practice Bundle PDF
a) 3355 c) 7:13

b) 3150 d) 13:7

c) 3350 e) None of these

d) 3500 50. Find the total number of PDFs of the entire


subject which got the 5-star rating?
e) 4550
a) 13750
49. Find the ratio between DI for Mains PDFs which
does not get 5 start rating to Current affairs PDFs b) 13880
which does not get 5-star rating?
c) 13850
a) 15:19
d) 13450
b) 7:12
e) None of these

6). DI Based on Table - Solutions and Detailed Explanations

1-5. Common Explanation: 19 : 21 : 27

Ratio between X: Y: Z for Best Price

Big Bazaar Z: X = 1:2 and Y: Z = 2:1

Z: X = 7:9 and Y: Z = 19:21 Combined ratio

Combined ratio Y : Z : X

Y : Z : X : 1: 2

: 7 : 9 2 : 1

19 : 21 ________________

________________ 2 : 1 : 2

Page 622 of 728

Subscribe the Xpress Video Course & Mock Test Package for Bank & Insurance Exams
If there are any suggestions/ errors in our PDFs Feel Free to contact us via this email: admin@exampundit.in
IBPS RRB PO Prelims – Ultra Practice Bundle PDF
Metro ________________

Z: X = 4:5 and Y: Z = 6:5 2 :3 : 3

Combined ratio Open Market

Y : Z : X Z: X = 4:5 and Y: Z = 5:7

: 4 : 5 Combined ratio

6 : 5 Y : Z : X

________________ : 4 : 5

24 : 20 : 25 5 : 7

Big Basket _____________

Z: X = 9:5 and Y: Z = 5:9 20 : 28 : 35

Combined ratio 1. Answer: B)

Y : Z : X Y type of water bottle sold by Big Basket = 1710*5/19 =


450
: 9 : 5
Total number of Z type of water bottle sold by Open
5 : 9 Market = 1494*28/83 = 504
________________ Required percentage = 450-504/504 * 100 = 10.71% less
5 :9 : 5
2. Answer: C)
7-Eleven Total water bottle of z type sold by Big Bazaar and
Z: X = 1:1 and Y: Z = 2:3 Metro together = 1005*21/67 + 1656*20/69 = 315+ 480
= 795
Combined ratio
Total water bottle of Y type sold by Open market =
Y : Z : X 1494*20/83 = 360

: 1 : 1 Required ratio = 795: 360 = 53: 24

2 : 3 3. Answer: A)

Page 623 of 728

Subscribe the Xpress Video Course & Mock Test Package for Bank & Insurance Exams
If there are any suggestions/ errors in our PDFs Feel Free to contact us via this email: admin@exampundit.in
IBPS RRB PO Prelims – Ultra Practice Bundle PDF
Average number of water bottle sold by 7-Elven and Total number of Covid-19 case in UP, MP and
Best Price of X type = 1/2 * [{1440*3/8} + {1350*2/5}] Maharashtra from the month of May =
= 540 12800+22000+95000 = 129800

4. Answer: D) Required difference = 147950 – 129800 = 18150

Difference between total number of water bottle of Y 8. Answer: D)


type sold by Best Price and 7-Eleven = {(1350*2/5) –
(1440*2/8)} = 540 – 360 = 180 Total number of Covid-19 case UP and MP in the month
of April = 11550 + 8700 = 20250
5. Answer: A)
Total number of Covid-19 case in UP, MP and
Revenue generated by Metro by Selling X type of water Maharashtra from the month of May =
bottle = 1656*25/69 * 25 = 15000 12800+22000+95000 = 129800

Revenue generated by Big Basket by Selling X type of Required ratio = 20250: 129800 = 810: 5192
water bottle = 1710 * 5/19 * 25 = 11250
9. Answer: A)
Total Revenue = 15000 + 11250 = 26250
% increase in total number of Covid-19 cases from
6 – 10: Common Explanation: March to April for

6. Answer: C) Maharashtra = 55000-25000/25000 * 100 = 120%

Average number of Covid-19 cases form the month of Delhi = 33500 – 24800 / 24800 * 100 = 35.08%
March = 25000+24800+12800+10020+5800+12500 / 6 Punjab = 22500-12800/12800 * 100 = 75.78%
= 90920/6
UP = 11550-10020/10020 * 100 = 15.27%
Average number of Covid-19 Cases from the month of
May = 95000+50500+32100+12800+22000+25200 = MP = 8700-5800/5800 * 100 = 50%
237600/6 Rajasthan = 16700 – 12500 / 12500 * 100 = 33.6%

Required answer = 237600 / 6 – 90920 / 6 = 24447 Required answer (2nd highest) = 75.78% (Punjab)
Approx. 10. Answer: A)
7. Answer: E) Total number of Covid-19 cases form Delhi in the month
of March and May = 24800+50500 = 75300
Total number of Covid-19 in the month of April =
55000+33500+22500+11550+8700+16700 = 147950 Total number of Covid-19 cases from Rajasthan in the
month of March and April = 12500 + 16700 = 29200

Page 624 of 728

Subscribe the Xpress Video Course & Mock Test Package for Bank & Insurance Exams
If there are any suggestions/ errors in our PDFs Feel Free to contact us via this email: admin@exampundit.in
IBPS RRB PO Prelims – Ultra Practice Bundle PDF
Required percentage = 75300/29200 * 100 = 257.87% B = 540*10/27 = 200

Average number of people who like Drama =


{420+200+240+198+200)/5 = 251.6
11-15. Common Explanation
Male visitors from
11. Answer: B)
X = 750*7/10 = 525
Total number of visitors of Theater A who like Drama
show = 385*18/35 = 198 Y = 420*4/7 = 240

Total number of visitors of theater B who like Classical Z = 720*7/10 = 504


music show = 540*17/27 =340
A = 385 * 3/7 = 165
Required difference = 340 – 198 = 142
B = 540*1/2 = 270
12. Answer: B)
Average number of male = (525+240+504+165+270)/5
Number of male Visitors of theater Y and Z = 420*4/7 + = 1704 / 5 = 340.8
720*7/10 = 240 + 504 = 744
Required difference = 334.8-251.6 = 89.2
Female visitor of X and A = 750 * 3/10 + 385*4/7 = 225
+ 220 = 445 14. Answer: D)

Total number of female from all the theater together =


Required ratio = 744 : 445
Total visitors – Total male visitors =
13. Answer: D) (750+420+720+385+540) – (525+240+504+165+270) =
1141
Visitors who like Drama from,
15. Answer: D)
X = 750*14/25 = 420
Female visitors of A= 385*4/7 = 220
Y = 420*20/42 = 200
Total visitors of Y who like Drama = 420*20/42 = 200
Z = 720*1/3 = 240
Required percentage = 200-140 / 200 * 100 = 10%
A = 385*18/35 = 198

16-20. Common Explanation:

Seller Total Laptop sold in 2020 Total Laptop Sold in 2021

Page 625 of 728

Subscribe the Xpress Video Course & Mock Test Package for Bank & Insurance Exams
If there are any suggestions/ errors in our PDFs Feel Free to contact us via this email: admin@exampundit.in
IBPS RRB PO Prelims – Ultra Practice Bundle PDF
Aman 12500 12500*112.4% = 14050

Raman 11440 11440*125% = 14300

Ravan 12800 12800*84% = 10752

Raghav 13400 13400*75% = 10050

Raju 22100 22100*89% = 19669

Rama 12100 12100*112% = 13552

16. Answer: A) Price of one laptop sold by Aman in 2020 = 12500

Total number of Laptop sold by Ravan in 2020 and 2021 Price of One laptop sold by Aman in 2021 = 12500/80 *
= 12800 + 10752 = 23552 100 = 15625

Total number of laptop sold by Raju and Rama in 2021 = Total revenue = 12500*12500 + 14050*15625 =
19669+13552 = 33221 156250000 + 219531250 = 375781250

Required difference = 33221 – 23552 = 9669 20. Answer: E)

17. Answer: B) Difference between laptop sold

Total number of Laptop Sold in 2021 = Aman = 12500 -14050 = 1550


14050+14300+10752+10050+19669+13552 = 82373
Raman = 11440 – 14300 = 2860
18. Answer: C)
Ravan = 12800 – 10752 = 2048
Total Laptop sold by Ravan, Raju and Aman together in
Raghav = 13400 – 10050 = 3350
2021 = 10752+19669+14050 = 44471
Raju = 22100 – 19669 = 2431
Total number of laptop sold by Raghav, Raju and Rama
in 2020 = 13400+22100+12100 = 47600 Rama = 12100 – 13552 = 1452
Required ratio = 44471:47600 Required answer = 1550 (Aman)
19. Answer: E)

21-25. Common Explanation:

Page 626 of 728

Subscribe the Xpress Video Course & Mock Test Package for Bank & Insurance Exams
If there are any suggestions/ errors in our PDFs Feel Free to contact us via this email: admin@exampundit.in
IBPS RRB PO Prelims – Ultra Practice Bundle PDF
State Total studentsAppeared Male students Female Students

Rajasthan 458000*12% = 54960 54960*5/12 = 22900 54960*7/12 = 32060

UP 458000*18% = 82440 82440*11/18 = 50380 82440*7/18 = 32060

HP 458000*15% = 68700 68700*7/15 = 32060 68700*8/15 = 36640

UK 458000*25% = 114500 114500*14/25 = 64120 114500*11/25 = 50380

Assam 458000*13% = 59540 59540*9/13 = 41220 59540*4/13 = 18320

Tamil Nadu 458000*17% = 77860 77860*9/17 = 41220 77860*8/17 = 36640


21. Answer: A)

Required Answer is = 64120 (UK)

22. Answer: B)

Average number of studentsappeared form HP, UP, Assam and UK = (68700+82440+59540+114500)/4 = 81295

23. Answer: A)

Total Male and Female Qualified form UP = 53350*10% + 33950*8% = 5335+2716 = 8051

24. Answer: D)

Total number of male who appeared form UK and Assam = 64120+41220 = 105340

Total Female who papered from Rajasthan and HP = 32060+36640 = 68700

Required ratio = 105340:68700 = 26335:17175

25. Answer: D)

We don’t know the data about MP

Answer is can’t be determined

26-30. Common Explanation 26. Answer: A)

Page 627 of 728

Subscribe the Xpress Video Course & Mock Test Package for Bank & Insurance Exams
If there are any suggestions/ errors in our PDFs Feel Free to contact us via this email: admin@exampundit.in
IBPS RRB PO Prelims – Ultra Practice Bundle PDF
Number of volleyball player in P and Q = 1050*(100-42) Required difference = 624*40% * (7-5)/12 = 104
% + 1400*(100-70) % = 609+420 = 1029
29. Answer: D)
Number of Football player in T = 1560*40% = 624
ATQ,
Required ratio = 1029 : 624 = 343 : 208
Volley Ball player who left Q = 4/7 * (1400*30%) = 240
27. Answer: A)
Total number of volley ball player R = 1950* (100-
Average number of Volley Ball player in R and S 33.33) % + 240 = 1300+240 = 1540
together = {1950*(100-33.33) % + 1800*(100-62.5)
%}*1/2 = (1300+675)*1/2 = 1975/2 Total number of Football player R = 1950 * 33.33% =
650
Average number of Football player in P and U =
(1050*42%+1380*55%)*1/2 = (441+759)*1/2= 600 Required percentage = 650/1540 * 100 = 42.207%

Required difference = 1975/2 – 600 = 775/2 = 387.5 30. Answer: E)

28. Answer: C) Volley Ball player of State T = 1560*(100-40) % = 936

Number of boys playing football in T = 140/100 * Football player of State S and U = 1800*62.5% +
Number of girls playing Football 1380*55% = 1125+759=1884

ATQ, Required % = 1884-936/1884 * 100 = 50.32% less

Ratio of Boys to girl who play Football = 7:5

31-35. Common Explanation And,

31. Answer: A) Export of Almond in Russia in 2019 is 16-2/3% more


than its import
Ratio between export to import of Almond in Japan in
year 2019 is 16:17 Import of almond in Russia in 2019 = 4200

ATQ, Export of Almond in Russia in 2019 = 4200*116.67 /


100 = 4900
Export of Almond in Japan in 2019 = 3400
Required answer = 4900-3200 = 1700
Import of almond in Japan in 2019 = 3400/17 * 16 =
3200 32. Answer: A)

Page 628 of 728

Subscribe the Xpress Video Course & Mock Test Package for Bank & Insurance Exams
If there are any suggestions/ errors in our PDFs Feel Free to contact us via this email: admin@exampundit.in
IBPS RRB PO Prelims – Ultra Practice Bundle PDF
Average quantity of Almond exported by Japan in year Almond imported by Russia in 2020 = 4200*(100+20) %
2015, 2016 and 2017 = (3300+3600+5100)/3 = 4000 = 5040

Average Quantity of Almond Imported by Russia in year Required percentage = 3000/5040 *100 = 59.52%
2016, 2017 and 2019 = (5500+3800+4200)/3 = 4500

% less = 4500-4000/4500 * 100 = 11.11%


36-40. Common Explanation
33. Answer: A)
36. Answer: A)
Let Total import and Export of Almond by Russia in
2018 = 100x Revenue Generated by A on Monday and Wednesday =
(1250 * 4 + 1250*6)*6 = 75000
Export of Almond by Russia in 2018 = 100x*20% = 20x
Revenue generated by D on Monday and Wednesday=
ATQ, (1144*6+1144*4)*6 = 68640

80x (Import of Almond by Russia in 2018) = 4600 Required answer = 75000 – 68640 = 6360

X = 4600/80 37. Answer: C)

Required answer = 20x = 4600/80 * 20 = 1150 Total number of Eggs given by all hens on Monday =
1250*4+1155*3+1235*5+1144*6+1530*2+1420*8 =
34. Answer: B) 35924

Almond imported by Russia in year 2015, 2016, 2017 Total number of Eggs given by all hens on Wednesday=
and 2018 together = 2800+5500+3800+4600 = 16700 1250*6+1155*3+1235*8+1144*4+1530*5+1420*3 =
37331
Almond Exported by USA in year 2018 and 2019 = can’t
be determine (data regarding USA is not given) Required ratio = 35924:37331

Required answer is can’t be determined 38. Answer: D)

35. Answer: B) Total number of Eggs produced by C and E on Tuesday


= 1235*4+1530*1 = 6470
Given that, ratio of Quantity of Almond exported to
Almond imported by Japan in 2015 is 11:10 Total number of Eggs produced by D and F on
Wednesday = 1144*4+1420*3 = 8836
Almond imported by Japan in 2015 = 3300/11 * 10 =
3000 Required % = 8836-6470/8836 * 100 = 26.77% less

Page 629 of 728

Subscribe the Xpress Video Course & Mock Test Package for Bank & Insurance Exams
If there are any suggestions/ errors in our PDFs Feel Free to contact us via this email: admin@exampundit.in
IBPS RRB PO Prelims – Ultra Practice Bundle PDF
39. Answer: A) C = 1235*8*6 = 59280

Total number of Eggs produced by A, B and C on D = 1144*4*6 = 27456


Wednesday = 1250*6+1155*3+1235*8 = 20845
E = 1530*5*6 = 45900
40. Answer: E)
F = 1420*3*6 = 25560
Revenue on Wednesday
Required answer = E = 45900 (2nd highest)
A = 1250*6* 6 = 45000

B = 1155*3*6 = 20790
41-45. Common Explanation

Invalid Vote by Invalid Vote By


State Total vote Vote by Male Vote by Female
Male Female
UP 15000 8250 6750 1650 675
MP 12000 5400 6600 1080 660
UK 18000 8820 9180 441 918
HP 10000 5800 4200 870 420
RJ 8000 3600 4400 720 440
41. Answer: A) Total number of vote casted by female form state of UK,
HP and MP =9180+4200+6600 = 19980
Valid vote by Male in UP = 8250 – 1650 =6600
Required % = 19980 – 20020 / 19980 * 100 = 0.2%
Valid vote by Male in MP = 5400-1080 = 4320 more

Required difference = 6600 – 4320 = 2280 44. Answer: B)

42. Answer: C) Total number of vote casted by Female form all the state
together = 6750+6600+9180+4200+4400 =31130
Average number of invalid vote by Female form all the
state together = (675+660+918+420+440)/5 = 623 45. Answer: E)
Approx.
Difference between invalid vote casted by male and
43. Answer: C) female form UK and HP = (441+870) – (918+420) = 27

Total number of vote casted by Male form state of UK,


HP and MP = 8820+5800+5400 = 20020 46-50. Common Explanation
Page 630 of 728

Subscribe the Xpress Video Course & Mock Test Package for Bank & Insurance Exams
If there are any suggestions/ errors in our PDFs Feel Free to contact us via this email: admin@exampundit.in
IBPS RRB PO Prelims – Ultra Practice Bundle PDF
46. Answer: A) 49. Answer: B)

Total number of DI for Mains and Reasoning PDF DI For mains which does not get 5-star rating = 3150 *
together got 5 start rating = 3150*77.77% + (100-77.77) % = 700
4500*88.88% = 2450+4000 = 6450
Current affairs PDFs which does not get 5-star rating =
Total number of DI for prelims and English PDF 4200 * (100-71.428) % = 1200
together got 5 start rating by Exam Pundit =
2700*55.55% + 5500*45.45% = 1500 + 2500 = 4000 Required Ratio = 700:1200 = 7:12

Required % more = 6450 – 4000 / 4000 * 100 = 61.25% 50. Answer: B)

47. Answer: D) PDF of which got 5 Star rating

GK PDFs which does not get 5-star rating = 2500(100- DI for mains = 3150*77.77% = 2450
37.2) % = 1570
DI Fro prelims = 2700 * 55.55% = 1500
English PDFs which does not get 5-star rating =
5500*(100-45.45) % = 3000 English = 5500*45.45% = 2000

Required % less = 3000-1570 / 3000 * 100 = 47.66% Reasoning = 4500 * 88.88% = 4000

48. Answer: D) Current Affairs = 4200 * 71.428% = 3000

Average number of Reasoning and Current affairs PDFs GK = 2500 * 37.2% = 930
got 5-star rating = (4500*88.88% + 4200*71.428%) *1/2
=4000+3000 = 7000/2 = 3500 Required sum = 2450+1500+2000+4000+3000+930 =
13880

7). Bar Graph DI

Directions (1 – 5): Given line graph shows the number of creams sold of various companies in 3 different
months. Read the following information carefully and answer the given questions:

Page 631 of 728

Subscribe the Xpress Video Course & Mock Test Package for Bank & Insurance Exams
If there are any suggestions/ errors in our PDFs Feel Free to contact us via this email: admin@exampundit.in
IBPS RRB PO Prelims – Ultra Practice Bundle PDF

November December January


650
550
450
350
250
Ponds VLCC Lakme Lotus Loreal Ayur

1. The quantity of L’Oreal creams sold in November a) 31: 78


is what percent of the total sale of all brands cream
b) 24: 113
in November?

a) 14.84% c) 19: 128

b) 24.25% d) 13: 121

c) 21.43% e) None of these

d) 17.34% 4. The quantity of VLCC creams sold in January is


what percent of the total sale of all brands cream in
e) None of these January?

2. What is the average number of creams sold in a) 16.35%


December?
b) 27.25%
a) 380
c) 21.09%
b) 440
d) 18.34%
c) 485
e) 24.75%
d) 350
5. Which of the following cream is sold most in all
e) 424 given months together?

3. Find the ratio of Lotus creams sold in January a) Ponds


sold and total number of creams sold in January?
b) VLCC

Page 632 of 728

Subscribe the Xpress Video Course & Mock Test Package for Bank & Insurance Exams
If there are any suggestions/ errors in our PDFs Feel Free to contact us via this email: admin@exampundit.in
IBPS RRB PO Prelims – Ultra Practice Bundle PDF
c) Lakme e) Ayur

d) L’Oreal

Directions (6 – 10): Answer the questions based on the information given below.

The given bar graph shows the total number of magazines (India Today + Filmfare) sold (in hundreds), and
the percentage of India Today magazines sold out of total number of magazines sold, in five different places.

90
80 80
80 75

70
60 60
60
50
50 45
40 40
40 35

30

20
Delhi Pune Nagpur Chennai Cuttack

Total Number of magazines sold (in hundreds)


Percentage of magazines sold of India Today out of total number of magazines sold

6. The number of Filmfare’s magazines sold in 7. Find the average number of Filmfare’s magazines
Chennai is how much percent more/less than number sold in Nagpur and Cuttack.
of India today’s magazines sold at Cuttack?
a) 3000
a) 24.5%
b) 3500
b) 37.5%
c) 2400
c) 32.5%
d) 2800
d) 42.5%
e) 3200
e) 40.5%

Page 633 of 728

Subscribe the Xpress Video Course & Mock Test Package for Bank & Insurance Exams
If there are any suggestions/ errors in our PDFs Feel Free to contact us via this email: admin@exampundit.in
IBPS RRB PO Prelims – Ultra Practice Bundle PDF
8. Find the ratio of number of India today’s b) 3600
magazines sold in Delhi and Chennai, respectively.
c) 4000
a) 7:8
d) 3800
b) 3:4
e) 4200
c) 1:2
10. Find the total number of India today’s magazines
d) 5:6 sold at Pune and Nagpur, together.

e) 3:2 a) 4200

9. Find the difference between total number of b) 4800


magazines sold in Nagpur, and number of Filmfare’s
c) 4600
magazines sold at Delhi and Pune together.

a) 2400 d) 5200

e) 3600

Directions (11 – 15): Study the following bar graph carefully and answer the following questions:

The bar graph shows selling price (in thousands) of portable laptops by 3 companies Lenovo, Asus and HP
from 2011 to 2015

Price of Laptops (in thousands)

2015
2014
2013
2012
2011

30 35 40 45 50 55
HP Asus Lenovo

Page 634 of 728

Subscribe the Xpress Video Course & Mock Test Package for Bank & Insurance Exams
If there are any suggestions/ errors in our PDFs Feel Free to contact us via this email: admin@exampundit.in
IBPS RRB PO Prelims – Ultra Practice Bundle PDF
11. In which of the given years, the average selling d) Lenovo and HP
price of all the 3 portable laptops was minimum?
e) Asus and HP
a) 2011
14. If a distributor purchases 40 Lenovo, 60 Asus and
b) 2012 80 HP laptops in 2014 at a selling price given in the
graph and then sells them each at Rs. 50400, then
c) 2013 what is the profit or loss percentage of the
d) 2014 distributor?

e) 2015 a) 1.70%

12. If the number of portable laptops sold by HP in b) 2.87%


2013 was 25000 and that by Lenovo was 24000 in the c) 1.89%
same year, then find the difference between the
revenue of HP and Lenovo? d) 1.34%

a) 13.9 crores e) 2.24%

b) 12.1 crores 15. If the selling price of Asus laptops increases by


30% in 2016 when compared to 2013, then the selling
c) 11.8 crores price of Asus laptops in 2016 is what percentage of
d) 10.2 crores the selling price of HP laptops in 2013?

e) 9.6 crores a) 143.67%

13. Which portable Laptop Company has the b) 138.54%


maximum average selling price during 2011-2015? c) 127.47%
a) Lenovo d) 124.45%
b) HP e) 139.28%
c) Asus

Directions (16 – 20): Answer the questions based on the information given below.
The given bar graph shows the number of accounts opened in different banks in Bijnor district in 2016.
Note: Total accounts = Savings accounts + Current accounts

Page 635 of 728

Subscribe the Xpress Video Course & Mock Test Package for Bank & Insurance Exams
If there are any suggestions/ errors in our PDFs Feel Free to contact us via this email: admin@exampundit.in
IBPS RRB PO Prelims – Ultra Practice Bundle PDF

16. Which of the following given bank has highest d) 175


ratio of Savings account to Current accounts?
e) None of these
a) SBI
18. Find the ratio of total number of current
b) PNB accounts opened in SBI and PNB taken together to
the total account opened in BOB.
c) BOB
a) 3:4
d) OBC
b) 3:5
e) Both SBI and OBC
c) 5:7
17. Average of number of savings accounts opened in
SBI and PNB taken together is how much more/less d) 5:9
than number of current accounts opened in BOB?
e) 7:12
a) 100
19. If Rs. 40 and Rs. 60, respectively, are submitted
b) 125 in each saving and current accounts opened in BOB,
then find the total revenue (in Rs.) generated by
c) 150 BOB.
Page 636 of 728

Subscribe the Xpress Video Course & Mock Test Package for Bank & Insurance Exams
If there are any suggestions/ errors in our PDFs Feel Free to contact us via this email: admin@exampundit.in
IBPS RRB PO Prelims – Ultra Practice Bundle PDF
a) Rs. 122500 a) 86.4%

b) Rs. 175000 b) 72.8%

c) Rs. 217500 c) 56.5%

d) Rs. 245000 d) 45.4%

e) None of these e) 36.2%

20. Number of savings accounts opened in OBC is


what percentage more or less than the number of
current accounts opened in BOB?

Directions (21 – 25): Given bar graph shows total number of confirmed cases of COVID-19 and number of
deaths in four different countries. Study the bar graph carefully and answer the questions given below.

China 4000
80000

USA 11000
350000

Italy 17500
130000

Spain 15000
140000

0 50000 100000 150000 200000 250000 300000 350000 400000


Deaths Total Confirmed Cases

𝑵𝒐. 𝒐𝒇 𝑫𝒆𝒂𝒕𝒉𝒔
𝑴𝒐𝒓𝒕𝒂𝒍𝒊𝒕𝒚 𝑹𝒂𝒕𝒆 = × 𝟏𝟎𝟎
𝑵𝒖𝒎𝒃𝒆𝒓 𝒐𝒇 𝑻𝒐𝒕𝒂𝒍 𝑪𝒐𝒏𝒇𝒊𝒓𝒎𝒆𝒅 𝒄𝒂𝒔𝒆𝒔

21. For which country mortality rate is lowest among a) Italy


the given four countries.
b) USA
Page 637 of 728

Subscribe the Xpress Video Course & Mock Test Package for Bank & Insurance Exams
If there are any suggestions/ errors in our PDFs Feel Free to contact us via this email: admin@exampundit.in
IBPS RRB PO Prelims – Ultra Practice Bundle PDF
c) Spain e) 13: 5

d) China 24. death in all countries is what % of china’s total


cases confirmed?
e) USA and China
a) 59.375
22. Total confirmed cases in USA are what percent
more than total deaths in Italy. b) 58.375

a) 1200% c) 56.375

b) 1350% d) 60.375

c) 2100% e) 63.375

d) 1900% 25. If number of confirmed cases in China is


increased by 25% and mortality rate remains same,
e) 1500%
what will be the new number of total deaths in
23. Find out the ratio between mortality rates of China?
Spain to that of China? a) 4400
a) 19: 11 b) 4500
b) 43:14 c) 4600
c) 15:7 d) 5200
d) 14:9 e) 5000
Directions (26 – 30): Study the following bar graph carefully and answer the following questions:

Following bar graph shows percentage of marks obtained by a studentsin his semester exam in five different
subjects. Total marks of exam are 500.

Page 638 of 728

Subscribe the Xpress Video Course & Mock Test Package for Bank & Insurance Exams
If there are any suggestions/ errors in our PDFs Feel Free to contact us via this email: admin@exampundit.in
IBPS RRB PO Prelims – Ultra Practice Bundle PDF

Percentage of Marks
20
18
16
14
12
10
8
6
4
2
0
Maths Science Social Science English Sanskrit

26. What are the marks obtained in Sanskrit? 28. Total marks obtained in all five subjects are –

a) 50 a) 405

b) 60 b) 395

c) 70 c) 375

d) 64 d) 385

e) 72 e) 390

27. Difference between the marks obtained in Maths 29. Find the ratio between the marks obtained in
and English is – Social Science to the marks obtained in five subjects.

a) 30 a) 21 : 79

b) 15 b) 19 : 79

c) 25 c) 21 : 67

d) 40 d) 19 : 67

e) 20 e) 23 : 67
Page 639 of 728

Subscribe the Xpress Video Course & Mock Test Package for Bank & Insurance Exams
If there are any suggestions/ errors in our PDFs Feel Free to contact us via this email: admin@exampundit.in
IBPS RRB PO Prelims – Ultra Practice Bundle PDF
30. The marks obtained in Science is what percent of c) 20.5%
total marks obtained in all five subjects?
d) 19.75%
a) 20.75%
e) 19.25%
b) 20.25%
Directions (31 – 35): Answer the questions based on the information given below.

The bar graph given below shows the number of window AC’s and split AC’s manufactured by four different
companies.

350

300

250

200

150

100

50

0
A B C D
Window AC Split AC

31. The number of window AC’s manufactured by A e) None of these


is what percent of the number of split AC’s
manufactured by it. 32. Find the ratio of the number of windows to split
AC’s manufactured by company C.
a) 56.25%
a) 4:5
b) 60%
b) 3:5
c) 62.5%
c) 4:7
d) 50%
d) 5:7
Page 640 of 728

Subscribe the Xpress Video Course & Mock Test Package for Bank & Insurance Exams
If there are any suggestions/ errors in our PDFs Feel Free to contact us via this email: admin@exampundit.in
IBPS RRB PO Prelims – Ultra Practice Bundle PDF
e) None of these b) 50

33. Find the average number of split AC’s c) 60


manufactured by each of the given company.
d) 30
a) 235
e) 20
b) 225
35. Find the total number of windows AC’s
c) 205 manufactured by all the given companies together.

d) 215 a) 850

e) None of these b) 800

34. Find the difference between the number of c) 720


window AC’s manufactured by companies C and D
d) 700
together, and the number of split AC’s manufactured
by companies B and C together. e) None of these
a) 40

Directions (36 – 40): The following bar graph shows the number of tickets booked (in hundred's) for train, bus
and cruise in the last five months of 2019.

Number of Tickets booked (in 00's)


50
40
30
20
10
0
August September October November December
Train Bus Cruise

Page 641 of 728

Subscribe the Xpress Video Course & Mock Test Package for Bank & Insurance Exams
If there are any suggestions/ errors in our PDFs Feel Free to contact us via this email: admin@exampundit.in
IBPS RRB PO Prelims – Ultra Practice Bundle PDF
36. By what percentage approximately is the total b) 800
number of tickets booked in October less than the
c) 300
total number of tickets booked in November?

a) 55% d) 500

b) 45% e) 100

c) 42% 39. What is the approximate percentage of the total


number of cruise tickets booked in September,
d) 58% October and November out of the total number of
cruise tickets booked in all the five months?
e) 50%
a) 68%
37. What is the ratio of the total number of train
tickets booked in August and September to the total b) 63%
number of bus tickets booked in November and
c) 58%
December respectively?

a) 18:29 d) 61%

b) 2:3 e) 65%

c) 9:14 40. What is the ratio of the total number of tickets


booked in August to the total number of bus tickets
d) 12:19 booked in October and December respectively?

e) 4:9 a) 29:11

38. What is the difference between the total number b) 19:11


of tickets booked in September and the total number
c) 29:22
of bus tickets booked in August and November
together? d) 14:11
a) 1000 e) 19:10

Directions (41 – 45): Study the data carefully and answer the following questions.

Bar Graph given below shows the number of cars produced (in thousands) by five different companies Maruti,
Suzuki, Tata Motors, Honda and Renault India in three different years.
Page 642 of 728

Subscribe the Xpress Video Course & Mock Test Package for Bank & Insurance Exams
If there are any suggestions/ errors in our PDFs Feel Free to contact us via this email: admin@exampundit.in
IBPS RRB PO Prelims – Ultra Practice Bundle PDF

Renault India

Honda

Tata Motors

Suzuki

Maruti

40 45 50 55 60 65 70
2018 2017 2016

41. Total car produced by Maruti, Suzuki and Honda b) 6000


in the year 2016 together is what percent more or less
c) 4000
than the sum of the total car produced by Tata
motors in 2017 and Maruti in 2018? d) 7000
a) 20% e) 5500
b) 24% 43. Find the ratio of the total car produced by
c) 26% Renault India in 2017 and 2018 together to the sum
of total car produced by Suzuki in 2016 and 2017
d) 25% together with total car produced by Honda in 2017
and 2018 together?
e) 28%
a) 7 : 8
42. The average number of cars produced by Maruti,
Renault India and Tata motors in 2018 is how much b) 5 : 8
more or less than the average number of cars
c) 8 : 3
produced by Tata motors and Honda in 2017?

a) 5000 d) 8 : 9

e) 5 : 9
Page 643 of 728

Subscribe the Xpress Video Course & Mock Test Package for Bank & Insurance Exams
If there are any suggestions/ errors in our PDFs Feel Free to contact us via this email: admin@exampundit.in
IBPS RRB PO Prelims – Ultra Practice Bundle PDF
44. If in 2019 total car produced by Suzuki and Tata 45. Total production produced by Suzuki and
increases by 10% and 30% respectively over 2018 Renault India together in these three years is what
and total car produced by Honda decreases by 40% percent of the total car produced by Maruti and Tata
in 2019 over 2018, then find the total production of together in these three years?
these three companies in 2019?
a) 97 1/7%
a) 170000
b) 98 1/7%
b) 180000
c) 98 1/3%
c) 160000
d) 97 1/11%
d) 174000
e) 97 6/7%
e) 168000

Directions (46 – 50): The following bar graph represents the profit percentage of company A and B during the
period 2015 to 2019.

B A

2019

2018

2017

2016

2015

15 20 25 30 35 40 45 50

Page 644 of 728

Subscribe the Xpress Video Course & Mock Test Package for Bank & Insurance Exams
If there are any suggestions/ errors in our PDFs Feel Free to contact us via this email: admin@exampundit.in
IBPS RRB PO Prelims – Ultra Practice Bundle PDF
𝑰𝒏𝒄𝒐𝒎𝒆 − 𝑬𝒙𝒑𝒆𝒏𝒅𝒊𝒕𝒖𝒓𝒆
𝑷𝒓𝒐𝒇𝒊𝒕% = × 𝟏𝟎𝟎
𝑬𝒙𝒑𝒆𝒏𝒅𝒊𝒕𝒖𝒓𝒆

46. If the expenditure of company A in year 2016 was b) 350 crore


240 crore and income of company B was 210 crore in
c) 400 crore
the same year, then the difference between the
income of company A and the expenditure of d) Cannot be determined
company B in that year was:
e) None of these
a) 170 crore
49. If Income of company A in the year 2018 is 280
b) 180 crore crore, then find the expenditure in the same year.
c) 174 crore a) 200 crore
d) 154 crore b) 210 crore
e) 164 crore c) 220 crore
47. The ratio of expenditure of company A to d) 240 crore
company B was 2:1 in year 2018. What was the ratio
of Income of company A to company B in same year? e) None of these

a) 73:38 50. If the difference between Income and expenditure


of company B in the year 2019 was 18 crore, then
b) 28: 19 find the Expenditure of company B in the same year.
c) 8: 7 a) 90 Crore
d) 56: 29 b) 80 Crore
e) None of these c) 75 Crore
48. If the income of company A in year 2015 was 300 d) 60 Crore
crores, then what was the expenditure of company A
in year 2017? e) None of these

a) 200 crore

Page 645 of 728

Subscribe the Xpress Video Course & Mock Test Package for Bank & Insurance Exams
If there are any suggestions/ errors in our PDFs Feel Free to contact us via this email: admin@exampundit.in
IBPS RRB PO Prelims – Ultra Practice Bundle PDF
7). Bar Graph DI - Solutions with Explanation:

Solution (1-5): 5. Answer: D)

1. Answer: C) Total number of Ponds creams sold = 360 + 310 + 450 =


1120
The quantity of L’Oreal sold in November = 510
Total number of VLCC creams sold = 420 + 430 + 540
Total number of all brand creams sold in November = = 1390
360 + 420 + 320 + 290 + 510 + 480 = 2380
Total number of Lakme creams sold = 320 + 520 + 270
∴ The required percentage = (510/2380) × 100 = = 1110
21.428% ≈ 21.43%.
Total number of Lotus creams sold = 290 + 550 + 380 =
2. Answer: B) 1220
Total creams sold in December = 310 + 430 + 520 + 550 Total number of L’Oreal creams sold = 510 + 480 +
+ 480 + 350 = 2640. 420 = 1410
∴ The average number of creams sold in December = Total number of Ayur creams sold = 480 + 350 + 500 =
2640/6 = 440. 1330
3. Answer: C) Hence, the sale of L’Oreal creams is maximum in given
The Lotus creams sold in January = 380 months

Total number of creams sold in January = 450 + 540 + Solution(6 – 10):


270 + 380 + 420 + 500 = 2560 Common Explanation:
∴ The required ratio = 380 : 2560 = 19 : 128. Number of
Total Number of
India
4. Answer: C) number of Filmfare’s
today’s
magazines magazines
The quantity of VLCC sold in January = 540 magazines
sold sold
sold
Total number of all brand creams sold in January = 0.75 × 6000 6000 – 4500
Delhi 6000
450+ 540 + 270 + 380 + 420 + 500 = 2560 = 4500 = 1500
0.4 × 4500 = 4500 – 1800
∴ The required percentage = (540/2560) × 100 = 21.09% Pune 4500
1800 = 2700

Page 646 of 728

Subscribe the Xpress Video Course & Mock Test Package for Bank & Insurance Exams
If there are any suggestions/ errors in our PDFs Feel Free to contact us via this email: admin@exampundit.in
IBPS RRB PO Prelims – Ultra Practice Bundle PDF
0.35 × 8000 8000 – 2800
Nagpur 8000
= 2800 = 5200 2012 43000 47000 44000 134000
0.6 × 5000 = 5000 – 3000
Chennai 5000
3000 = 2000
2013 48000 45000 42000 135000
0.8 × 4000 = 4000 – 3200
Cuttack 4000
3200 = 800
2014 47000 52000 49000 148000
6. Answer: B)

Required percentage = {(3200 – 2000)/3200} × 100 = 2015 46000 40000 54000 140000
37.5%

7. Answer: A) Total 229000 234000 232000


Required average = (5200 + 800)/2 = 3000

8. Answer: E)
11. Answer: B)
Required ratio = 4500:3000 = 3:2
In 2012, the average selling price of all 3 portable
9. Answer: D) laptops was minimum as the selling price was the
minimum.
Required difference = 8000 – (1500 + 2700) = 3800
12. Answer: D)
10. Answer: C)
Total selling price of 25000 HP laptops in 2013 = 25000
Required sum = 1800 + 2800 = 4600 * 42000 = 105 crores
Solution (11 – 15): Total selling price of 24000 Lenovo laptops in 2013 =
24000 * 48000 = 115.2 crores

Therefore, required difference = 10.2 crores


Common Explanation:
13. Answer: C)
Year Lenovo Asus HP Total The selling price of all the companies during 2011-2015
can be calculated as follows:

2011 45000 50000 43000 138000 Selling price for Lenovo during 2011-2015 = 45000 +
43000 + 48000 + 47000 + 46000 = 229000

Page 647 of 728

Subscribe the Xpress Video Course & Mock Test Package for Bank & Insurance Exams
If there are any suggestions/ errors in our PDFs Feel Free to contact us via this email: admin@exampundit.in
IBPS RRB PO Prelims – Ultra Practice Bundle PDF
Selling price for Asus 2011-2015 = 50000 + 47000 + Solution (16-20):
45000 + 52000 + 40000 = 234000
Common Explanation:
Selling price for HP 2011-2015 = 43000 + 44000 +
42000 + 49000 + 54000 = 232000

Since Total selling price for Asus is the maximum that


means average selling price would also be the maximum
for Asus.

Hence Asus has the maximum average selling price


during 2011-2015

14. Answer: A)

Purchasing price of Lenovo for the distributor = 47000 *


40 = 18.8 lakhs

Purchasing price of Asus for the distributor = 52000 *


60 = 31.2 lakhs

Purchasing price of HP for the distributor = 49000 * 80 16. Answer: A)


= 39.2 lakhs
17. Answer: B)
So, total purchasing price for the distributor = 89.2 lakhs
Average of number of savings accounts opened in SBI
Total selling price for 180 laptops = 50400 * 180 = and
90.72 lakhs
PNB taken together = (1500 + 2000)/2 = 1750
Hence Profit = 90.72 - 89.2 = 1.52
Therefore, required difference = 1875 – 1750 = 125
Therefore, profit percentage = (90.72 - 89.2) / 89.2 *
100 = 1.70% 18. Answer: D)

15. Answer: E) Required ratio = (900 + 1600): 4500 = 2500: 4500 = 5: 9

Selling price of Asus laptops in 2016 = 45000 x 1.3 = 19. Answer: C)


58500 Therefore, required revenue = 2625 × 40 + 1875 × 60 =
Selling price of HP laptops in 2013 = 42000 105000 + 112500 = Rs. 217500
Therefore, percentage = 58500 /42000 * 100 = 139.28%
Page 648 of 728

Subscribe the Xpress Video Course & Mock Test Package for Bank & Insurance Exams
If there are any suggestions/ errors in our PDFs Feel Free to contact us via this email: admin@exampundit.in
IBPS RRB PO Prelims – Ultra Practice Bundle PDF
20. Answer: B)

Therefore, required percentage = [(3240 – 1875)/1875]


× 100

= 72.8%
Solution (26 -30):
Solution (21 – 25):
Common Explanation:
Common Explanation:
26. Answer: B)

Percentage of marks obtained in Sanskrit = 12

Total marks of an exam = 500

∴ Marks obtained in Sanskrit = 12/100 × 500 = 60

27. Answer: E)
21. Answer: B)
Total marks of an exam = 500
USA has lowest mortality rate, which is 3.14%
Percentage marks obtained in Maths = 18%
22. Answer: D)
⇒ Marks obtained in Maths = 18/100 × 500 = 90

Percentage marks obtained in English = 14%

23. Answer: C) ⇒ Marks obtained in English = 14/100 × 500 = 70

∴ Required Difference = 90 – 70 = 20

28. Answer: B)
24. Answer: A) Total marks of an exam = 500

Total marks obtained in all five subjects is

⇒ (18/100 + 16/100 + 19/100 + 14/100 + 12/100) × 500


25. Answer: E)
⇒ 395

29. Answer: B)

Page 649 of 728

Subscribe the Xpress Video Course & Mock Test Package for Bank & Insurance Exams
If there are any suggestions/ errors in our PDFs Feel Free to contact us via this email: admin@exampundit.in
IBPS RRB PO Prelims – Ultra Practice Bundle PDF
Total marks of an exam = 500 Required difference = (120 + 300) – (170 + 210) = 40

Total marks obtained in all five subjects is 35. Answer: E)

⇒ (18/100 + 16/100 + 19/100 + 14/100 + 12/100) × 500 Required total number of window AC’s = (180 + 150 +
120 + 300) = 750
⇒ 395
Solution (36 – 40):
Marks obtained in Social Science = 19/100 × 500 = 95
Common Explanation:
∴ Required ratio = 95 : 395 = 19 : 79
In August:
30. Answer: B)
Number of bus tickets booked =4000
Total marks of an exam = 500
Number of train tickets booked =1200
Total marks obtained in all five subjects is
Number of cruise tickets booked =600
⇒ (18/100 + 16/100 + 19/100 + 14/100 + 12/100) × 500
Total number of tickets booked in August = 5800
⇒ 395
Similarly, we calculate for other months and tabulate the
Marks obtained in Science = (16/100) × 500 = 80 data as follows:
∴ required percentage = (80/395) × 100 = 20.25% Train Bus Cruise
Month Total
tickets tickets tickets
Solution (31-35):
August 1200 4000 600 5800
Common Explanation:
September 2400 3500 1000 6900
31. Answer: A)
October 1100 1600 500 3200
Required percentage = 180/320 × 100 = 56.25%

32. Answer: C) November 4200 3000 400 7600

Required ratio = 120:210 = 4:7 December 3200 2800 500 6500

33. Answer: D)
36. Answer: D)
Required average = (320 + 170 + 210 + 160)/4 = 215
Total number of tickets booked in October = 3200
34. Answer: A)
Total number of tickets booked in November = 7600
Page 650 of 728

Subscribe the Xpress Video Course & Mock Test Package for Bank & Insurance Exams
If there are any suggestions/ errors in our PDFs Feel Free to contact us via this email: admin@exampundit.in
IBPS RRB PO Prelims – Ultra Practice Bundle PDF
Percentage = (7600 - 3200)/7600 * 100% = 57.8% ~ Common Explanation :
58%
41. Answer: D)
37. Answer: A)

Total number of train tickets booked in August and


September = 1200 + 2400 = 3600

Total number of bus tickets booked in November and


December = 3000 + 2800 = 5800

Ratio = 3600:5800 = 18:29


42. Answer: A)
38. Answer: E)

Total number of tickets booked in September = 6900

Total number of bus tickets booked in August and


November = 4000 + 3000 = 7000

Required Difference = 100

39. Answer: B)

Total number of cruise tickets booked in September, 43. Answer: B)


October and November = 1000 + 500 + 400 = 1900
Total car produced by Renault India in 2017 & 2018
Total number of cruise tickets booked in all the five together = 60000 + 65000
months = 1900 + 600 + 500 = 3000
= 125000
Required Percentage = 1900/3000 * 100% = 63.33% ~
And sum of the total car produced by Suzuki in 2016 &
63%
2017 and Honda in 2017 & 2018 together =55000 +
40. Answer: C) 50000 + 45000 + 50000

Total number of tickets booked in August = 5800 = 200000

Total number of bus tickets booked in October and Required ratio =125: 200
December = 1600 + 2800 = 4400
=5 : 8
Required Ratio = 5800:4400 = 29:22
44. Answer: D)
Solution (41-45):
Page 651 of 728

Subscribe the Xpress Video Course & Mock Test Package for Bank & Insurance Exams
If there are any suggestions/ errors in our PDFs Feel Free to contact us via this email: admin@exampundit.in
IBPS RRB PO Prelims – Ultra Practice Bundle PDF

45. Answer: A)

48. Answer: D)
Solution (46-50): We cannot stabilize relation between in year 2015 and in
year 2017. So we cannot determine the answer.
Common Explanation :

46. Answer: C) 49. Answer: A)

Crore

Crore

50. Answer: A)
Difference = 324 – 150 =174 crore

47. Answer: D)

Crore

Page 652 of 728

Subscribe the Xpress Video Course & Mock Test Package for Bank & Insurance Exams
If there are any suggestions/ errors in our PDFs Feel Free to contact us via this email: admin@exampundit.in
IBPS RRB PO Prelims – Ultra Practice Bundle PDF
8). DI Based on Line Graph

Directions (1 – 5): Study the following information and answer the following questions:
There are 15000 students applied for Scholarship in 2019. The line graph given below shows the %
distribution of the studentswho applied from 5 different state, also the second line graph show the % of
studentswho got scholarship in 2019 out of total number of students who applied from the same state.

12%

35% 20%
10% 18%
12%
22%
15% 16%
12%

UP MP HP UK MH

% of student who got scholoar ship out of total number of student who applied
% distribution of total number of student who applied for Scholarship

1. Find the ratio between number of studentswho got scholarship from HP and UK?
a) 33:18
b) 35:18
c) 31:18
d) 18:31
e) None of the above
2. Total number of studentswho got Scholarship from MP is what % of the number of studentswho applied for
scholarship from MP?
a) 41%
b) 23%
c) 12%

Page 653 of 728

Subscribe the Xpress Video Course & Mock Test Package for Bank & Insurance Exams
If there are any suggestions/ errors in our PDFs Feel Free to contact us via this email: admin@exampundit.in
IBPS RRB PO Prelims – Ultra Practice Bundle PDF
d) 23%
e) None of these
3. Find the average number of studentswho got scholarship from all the state together?
a) 406.8
b) 590.8
c) 525.8
d) 530.8
e) None of these
4. Total number of studentswho applied for scholarship from the state of Delhi is 20% more than the number
of studentswho applied from UP, while % of studentswho got scholarship out of total number of studentswho
applied from Delhi is 25%. Then find out how many students from Delhi got the scholarship?
a) 990
b) 880
c) 920
d) 920
e) None of these
5. If one studentsfrom HP got the scholarship of rupees 5000 then find out total amount of scholarship
distributed amongst the studentsof HP?
a) 3150000
b) 2880000
c) 2750000
d) 2500000
e) None of these

Directions (6 – 10): Answer the questions based on the information given below.
Line graph given below show the data regarding total number of death in 6 different states due to ongoing
Covid-19 Pandemic. While another line of graph show the % of death out of total infected People from the
same state.
Page 654 of 728

Subscribe the Xpress Video Course & Mock Test Package for Bank & Insurance Exams
If there are any suggestions/ errors in our PDFs Feel Free to contact us via this email: admin@exampundit.in
IBPS RRB PO Prelims – Ultra Practice Bundle PDF
Note = % of death = Total number of death / Total number of infected people *100

1750
1600
1500
1400
1250 1200

25 10 20 35 10 17.5
A B C D E F

Totla number of Death % of Death

6. Find the ratio between total number of people form 8. Find the average number of people form A, C and
state E and D, who are infected form Covid-19 in E state together in 2019 that are infected form Covid-
2019? 19?
a) 2:3 a) 26000/3
b) 2:1 b) 10060/3
c) 3:1 c) 22400/3
d) 1:3 d) 20630/3
e) None of these e) None of these
7. Total numbers of death form state C and D 9. Average number of deaths form A, C and D is what
together is how much percentage more or less than % of total number of death form B, E and F state
the total number of death form State A and B together in 2019?
together? a) 40.20%
a) 13.5% b) 22.72%
b) 14% c) 25.71%
c) 10% d) 35.71%
d) 17% e) None of these
e) None of these
Page 655 of 728

Subscribe the Xpress Video Course & Mock Test Package for Bank & Insurance Exams
If there are any suggestions/ errors in our PDFs Feel Free to contact us via this email: admin@exampundit.in
IBPS RRB PO Prelims – Ultra Practice Bundle PDF
10. Total number of people who are infected form a) 480
Covid-19 from the state of M is 25% more than the b) 450
total number of people who are infected from state D. c) 440
and death % in state M is 20% less than the death% d) 450
of state E. Then find out number of Deaths from State e) None of these
M in 2019?
Directions (11 – 15): Read the Data carefully and answer the Question given below.
Total number of Vote casted in 6 state of India election 2020 is 125000. Line graph given below shows the %
distribution of total number of vote casted by 6 different states, while another line graph shows the % of
invalid vote out of total vote casted in the same state in 2020.

% distribution of totla number of Vote casted % of invalid voted casted out of totla vote casted

30

25 25
22
20 20
18 18
15 14 15
13

Maharasthra Gujrat Delhi Tamil Nadu West Bangal UP

Note – Total Vote casted = Total valid vote + Total invalid vote casted

Page 656 of 728

Subscribe the Xpress Video Course & Mock Test Package for Bank & Insurance Exams
If there are any suggestions/ errors in our PDFs Feel Free to contact us via this email: admin@exampundit.in
IBPS RRB PO Prelims – Ultra Practice Bundle PDF
11. Find the difference between total number of valid c) 91.27%
vote casted by Delhi and UP together to total votes d) 104.167%
casted by Maharashtra and Gujrat together? e) Can’t be determined
a). 25500 14. Total number of Vote casted in Tamil Nadu is
b). 22500 how much more or less than the total number of vote
c). 20500 casted in the Delhi and UP state together in 2020?
d). 28500 a) 18000
e). 27500 b) 12000
12. Valid Vote casted is maximum in which of the c) 9000
following state in the year 2020? d) 15000
a) Delhi e) None of these
b) Maharashtra 15.Total valid vote and casted by Female from the
c) UP state of Delhi s 50% more than the total valid vote
d) West Bengal casted by male than find out the ratio between valid
e) None of these vote casted by male to total number of invalid vote
13. Average number of invalid vote casted in Tamil casted from the same state in 2020?
Nadu and West Bengal together is what percentage of a) 11:12
average number of invalid vote casted in UP and b) 1:2
Maharashtra together? c) 2:1
a) 110.275% d) 1:1
b) 88.57% e) None of these
Directions (16-20): Answer the questions based on the information given below.
There are 1500 studentsstudying in 6 different classes. Line graph given below shows the % distribution on
total Male studentsand Female studentsstudying in the same class.
Note – Difference between total female studentsand Total male studentsform Class V is 34.

Page 657 of 728

Subscribe the Xpress Video Course & Mock Test Package for Bank & Insurance Exams
If there are any suggestions/ errors in our PDFs Feel Free to contact us via this email: admin@exampundit.in
IBPS RRB PO Prelims – Ultra Practice Bundle PDF
30%

25%
24%

20% 20% 20%


18% 18%
17%
16%
15% 15%
12% 12%
10% 10%

5%

0%
Class I Class II Class III Class IV Class V Class VI

% distribution of Male student % distributio of Female student

16. Find the ratio between male studentsstudying in e) None of these


Class III and IV together to total number of Female 18. Total number of male studentswho studying in
studentsstudying in class I and VI together? class VI is how much % more or less than the total
a) 231:272 number of studentswho studying class III?
b) 356:271 a) 57.93%
c) 290:371 b) 45.92%
d) 255:231 c) 55.93%
e) None of the above d) 55.50%
17. Find the difference between total number of e) Can’t be determined
studentsstudying in class II and Class III? 19. Average number of studentswho studying in Class
a) 64 II and III is how much more or less than the total
b) 58 number of studentsstudying in class III and IV?
c) 16 a) 440
d) 12 b) 640

Page 658 of 728

Subscribe the Xpress Video Course & Mock Test Package for Bank & Insurance Exams
If there are any suggestions/ errors in our PDFs Feel Free to contact us via this email: admin@exampundit.in
IBPS RRB PO Prelims – Ultra Practice Bundle PDF
c) 780 a) 1120
d) 450 b) 640
e) None of these c) 540
20. Find the total number of studentsstudying in d) 440
Class III, IV and V together? e) None of these

Directions (21 – 25): Answer the questions based on the information given below.
Five different sellers sell two types of food packets i.e. vegetarian and non-vegetarian food. The Line graph
given below shows the total number of food packet sold, and the % of the number of Vegetarian food packet
sold by each seller.
6000

5000 5000 4800


4000
3600
3300
3000
2700
2000

1000

0 25% 35% 55% 60% 45%


A B C D E

Total Number of Food Packet sold % of Vegetarian food packet sold

21. The number of Vegetarian food packet sold by C e) None of these


is how much percent more or less than the number of 22. The price of the vegetarian and non-vegetarian
non-vegetarian food packet sold by E? food packet is 40 and 50 rupees respectively. The find
a) 4.167% more out the total amount collected by D is how much more
b) 4.525% more or less than the total amount collected by C after
c) 2.151% more selling all the units of food packets?
d) 5.012% more a) 123700
Page 659 of 728

Subscribe the Xpress Video Course & Mock Test Package for Bank & Insurance Exams
If there are any suggestions/ errors in our PDFs Feel Free to contact us via this email: admin@exampundit.in
IBPS RRB PO Prelims – Ultra Practice Bundle PDF
b) 103700 a) 1165
c) 105700 b) 2167
d) 122570 c) 3251
e) None of these d) 2157
23. Find the ratio between numbers of Vegetarian e) None of these
food packet sold by D to number of non-vegetarian 25. Difference between Vegetarian and non-
food packet sold by C? vegetarian food Packet is maximum for which of the
a) 18:25 following seller?
b) 15:18 a) D
c) 14:23 b) B
d) 15:28 c) C
e) 15:51 d) E
24. Find the average number of non-vegetarian food e) None of these
packet sold by all sellers together?
Directions (26-30) : Answer the question based on the information given below.
The Given Bar graph shows the number of Male and female people from five different city. male and female
from each city earn rupees 15000 and rupees 20000 respectively.

Page 660 of 728

Subscribe the Xpress Video Course & Mock Test Package for Bank & Insurance Exams
If there are any suggestions/ errors in our PDFs Feel Free to contact us via this email: admin@exampundit.in
IBPS RRB PO Prelims – Ultra Practice Bundle PDF
140

120 120

100 100 100


90
85
80 80
75
70
60 60
55
40

20

0
A B C D E

Total Males Total Female

Note –Earning of male and female is same in each city.

26. Average number of male people from all the city c) 720000
together is how much percentage more or less than d) 750200
the average number of female form city A and B e) 805000
together? 28. Find the ratio between total number of male from
a) 80% City A, B and C together to total number of Female
b) 82% from D, E and A?
c) 70% a) 1:3
d) 72% b) 5:4
e) None of these c) 1:1
27. Total earning of city A is how much more or less d) 2:1
than total earning of city B? e) None of these
a) 750000 29. Total earning of Female is maximum form which
b) 700000 of the following City?

Page 661 of 728

Subscribe the Xpress Video Course & Mock Test Package for Bank & Insurance Exams
If there are any suggestions/ errors in our PDFs Feel Free to contact us via this email: admin@exampundit.in
IBPS RRB PO Prelims – Ultra Practice Bundle PDF
a) D a) 22.22%
b) C b) 17.17%
c) B c) 18.28%
d) A d) 15.15%
e) None of these e) None of these
30. Total income of male form City A and B is what
percentage more or less than the total Income of
female form C and D together?
Directions (31 – 35): Answer the question based on the information given below
Total population of five different villages is X. Line graphs given below show the % of literate population out
of total population of particular village and difference between literate and illiterate population from each
village
250

200
192

150
140

114
100 100

50 50
25 30
15 12 18
0
Village A Village B Village C Village D Village E

% of Literate population out of total populaion Difference betweeen litrate and illitrate population

Note – Total population = Literate + Illiterate population


31. What is the value of X? b) 1200
a) 1000 c) 1650

Page 662 of 728

Subscribe the Xpress Video Course & Mock Test Package for Bank & Insurance Exams
If there are any suggestions/ errors in our PDFs Feel Free to contact us via this email: admin@exampundit.in
IBPS RRB PO Prelims – Ultra Practice Bundle PDF
d) 1700 34. Total number of population in the village F is
e) None of these 20% more than the total number of population from
32. Average number of literate population form village C, while total number of illiterate population
village A and B is how much more or less than the from village F is 20% less than that of village B. Then
average number of illiterate population form village find out total literate population form village F?
C and D together? a) 53
a) 162 b) 44
b) 161 c) 55
c) 112 d) 65
d) 115 e) None of these
e) None of these 35. Ratio of Literate male and female form village E
33. Literate population of village A is how much is 2:1. Then find out the difference between literate
percentage more or less than the illiterate population male and female from village E?
of village B? a) 35
a) 85.29% b) 25
b) 85.55% c) 15
c) 112.25% d) 55
d) 115.55% e) None of the above
e) None of these
Directions (36 – 40): Answer the question based on the information given below
The Line graph given below shows the ratio of factory workers who are working in day to a night shift worker
of the five different factories. It is also shows the total number of worker in these companies.

Page 663 of 728

Subscribe the Xpress Video Course & Mock Test Package for Bank & Insurance Exams
If there are any suggestions/ errors in our PDFs Feel Free to contact us via this email: admin@exampundit.in
IBPS RRB PO Prelims – Ultra Practice Bundle PDF

2574

1782
1680
1563 1592

2 0.8 1.25 1 0.875


A S D F G

Total number of Workers


ratio between number of worker workin in day shift to number of worker working in night shift

Note : Total number of worker in the factory = Total number of worker working in Day shift + Total number
of worker working in day shift.
36. What is the ratio of number of worker working in d) 575
Day shift from factory G to same in factory D? e) None of these
a) 392:715 38. Number of workers who are working in night
b) 405:713 shift from factory S is how much percentage more or
c) 552:719 less than the same form factory F?
d) 509:663 a) 27.80% Less
e) None of these b) 15.80% Less
37. What is the difference between the number of c) 22.80% More
workers who are working in night shift from factory d) 24.37% More
D and that of Factory A? e) 25.50% More
a) 552 39. What is the average number of worker in each of
b) 625 the given five factories?
c) 623 a) 1838

Page 664 of 728

Subscribe the Xpress Video Course & Mock Test Package for Bank & Insurance Exams
If there are any suggestions/ errors in our PDFs Feel Free to contact us via this email: admin@exampundit.in
IBPS RRB PO Prelims – Ultra Practice Bundle PDF
b) 1268 a) 1180
c) 1258 b) 1530
d) 1548 c) 1240
e) None of these d) 3130
40. What is the total number of workers who are e) 1826
working in Day shift from factories A and G
together?
Directions (41 – 45) – Answer the question based on information given below.
The line graph given below shows the number of male visitors and difference between male and female visitors
to the Statue of Unity on different days of a week.
Note – Number of male > Number of Female
500
450 450
420
400
350 350
330
300
250
230
200 200
180 180
150 150
100 100
50
0
Monday Tuesday Wednesday Thursday Friday

Number of Male visitors Difference between male and female visitors

41. If 25% of the number of female visitors on number of male and female visitors on Thursday who
Thursday, and 30% of the male visitors on the same are from Delhi state?
day are belong to Delhi state, then find the sum of a) 186
b) 180

Page 665 of 728

Subscribe the Xpress Video Course & Mock Test Package for Bank & Insurance Exams
If there are any suggestions/ errors in our PDFs Feel Free to contact us via this email: admin@exampundit.in
IBPS RRB PO Prelims – Ultra Practice Bundle PDF
c) 181 e) Can’t be determined
d) 182 44. Ticket of one male and one female on Tuesday is
e) None of these 25 and 22 rupees respectively, then find out total
42. What is the ratio of number of female visitors on amount collected from sell of ticket on Tuesday?
Tuesday to that on Friday? a) 20750
a) 22:29 b) 16750
b) 32:21 c) 21750
c) 25:18 d) 11750
d) 30:31 e) 23750
e) None of these 45. What is the average number of Female visitors on
43. Number of female visitors on Friday and Monday Tuesday, Wednesday, Thursday and Friday
together is how much more or less than the total together?
number of visitors on Monday? a) 220
a) 13 b) 210
b) 55 c) 420
c) 50 d) 330
d) 47 e) None of these

Directions (46 – 50): Answer the question based on information given below.
The line graph given below shows the foreign exchange of a country (in million US $) from 2015 to 2021.

Page 666 of 728

Subscribe the Xpress Video Course & Mock Test Package for Bank & Insurance Exams
If there are any suggestions/ errors in our PDFs Feel Free to contact us via this email: admin@exampundit.in
IBPS RRB PO Prelims – Ultra Practice Bundle PDF
6000

5000 5040

4320
4000
3720
3360
3000 3120
2640 2520
2000

1000

0
2015 2016 2017 2018 2019 2020 2021

Foreign exchange reserve of a country in (Milliion US $)

46. The ratio of number of year, in which the foreign e) Can’t be determined
exchange reserve is above the average to those in 48. For which year the percentage increase of foreign
which the reserve are below the average reserve is? exchange reserve over the previous year is the
a) 3:4 highest?
b) 1:6 a) 2020
c) 2:5 b) 2019
d) 4:3 c) 2018
e) None of these d) 2016
47. The foreign exchange reserve in 2020 was how e) 2015
many time that in 2018? 49. The foreign exchange reserves in 2018 were
a) 2.8 Times approximately what percentage of the average
b) 1.9 Times foreign exchange reserve from all the years together?
c) 1.8 Times a) 91.14%
d) 1.5 Times b) 95.14%
Page 667 of 728

Subscribe the Xpress Video Course & Mock Test Package for Bank & Insurance Exams
If there are any suggestions/ errors in our PDFs Feel Free to contact us via this email: admin@exampundit.in
IBPS RRB PO Prelims – Ultra Practice Bundle PDF
c) 93.99% a) 41.55%
d) 91.91% b) 40.9%
e) 95.85% c) 45.52%
50. What is the percentage increase in the foreign d) 52.25%
exchange reserve in 2016 as compared to previous e) None of these
year?

8). DI Based on Line Graph – Solution with Explanation


1-5: Common Explanation

Studentsapplied for Scholarship in


State Studentswho got scholar ship
2019
UP 15000*22% = 3300 3300*10% = 330
MP 15000*15% = 2250 2250 * 12% = 270
HP 15000*35% = 5250 5250*12% = 630
UK 15000*12% = 1800 1800*18% = 324
MH 15000*16% = 2400 2400*20% = 480

1. Answer: B)

Required ratio = 630:324 = 35:18

2. Answer: C)

Total number of studentswho got scholarship from MP = 270

The number of studentswho applied for scholarship from MP = 2250

Required percentage = 270/2250 * 100 = 12%

3. Answer: A)
Page 668 of 728

Subscribe the Xpress Video Course & Mock Test Package for Bank & Insurance Exams
If there are any suggestions/ errors in our PDFs Feel Free to contact us via this email: admin@exampundit.in
IBPS RRB PO Prelims – Ultra Practice Bundle PDF
Average number of studentswho got scholarship from all the state is = (330+270+630+324+480)/ 5 = 406.8

4. Answer: A)

Total number of studentswho applied form Delhi = 3300*120% = 3960

Total number of studentswho got scholar ship = 3960 * 25% = 990

5. Answer: A)

Total amount of scholarship from HP = 5000 * 630 = 3150000

(6 – 10): Common explanation

State Total number of death Total number of infected people


A 1500 1500/25*100=6000
B 1250 1250/10*100 = 12500
C 1600 1600/20*100 = 8000
D 1400 1400/35*100 = 4000
E 1200 1200/10 * 100 = 12000
F 1750 1750/17.5 * 100 = 10000

6. Answer: C)

Total number of people from state E and D who are infected from Covid-19 = 12000 and 4000 respectively

Required ratio = 12000:4000 = 3:1

7. Answer: E)

Total number of deaths from C and D together = 1600+1400 = 3000

Total number of deaths from state A and B together = 1500+1250 = 2750

Required percentage = 3000-2750/2750 * 100 = 9.09%

8. Answer: D)

Page 669 of 728

Subscribe the Xpress Video Course & Mock Test Package for Bank & Insurance Exams
If there are any suggestions/ errors in our PDFs Feel Free to contact us via this email: admin@exampundit.in
IBPS RRB PO Prelims – Ultra Practice Bundle PDF
Average number of People from A, C and E state who are infected from covid-19 = (6000+8000+12000)/3 = 26000/3

9. Answer: D)

Average number of deaths form A, C and D = (1500+1600+1400)/3 = 1500

Total numbers of death form B, E and F state together = 1250+1200+1750 = 4200

Required percentage = 1500/4200 * 100 = 35.71%

10. Answer: A)

Total number of people who are infected form Covid-19 from the state of M = 125/100 * 4000 = 5000

Death % in state M = 80/100 * 12% = 9.6%

Number of deaths from State M = 5000*9.6% = 480

11-15. (Common Explanation)

Valid vote = Total


State Total Voted Casted Invalid vote casted
vote – Invalid vote
Maharashtra 125000*22% = 27500 27500*15% = 4125 23375
Gujrat 125000*18% = 22500 22500*20% = 4500 18000
Delhi 125000*14% = 17500 17500 * 25% =4375 13125
Tamil Nadu 125000*15% = 18750 18750*20% = 3750 15000
West Bengal 125000*18% = 22500 22500*25% = 5625 16875
UP 125000*13% = 16250 16250*30% = 4875 11375

11. Answer: A)

Total number of valid vote casted by Delhi and UP = 13125+11375 = 24500

Total vote casted by Maharashtra and Gujrat = 27500+22500 = 50000

Required difference = 50000 – 24500 = 25500

12. Answer: B)

Page 670 of 728

Subscribe the Xpress Video Course & Mock Test Package for Bank & Insurance Exams
If there are any suggestions/ errors in our PDFs Feel Free to contact us via this email: admin@exampundit.in
IBPS RRB PO Prelims – Ultra Practice Bundle PDF
Valid voted casted (maximum) = 23375 in the state of Maharashtra

(Note – See Table)

13. Answer: D)

Average number of invalid vote casted in Tamil Nadu and west Bengal = (3750+5625)/2 = 9375/2=4687.5

Average number of invalid vote casted in UP and Maharashtra = (4875+4125)/2 = 9000/2 = 4500

Required percentage = 4687.5/4500 * 100 = 104.167%

14. Answer: D)

Total number of vote casted in Tamil Nadu = 18750

Total number of vote casted in Delhi and UP together = 17500+16250 = 33750

Required difference = 33750 – 18750 = 15000

15. Answer: D)

Total valid vote and casted by Female from the state of Delhi s 50% more than the total valid vote casted by male

ATQ, ratio between valid vote by female and male is = 2:1

And, Valid vote casted by male from Delhi = 13125*1/3=4375

Required ratio is = 4375:4375 = 1:1

16-20. Common Explanation:

Let total number of Male students= 100X

And, Let total number of Female = 100Y

ATQ,

100X + 100Y = 1500

Total Male form V = 100x * 18% = 18X

Total Female form V = 100Y * 20% = 20Y

Page 671 of 728

Subscribe the Xpress Video Course & Mock Test Package for Bank & Insurance Exams
If there are any suggestions/ errors in our PDFs Feel Free to contact us via this email: admin@exampundit.in
IBPS RRB PO Prelims – Ultra Practice Bundle PDF
ATQ,

100X +100Y = 1500 ------ (1)

20Y-18X = 34 ----------- (2)

By solving above equation

X = 7 and Y = 8

Class Total male = 100x = 100*7 = 700 Total Female = 100y = 100*8 =800
Class I 700*20% = 140 800*24% = 192
Class II 700*12% = 84 800*16% = 128
Class III 700*18% = 126 800*18% = 144
Class IV 700*15% = 105 800*12% = 96
Class V 700*18% = 126 800*20% = 160
Class VI 700*17% = 119 800*10% = 80

16. Answer: A)

Male studentsstudying in Class III and IV together = 126+105=231

Female studentsstudying in class I and VI together = 192+80 = 272

Required ratio = 231:272

17. Answer: B)

Difference between total number of studentsstudying in class II and Class III = (84+128) – (126+144) = 58

18. Answer: C)

Total number of male studentswho studying in class VI = 119

Total number of studentswho studying class III = 126 + 144 = 270

Required % = 119-270/270 * 100 = 55.93% Less

Page 672 of 728

Subscribe the Xpress Video Course & Mock Test Package for Bank & Insurance Exams
If there are any suggestions/ errors in our PDFs Feel Free to contact us via this email: admin@exampundit.in
IBPS RRB PO Prelims – Ultra Practice Bundle PDF
19. Answer: E)

Average number of studentswho studying in Class II and III = {(84+128) + (126+144)}/2 = 241

Total number of studentsstudying in class III and IV = 126 + 144 +105+ 96 = 471

Required difference = 471 – 241 = 230

20. Answer: E)

Total number of studentsstudying in class III, IV and V together = (126+144) + (105+96) + (126+160) =757

21-25. (Common Explanation:)

21. Answer: A)

The number of Vegetarian food packet sold by C = 5000*55% = 2750

Non-vegetarian food packet sold by E = 4800*(100-45) % = 2640

Required percentage = 2750-2640/2640 * 100 = 4.167% more

22. Answer: B)

The price of the vegetarian and non-vegetarian food packet is 40 and 50 rupees respectively

Total amount collected by D = 2700*60% * 40 + 2700*40%*50 = 64800+54000 = 118800

Total amount collected by C = 5000*55%*40 + 5000*45%*50 = 110000+112500 = 222500

Required difference = 222500-118800 = 103700

23. Answer: A)

Numbers of Vegetarian food packet sold by D = 2700*60% = 1620

Number of non-vegetarian food packet sold by C = 5000*(100-55) % = 2250

Required ratio = 1620:2250 = 18:25

24. Answer: D)

Average number of Non-vegetarian food packet sold by all the seller =


1/5{3300*75%+3600*65%+5000*45%+2700*40%+4800*55%} = 2157
Page 673 of 728

Subscribe the Xpress Video Course & Mock Test Package for Bank & Insurance Exams
If there are any suggestions/ errors in our PDFs Feel Free to contact us via this email: admin@exampundit.in
IBPS RRB PO Prelims – Ultra Practice Bundle PDF
25. Answer: E)

Difference between vegetarian and Non-vegetarian food from

A = 3300* (75-25) % = 1650

B = 3600 * (65-35) % = 1080

C = 5000* (55-45) % = 500

D = 2700 * (60-40) % = 540

E = 4800* (55-45) % = 480

Required answer is = A = 1650 (maximum difference)

26-30.

26. Answer: E)

Average number of male people from all the city together = (100+80+70+85+60)/5 = 79

Average number of female form city A and B together = (120+100)/2 = 110

Required percentage = 110-79/110 * 100 = 28.18% less

27. Answer: B)

Total earning of A = 100*15000 + 120*20000 = 3900000

Total earning of B = 80*15000+100*20000 = 3200000

Required difference = 3900000-3200000 = 700000

28. Answer: C)

Total number of male from City A, B and C together = (100+80+70) = 250

Female from D, E and A = 75+55+120 = 250

Page 674 of 728

Subscribe the Xpress Video Course & Mock Test Package for Bank & Insurance Exams
If there are any suggestions/ errors in our PDFs Feel Free to contact us via this email: admin@exampundit.in
IBPS RRB PO Prelims – Ultra Practice Bundle PDF
Required ratio = 250:250 = 1:1

29. Answer: D)

Earning of female form

A = 120*20000 = 2400000

B = 100 * 20000 = 2000000

C = 90*20000 = 1800000

D = 75*20000 = 1500000

E = 55*20000 = 1100000

Required answer is A.

(Note: we can also find the answer directly, as maximum number of females is from A = 120. As we know the
earning is same form all the city)

30. Answer: E)

Total income of male form City A and B = (100+80) *15000 = 2700000

Total Income of female form C and D together = (90+75) *20000 = 3300000

Required percentage = 3300000-2700000/3300000 * 100 = 18.18%

31-35. (Common Explanation)

For Village A

Let total population = 100x

Literate population = 100x*25% = 25x

Illiterate population = 100x – 25x = 75x

ATQ, 75x – 25x = 50

Page 675 of 728

Subscribe the Xpress Video Course & Mock Test Package for Bank & Insurance Exams
If there are any suggestions/ errors in our PDFs Feel Free to contact us via this email: admin@exampundit.in
IBPS RRB PO Prelims – Ultra Practice Bundle PDF
X=1

Accordingly

Literate population = 25x = 25

Illiterate population = 75x = 75

For Village B

Let total population = 100Y

Literate population = 100y*15% = 15y

Illiterate population = 100y – 15y = 85y

ATQ, 85y – 15 y = 140

Y=2

Accordingly

Literate population = 15Y = 30

Illiterate population = 85Y = 170

For Village C

Let total population = 100x

Literate population = 100x*12% = 12x

Illiterate population = 100x – 12x = 88x

ATQ, 88x – 12x = 114

X = 1.5

Accordingly

Literate population = 12x = 18

Page 676 of 728

Subscribe the Xpress Video Course & Mock Test Package for Bank & Insurance Exams
If there are any suggestions/ errors in our PDFs Feel Free to contact us via this email: admin@exampundit.in
IBPS RRB PO Prelims – Ultra Practice Bundle PDF
Illiterate population = 88x = 132

For Village D

Let total population = 100x

Literate population = 100x*18% = 18x

Illiterate population = 100x – 18x = 82x

ATQ, 82x-18x = 192

X=3

Accordingly

Literate population = 18x = 54

Illiterate population = 82x = 246

For Village E

Let total population = 100x

Literate population = 100x*30% = 30x

Illiterate population = 100x – 30x = 70x

ATQ, 70x – 30x = 100

X =2.5

Accordingly

Literate population = 30x = 75

Illiterate population = 70x = 175

Village Literate population Illiterate Population


A 25 75
B 30 170
Page 677 of 728

Subscribe the Xpress Video Course & Mock Test Package for Bank & Insurance Exams
If there are any suggestions/ errors in our PDFs Feel Free to contact us via this email: admin@exampundit.in
IBPS RRB PO Prelims – Ultra Practice Bundle PDF
C 18 132
D 54 246
E 75 175

31. Answer: A)

Total population form all the village = 25+75+30+170+18+132+54+246+75+175 = 1000

32. Answer: E)

Average number of literate population from A and B = (25+30)/2 = 27.5

Average number of illiterate population from C and D together = (132+246)/2 = 189

Required difference = 189 – 27.5 = 161.5

33. Answer: A)

Literate population of village A = 25

Illiterate population of village B = 170

Required % = 170-25/170 * 100 = 85.29%

34. Answer: B)

Total number of populations in the village F is 25% more than the total number of populations from village C =
120/100 * (18+132) = 180

Total number of illiterate populations from village F is 20% less than that of village B = 80/100 *170 = 136

Literate population form F = 180 – 136 = 44

35. Answer: B)

Required difference = 75*(2-1)/3 = 25

36-40. (Common Explanation)

Total worker working Total worker working


Factory Total number of workers
in Day shift in Night shift
Page 678 of 728

Subscribe the Xpress Video Course & Mock Test Package for Bank & Insurance Exams
If there are any suggestions/ errors in our PDFs Feel Free to contact us via this email: admin@exampundit.in
IBPS RRB PO Prelims – Ultra Practice Bundle PDF
A 1563 1563*2/3 = 1042 1563*1/3 = 521
S 1782 1782*0.8/1.8 = 792 1782*1/1.8 = 990
D 2574 2574*1.25/2.25 = 1430 2574*1/2.25=1144
F 1592 1592*1/2 = 796 1592*1/2 = 796
G 1680 1680*0.875/1.875 = 784 1680*1/1.875 = 896

36. Answer: A)

Number of workers working in Day shift form G = 784

Number of workers working in Day shift from D = 1430

Required ratio = 784:1430 = 392:715

37. Answer: C)

Difference between the number of workers who are working in night shift from factory D and that of Factory A =
1144-521 = 623

38. Answer: D)

Number of workers who are working in night shift from factory S = 990

Number of workers who are working in night shift from factory F = 796

Required percentage = 990-796/796 * 100 = 24.37% more

39. Answer: A)

Average number of worker working in Each of the five factories = {1563+1782+2574+1592+1680}/5 = 1838
Approx.

40. Answer: E)

Total number of workers who are working in Day shift from factories A and G together = 1042+784 = 1826

41-45. (Common Explanation)

Page 679 of 728

Subscribe the Xpress Video Course & Mock Test Package for Bank & Insurance Exams
If there are any suggestions/ errors in our PDFs Feel Free to contact us via this email: admin@exampundit.in
IBPS RRB PO Prelims – Ultra Practice Bundle PDF
Number of Female
Day Number of male visitors Total visitors
visitors
Monday 230 230-100 = 130 230+130 = 360
Tuesday 450 450-200 = 250 450+250 = 700
Wednesday 350 350-180 = 170 350+170 = 520
Thursday 420 420-180 = 240 420+240 = 660
Friday 330 330-150 = 180 330+180 = 510

41. Answer: A)

Sum of number of male and female visitors on Thursday who are from Delhi state = 420*30% + 240*25% = 126+60
= 186

42. Answer: C)

Ratio of number of female visitors on Tuesday to that on Friday = 250:180 = 25:18

43. Answer: C)

Number of female visitors on Friday and Monday together = 180+130 = 310

Total number of visitors on Monday = 360

Required answer = 360 – 310 = 50

44. Answer: B)

Total amount collected on Tuesday = 450 * 25 + 250 * 22 = 16750

45. Answer: B)

Average number of Female visitors on Tuesday, Wednesday, Thursday and Friday together = (250+170+240+180)/4
= 210

46-50.

46. Answer: A)

Page 680 of 728

Subscribe the Xpress Video Course & Mock Test Package for Bank & Insurance Exams
If there are any suggestions/ errors in our PDFs Feel Free to contact us via this email: admin@exampundit.in
IBPS RRB PO Prelims – Ultra Practice Bundle PDF
Average reserve = (2640+3720+2520+3360+4320+5040+3120)/7 = 3531.43

Number of years in which reserve are more than average = (2016, 2019, 2020) = 3

Number of years in which reserve are less than average = 7 – 3 = 4

Required ratio = 3:4

47. Answer: D)

Required answer = 5040/3360 = 1.5 times

48. Answer: D)

% increase in Foreign exchange as compare to previous year

For

2016 = 3720-2640 / 2640 * 100 = 40.9%

2018 = 3360-2520/2520 * 100 = 33.33%

2019 = 4320-3360/3360 * 100 = 28.57%

2020 = 5040-4320/4320 * 100 = 16.67%

Required answer = 2016 (maximum % increase as compare to previous year)

49. Answer: B)

Average foreign exchange reserve = (2640+3720+2520+3360+4320+5040+3120)/7 = 3531.43

Required answer = 3360/3531.43 * 100 = 95.14 %

50. Answer: B)

Required % increase = 2016 = 3720-2640 / 2640 * 100 = 40.9%

Page 681 of 728

Subscribe the Xpress Video Course & Mock Test Package for Bank & Insurance Exams
If there are any suggestions/ errors in our PDFs Feel Free to contact us via this email: admin@exampundit.in
IBPS RRB PO Prelims – Ultra Practice Bundle PDF
9). DI Based on Pie Chart

Directions (1 – 5): Study the following information and answer the following questions:

The following Pie Chart show the Percentage of Professors teaching six different subjects in University X.

Subject

18% 24%

10%

20%
13%

Chemistry Computer Economics History Hindi Maths

total number of Professors are 1500.

1. Find the ratio between number of teacher who teach history and Hindi together to number of teacher
who teach Economics and Maths?
a) 31:26
b) 25:31
c) 19:25
d) 25:19
e) None of the above
2. Total number of teacher who teach maths and Computer is how much % of number of teacher who
teach history?
a) 211.15%
Page 682 of 728

Subscribe the Xpress Video Course & Mock Test Package for Bank & Insurance Exams
If there are any suggestions/ errors in our PDFs Feel Free to contact us via this email: admin@exampundit.in
IBPS RRB PO Prelims – Ultra Practice Bundle PDF
b) 235.1%
c) 223.53%
d) 232.25%
e) None of these
3. Average number of teachers who teach economics and Maths is how much more or less than the
number of teacher who teach Hindi? (Find approx. value)
a) 83
b) 85
c) 52
d) 53
e) None of these
4. Total number of teachers who teach Geography is 20 more than the average number of teacher who
teach Hindi and Math together. Then find out number of teachers who teach geography is what % of total
number of teacher who teach Computer?
a) 76.67%
b) 67.5%
c) 75%
d) 70%
e) None of the above
5. If % of teacher who teach Economics and Math is interchange with each other. Then find out
number of teacher who teach economic is increase or decrease by how much?
a) 38.46% More
b) 38.80% More
c) 48.46% more
d) 28.46% more
e) None of these
Directions (6 – 10): Answer the questions based on the information given below.

Page 683 of 728

Subscribe the Xpress Video Course & Mock Test Package for Bank & Insurance Exams
If there are any suggestions/ errors in our PDFs Feel Free to contact us via this email: admin@exampundit.in
IBPS RRB PO Prelims – Ultra Practice Bundle PDF
Study the following Pie chart carefully and answer the question. Pie chart given below show the
Average monthly expenditure of an organization under various head.
Total Expenditure = 3750000

EXPENDITURE
Salary to staff Transport subsidy Water Telephone Electricity Medical to Staff Canteen Subsidy Loan to staff

18% 20%

8%
10%

6%

11%
12%

15%

6. What is the amount spent on transport subsidy and canteen subsidy together?
a) 675500
b) 622500
c) 675000
d) 650000
e) None of these
7. What is the difference between the expenditure on salary to staff and that on loans to staff?
a) 85000
b) 72800
c) 72500
d) 171500

Page 684 of 728

Subscribe the Xpress Video Course & Mock Test Package for Bank & Insurance Exams
If there are any suggestions/ errors in our PDFs Feel Free to contact us via this email: admin@exampundit.in
IBPS RRB PO Prelims – Ultra Practice Bundle PDF
e) None of these
8. The amount spent on medical staff is what percentage more or less than the average amount spent on
salary to staff and Electricity?
a) 70.50% less
b) 65.5% less
c) 75.5% less
d) 62.5% less
e) None of these
9. Amount spent on medical and water is increased by 25% to next year, then find out amount spent
on medical and water next year?
a) 796875
b) 897550
c) 798550
d) 768240
e) 755500
10. Amount spent on Canteen Subsidy and Loan to staff is how much % of amount spent on Telephone,
Water and Electricity?
a) 68.42%
b) 61.83%
c) 62.83%
d) 67.55%
e) None of these
Directions (11 – 15): Answer the questions based on the information given below.
Pie chart given below shows the total export of Mango to 13 different countries. Read the data carefully
and answer the given question. Data given below is in degrees.

Page 685 of 728

Subscribe the Xpress Video Course & Mock Test Package for Bank & Insurance Exams
If there are any suggestions/ errors in our PDFs Feel Free to contact us via this email: admin@exampundit.in
IBPS RRB PO Prelims – Ultra Practice Bundle PDF
TOTAL EXPORT = $144000
USA

45*

Other

104.4* Japan

Spain
32.4*
9*
UK
Br
azil
21.6*
10.
Hog Kong Austrelia China
Russia England 18*
10.8* 7.2* 10.8*
28.8* 7.2*

11. Value of export to USA is how much more or less than the value of export of Germany?
a) $5135
b) $5142
c)$514
d). $5040
e) None of these
12. Find the difference of the value of the export of Japan and France together to the value of export of
UK and Taiwan together?
a) $5750
b) $5760
c) $6550
d) $7650
e) None of these
13. The value of export of the England, UK and Taiwan is how much percentage more or less than the

Page 686 of 728

Subscribe the Xpress Video Course & Mock Test Package for Bank & Insurance Exams
If there are any suggestions/ errors in our PDFs Feel Free to contact us via this email: admin@exampundit.in
IBPS RRB PO Prelims – Ultra Practice Bundle PDF
value of export of Brazil, Hog Kong and Australia together?
a) 68.5%
b) 85.6%
c) 95.2%
d) 62.5%
e) 32.5%
14. Find the ratio between total export form USA, Japan and other countries together to total export
from Russia and china together?
a) 125:23
b) 115:23
c) 119:22
d) 103:22
e) None of these
15. Find the 36% of total expenditure form other countries?
a) $15333.6
b) $15233.6
c) $15133.6
d) $15033.6
e) None of these
Directions (16-20): Answer the questions based on the information given below.
Pie chart given below show the percentage distribution of total number of Female employees working in SBI
at different work post like Customer support, Loan, Cashier, RTGS, NEFT and CTS.

Page 687 of 728

Subscribe the Xpress Video Course & Mock Test Package for Bank & Insurance Exams
If there are any suggestions/ errors in our PDFs Feel Free to contact us via this email: admin@exampundit.in
IBPS RRB PO Prelims – Ultra Practice Bundle PDF
Total female employees working in the organization = 14000

TOTAL FEMALE EMPLOYEES


Customer Support
15%

CTS

Loan

12%

Cashier

13%
NEFT
RTGS
22%
10%

Note - Total number of Male employees working in each work post is 20% more than the number of women
in each of the given work post.
16. Find the difference between total number Men and Women working in RTGS and NEFT work
post in SBI?
a) 3696
b) 3356
c) 3290
d) 3255
e) None of the above
17. Number of men working as Cashier in SBI is what percentage more or less than the number of
Men working as CTS in SBI?
a) 55.55%
b) 53.57%
Page 688 of 728

Subscribe the Xpress Video Course & Mock Test Package for Bank & Insurance Exams
If there are any suggestions/ errors in our PDFs Feel Free to contact us via this email: admin@exampundit.in
IBPS RRB PO Prelims – Ultra Practice Bundle PDF
c) 66.52%
d) 53.25%
e) None of these
18. Find the average number of male and female working in SBI from all the work post?
a) 12400
b) 15800
c) 15400
d) 25200
e) Can’t be determined
19. 25 % of Male who work in Loan is promoted as senior loan managers, then find out how many men
does not get promotion form the same?
a) 1240
b) 1580
c) 1540
d) 2520
e) None of these
20. Difference between male and female working as customer support is what percentage more or less than
the difference between male and female working as NEFT?
a) 35.23%
b) 33.25%
c) 33.55%
d) 32.81%
e) None of these
Directions (21 – 25): Answer the questions based on the information given below.
The pie chart given below show the percentage distribution of total number of Employees working in SBI
of different age group

Page 689 of 728

Subscribe the Xpress Video Course & Mock Test Package for Bank & Insurance Exams
If there are any suggestions/ errors in our PDFs Feel Free to contact us via this email: admin@exampundit.in
IBPS RRB PO Prelims – Ultra Practice Bundle PDF
Total number of employees working in SBI is 125500 in year 2019.

TOTAL EMPLOYEES = 125500


40=>50 50 and above
15%
20%

18=>20

30=>40 25%

26%

20=>30

14%

21. What is Total number of employees of age group of more than or equals to 20 year?
a) 94125
b) 92250
c) 94150
d) 92250
e) None of these
22. For which of the following age group number of employees is maximum?
a) 20=>40.
b) 30=>40
c) 40 and above
d) 30 and above
e) None of these
23. Find the total number of employees of age of 40 years and above?
a) 80515
b) 55810
Page 690 of 728

Subscribe the Xpress Video Course & Mock Test Package for Bank & Insurance Exams
If there are any suggestions/ errors in our PDFs Feel Free to contact us via this email: admin@exampundit.in
IBPS RRB PO Prelims – Ultra Practice Bundle PDF
c) 43925
d) 75515
e) 54545
24. For the age group of 20 or above, difference between male and female employees are 2500. Then
number of male employees of age group of 20 or above is how much percentage more or less than the female
of same age group?
a) 52.25%
b) 45.45%
c) 51.25%
d) 54.12%
e) can’t be determined
25. Employees of age group of 50 or above is going to retire next year and also there are new vacancy of
2540 employees next year, then find out total number of employees working in SBI in next year?
a) 115200
b) 108550
c) 109215
d) Can’t be determined
e) None of these

Direction (26-30) Direction: Answer the question based on the information given belowPie chart given below
shows the % distribution of Rummy Player out of total number of Rummy Players in 6 different casinos. There
are only two types of card game is played i.e. Rummy and Tin Pati. Read the data carefully and answer the
question given below. (Total Player = Rummy Player + Tin Pati Player)
Note – Total Rummy Palayer from all the Casino is 2500 more than the total Tin pati player form all
the casino.

Page 691 of 728

Subscribe the Xpress Video Course & Mock Test Package for Bank & Insurance Exams
If there are any suggestions/ errors in our PDFs Feel Free to contact us via this email: admin@exampundit.in
IBPS RRB PO Prelims – Ultra Practice Bundle PDF

RUMMY PLAYER
Q W E R T Y

12
26 %
%
18
%

18 14
% %
12
%

- Percentage distribution of Tin pati palaye is 12%, 18%, 14%, 12%, 18% and 26% for Casino Q, W,
E, R, T, and Y respectively out of total number of Tin pati player.

26. Find the ratio between total number of Rummy players from R and total number of Tin pati player
from same casino. If the Total player from casino R is 5700?
a) 228.225
b) 75:225
c) 271:252
d) 153:222
e) None of these
27. Total number of Tin pati player from T is what % more or less than the total number player form Q
who play the same game?
a) 37.5% more
b) 50% more
c) 55.25% more
d) 35.25% more
e) 25.25% less

Page 692 of 728

Subscribe the Xpress Video Course & Mock Test Package for Bank & Insurance Exams
If there are any suggestions/ errors in our PDFs Feel Free to contact us via this email: admin@exampundit.in
IBPS RRB PO Prelims – Ultra Practice Bundle PDF
28. Find the difference between total number of Rummy players to total number of Tin pati players form
W, E and R?
a) 1200
b) 840
c) 1100
d) 1020
e) 1050
29. If there are 7500 player who play tin pati form R and T together then find out total number of player
who play Rummy from all the casino together?
a) 15500
b) 23500
c) 32200
d) 27500
e) 25500
30. Difference between Rummy player and Tin pati player is maximum for which Casino?
a) R
b) E
c) W
d) Q
e) None of these

Directions (31 – 35): Answer the question based on the following information

Page 693 of 728

Subscribe the Xpress Video Course & Mock Test Package for Bank & Insurance Exams
If there are any suggestions/ errors in our PDFs Feel Free to contact us via this email: admin@exampundit.in
IBPS RRB PO Prelims – Ultra Practice Bundle PDF
Pie chart given below shows population of 6 village. Study the chart carefully & answer the following
question.

TOTAL POPULATION = 37550


L F

14% 12%

20% 22%

14%
18%

31. What is the difference between the population of village F & H together to the population of village K & L
together?
a) 1502
b) 1280
c) 1320
d) 1350
e) None of these
32. In village H, 33.33% are male, 33.33% are female and remaining are transgender. Find the
difference between males and transgender of village H?
a) 0
b) 500
c) 3755

Page 694 of 728

Subscribe the Xpress Video Course & Mock Test Package for Bank & Insurance Exams
If there are any suggestions/ errors in our PDFs Feel Free to contact us via this email: admin@exampundit.in
IBPS RRB PO Prelims – Ultra Practice Bundle PDF
d) 2150
e) None of these
33. Population of village L & G together is how much percent more than the population of village J?
a) 57.14%
b) 47.25%
c) 55.75%
d) 57.55%
e) None of these
34. Average population of G, H & J all together is how much less than the total population of L
and K together?
a) 7008
b) 6008
c) 8008
d) 9008
e) 1108
35. 30% of population of village K doing govt. Job, 40% of population doing Private Job and remaining
are unemployed. Find the number of person not doing govt. job?
a) 7557
b) 5257
c) 49.57
d) 5152
e) None of the above

Directions (36 – 40): Answer the question based on the information given below

Page 695 of 728

Subscribe the Xpress Video Course & Mock Test Package for Bank & Insurance Exams
If there are any suggestions/ errors in our PDFs Feel Free to contact us via this email: admin@exampundit.in
IBPS RRB PO Prelims – Ultra Practice Bundle PDF
The following pie chart gives the marks obtained by Pritam in SBI PO Mains in five subjects. The total
marks obtained is 360. Read the data carefully and answer the question base on it.

TOTLA MARKS OBTAINED = 360


Essay writing
Data Interpretation
55*
70*

CA

90*

English 65*

80*

Note – Data given is in Degree.


36. What percent of the total marks were obtained by Pritam in Data interpretation?
a) 70
b) 60
c) 65
d) 75
e) None of these
37. The marks scored by Pritam in Essay Writing and Data interpretation together is how much more or
less than the marks scored in Reasoning and English together by Pritam?
a) 25
b) 15.5
c) 20
Page 696 of 728

Subscribe the Xpress Video Course & Mock Test Package for Bank & Insurance Exams
If there are any suggestions/ errors in our PDFs Feel Free to contact us via this email: admin@exampundit.in
IBPS RRB PO Prelims – Ultra Practice Bundle PDF
d) 12.5
e) 17.5
38. Marks obtained in English and CA is how much % more or less than the marks obtained in
Data interpretation and Reasoning?
a) 27.82%
b) 25.50%
c) 22.80%
d) 25.93%
e) None of these
39. Passing cutoff of the exam is 80% and Pritam failed the exam by 20 marks. Then find out the
maximum mark of the SBI PO Exam?
a) 475
b) 865
c) 845
d) 695
e) None of these
40. Find the ratio between Total marks obtained in CA and English together to Total marks obtained in
Data interpretation and Essay writing?
a) 17:33
b) 34:27
c) 22:31
d) 17:25
e) 34:25
Direction (41-45) – Answer the question based on information given below.
Pie chart given below shows the data of foreigners who toured of various Places of India. Read the given data
carefully and answer the question based on it.

Page 697 of 728

Subscribe the Xpress Video Course & Mock Test Package for Bank & Insurance Exams
If there are any suggestions/ errors in our PDFs Feel Free to contact us via this email: admin@exampundit.in
IBPS RRB PO Prelims – Ultra Practice Bundle PDF
Total Number of foreigners = 6300

Total Foreign Visitors =


6300

20% 18%

14%
13%

12%
23%

Taj Mahal Elephanta Statue of Unity Red Hawa Mahal Shastra


Cave fort Dahra

41. What is the number of male foreigners who toured of Taj Mahal and Statue of Unity together, it is
also know that ratio between male and female foreigner who visited Taj Mahal and Statue of Unity are
7:2 and 11:12 respectively?
a) 1575
b) 1508
c) 2250
d) 3050
e) None of these

42. The number of foreigners who toured of Elephanta cave is approximately what per cent of the
total number of foreigners who toured of all the places except Elephanta cave?
a) 22.25%

Page 698 of 728

Subscribe the Xpress Video Course & Mock Test Package for Bank & Insurance Exams
If there are any suggestions/ errors in our PDFs Feel Free to contact us via this email: admin@exampundit.in
IBPS RRB PO Prelims – Ultra Practice Bundle PDF
b) 32.45%
c) 16.28%
d) 30.25%
e) None of these
43. What is the respective ratio of the number of female foreigners who visited Red Fort and Shastra
Dhara together and the total number of foreigners toured of these states together it is also known that total
number of male who visited Red fort is 550 which is 250 less than the male foreigner who visited Shastra
Dhara?
a) 74:224
b) 112:39
c) 37:112
d) Both A and C
e) Can’t be determined
44. What is the approximate average no. of male foreigners who visited all the places together, if
male and female visitors are equal in total?
a) 523
b) 555
c) 525
d) 325
e) None of these
45. Find the difference between total foreign visitors who visited Hawa Mahal and Red Fort together to
total number of foreign visitors who visited Statue of unity and Taj Mahal together?
a) 1002
b) 1006
c) 1005
d) 1007
e) None of these
Directions (46 – 50): Study the following graph and answer the following questions.
Page 699 of 728

Subscribe the Xpress Video Course & Mock Test Package for Bank & Insurance Exams
If there are any suggestions/ errors in our PDFs Feel Free to contact us via this email: admin@exampundit.in
IBPS RRB PO Prelims – Ultra Practice Bundle PDF

A survey was done on 688000 people having bank accounts in 6 different banks i.e. P, O, I, U, Y and T.
Pie chart given below show the percentage distribution of total number of people whose bank accounts
are 6 different banks.

TOTAL BANK A/C = 688000


10%

18%

28%

14%

12% 18%

46. If the ratio of the number of males and the number of females in bank P is 1 : 2, what is the number of
females having bank account in that bank?
a) 82560
b) 58260
c) 85260
d) 72560
e) None of these
47. If the percentage of people aged below 18 is 35% of the total no. of people having accounts in banks
U & Y together, find the number of people aged below 18 in both the banks together?
a) 56920

Page 700 of 728

Subscribe the Xpress Video Course & Mock Test Package for Bank & Insurance Exams
If there are any suggestions/ errors in our PDFs Feel Free to contact us via this email: admin@exampundit.in
IBPS RRB PO Prelims – Ultra Practice Bundle PDF

b) 65290
c) 63920
d) 96320
e) Can’t be determined
48. If the percentage of females having accounts in bank Y is 40% of the total accounts there and
females having accounts in bank O is 45%, find the ratio of the total number of males in banks Y and O
to the total number of females in banks Y and O?
a) 1120:2232
b) 1504:2107
c) 2108:1505
d) 2107:1505
e) None of these
49. What is the percentage increase in the people having accounts in banks I and O in next year, if total
number of people having bank account in I and O next year is 256500?
a) 15.28%
b) 16.28%
c) 25.28%
d) 22.28%
e) 16.58%
50. Find the central angle of total number of bank account in bank Y, U and O?
a) 125.5*
b) 194.4*
c) 144.4*
d) 127.7*
e) None of these

Page 701 of 728

Subscribe the Xpress Video Course & Mock Test Package for Bank & Insurance Exams
If there are any suggestions/ errors in our PDFs Feel Free to contact us via this email: admin@exampundit.in
IBPS RRB PO Prelims – Ultra Practice Bundle PDF
9). DI Based on Pie Chart - Solutions and Detailed Explanations
1-5. Common Explanation: *1500*(10+18) %} = 230 Required percentage =

230 / (1500*20%) = 210/300 * 100 = 76.67%


1. Answer: B)
5. Answer: A)
Number of teachers who teach history and Hindi =
Actual number of teacher who teach economics =
1500* (15+10) % = 375 Number of teacher who
1500*13% = 195 Now, after change
teach Economics and Math = 1500*(13+18) % =
Number of teacher who teach economics = 1500*18%
465 Require ratio = 375:465 = 25:31 = 270
Required % more = 270 – 195 / 195 *100 = 38.46%
Alternatively, more
Required ratio = (15+10) % : (13+18) % = 25:31 6 to 10: Common Explanation:
2. Answer: E)
6) Answer: C)
Total number of teacher who teach math and
Amount spent on transport subsidy and canteen subsidy
Computer = 1500*(18+20) % = 570 Total number of
together = 3750000*(10+8) % = 675000
teacher who teach history = 1500*15% = 225
7). Answer: E)
Required % = 570/225 * 100 = 253.33%
Difference between the expenditure on salary to staff
3. Answer: A) and that on loans to staff = 3750000 * (20-18) % =
75000
Average number of teacher who teach economics and
8). Answer: D)
math = 1/2 * 1500 * (13+18) % = 232.5 Total number
Amount spent on medical staff = 3750000*6% =
of teacher who teach Hindi = 1500 *10% = 150 225000

Required difference = 232.5 – 150 = 82.5 or 83 approx. Average amount spent on salary and Electricity = 1/2

4. Answer: A) (20+12) % * 3750000 = 600000 Required difference =

Number of teacher who teach geography = 20 + {1/2 600000-225000/600000 * 100 = 62.5% less
Page 702 of 728

Subscribe the Xpress Video Course & Mock Test Package for Bank & Insurance Exams
If there are any suggestions/ errors in our PDFs Feel Free to contact us via this email: admin@exampundit.in
IBPS RRB PO Prelims – Ultra Practice Bundle PDF
9). Answer: A) Value of export of the England, UK and Taiwan =
144000 * (7.2+21.6+18)/360 = 18720
Amount spent on medical and water is increased by
25% to next year Value of export of Brazil, Hog Kong and Australia

Amount spent on medical and water next year = together = 144000 * (10.8+10.8+7.2)/360 = 11520
125/100 * (6+11) % * 3750000 = 796875
Required percentage = 18720-11520/11520 * 100 =
10). Answer: A)
62.5%
Amount spent on Canteen subsidy and loan to staff =
14.Answer: D)
3750000 * (8+18) % = 975000 Amount spent on
Total export form USA, Japan and other countries
Telephone, Water and Electricity = 3750000 *
together = 144000*(45+36+104.4)/360 = 74160 Total
(15+11+12) % = 1425000 Required percentage =
export from Russia and china together = 144000 *
975000/1425000 * 100 = 68.42%
(28.8+10.8)/360 = 15840

11-15. Common Explanation Required ratio = 74160: 15840 = 1854:396 = 309:66 =


103: 22

11.Answer: D) 15.Answer: D)

36% of total expenditure form other countries = 36/100


Value of export of USA is how much more or less
* 144000 * 104.4/360 = $15033.6
than the value of export of Germany Required answer

= 144000* (45-32.4)/360 = $5040 16-20. Common Explanation:

12.Answer: B) 16.Answer: A)
Difference of the value of the export of Japan and
Total number Men and Women working in RTGS =
France together to the value of export of UK and
14000*10% + 14000*10%*120/100 =
Taiwan together Required answer = 144000 *
1400+1680=3080 Total number Men and Women
{(36+18) – (21.6+18)}/360 = $5760
working in NEFT = 14000*22% +
13.Answer: D)
14000*22%*120/100 = 3080+3696 = 6776 Required
Page 703 of 728

Subscribe the Xpress Video Course & Mock Test Package for Bank & Insurance Exams
If there are any suggestions/ errors in our PDFs Feel Free to contact us via this email: admin@exampundit.in
IBPS RRB PO Prelims – Ultra Practice Bundle PDF
difference = 6776-3080 = 3696 or equals to 20 year = 125500 * (14+26+20+15) % =
94125
17.Answer: B)
22.Answer: D)
Number of men working as Cashier in SBI =
For the answer we have to check the Given Option
14000*13%*120% = 2184 Number of Men working
For option no. A = 20=>40 = 14%+26% = 40% For
as CTS in SBI = 14000*28%*120% = 4704 Required
option no. B = 30=>40 = 26%
percentage = 4704-2184/4704 * 100 = 53.57%
For option no. C = 40 and above = 20% + 15% = 35%
18.Answer: C)
19.Answer: E) For option no. D = 30 and above = 26%+20%+15%

25 % of Male who work in Loan is promoted as senior = 61% Required answer is D (as more % as

loan managers Required answer = compared to other option)

14000*12%*120/100 *75/100 = 1512 23.Answer: C)

20.Answer: E) Total numbers of employees whose age is 40 year and


above = 125500*(20+15) % = 43925
Difference between male and female working as
customer support = 14000*15%*120/100 - 24.Answer: E)
14000*15% = 2520 – 2100 = 420
Only difference between male and female are known.
Difference between male and female working as
We don’t know the value of male and female.
NEFT = 14000*22%*120/100 – 14000*22% = 3696
25.Answer: C)
– 3080 = 616. Required percentage = 616-420/616 *
Employees of age group of 50 or above = 125500 *
100 = 31.81% 15% = 18825

Now ATQ, Employees working in SBI this year =


21-25. Common Explanation: 125500 – 18825 + 2540 = 109215

21.Answer: A) 26-30. Common Explanation

Total number of employees of age group of more than

Page 704 of 728

Subscribe the Xpress Video Course & Mock Test Package for Bank & Insurance Exams
If there are any suggestions/ errors in our PDFs Feel Free to contact us via this email: admin@exampundit.in
IBPS RRB PO Prelims – Ultra Practice Bundle PDF
Let, total number of Tin pati Player = X ATQ, *100= 25000
Total number of Rummy player = 25000+2500 = 27500
And Total number of Rummy Player = X + 2500
30.Answer: E)
26.Answer: A)
Difference between Rummy player and Tin pati player
Total number of Rummy Player from R = X+2500 *
is maximum for which of the following Casino (As
12% = 0.12X + 300 Total number of Tin Pati player
the % is same for both the game that means Maximum
from R = X *12% = 0.12x
%es means Maximum Difference)-
ATQ,
So required answer = 2500 * 26% = 650 for Y
0.12X +300 + 0.12X = 5700

X = 22500 31-35. Common Explanation


Required Ratio = (22500+300)*12% : 22500 *12% =
22800:22500 = 228:225 31.Answer: A)
27.Answer: B) Required difference = 37550 * (12+18-20-14) % =
1502
Total number of Tin Pati Player from T = X * 18% =
32.Answer: A)
0.18X Total number of Tin Pati Player form Q = X
Required difference = 375550 * 18% (33.33% -
*12% = 0.12X Required %= (0.18X – 0.12X)/0.12X 33.33%) = 0
= 50% more (hint - Transgender = 100% - 33.33% - 33.33%)
28.Answer: C) 33.Answer: A)
Difference between total number of Rummy player to
Required % = (14%+22%) – (14%) / 14% = 57.14%
total number of Tin pati players form W, E and R =
{(X+2500)
(Hint Common Population is same so, there is no need
– (X) } * (18+14+12) % = 2500 * 44% = 1100
to find the value. Use only the % to find the answer)
29.Answer: D)
34.Answer: B)
Total number of Tin Pati player = 7500/(12+18) %
Average population of G, H & J all together = 37550
Page 705 of 728

Subscribe the Xpress Video Course & Mock Test Package for Bank & Insurance Exams
If there are any suggestions/ errors in our PDFs Feel Free to contact us via this email: admin@exampundit.in
IBPS RRB PO Prelims – Ultra Practice Bundle PDF
* (22+18+14)/3 = 6759 Total population of L and K = 360 * (70+65)/360 = 135 Required % = 170-

together = 37550 * (14+20) % = 12767 Required 135/135 * 100 – 25.93% less

answer = 12767 – 6759 = 6008 39.Answer: A)

35.Answer: B) Marks scored by Pritam = 360 Passing marks = 360

Number of person not doing govt. job = 37550 * +20 = 380

20% * (100-30) % = 5257 (Note – Number of person Maximum marks in exam = 380/80 * 100 = 475

not doing Govt. Job means = PVT. + Other) 40.Answer: E)

Total marks obtained in CA and English = 360 *


36-40. Common Explanation (90+80)/360 = 170

36.Answer: A) Total marks obtained in Data interpretation and Essay

Total Marks obtained by Pritam in Data interpretation = writing = 360*(70+55)360 = 125 Required ratio =
360 * 70/360 = 70 marks
170:125 = 34:25
37.Answer: C)

Marks scored by Pritam in Essay Writing and Data


41-45. Common Explanation
interpretation together = 360 * (55+70)/360 = 125

Marks scored in Reasoning and English together = 41.Answer: A)

360 * (65+80)/360 = 145 Male Foreign visitors who visited Taj Mahal =
6300*18%7/9 = 882
Required answer = 145 – 125 = 20 Male Foreign visitors who visited Statue of unity =

38.Answer: D 6300*23%*11/23 = 693 Required answer = 882 + 693


Marks obtained in English and CA = 360 * (80+90)/360 = 1575
= 170
42.Answer: C)
Marks obtained in Data interpretation and Reasoning
Foreigners who toured of Elephanta cave is
Page 706 of 728

Subscribe the Xpress Video Course & Mock Test Package for Bank & Insurance Exams
If there are any suggestions/ errors in our PDFs Feel Free to contact us via this email: admin@exampundit.in
IBPS RRB PO Prelims – Ultra Practice Bundle PDF
approximately what per cent of the total number of Statue of unity and Taj Mahal together = 6300*
foreigners who toured of all the places except
Elephanta Cave. (23+18) % = 2583 Required difference = 2583

Required % = 14%/(100-14) % = 14/86 * 100 = 16.28% – 1575 = 1008

43.Answer: C)

Total number of male who visited Red fort is 550 46-50. Common Explanation

Total number of male who visited Shastra Dhara = 46.Answer: A)


550+250 = 800
Number of female having bank account in bank P =
Total number of foreign visitors who visited Red Fort = 688000*18% * 2/3 = 82560
6300*12% = 756
47.Answer: D)
Total number of Foreign visitors who visited Shastra
Number of people whose age is below 18 from U and Y
Dhara = 6300*20% = 1260 ATQ, is = 688000 * (12+28) % * 35% = 96320

Total female visitors of Red Fort and Shastra Dhara 48.Answer: D)

together = (756-550) + (1260-800) = 666 Required Total number of people having bank Account in Y =

ratio = 666 : (756+1260) = 666:2016 = 37:112 688000 * 28% = 192640 Total number of Female

having bank account in Y = 192640 * 40% = 77056


44.Answer: C)

Average Male foreign visitors form all the places = Total number of male having bank account in Y =
6300/2 * 1/6 = 525
192640 – 77056 = 115584 Total number of people
(Hint – as male and female are equal in total, so to find
total number of male we have to just divide by 2) having bank account in O = 688000 * 14% = 96320

Total number of female having bank account in O =


45.Answer: E)
96320*45% = 43344 Total number of male having
Total foreign visitors who visited Hawa Mahal and Red
Fort together = 6300 * (13+12) % = 1575
bank account in O = 96320 – 43344 = 52976
Total number of foreign visitors who visited
Required ratio = (115584+52976) : (77056+43344) =
168560:120400 = 4214:3010 = 2107:1505
Page 707 of 728

Subscribe the Xpress Video Course & Mock Test Package for Bank & Insurance Exams
If there are any suggestions/ errors in our PDFs Feel Free to contact us via this email: admin@exampundit.in
IBPS RRB PO Prelims – Ultra Practice Bundle PDF
220160/220160 * 100 = 16.28% more
49.Answer: B)
50.Answer: B)
Total number of people Having bank account from I
Required angle = (28+12+14) % * 18/5 = 194.4*
and O = 688000 * (18+14) % = 220160 Total number
(Note – to convert % into degree simply multiply %
of people having bank account from I and O next year with 18/5)
(given) = 256500 Required % increase = 256000 -

10. Caselet DI Questions


Directions (1 – 5): Study the following information c) 1: 2
carefully and answer the related questions.
d) 2: 3
Population (Male + Female) of three towns A, B and C
in 2018 is 12000, 15000 and 13500 respectively. In each e) 3: 2
town, female population is less than male population. 2. What is the difference between total literate male
The difference between total male and female population population and total literate female population in all
in town A is 2000 which is 2000 less than the difference three towns taken together?
between male and female population in town B. In town
C, the ratio of male to female population is 5: 4 a) 3110
respectively. 75% females in town A are literate which
b) 3210
are 250 more than the literate female population of town
B. 46% population of town B is illiterate, and total c) 3310
literate male population of town A is 4660. 4250 females
in town C are literate which are 2500 more than the d) 3410
illiterate male population of same town. In every town,
e) 3510
the number of total males is more than that of females.
3. Total number of males in town B is approximately
1. What is the ratio of total illiterate male population
what percent more/less than total number of females
to total illiterate female population in town C?
in town A?
a) 1: 1
a) 75%
b) 2: 1
b) 90%
Page 708 of 728

Subscribe the Xpress Video Course & Mock Test Package for Bank & Insurance Exams
If there are any suggestions/ errors in our PDFs Feel Free to contact us via this email: admin@exampundit.in
IBPS RRB PO Prelims – Ultra Practice Bundle PDF
c) 85% the total number of males is from Bihar. Respective ratio
of number of females from Jharkhand and number of
d) 70% males from Bihar is 4:7. Number of males from
e) 80% Jharkhand is 10% more than the number of females from
that state. 18% of the females are from Kerala. 25% of
4. Approximately what percent of total population in the total number of workers is from Kerala. Number of
town C are illiterate? males from Punjab is 80% of the number of males from
Kerala. Number of females from Karnataka is 20% more
a) 23%
than the number of males from that state. 30% of the
b) 29% total number of females is from Bihar.

c) 33% 6. Total number of males from Bihar is


approximately what percent of the total number of
d) 26% workers from Jharkhand?
e) 31% a) 69%
5. If the population of town A in 2017 was 20% less b) 63%
than 2018 and the ratio of male to female was reverse
of the ratio of male to female in 2018, then what will c) 83%
be the difference between female population of town
d) 67%
A in 2017 and female population of town B in 2018?
e) 49%
a) 140
7. Find the respective ratio of total number of
b) 130
workers from Kerala and total number of workers
c) 120 from Bihar.

d) 110 a) 13:14

e) 100 b) 12:13

Directions (6 – 10): Study the following information c) 11:12


carefully and answer the questions given below:
d) 10:11
There are total 24000 workers working in a company,
e) None of these
each one is from one among the five states viz. Kerala,
Karnataka, Jharkhand, Bihar and Punjab. Respective 8. Find the average of the number of male workers
ratio of males and females among them is 7:5. 25% of Kerala, Karnataka, Jharkhand and Punjab.
Page 709 of 728

Subscribe the Xpress Video Course & Mock Test Package for Bank & Insurance Exams
If there are any suggestions/ errors in our PDFs Feel Free to contact us via this email: admin@exampundit.in
IBPS RRB PO Prelims – Ultra Practice Bundle PDF
a) 4025 exams. Number of students preparing for CAT exams
was twice the number of male students preparing for
b) 3125 UPSC exams. Average weight of students preparing for
c) 2875 UPSC is 59 kg and total weight of male students
preparing for UPSC exams is 20475 kg. Ratio of number
d) 2625 of students preparing for SSC and CAT are in the ratio 8:
7 respectively. 60% students who are preparing for CAT
e) None of these
are males and remaining 252 are females. Female
9. Number of female workers are highest from which students who are preparing for SSC are 80 less than
state? number of males preparing for same exam.

a) Kerala 11. If total weight of female students who are


preparing for SSC is 11520 kg, then what is the
b) Karnataka respective ratio of average weight of female students
who are preparing for UPSC to SSC?
c) Jharkhand
a) 4: 3
d) Bihar
b) 15: 14
e) Punjab
c) 11: 10
10. Find the difference between number of female
workers from Jharkhand and Bihar together and d) 5: 4
number of female workers from Karnataka and
Punjab together. e) 8: 5

a) 2000 12. Number of boys preparing for CAT is what


percent of number of boys preparing for SSC?
b) 2400
a) 95.5%
c) 1600
b) 94.5%
d) 1800
c) 93.5%
e) None of these
d) 92.5%
Directions (11 – 15): Study the following information
carefully and answer the related questions. e) 91.5%

In a coaching institute, there are total 1800 students who 13. Approximately what percent students in coaching
are preparing for three exams CAT, SSC and UPSC institute are females?
Page 710 of 728

Subscribe the Xpress Video Course & Mock Test Package for Bank & Insurance Exams
If there are any suggestions/ errors in our PDFs Feel Free to contact us via this email: admin@exampundit.in
IBPS RRB PO Prelims – Ultra Practice Bundle PDF
a) 42% There are three schools - A, B and C. In each school
there are three classes - 8th class, 10th class and 12th class.
b) 48%
School A: In this school, the number of students in 8 th
c) 45% class and 10th class each is 14 more than that in 8th class
d) 36% and 10th class of school B respectively. The number of
students in 12th class is 14 less than that in 12th class of
e) 39% school B.

14. What is the difference between number of female School B: The ratio between the number of students in
students preparing for CAT and number of male 8th class and 10th class is 13: 11 respectively. The total
students preparing for UPSC? number of students in this school is 346 and the number
of students in 8th class is (65/173) times of the total
a) 57
number of students in that school.
b) 45
School C: The number of students in 10th class in 12/11
c) 63 times of that in 10th class of school B. The number of
students in 8th class is more than that in 10th class by 16.
d) 39 The number of students in 12th class is 75% of that in 8th
class.
e) 71
16. What is the total number of students in school C?
15. If monthly fee submitted by each student of
coaching institute in June is Rs.2400, then find the a) 360
per day earnings of coaching institute from all male
students. b) 346

a) Rs.87280 c) 358

b) Rs.87520 d) 354

c) Rs.87360 e) None of these

d) Rs.87440 17. The total number of students in school A is what


per cent of the total number of students in 12th class
e) Rs.87600 of all the three schools together?
Directions (16 – 20): Read the data carefully and a) 120%
answer the following questions.
b) 112%

Page 711 of 728

Subscribe the Xpress Video Course & Mock Test Package for Bank & Insurance Exams
If there are any suggestions/ errors in our PDFs Feel Free to contact us via this email: admin@exampundit.in
IBPS RRB PO Prelims – Ultra Practice Bundle PDF
c) 125% e) None of these

d) 110% Directions (21 – 25): Answer the questions based on


the information given below.
e) None of these
There are three cities (A, B and C) where a political
18. What is the ratio between the numbers of students party did a campaign to ask people to join the party.
in 10th class of school A to the number of students in Number of people who joined party in city A was twice
10th class of school C? the number of people who joined party in city C. Total
a) 21: 20 number of people who joined party in three cities
together was 1700. Ratio of number of male to female
b) 31: 30 who joined party in city A was 11:5, and number of
female who joined party in city B was 200. Number of
c) 41: 40
male who joined party in city B was 100 less than
d) 51: 50 number of people who joined party C. Number of female
who joined party in city C was 280.
e) None of these
21. Find the number of people who joined party in
19. What is the average of the total number of city A.
students in school A in all the three classes together?
a) 800
a) 100
b) 600
b) 105
c) 1000
c) 110
d) 700
d) 115
e) 400
e) None of these
22. Find the difference between number of male who
20. What is the total number of students in all the joined party in city A and that of same in city C.
three schools A, B and C together?
a) 450
a) 980
b) 410
b) 1022
c) 430
c) 1064
d) 460
d) 1098

Page 712 of 728

Subscribe the Xpress Video Course & Mock Test Package for Bank & Insurance Exams
If there are any suggestions/ errors in our PDFs Feel Free to contact us via this email: admin@exampundit.in
IBPS RRB PO Prelims – Ultra Practice Bundle PDF
e) 480 Directions (26 – 30): Answer the questions based on
the information given below.
23. Find the ratio of number of female who joined
party in city A to that of same in city B. The given data is about number of male and female staffs
at three different hotels (A, B and C). The number of
a) 7:5 male staff at ‘A’ is 80 more than the number of female
b) 3:2 staffs at ‘C’. The number of male staffs at ‘C’ is 25%
more than that at ‘A’. The number of male staffs at ‘B’ is
c) 5:4 100. Total number of male staffs at given three hotels is
370. The number of female staffs at ‘A’ is 1/3rd of male
d) 6:5
staffs at ‘C’. Total number of staffs at ‘B’ is 160.
e) 8:7
26. Find the average number female staffs at the
24. Number of people who joined party in city C is three hotels.
what percentage of number of people who joined
a) 45
party in city B?
b) 50
a) 50%
c) 55
b) 120%
d) 60
c) 60%
e) 52
d) 80%
27. The total number of staffs at ‘A’ is how much
e) 100%
percent more/less than the number of male staffs at
25. In city C, there were already 200 male members ‘B’?
in the party before campaign. Find the percentage
a) 70%
increase in the male members of party after joining of
the new male members. b) 50%

a) 80% c) 80%

b) 40% d) 65%

c) 70% e) 85%

d) 50% 28. Find the ratio of the number of male staffs at ‘C’
to female staffs at ‘B’.
e) 60%
Page 713 of 728

Subscribe the Xpress Video Course & Mock Test Package for Bank & Insurance Exams
If there are any suggestions/ errors in our PDFs Feel Free to contact us via this email: admin@exampundit.in
IBPS RRB PO Prelims – Ultra Practice Bundle PDF
a) 6:5 subjects. The number of students passed only in
chemistry is 3 times the number of students passed only
b) 3:1 in Physics. The ratio of the number of students who
c) 7:4 passed in both Chemistry and Mathematics but not in
Physics to the number of students who passed only in
d) 5:2 Mathematics is 5:3. 25 students passed in both Physics
and Mathematics but not in Chemistry and 465 students
e) None of these
passed in Mathematics. 115 students failed in all 3
29. Find the difference between the total staffs at ‘B’ subjects and 60 students passed in both Physics and
and ‘C’. Chemistry but not in Mathematics.

a) 30 31. What is the ratio of the number of students who


passed in Physics to the number of students who
b) 60 passed in Chemistry respectively?
c) 45 a) 73:109
d) 40 b) 71:111
e) 55 c) 77:113
30. At hotel ‘B’, if 20% of male staffs and 25% of d) 81:121
female staffs are managers then find the number of
staffs at ‘B’ other than managers. e) 63:122

a) 150 32. What percent of students appeared for the exam


had passed in at least 2 subjects?
b) 135
a) 52.25%
c) 140
b) 56.75%
d) 120
c) 52.75%
e) 125
d) 56.25%
Directions (31 – 35): Answer the questions based on
the information given below. e) 55.55%

800 students appeared for examination in three subjects: 33. What percent of students appeared for exam had
Physics, Chemistry and Mathematics. 30% of the passed in exactly 1 subject?
students appeared for exam had passed in all three
Page 714 of 728

Subscribe the Xpress Video Course & Mock Test Package for Bank & Insurance Exams
If there are any suggestions/ errors in our PDFs Feel Free to contact us via this email: admin@exampundit.in
IBPS RRB PO Prelims – Ultra Practice Bundle PDF
a) 22.275% The given data is about number of males and females
who are JRF holders in three universities i.e. JNU, DU
b) 18.375% and AMU. The number of male JRF holder in DU is
c) 26.375% 20% more than the number of female JRF holder in JNU.
Total number of male JRF holder in DU and AMU is
d) 32.275% 1400. Total number of JRF holders (male + female) in
JNU is 1300. The number of male JRF holders in JNU
e) 29.375%
and AMU are equal. The number of female JRF holder in
34. Out of students who passed in both Physics and AMU is 700 which is 200 less than that in DU.
Chemistry, what percent passed in physics and
36. Find the average number of female JRF holders
chemistry but not in Mathematics?
in the given three universities.
a) 15%
a) 500
b) 20%
b) 700
c) 25%
c) 800
d) 30%
d) 600
e) 10%
e) 400
35. What is the respective ratio of the number of
37. The number of female JRF holders in DU is how
students who passed only in Mathematics to the
much percent more/less than the number of male JRF
number of students who passed in Physics but not in
holders in AMU?
Chemistry?
a) 12.5%
a) 15:13
b) 20%
b) 13:11
c) 10.75%
c) 11:9
d) 15%
d) 17:15
e) None of these
e) 9:7
38. Find the ratio of number of male JRF holders in
Directions (36 – 40): Answer the questions based on
DU to total number of JRF holders in AMU.
the information given below.
a) 5:9

Page 715 of 728

Subscribe the Xpress Video Course & Mock Test Package for Bank & Insurance Exams
If there are any suggestions/ errors in our PDFs Feel Free to contact us via this email: admin@exampundit.in
IBPS RRB PO Prelims – Ultra Practice Bundle PDF
b) 1:3 and number of male participants from school C are
equal. Respective ratio of number of male and female
c) 5:7 students participated from school A is 2: 3 and number
d) 3:4 of female participants from school A are 18 more than
the number of female participants from school C. Sum of
e) 2:5 number of participants from school B and C is 490 and
180 students participated from school A. 100 male
39. Find the difference between total number of JRF
students participated from school B.
holders in DU and number of male JRF holders in
JNU. 41. Total number of participants from school A are
approximately what percent of the number of male
a) 500
participants from school B and C together?
b) 600
a) 68%
c) 700
b) 65%
d) 800
c) 72%
e) 400
d) 78%
40. 40% of JRF holders from all three given
e) 75%
universities are of Science. Find the number of JRF
holders of subjects other than Science from all three 42. What is the respective ratio of number of male
given universities. participants from school A to the number of female
participants from school C?
a) 2580
a) 3: 4
b) 2420
b) 8: 9
c) 2640
c) 2: 3
d) 2360
d) 6: 7
e) 2290
e) 4: 5
Directions (41 – 45): Study the following information
carefully and answer the related questions. 43. Approximately what percent participants from
school B are females?
Following information gives the data regarding number
of students participated in sports league from different a) 75%
schools. Number of female participants from school B
Page 716 of 728

Subscribe the Xpress Video Course & Mock Test Package for Bank & Insurance Exams
If there are any suggestions/ errors in our PDFs Feel Free to contact us via this email: admin@exampundit.in
IBPS RRB PO Prelims – Ultra Practice Bundle PDF
b) 70% buffaloes in farm ‘C’ is 200 more than number of cows
in farm ‘A’. Total number of animals in farm ‘C’ is
c) 54% 1000. Number of cows in farm ‘D’ is 25% more than that
d) 66% in ‘C’. The ratio of number of buffaloes in farm ‘A’ and
‘C’ is 6:5, respectively. Total number of cows in all the
e) 60% farms together is 1800. Number of buffaloes in farm ‘B’
is 350 which is 150 less than that in farm ‘D’.
44. What is the difference between number of
students participated from school C and number of 46. Find the ratio of the number of buffaloes in farm
female students participated from school A and B ‘A’ to the total number of animals in farm ‘B’.
together?
a) 2:5
a) 18
b) 3:7
b) 24
c) 4:5
c) 20
d) 1:2
d) 26
e) 5:8
e) 16
47. Find the difference between the total number of
45. If average of male students participated from animals in farm ‘D’ and number of cows in farm ‘B’.
school A, C and D taken together is 122, then how
many male students participated from school D? a) 1000

a) 128 b) 1200

b) 140 c) 750

c) 132 d) 900

d) 144 e) 1060

e) 136 48. The number of cows in farm ‘D’ is how much


percent more/less than total number of animals in
Directions (46 – 50): Answer the questions based on ‘C’?
the information given below.
a) 20%
The given data is about number of animals (cows and
buffaloes) in four different farms (A, B, C and D). The b) 25%
number of cows in farm ‘A’ is 200. The number of
Page 717 of 728

Subscribe the Xpress Video Course & Mock Test Package for Bank & Insurance Exams
If there are any suggestions/ errors in our PDFs Feel Free to contact us via this email: admin@exampundit.in
IBPS RRB PO Prelims – Ultra Practice Bundle PDF
c) 35% e) 450

d) 30% 50. If 20% of buffaloes in farm ‘B’ does not give milk,
then find the number of milk giving buffaloes in farm
e) 18% ‘B’.
49. Find the average of the number of buffaloes in a) 320
farm ‘C’ and ‘D’ together.
b) 280
a) 500
c) 260
b) 400
d) 240
c) 480
e) None of these
d) 420

10. Caselet DI Questions - Answers with Explanation:


(1 – 5): Common Explanation: Town B

Town A Then, total male population in town B = total female


population in town B + 4000
Population of town A = 12000
Total male population in town B = 15000 - total female
Population of town B = 15000 population in town B
Population of town C = 13500 Then, total female population in town B = 5500
The difference between total male and female population And, total male population in town B = 5500 + 4000 =
in town A is 2000. Then, 9500
Total male population in town A = total female Town C
population in town A + 2000
Total male population in town C = 13500 x 5/9 = 7500
Total male population in town A = 12000 - total female
population in town A And, total female population in town C = 13500 - 7500 =
6000
Then, total female population in town A = 5000
Literate and Illiterate population Calculation for
And, total male population of town A = 2000 + 5000 =
Town A, B and C
7000

Page 718 of 728

Subscribe the Xpress Video Course & Mock Test Package for Bank & Insurance Exams
If there are any suggestions/ errors in our PDFs Feel Free to contact us via this email: admin@exampundit.in
IBPS RRB PO Prelims – Ultra Practice Bundle PDF
Now, literate female population in town A = 75% of 0 0
5000 = 3750 750 600
C 5750 1750 4250 1750
0 0
Illiterate female population in town A = 5000 - 3750 =
1250
1. Answer: A)
Literate female population of town B = 3750 - 250 =
3500 Total illiterate male population in town C = 1750

Illiterate female population in town B = 5500 - 3500 = Total illiterate female population in town C = 1750
2000 Ratio = 1750: 1750 = 1: 1
Illiterate population of town B = 46% of 15000 = 6900 2. Answer: E)
Illiterate male population in town B = 6900 - 2000 = Total literate male population in all three towns taken
4900 together
Literate male population in town B = 9500 - 4900 = 4600 = 4660 + 4600 + 5750
Literate male population in town A = 4660 = 15010
Illiterate male population in town A = 7000 - 4660 = Total literate female population in all three towns taken
2340 together
Literate female population in town C = 4250 = 3750 + 3500 + 4250
Illiterate female population in town C = 6000 - 4250 = = 11500
1750
Difference = 15010 - 11500 = 3510
Illiterate male population in town C = 4250 - 2500 =
1750 3. Answer: B)

Literate male population in town C = 7500 - 1750 = 5750 Total number of males in town B = 9500

In tabular form: Total number of females in town A = 5000

Male population Female population Difference = 9500 - 5000 = 4500


Tow
Tota Literat Illiterat Tota Literat Illiterat
n Percentage = (4500/5000) x 100 = 90%
l e e l e e
700 500
A 4660 2340 3750 1250 4. Answer: D)
0 0
B 950 4600 4900 550 3500 2000
Total population of town C = 13500
Page 719 of 728

Subscribe the Xpress Video Course & Mock Test Package for Bank & Insurance Exams
If there are any suggestions/ errors in our PDFs Feel Free to contact us via this email: admin@exampundit.in
IBPS RRB PO Prelims – Ultra Practice Bundle PDF
Total illiterate population of town C = 1750 + 1750 = Total number of workers from Kerala = 25/100 x 24000
3500 = 6000

Percentage = (3500/13500) x 100 = 26% (approx.) Number of males from Kerala = 6000 - 1800 = 4200

5. Answer: E) Number of males from Punjab = 80/100 x 4200 = 3360

Ratio of male to female population in town A in 2018 = Number of males from Karnataka = 14000 - 4200 - 2200
7000: 5000 = 7: 5 - 3500 - 3360 = 740

In 2017: Number of females from Karnataka = 740 x 120/100 =


888
Population of town A = 80% of 12000 = 9600
Number of females from Bihar = 30/100 x 10000 = 3000
Ratio of male to female population in town A in 2017 =
5: 7 Number of females from Punjab = 10000 - 1800 - 888 -
2000 - 3000 = 2312
Total female population in town A = 9600 x 7/12 = 5600
Total number of workers from Karnataka = 740 + 888 =
Female population of town B in 2018 = 5500 1628
Difference = 5600 - 5500 = 100 Total number of workers from Jharkhand = 2200 + 2000
= 4200

(6 – 10): Common Explanation: Total number of workers from Bihar = 3500 + 3000 =
6500
Total number of workers = 24000
Total number of workers from Punjab = 3360 + 2312 =
Number of males = 7/12 x 24000 = 14000 5672

Number of females = 5/12 x 24000 = 10000 States Male Female Total


Kerala 4200 1800 6000
Number of males from Bihar = 25/100 x 14000 = 3500 Karnataka 740 888 1628
Jharkhand 2200 2000 4200
Number of females from Jharkhand = 4/7 x 3500 = 2000
Bihar 3500 3000 6500
Number of males from Jharkhand = 2000 x 110/100 = Punjab 3360 2312 5672
2200 Total 14000 10000 24000

Number of females from Kerala = 18/100 x 10000 =


6. Answer: C)
1800

Page 720 of 728

Subscribe the Xpress Video Course & Mock Test Package for Bank & Insurance Exams
If there are any suggestions/ errors in our PDFs Feel Free to contact us via this email: admin@exampundit.in
IBPS RRB PO Prelims – Ultra Practice Bundle PDF
Required percentage = 3500/4200 x 100 = 83.33% = Number of male students preparing for SSC = (720 +
83% approx. 80)/2 = 400

7. Answer: B) Number of female students preparing for SSC = 720 -


400 = 320
Required ratio = 6000 : 6500 = 12 : 13
Number of students preparing for UPSC = 1800 - 630 -
8. Answer: D) 720 = 450
Required average = (4200 + 740 + 2200 + 3360)/4 = Total weight of students preparing for UPSC = 450 * 59
10500/4 = 2625 = 26550 kg
9. Answer: D) Total weight of male students preparing for UPSC =
Number of female workers is highest from Bihar which 20475 kg
is 3000. Then, total weight of female students preparing for
10. Answer: D) UPSC = 26550 - 20475 = 6075 kg

Number of female workers from Jharkhand and Bihar Now, number of male students preparing for UPSC =
together = 2000 + 3000 = 5000 630/2 = 315

Number of female workers from Karnataka and Punjab Then, number of female students preparing for UPSC =
together = 888 + 2312 = 3200 450 - 315 = 135

Required difference = 5000 - 3200 = 1800 And, average weight of female students preparing for
UPSC = 6075/135 = 45 kg

11. Answer: D)
(11 – 15): Common Explanation:
Average weight of female students preparing for SSC =
Total number of students = 1800 11520/320 = 36 kg

Number of female students preparing for CAT = 252 Average weight of female students preparing for UPSC =
45 kg
Number of students preparing for CAT = 252 * 100/40 =
630 Therefore, ratio = 45: 36 = 5: 4

Number of male students preparing for CAT = 630 - 252 12. Answer: B)
= 378
Number of boys preparing for CAT = 378
Number of students preparing for SSC = 630 * 8/7 = 720
Number of boys preparing for SSC = 400
Page 721 of 728

Subscribe the Xpress Video Course & Mock Test Package for Bank & Insurance Exams
If there are any suggestions/ errors in our PDFs Feel Free to contact us via this email: admin@exampundit.in
IBPS RRB PO Prelims – Ultra Practice Bundle PDF
Therefore, percentage = (378/400) * 100 = 94.5% School A:

13. Answer: E) The number of students in 8th class = 130 + 14 = 144

Total number of students = 1800 The number of students in 10th class = 110 + 14 = 124

Number of female students = 252 + 320 + 135 = 707 The number of students in 12th class = 106 - 14 = 92

Therefore, percentage = (707/1800) * 100 = 39% School C:


(approx.)
The number of students in 10th class = 110 * (12/11) =
14. Answer: C) 120

Number of female students preparing for CAT = 252 The number of students in 8th class = 120 + 16 = 136

Number of male students preparing for UPSC = 315 The number of students in 12th class = 75% of 136 = 102

Therefore, difference = 315 - 252 = 63 Number of Number of Number of


School/Class students in students in students in
15. Answer: D) 8th class 10th class 12th class
A 144 124 92
One day earnings of coaching institute from all male B 130 110 106
students = 2400 * (378 + 400 + 315)/30 C 136 120 102
= Rs.87440
16. Answer: C)

The number of students in 8th class = 136


(16 – 20): Common Explanation:
The number of students in 10th class = 120
School B:
The number of students in 12th class = 102
The total number of students in this school = 346
The total number of students in school C = 120 + 136 +
The number of students in 8th class = 346 * (65/173) =
102 = 358
130
17. Answer: A)
So, the number of students in 10th class = 130 * (11/13) =
110 The total number of students in school A = 144 + 124 +
92 = 360
And the number of students in 12th class = 346 - 130 -
110 = 106 The total number of students in 12th class of all the three
schools together = 92 + 106 + 102 = 300
Page 722 of 728

Subscribe the Xpress Video Course & Mock Test Package for Bank & Insurance Exams
If there are any suggestions/ errors in our PDFs Feel Free to contact us via this email: admin@exampundit.in
IBPS RRB PO Prelims – Ultra Practice Bundle PDF
Required percentage = (360/300) * 100 = 120% So, x + 2x + y = 1700

18. Answer: B) 3x + y = 1700 -------(i)

The number of students in 10th class of school A = 124 Number of male who joined party in city B = x – 100

The number of students in 10th class of school C = 120 So, x – 100 + 200 = y

Required ratio = 124: 120 = 31: 30 x + 100 = y

19. Answer: E) Put value of y in (i)

The number of students in 8th class in school A = 144 So, 3x + x + 100 = 1700

The number of students in 10th class in school A = 124 4x = 1600

The number of students in 12th class in school A = 92 x = 400

Required average = (144 + 124 + 92)/3 = 120 And, y = 400 + 100 = 500

20. Answer: C) Number of Number of Total number


male female of people
The total number of students in school A = 144 + 124 + City 11/16 × 800 5/16 × 800 =
800
92 = 360 A = 550 250
City 500 – 200 =
The total number of students in school B = 130 + 110 + 200 500
B 300
106 = 346 City 400 – 280 =
280 400
C 120
The total number of students in school C = 136 + 120 +
102 = 358
21. Answer: A)
The total number of students in all the schools together =
360 + 346 + 358 = 1064 Required number of people = 800

22. Answer: C)

(21 – 25): Common Explanation: Required difference = 550 – 120 = 430

Let, number of people who joined party in city C be ‘x’ 23. Answer: C)

So, number of people who joined party in city A = 2x Required ratio = 250:200 = 5:4

And, number of people who joined party in city B = ‘y’ 24. Answer: D)

Page 723 of 728

Subscribe the Xpress Video Course & Mock Test Package for Bank & Insurance Exams
If there are any suggestions/ errors in our PDFs Feel Free to contact us via this email: admin@exampundit.in
IBPS RRB PO Prelims – Ultra Practice Bundle PDF
Required percentage = 400/500 × 100 = 80% Required average = 150/3 = 50

25. Answer: E) 27. Answer: A)

Required percentage increase = 120/200 × 100 = 60% Required percentage = {(170 – 100)/100} × 100 = 70%

(26 – 30): Common Explanation: 28. Answer: D)

Let the number of female staffs at ‘C’ be ‘x’ Required ratio = 150:60 = 5:2

Therefore, number of male staffs at ‘A’ = (x + 80) 29. Answer: A)

Number of male staffs at ‘C’ = 1.25(x + 80) Required difference = 190 – 160 = 30

According to the question, 30. Answer: E)

1.25(x + 80) + (x + 80) = 370 – 100 Required number of staffs = (0.8 × 100) + (0.75 × 60) =
125
Or, x + 80 = 120
(31 – 35): Common Explanation:
Or, x = 40
Number of students who passed in all 3 subjects =
Therefore, number of male staffs at ‘A’ = (x + 80) = 120 (30/100)*800 = 240
Number of male staffs at ‘B’ = 100 Drawing the Venn-diagram:
Number of male staffs at ‘C’ = 1.25(x + 80) = 150

Number of female staffs at ‘A’ = (1/3) × 150 = 50

Number of female staffs at ‘B’ = 160 – 100 = 60

Number of female staffs at ‘C’ = x = 40

Number of male Number of female


Total
staffs staffs
A 120 50 170
B 100 60 160
C 150 40 190
Total 370 150
So, 25 + 240 + 5k + 3k = 465

26. Answer: B) 8k = 200

Page 724 of 728

Subscribe the Xpress Video Course & Mock Test Package for Bank & Insurance Exams
If there are any suggestions/ errors in our PDFs Feel Free to contact us via this email: admin@exampundit.in
IBPS RRB PO Prelims – Ultra Practice Bundle PDF
k = 25 Number of students who passed in exactly 1 subject

Also, p + 60 + 3p + 465 + 115 = 800 = 40 + 120 + 75

4p = 160 = 235

p = 40 Required percentage = (235/800)*100% = 29.375%

Completing the Venn-diagram: 34. Answer: B)

Required percentage = (60/(60 + 240))*100% = 20%

35. Answer: A)

Number of students who passed only in Mathematics =


75

Number of students who passed in Physics but not in


Chemistry

= 40 + 25 = 65

Required ratio = 75:65 = 15:13

31. Answer: A) (36 – 40): Common Explanation:

Number of students who passed in Physics = 40 + 60 + Let the number of female JRF holders in JNU be ‘x’
25 + 240 = 365
Therefore, number male JRF holders in DU = 1.2x
Number of students who passed in Chemistry = 60 + 120
Number of male JRF holders in AMU = (1400 – 1.2x)
+ 240 + 125 = 545
Number of male JRF holders in JNU = (1300 – x)
Required ratio = 365 : 545 = 73:109
According to the question,
32. Answer: D)
1400 – 1.2x = 1300 – x
Number of students who passed in at least 2 subjects =
60 + 240 + 25 + 125 = 450 Or, x = 100/0.2 = 500

Required percentage = (450/800)*100% = 56.25% Therefore, number of female JRF holders in JNU = x =
500
33. Answer: E)
Number of female JRF holders in AMU = 700
Page 725 of 728

Subscribe the Xpress Video Course & Mock Test Package for Bank & Insurance Exams
If there are any suggestions/ errors in our PDFs Feel Free to contact us via this email: admin@exampundit.in
IBPS RRB PO Prelims – Ultra Practice Bundle PDF
Number of female JRF holders in DU = 700 + 200 = 900 Respective ratio of number of male and female students
participated from school A is 2: 3 and 180 students
Number of male JRF holders in JNU = 1300 – x = 800 participated from school A. Then,
Number of male JRF holders in DU = 1.2x = 600 Total number of participants from school A = 180
Number of male JRF holders in AMU = 1400 – 1.2x = Number of male participants from school A = 180 * 2/5
800 = 72
Therefore, And, number of female participants from school A = 180
Number of male Number of female - 72 = 108
Total
JRF holders JRF holders
Number of female participants from schools A are 18
JNU 800 500 1300
DU 600 900 1500 more than the number of female participants from school
AMU 800 700 1500 C. Then,
Total 2200 2100
Number of female participants from school C = 108 - 18
= 90
36. Answer: B)
Number of male participants from school B = 100
Required average = 2100/3 = 700
Number of female participants from school B and
37. Answer: A) number of male participants from school C are equal.
Sum of number of participants from school B and C is
Required percentage = {(900 – 800)/800} × 100 = 12.5% 490. Then,
38. Answer: E) 490 = 100 + number of female participants from school
Required ratio = 600:1500 = 2:5 B + 90 + number of male participants from school C

39. Answer: C) Number of female participants from school B + number


of male participants from school C = 300
Required difference = 1500 – 800 = 700
Number of female participants from school B = number
40. Answer: A) of male participants from school C = 300/2 = 150

Required number = 0.6 × (1500 + 1500 + 1300) = 2580 Then, total number of participants from school B = 100 +
150 = 250

And, total number of participants from school C = 490 -


(41 – 45): Common Explanation:
250 = 240

Page 726 of 728

Subscribe the Xpress Video Course & Mock Test Package for Bank & Insurance Exams
If there are any suggestions/ errors in our PDFs Feel Free to contact us via this email: admin@exampundit.in
IBPS RRB PO Prelims – Ultra Practice Bundle PDF
In tabular form: 45. Answer: D)

School Total Male Female Number of male students participated from school A =
participants participants participants 72
A 180 72 108
B 250 100 150 Number of male students participated from school C =
C 240 150 90 150

Then, number of male students participated from school


41. Answer: C) D = 3 * 122 - 72 - 150 = 144

Total number of participants from school A = 180

And, number of male participants from school B and C (46 – 50): Common Explanation:
together = 100 + 150 = 250
Number of cows in farm ‘A’ = 200
Therefore, percentage = (180/250) * 100 = 72%
Number of buffaloes in farm ‘C’ = 200 + 200 = 400
42. Answer: E)
Number of cows in farm ‘C’ = 1000 – 400 = 600
Here, number of male participants from school A = 72
Number of cows in farm ‘D’ = 1.25 × 600 = 750
And, number of female participants from school C = 90
Number of cows in farm ‘B’ = 1800 – (750 + 600 + 200)
Therefore, ratio = 72: 90 = 4: 5 = 250

43. Answer: E) Number of buffaloes in farm ‘A’ = 400 × (6/5) = 480

Number of participants from school B = 250 Number of buffaloes in farm ‘B’ = 350

Number of female participants from school B = 150 Number of buffaloes in farm ‘D’ = 350 + 150 = 500

Therefore, percentage = (150/250) * 100 = 60% Therefore,

44. Answer: A) Number of Number of Total


cows buffaloes animals
Here, number of students participated from school C = A 200 480 680
240 B 250 350 600
C 600 400 1000
And, number of female students participated from school 750 500 1250
D
A and B together = 108 + 150 = 258 Total 1800 1730
Therefore, difference = 258 - 240 = 18
Page 727 of 728

Subscribe the Xpress Video Course & Mock Test Package for Bank & Insurance Exams
If there are any suggestions/ errors in our PDFs Feel Free to contact us via this email: admin@exampundit.in
IBPS RRB PO Prelims – Ultra Practice Bundle PDF
46. Answer: C) Required percentage = {(1000 – 750)/1000} × 100 =
25%
Required ratio = 480:600 = 4:5
49. Answer: E)
47. Answer: A)
Required average = (400 + 500)/2 = 450
Required difference = 1250 – 250 = 1000
50. Answer: B)
48. Answer: B)
Required number of buffaloes = 0.8 × 350 = 280

IBPS RRB Free Practice Model Question Paper PDF – Download Now

THE COMPLETE Static GK Capsule for Upcoming Exams

The COMPLETE Static Banking Awareness PDF

Join Our What’s App Group & Get Instant Notification on Study
Materials & PDFs

Click Here to Join Our Official Telegram Channel

Page 728 of 728

Subscribe the Xpress Video Course & Mock Test Package for Bank & Insurance Exams
If there are any suggestions/ errors in our PDFs Feel Free to contact us via this email: admin@exampundit.in

You might also like